You are on page 1of 529

McGILL UNIVERSITY

FACULTY OF SCIENCE

DEPARTMENT OF
MATHEMATICS AND STATISTICS

MATH 141 2009 01


CALCULUS 2

Information for Students


(Winter Term, 2008/2009)
Pages 1 - 19 of these notes may be considered the Course Outline for this course. The page

numbers shown in the table of contents and in the upper right hand corners of pages are not the

same as the numbers of pages in the PDF document. If you wish to print out specific pages, you

should first view the relevant pages at your screen, and determine what are the numbers of the

corresponding PDF pages.

W. G. Brown
April 12, 2009
Information for Students in MATH 141 2009 01

Contents 2 Sketch of Draft Solutions to Quiz Q1 20


2.1 Instructions to students . . . . . 20
1 General Information 1 2.1.1 Monday Versions . . . . 20
1.1 Instructors and Times . . . . . . 1 2.1.2 Tuesday Versions . . . . 21
1.2 Calendar Description . . . . . . 2 2.1.3 Wednesday Versions . . 23
1.2.1 Calendar Description . . 2 2.1.4 Thursday Versions . . . 24
1.2.2 Late transfer from MATH
151/MATH 152 . . . . . 2 A Information Specifically for Students in
1.3 Tutorials . . . . . . . . . . . . . 2 Lecture Section 001 1001
1.3.1 Tutorial Times, Locations, A.1 Timetable for Lecture Section 001
and Personnel (subject to of MATH 141 2009 01 . . . . . 1001
change) . . . . . . . . . 2
1.3.2 Teaching Assistants (TA’s) 4 B Information Specifically for Students in
1.3.3 Thursday, April 09th, 2009 5 Lecture Section 002 2001
1.4 Evaluation of Your Progress . . 5 B.1 Timetable for Lecture Section 002
1.4.1 Your final grade . . . . . 5 of MATH 141 2009 01 . . . . . 2001
1.4.2 WeBWorK . . . . . . . 6 B.2 Supplementary Notes for the Lec-
1.4.3 Written Assignments . . 7 ture of January 05th, 2009 . . . 2003
1.4.4 Quizzes at the Tutorials. 7 B.2.1 §5.1 Areas and Distances 2003
1.4.5 Final Examination . . . 8 B.2.2 §5.2 The Definite Integral 2006
1.4.6 Supplemental Assessments 8 B.3 Supplementary Notes for the Lec-
1.4.7 Machine Scoring . . . . 8 ture of January 09th, 2008 . . . 2017
1.4.8 Plagiarism. . . . . . . . 8 B.3.1 Summary of the last lecture 2017
1.4.9 Corrections to grades . . 9 B.3.2 §5.3 The Fundamental The-
1.5 Published Materials . . . . . . . 9 orem of Calculus . . . . 2018
1.5.1 Required Text-Book . . 9 B.4 Supplementary Notes for the Lec-
1.5.2 Optional Reference Books 10 ture of January 12th, 2009 . . . 2025
1.5.3 Recommended Video Ma- B.4.1 §5.4 Indefinite Integrals
terials . . . . . . . . . . 11 and the “Net Change” The-
1.5.4 Other Calculus Textbooks 12 orem . . . . . . . . . . 2025
1.5.5 Website . . . . . . . . . 12 B.4.2 §5.5 The Substitution Rule 2032
1.6 Syllabus . . . . . . . . . . . . . 13 B.5 Supplementary Notes for the Lec-
1.7 Preparation and Workload . . . 14 ture of January 14th, 2009 . . . 2035
1.7.1 Prerequisites. . . . . . . 14 B.5.1 §5.5 The Substitution Rule
1.7.2 Calculators . . . . . . . 15 (conclusion) . . . . . . . 2035
1.7.3 Self-Supervision . . . . 15 B.5.2 5 Review . . . . . . . . 2047
1.7.4 Escape Routes . . . . . 17 B.6 Supplementary Notes for the Lec-
1.7.5 Terminology . . . . . . 17 ture of January 19th, 2009 . . . 2050
1.8 Communication with Instructors B.6.1 §6.1 Areas between Curves 2050
and TA’s . . . . . . . . . . . . . 18 B.7 Supplementary Notes for the Lec-
1.9 Special Office Hours and Tutorials 19 ture of January 21st, 2009 . . . . 2059
B.7.1 §6.2 Volumes . . . . . . 2059
Information for Students in MATH 141 2009 01

B.8 Supplementary Notes for the Lec- B.14.3 §7.7 Approximate Inte-
ture of January 26th, 2008 . . . 2067 gration (OMIT) . . . . . 2122
B.8.1 §6.3 Volumes by Cylin- B.14.4 §7.8 Improper Integrals . 2122
drical Shells . . . . . . . 2068 B.15 Supplementary Notes for the Lec-
B.8.2 §6.4 Work . . . . . . . . 2074 ture of February 18th, 2009 . . . 2132
B.9 Supplementary Notes for the Lec- B.15.1 §8.1 Arc Length . . . . 2132
ture of January 26th, 2009 . . . 2075 B.15.2 §8.2 Area of a Surface
B.9.1 §6.5 Average value of a of Revolution . . . . . . 2137
function . . . . . . . . . 2076 B.15.3 §8.3 Applications to Physics
B.9.2 §7.1 Integration by Parts 2081 and Engineering (OMIT) 2140
B.10 Supplementary Notes for the Lec- B.15.4 §8.4 Applications to Eco-
ture of February 02nd, 2009 . . 2085 nomics and Biology (OMIT) 2140
B.10.1 §7.1 Integration by Parts B.15.5 §8.5 Probability (OMIT) 2140
(conclusion) . . . . . . . 2085 B.16 Supplementary Notes for the Lec-
B.10.2 §7.2 Trigonometric Inte- ture of March 02nd, 2009 . . . . 2141
grals . . . . . . . . . . . 2090 B.16.1 §10.1 Curves Defined by
B.11 Supplementary Notes for the Lec- Parametric Equations . . 2141
ture of February 04th, 2009 . . . 2092 B.16.2 §10.2 Calculus with Para-
B.11.1 §7.2 Trigonometric Inte- metric Curves . . . . . . 2144
grals (conclusion) . . . . 2092 B.17 Supplementary Notes for the Lec-
B.11.2 §7.3 Trigonometric Sub- ture of March 04th, 2009 . . . . 2151
stitution . . . . . . . . . 2097 B.17.1 §10.2 Calculus with Para-
B.12 Supplementary Notes for the Lec- metric Curves (conclusion) 2151
ture of February 09th, 2009 . . . 2100 B.17.2 §10.3 Polar Coordinates 2152
B.12.1 §7.3 Trigonometric Sub- B.18 Supplementary Notes for the Lec-
stitution (conclusion) . . 2100 ture of March 09th, 2009 . . . . 2160
B.12.2 §7.4 Integration of Ra- B.18.1 §10.3 Polar Coordinates
tional Functions by Par- (continued) . . . . . . . 2160
tial Fractions (Introduc- B.18.2 §10.4 Areas and Lengths
tion, to be continued) . . 2106 in Polar Coordinates . . 2166
B.13 Supplementary Notes for the Lec- B.19 Supplementary Notes for the Lec-
ture of February 11th, 2009 . . . 2108 ture of March 11th, 2009 . . . . 2180
B.13.1 §7.4 Integration of Ra- B.19.1 §10.4 Areas and Lengths
tional Functions by Par- in Polar Coordinates (con-
tial Fractions (conclusion) 2108 clusion) . . . . . . . . . 2180
B.14 Supplementary Notes for the Lec- B.19.2 §10.5 Conic Sections . . 2181
ture of February 16th, 2009 . . . 2117 B.19.3 §11.1 Sequences . . . . 2181
B.14.1 §7.5 Strategy for Integra- B.19.4 §11.2 Series . . . . . . . 2186
tion . . . . . . . . . . . 2117 B.20 Supplementary Notes for the Lec-
B.14.2 §7.6 Integration Using Ta- ture of March 16th, 2009 . . . . 2189
bles and Computer Al- B.20.1 §11.2 Series (conclusion) 2189
gebra Systems (OMIT) . 2122
Information for Students in MATH 141 2009 01

B.20.2 §11.3 The Integral Test B.25.1 Final Examination in MATH


and Estimates of Sums . 2193 141 2008 01 (one version) 2262
B.20.3 Sketch of Solutions to Prob- B.26 Supplementary Notes for the Lec-
lems on the Final Exam- ture of Monday, April 06th, 2009 2270
ination in MATH 141 2005 B.26.1 Final Examination in MATH
01 . . . . . . . . . . . . 2194 141 2008 01 (Version 1,
B.21 Supplementary Notes for the Lec- continued) . . . . . . . 2270
ture of March 18th, 2009 . . . . 2203 B.27 Supplementary Notes for the Lec-
B.21.1 §11.3 The Integral Test ture of Wednesday, April 08th,
and Estimates of Sums 2009 . . . . . . . . . . . . . . . 2274
(conclusion) . . . . . . . 2203 B.27.1 Final Examination in MATH
B.21.2 §11.4 The Comparison 141 2008 01 (Version 1,
Tests . . . . . . . . . . 2208 conclusion) . . . . . . . 2274
B.21.3 Sketch of Solutions to Prob-
lems on the Final Exam- C Problem Assignments from Previous Years 3001
ination in MATH 141 2006 C.1 1998/1999 . . . . . . . . . . . . 3001
01 . . . . . . . . . . . . 2210 C.1.1 Assignment 1 . . . . . . 3001
B.22 Supplementary Notes for the Lec- C.1.2 Assignment 2 . . . . . . 3001
ture of March 23rd, 2009 . . . . 2224 C.1.3 Assignment 3 . . . . . . 3002
B.22.1 §11.4 The Comparison C.1.4 Assignment 4 . . . . . . 3002
Tests (conclusion) . . . 2224 C.1.5 Assignment 5 . . . . . . 3002
B.22.2 §11.5 Alternating Series 2228 C.2 1999/2000 . . . . . . . . . . . . 3003
B.22.3 Solutions to Problems on C.2.1 Assignment 1 . . . . . . 3003
the Final Examination in C.2.2 Assignment 2 . . . . . . 3004
MATH 141 2007 01 . . 2231 C.2.3 Assignment 3 . . . . . . 3006
B.23 Supplementary Notes for the Lec- C.2.4 Assignment 4 . . . . . . 3007
ture of March 25th, 2009 . . . . 2247 C.2.5 Assignment 5 . . . . . . 3009
B.23.1 §11.5 Alternating Series C.2.6 Assignment 6 . . . . . . 3010
(conclusion) . . . . . . . 2247 C.3 2000/2001 . . . . . . . . . . . . 3012
B.23.2 §11.6 Absolute Conver- C.4 2001/2002 . . . . . . . . . . . . 3012
gence and the Ratio and C.5 MATH 141 2003 01 . . . . . . . 3012
Root Tests . . . . . . . . 2249 C.6 MATH 141 2004 01 . . . . . . . 3012
B.24 Supplementary Notes for the Lec- C.7 MATH 141 2005 01 . . . . . . . 3013
ture of March 30th, 2009 . . . . 2253 C.7.1 Written Assignment W1 3013
B.24.1 §11.6 Absolute Conver- C.7.2 Written Assignment W 2 3014
gence and the Ratio and C.7.3 Written Assignment W3 3016
Root Tests (conclusion) . 2253 C.7.4 Written Assignment W 4 3017
B.24.2 §11.7 Strategy for Test- C.7.5 Written Assignment W5 3019
ing Series . . . . . . . . 2256 C.8 MATH 141 2006 01 . . . . . . . 3021
B.25 Supplementary Notes for the Lec- C.8.1 Solution to Written As-
ture of Wednesday, April 01st, signment W1 . . . . . . 3021
2009 . . . . . . . . . . . . . . . 2262
Information for Students in MATH 141 2009 01

C.8.2Solution to Written As- E.9 Supplemental/Deferred Examina-


signment W2 . . . . . . 3024 tion in Mathematics 189-141B
C.8.3 Solutions to Written As- (2000/2001) . . . . . . . . . . . 3147
signment W3 . . . . . . 3025 E.10 Final Examination in Mathemat-
C.8.4 Solutions to Written As- ics 189-141B (2001/2002) . . . 3148
signment W4 . . . . . . 3028 E.11 Supplemental/Deferred Examina-
C.8.5 Solutions to Written As- tion in Mathematics 189-141B
signment W5 . . . . . . 3030 (2001/2002) . . . . . . . . . . . 3150
C.9 MATH 141 2007 01 . . . . . . . 3032 E.12 Final Examination in MATH 141
2003 01 . . . . . . . . . . . . . 3152
D Quizzes from Previous Years 3033 E.13 Supplemental/Deferred Examina-
D.1 MATH 141 2007 01 . . . . . . . 3033 tion in MATH 141 2003 01 . . . 3154
D.1.1 Draft Solutions to Quiz Q1 3033 E.14 Final Examination in MATH 141
D.1.2 Draft Solutions to Quiz Q2 3043 2004 01 . . . . . . . . . . . . . 3155
D.1.3 Draft Solutions to Quiz Q3 3055 E.15 Supplemental/Deferred Examina-
D.1.4 Draft Solutions to Quiz Q4 3070 tion in MATH 141 2004 01 . . . 3164
D.2 MATH 141 2008 01 . . . . . . . 3086 E.16 Final Examination in MATH 141
D.2.1 Draft Solutions to Quiz Q1 3086 2005 01 . . . . . . . . . . . . . 3168
D.2.2 Draft Solutions to Quiz Q2 3099 E.17 Supplemental/Deferred Examina-
D.2.3 Draft Solutions to Quiz Q3 3110 tion in MATH 141 2005 01 . . . 3172
D.2.4 Draft Solutions to Quiz Q4 3121 E.18 Final Examination in MATH 141
2006 01 (One version) . . . . .3175
E Final Examinations from Previous Years 3135 E.19 Supplemental/Deferred Examina-
E.1 Final Examination in Mathemat- tion in MATH 141 2006 01 . . . 3178
ics 189-121B (1996/1997) . . . 3135 E.20 Final Examination in MATH 141
E.2 Final Examination in Mathemat- 2007 01 (One version) . . . . .3182
ics 189-141B (1997/1998) . . . 3136 E.21 Supplemental/Deferred Examina-
E.3 Supplemental/Deferred Examina- tion in MATH 141 2007 01 (One
tion in Mathematics 189-141B version) . . . . . . . . . . . . .
3186
(1997/1998) . . . . . . . . . . . 3138 E.22 Final Examination in MATH 141
E.4 Final Examination in Mathemat- 2008 01 (one version) . . . . . .
3190
ics 189-141B (1998/1999) . . . 3139 E.23 Supplemental/Deferred Examina-
E.5 Supplemental/Deferred Examina- tion in MATH 141 2008 01 (one
tion in Mathematics 189-141B version) . . . . . . . . . . . . .
3200
(1998/1999) . . . . . . . . . . . 3141
E.6 Final Examination in Mathemat- F WeBWorK 4001
ics 189-141B (1999/2000) . . . 3143 F.1 Frequently Asked Questions (FAQ) 4001
E.7 Supplemental/Deferred Examina- F.1.1 Where is WeBWorK? . 4001
tion in Mathematics 189-141B F.1.2 Do I need a password to
(1999/2000) . . . . . . . . . . . 3144 use WeBWorK? . . . . 4001
E.8 Final Examination in Mathemat-
ics 189-141B (2000/2001) . . . 3145
Information for Students in MATH 141 2009 01

F.1.3 Do I have to pay an ad- F.1.15 How many attempts may


ditional fee to use WeB- I make to solve a partic-
WorK? . . . . . . . . . 4002 ular problem on WeB-
F.1.4 When will assignments WorK? . . . . . . . . . 4005
be available on WeBWorK? 4002 F.1.16 Will all WeBWorK as-
F.1.5 Do WeBWorK assign- signments have the same
ments cover the full range length? the same value? 4006
of problems that I should F.1.17 Is WeBWorK a good in-
be able to solve in this dicator of examination per-
course? . . . . . . . . . 4002 formance? . . . . . . . . 4006
F.1.6 May I assume that the
distribution of topics on G Contents of the DVD disks for
quizzes and final exam- Larson/Hostetler/Edwards 5001
inations will parallel the
distribution of topics in H References 6001
the WeBWorK assign- H.1 Stewart Calculus Series . . . . . 6001
ments? . . . . . . . . . 4002 H.2 Other Calculus Textbooks . . . . 6002
F.1.7 WeBWorK provides for H.2.1 R. A. Adams . . . . . . 6002
different kinds of “Dis- H.2.2 Larson, Hostetler, et al. . 6003
play Mode”. Which should H.2.3 Edwards and Penney . . 6003
I use? . . . . . . . . . . 4003 H.2.4 Others, not “Early Tran-
F.1.8 WeBWorK provides for scendentals” . . . . . . 6004
printing assignments in H.3 Other References . . . . . . . . 6004
“Portable Document Format”
(.pdf), “PostScript” (.ps)
forms. Which should I
List of Tables
use? . . . . . . . . . . . 4003 1 Instructors and Times . . . . . . 1
F.1.9 What is the relation be- 2 Schedule and Locations of Tu-
tween WeBWorK and We- torials, as of April 12, 2009. . . 3
bCT? . . . . . . . . . . 4003 3 Tutors’ Coordinates, as of April
F.1.10 Which browser should I 12, 2009 . . . . . . . . . . . . . 4
use for WeBWorK? . . 4004 4 Summary of Course Requirements,
F.1.11 What do I have to do on as of April 12, 2009; (all dates
WeBWorK? . . . . . . 4004 are subject to change) . . . . . . 10
F.1.12 How can I learn how to 5 Some Antiderivatives . . . . . . 2027
use WeBWorK? . . . . 4004 6 Very Short Table of Indefinite In-
F.1.13 Where should I go if I tegrals . . . . . . . . . . . . . . 2028
have difficulties with WeB-
WorK ? . . . . . . . . . 4005
F.1.14 Can the WeBWorK sys-
tem ever break down or
degrade? . . . . . . . . 4005
Information for Students in MATH 141 2009 01

List of Figures 22 The curves with equations r =


1 − cos θ, (θ ≤ 0), and r = 1 +
1 The region(s) bounded by y = sin θ, and the point 12 , π2 . . . . 2222
x2 and y = x4 . . . . . . . . . . 2051 23 The cardioids with equations r =
2 The region(s) bounded by y = 2 + 2 sin θ, r = 6 − 6 sin θ . . . . 2243
sin x, y = sin 2x between x = 0 24 The region bounded by cardioids
and x = π2 . . . . . . . . . . . . 2053 r = 2+2 sin θ, r = 6−6 sin θ and
3 The region(s) bounded by y = containing the point (r, θ) = (1, 0) 2245
8 − x2 , y = x2 between x = ±3 . 2054 25 The curves with equations r =
4 The region(s) bounded by y =
√ 4 + 2 cos θ, r = 4 cos θ + 5 . . . 2276
x + 2, y = x between x = 0 26 The limaçon r = 1 + 2 sin θ . . . 3071
and x = 4 . . . . . . . . . . . . 2056
5 Regions for Example B.31 . . . 2059
6 The region(s) bounded by x +
y = 3 and x = 4 − (y − 1)2 . . . 2070
7 The curve x = cos θ+sin 2θ, y =
sin θ + cos 2θ . . . . . . . . . . 2146
8 The cardioid with equation r =
2(1 − sin θ) . . . . . . . . . . . 2157
9 The lima¸con r = 1 − 3 cos θ, . . 2158
10 The spiral with equation r = θ,
(θ ≥ 0) . . . . . . . . . . . . . . 2161
11 The spiral with equation r = θ,
(θ ≤ 0) . . . . . . . . . . . . . . 2162
12 The full spiral with equation r =
θ, −∞ < θ < +∞ . . . . . . . . 2163
13 The “4-leafed rose” with equa-
tion r = sin 2θ, . . . . . . . . . . 2164
14 The “5-leafed rose” with equa-
tion r = sin 5θ, . . . . . . . . . . 2165
15 The lemniscates r2 = sin 2θ, r2 =
cos 2θ . . . . . . . . . . . . . . 2166
16 Intersecting polar curves r = 1+
sin θ, r2 = 4 sin θ . . . . . . . . 2168
17 Curves r = sin θ, r = cos θ . . . 2170
18 The strophoid r = 2 cos θ − sec θ 2172
19 Curves r = 2 + sin θ, r = 3 sin θ . 2174
20 Intersections of the limaçon r =
1 − 2 cos θ with the circle r = 1 . 2177
21 Intersections of the curve r =
sec θ with the circle r = 1 . . . . 2178
Information for Students in MATH 141 2009 01 1

1 General Information
Distribution Date: January 05th, 2009
(all information is subject to change)
Pages 1 - 19 of these notes may be considered the Course Outline for this course.

These notes may undergo minor corrections or updates during the term: the defini-
tive version will be the version accessible at

http://www.math.mcgill.ca/brown/math141a.html

or on myCourses, at

http://www.mcgill.ca/mycourses/ 1

Students are advised not to make assumptions based on past years’ operations,
as some of the details concerning this course could be different from past years.
Publications other than this document may contain unreliable information about
this course.
All details of the course could be subject to discretionary change in case of force
majeure.

1.1 Instructors and Times

INSTRUCTOR: S. Shahabi Prof. W. G. Brown


(Course Coordinator)
LECTURE SECTION: 1 2
CRN: 576 577
OFFICE: BURN 1243 BURN 1224
OFFICE HOURS: W 13:15 - 14:15
(subject to Th 09:00 - 10:30 F 10:30 - 11:30
change) or by appointment
TELEPHONE: 514-398-3803 514-398–3836
E-MAIL: SHAHABI@ BROWN@
MATH.MCGILL.CA MATH.MCGILL.CA
CLASSROOM: ADAMS AUD ADAMS AUD
CLASS HOURS: MWF 8:35–9:25 h. MW 16:35–17:55 h.

Table 1: Instructors and Times


1
Please note that the statements about MATH 141 in an SUS publication called Absolute Zero were not given
to instructors of this course to check, and some of them may not be currently correct.
Information for Students in MATH 141 2009 01 2

1.2 Calendar Description


1.2.1 Calendar Description
MATH 1412 CALCULUS 2. (4 credits; 3 hours lecture; 2 hours tutorial. Prerequisites:
MATH 139 or MATH 140 or MATH 150. Restriction: Not open to students who have taken
MATH 121 or CEGEP objective 00UP or equivalent; not open to students who have taken or
are taking MATH 122 or MATH 130 or MATH 131, except by permission of the Department of
Mathematics and Statistics. Each Tutorial section is enrolment limited.) The definite integral.
Techniques of integration. Applications. Introduction to sequences and series.

Students Lacking the Prerequisite will, when discovered, be removed from the course.
Students without the prerequisite (or standing in a course recognized by the Admissions
Office as being equivalent to MATH 140) should not assume that, in possibly permitting
MINERVA to accept their registration for MATH 141, the University was tacitly approv-
ing their registration without the prerequisite. In particular, students who obtained a grade
of F in MATH 139/140/150 are expressly excluded from registration in MATH 141, even if
they registered in the course before the failed or missed examination.3

1.2.2 Late transfer from MATH 151/MATH 152


Some students from MATH 151 or MATH 152 may be permitted to transfer into MATH 141
after the end of the Change of Course Period. If your instructor in MATH 151 or 152 advises
you that you are in this category, please send an e-mail message to Professor Brown as soon
as your transfer has been approved.4

1.3 Tutorials
1.3.1 Tutorial Times, Locations, and Personnel (subject to change)
Every student must be registered in one lecture section and one tutorial section for this course.
Tutorials begin in the week of January 12th, 2009. The last tutorials in all tutorial sections will
be in the week beginning Monday, April 06th, 2009. Table 2 gives times, locations, and the
tutor’s name for each of the tutorials; Table 3 gives the tutors’ coordinates. The information
2
The previous designation for this course was 189-141, and the version given in the winter was labelled
189-141B; an earlier number for a similar course was 189-121.
3
Exceptionally, a student whose Faculty has authorized her to write a deferred (not a supplemental) examina-
tion MATH 139/140 may be permitted to remain, at her own risk, in MATH 141, but should discuss her situation
with advisors and instructors.
4
This is to ensure that your WeBWorK account is opened, and that your date of entry to the course is recorded.
Information for Students in MATH 141 2009 01 3

in these tables is subject to change. We try to publicize changes but sometimes we are not
informed in advance.5
You are expected to write quizzes only in the tutorial section in which you are reg-
istered.6 You do not have a licence to move from one tutorial section to another at will,
even if you find the time, location, or personnel of your tutorials either temporarily or
permanently inconvenient; in the latter case the onus is on you to transfer formally to an-
other tutorial section, to change your other classes, or to drop MATH 141 2009 01. Please
remember that transfers must be completed by the Course Change (drop/add) deadline
(January 20th, 2009), and are subject to the maximum capacities established for each
tutorial section7 .

# CRN Day Begins Ends Room Tutor


T003 578 Tue 08:05 09:55 BURN 1B24 S. Nashaat
T004 579 Tue 12:05 01:55 EDUC 624 R. Wilds
T005 580 Tue 14:05 15:55 EDUC 211 A. Mehrabian
T006 581 Tue 16:05 17:55 BURN 1B39 M.-A. Rousseau
T007 582 Tue 16:05 17:55 BURN 1B23 X. Zhang
T008 583 Thurs 14:05 15:55 BURN 1214 L. El-Hajj
T009 584 Thurs 16:05 17:55 BURN 1B39 J. Macdonald
T010 585 Thurs 16:05 17:55 BURN 1B23 L. Candelori
T011 586 Mon 13:35 15:25 EDUC 216 Y. Rabhi
T012 587 Mon 14:35 16:25 ARTS W-20 L. Takei
T013 588 Mon 14:35 16:25 EDUC 624 S. Vassileva
T014 589 Wed 13:35 15:25 LEA 14 A. Botros
T015 590 Wed 14:35 16:25 WONG 1030 P. Rempel
T016 591 Wed 14:35 16:25 EDUC 216 M. Masdeu-Sabate
T017 2071 Mon 13:35 15:25 BURN 1B39 P. Chaput
T018 2072 Wed 13:35 15:25 ARTS W-20 G. Gauthier-Shalom

Some of these room assignments could change before or early in the beginning of the term, as we
have a pending request to upgrade some of the rooms. In any case, all assignments are subject to
change.

Table 2: Schedule and Locations of Tutorials, as of April 12, 2009.

5
The current room for your tutorial should always be available by clicking on “Class Schedule” on MINERVA
FOR STUDENTS, http://www.mcgill.ca/minerva-students/.
6
In some time slots there may be several tutorial sections, meeting in different rooms.
7
Your instructors do not have the ability to change the maximum capacities of tutorials.
Information for Students in MATH 141 2009 01 4

1.3.2 Teaching Assistants (TA’s)


The tutors in MATH 141 2009 01 are graduate students in Mathematics and Statistics. Like
you, they are students, albeit at the graduate level; they have deadlines and commitments and
personal lives, and the time they have available for MATH 141 is limited and controlled by a
collective agreement (union contract). Please respect the important functions that our tutors
provide, and do not ask them for services they are not expected to perform:

• Outside of the normal quiz times in their tutorials, tutors are neither expected nor au-
thorized to administer a special quiz or a quiz that has already been administered to
others.

• Tutors in MATH 141 2009 01 are not permitted to offer paid, private tuition to students
in any tutorial section of this course.

Tutor E-mail address Office Office Hours


BURN Day Begins Ends Day Begins Ends
Botros, A. abotros@math.mcgill.ca 1132 T 14:00 15:30 Th 14:00 15:30
Candelori, L. candelori@math.mcgill.ca 1032 W 13:30 16:30
Chaput, P. chaput@math.mcgill.ca 1032 M 10:00 11:00 M 12:30 13:30
1032 M 15:30 16:30
El-Hajj, L. elhajj@math.mcgill.ca 1133 M 11:00 12:30 Th 16:00 17:30
Gauthier-Shalom, G. ggauthier@math.mcgill.ca 1036 T 12:00 15:00
Macdonald, J. jmacdonald@math.mcgill.ca 1030 M 13:30 16:30
Mehrabian, A. mehrabian@math.mcgill.ca 1023 W 09:30 11:00 Th 09:30 11:00
Nashaat, S. nashaat@math.mcgill.ca 1132 M 10:00 11:30 F 10:00 11:30
Rabhi, Y. yrabhi@math.mcgill.ca 1021 T 09:00 12:00
Rempel, P. rempel@math.mcgill.ca 1140 Th 08:00 09:30 F 11:30 13:30
Rousseau, M.-A. mrousseau@math.mcgill.ca 1021 Th 11:00 14:00
Masdeu-Sabate, M. mmasdeu@math.mcgill.ca 1033 T 17:00 20:00
Takei, L. takei@math.mcgill.ca 1033 F 12:00 15:00
Vassileva, S. svassileva@math.mcgill.ca 1133 M 13:00 14:00 Th 16:00 18:00
Wilds, R. wilds@math.mcgill.ca McInt 1017
Location of Office hours = McInt 1019 W 09:30 11:30 W 13:00 14:00
Zhang, X. xzhang@math.mcgill.ca 1020 T 08:00 09:00 Th 08:00 10:00

During her/his office hours, a tutor is available to all students in the course,
not only to the students of her/his tutorial section.
For last minute changes, see myCourses (WebCT).

Table 3: Tutors’ Coordinates, as of April 12, 2009


Information for Students in MATH 141 2009 01 5

1.3.3 Thursday, April 09th, 2009


The University has decided that this day will be considered a Monday, for timetable purposes,
to replace the missed Easter Monday, April 13th, 2009, which, like Good Friday, April 10th,
2009, is a University holiday. Students in tutorial sections T008, T009, T010 will have a
number of options for this particular tutorial:

1. While the University has officially cancelled tutorials T008, T009, T010 previously
scheduled for April 09th, students of those three tutorial sections are invited — space
permitted — to attend any of the other tutorials on Monday, April 06th, 2009 through
Wednesday, April 08th, 2009.

2. It is hoped that two of the tutors of Thursday tutorials will offer a tutorials, one on
Thursday, 09 April 2009, another on another day of that week.

There will be no quiz that week.

1.4 Evaluation of Your Progress


1.4.1 Your final grade
(See Table 4, p. 10) Your grade in this course will be a letter grade, based on a percentage
grade computed from the following components:

1. Assignments submitted over the Web: Six8 (6) WeBWorK homework assignments —
counting together for 10%.

2. Quizzes given at the tutorials: Four (4), counting together for 20%.9

3. The final examination — counting for 70%.

Where a student’s performance on the final examination is superior to her performance on the
tutorial quizzes, the final examination grade will replace the quiz grades in the calculations;
in that case the grade on the final examination will count for 90% of the final grade. It is not
planned to permit the examination grade to replace the grades on WeBWorK assignments.
8
Numbers of assignments, quizzes, etc., are as planned as of the date of this version of these notes. Students
must be prepared for the possibility that it could be necessary to adjust these numbers during the term. If there
are any changes, these will be announced on myCourses, by broadcast e-mail messages, or by announcements at
the lectures.
9
But be warned: students who fail to write quizzes are often at risk in this course. The quizzes are mainly
a learning, rather than a testing experience. You need the information that comes from writing quizzes in a
group, and observing whether your performance was at an appropriate level. Students who deny themselves this
experience often undergo a rude awakening at the final examination.
Information for Students in MATH 141 2009 01 6

1.4.2 WeBWorK
1. The WeBWorK system, developed at the University of Rochester — is designed to
expose you to a large number of drill problems, and where plagiarism is discouraged.
WeBWorK is accessible only over the Internet. Details on how to sign on to WeBWorK
are contained in Appendix F to these notes, page 4001.
Only answers submitted by the due date and time will count. The WeBWorK assign-
ments which count in your term mark will be labelled A1 , . . ., A6 .

2. Due dates and times for WeBWorK assignments. The due dates for WeBWorK as-
signments will be on specified Sundays, about 23:30h; last minute changes in the due
dates may be announced either on WeBWorK, on myCourses, or by an e-mail message10
As mentioned in the WeBWorK FAQ (cf. Appendix F), if you leave your WeBWorK
assignment until the hours close to the due time on the due date, you should not be
surprised if the system is slow to respond. This is not a malfunction, but is simply a
reflection of the fact that other students have also been procrastinating! To benefit from
the speed that the system can deliver under normal conditions, do not delay your WeB-
WorK until the last possible day! If a systems failure interferes with the due date of
an assignment, arrangements may be made to change that date, and an e-mail message
may be broadcast to all users (to the e-mail addresses on record), or a note posted in the
course announcements on myCourses; but slowness in the system just before the due
time will not normally be considered a systems failure.11

3. Numbers of permitted attempts at WeBWorK questions. While the number of times


you may attempt each problem on WeBWorK An will be limited, there will be a com-
panion “Practice” Assignment Pn (n = 1, 2, . . . , 6) with an unlimited number of attempts
at similar problems, but in which the specific data may be different. Thus you have
the opportunity to prepare yourself on the Practice assignment before attempting the ac-
tual assignment. The practice assignments DO NOT COUNT in your term mark, even
though a grade is recorded.. Practice assignment Pn is normally due one week before
assignment An . Another assignment which will not count will be Practice Assignment
P0 , which is directed to students who are not familiar with the WeBWorK system.
10
Be sure that your e-mail addresses are correctly recorded. See 4, p. 18 of these notes.
11
Should you find that the system is responding slowly, do not submit your solutions more than once; you may
deplete the number of attempts that have been allowed to you for a problem: this will not be considered a systems
failure.
Information for Students in MATH 141 2009 01 7

1.4.3 Written Assignments


There will be no Written Assignments in MATH 141 2009 01.12

1.4.4 Quizzes at the Tutorials.


1. There will be 4 quizzes, numbered Q1 , Q2 , Q3 , Q4 , administered at the tutorials. These
quizzes will be graded, and returned. The primary purpose of a quiz is to diagnose
possible gaps in your understanding. In the grading formula the quiz component of
the final grade will be replaced by the final examination grade, if that is to a student’s
advantage.
2. Students may write a quiz only in the tutorial in which they are registered.
3. Medical absences. If you have missed or expect to miss a quiz for a valid reason (med-
ical or otherwise), please communicate directly with Professor Brown, providing a copy
of the medical or other supporting documents; do not contact your TA. Authorized med-
ical absences can be accommodated only through averaging, as students in MATH 141
are never permitted to write a quiz in any tutorial section other than the one in which
they are registered.
4. To prepare for a quiz you should be working exercises in the textbook based on the ma-
terial currently under discussion at the lectures, and you should have attempted any open
WeBWorK assignments. But, unlike the WeBWorK assignments — where the empha-
sis is on correct answers alone — students may be expected to provide full solutions to
some or all problems on quizzes.13
The quizzes may examine on only a sampling of topics. Students should not assume that
topics not examined are in any subsidiary parts of the syllabus.
5. Your tutors will normally bring graded quizzes to the tutorial to be returned to you.
University regulations do not permit us to leave unclaimed materials bearing names and
student numbers in unsupervised locations; you may be able to recover an unclaimed
quiz from the tutor who graded it, during her/his regular office hours. Be sure to attend
the tutorial following a quiz14 , as claims of incorrect recording of a quiz or assignment
grade will need to be substantiated by a graded paper.
12
While it is not required for grading purposes, students are urged to keep a systematic record of written
solutions to problems in the textbook. This could be in the form of a workbook, or a file, but should be orderly
enough that you can look back at a later time to see your solutions. You are invited to bring such a file to TA’s or
instructors at their office hours, to receive advice about the quality and correctness of your solutions.
13
In Math 141 the general rule for quizzes is that full solutions are expected to all problems, unless you receive
explicit instructions to the contrary: ALWAYS SHOW YOUR WORK! The solutions in the Student Solutions
Manual [9] to the textbook can serve as a guide to what should be included in a “full” solution.
14
The return of Quiz Q1 may be delayed to the 2nd week after the quiz was written.
Information for Students in MATH 141 2009 01 8

1.4.5 Final Examination


A 3-hour-long final examination will be scheduled during the regular examination period for
the winter term (April 15th, 2009 through April 30th, 2009). You are advised not to make any
travel arrangements that would prevent you from being present on campus at any time during
this period.15

1.4.6 Supplemental Assessments


1. Supplemental Examination. There will be a supplemental examination in this course.
(For information about Supplemental Examinations, see

http://www.mcgill.ca/artscisao/departmental/examination/supplemental/.)

2. There is No Additional Work Option. “Will students with marks of D, F, or J have the
option of doing additional work to upgrade their mark?” No. (“Additional Work” refers
to an option available in certain Arts and Science courses, but not available in MATH
141 2009 01.)

1.4.7 Machine Scoring


“Will the final examination be machine scored?” It is possible that the final examination, or
part of it, could be machine scored. Multiple choice problems, possibly machine scored, could
also appear on some quizzes. (Machine grading, if implemented in whole or in part, would be
a change from the practice of past years.)

Answer-only Problems. Some of the problems on your final examination — possibly a sub-
stantial number of them — may request that the answer only be given, and may not carry part
marks which could be based on the work leading up to the answer.

1.4.8 Plagiarism.
While students are not discouraged from discussing methods for solving WeBWorK assign-
ment problems with their colleagues, all work that you submit must be your own. The Senate
of the University requires the following message in all course outlines:

“McGill University values academic integrity. Therefore all students must understand the
meaning and consequences of cheating, plagiarism and other academic offences under the
Code of Student Conduct and Disciplinary Procedures. (See http://www.mcgill.ca/integrity
15
Your instructors learn the date of your examination at the same time as you do — when the Provisional
examination timetable is published.
Information for Students in MATH 141 2009 01 9

for more information).


“L’université McGill attache une haute importance à l’honnêteté académique. Il incombe
par conséquent à tous les étudiants de comprendre ce que l’on entend par tricherie, plagiat
et autres infractions académiques, ainsi que les conséquences que peuvent avoir de telles
actions, selon le Code de conduite de l’étudiant et des procédures disciplinaires. (Pour de
plus amples renseignements, veuillez consulter le site http://www.mcgill.ca/integrity).”
It is a violation of University regulations to permit others to solve your WeB-
WorK problems, or to extend such assistance to others; you could be asked
to sign a statement attesting to the originality of your work. The Handbook
on Student Rights and Responsibilities16 states in ¶A.I.15(a) that
“No student shall, with intent to deceive, represent the work of another person
as his or her own in any academic writing, essay, thesis, research report, project
or assignment submitted in a course or program of study or represent as his or
her own an entire essay or work of another, whether the material so represented
constitutes a part or the entirety of the work submitted.”
You are also referred to the following URL:
http://www.mcgill.ca/integrity/studentguide/

Other Fraud. It is a serious offence to alter a graded quiz paper and return it to the tutor
under the pretense that the work was not graded properly.

1.4.9 Corrections to grades


Grades will eventually be posted on myCourses. If you believe a grade has been recorded
incorrectly, you must advise your tutor not later than 4 weeks after the grade has been posted,
and not later than the day before of the final examination whichever of these dates is earlier. It
is hoped that grades will be posted within 2 weeks of the due date. You will have to present the
graded quiz to support your claim, which must be submitted to the tutor that graded the quiz. If
he/she believes there has been an error, the tutor will advise Professor Brown. New corrections
to the myCourses posting will appear the next time grades are uploaded to myCourses.

1.5 Published Materials


1.5.1 Required Text-Book
The textbook for the course is J. Stewart, SINGLE VARIABLE CALCULUS: Early Tran-
scendentals, Sixth Edition, Brooks/Cole (2008), ISBN 0-495-01169-X, [1]. This book is the
16
http://upload.mcgill.ca/secretariat/greenbookenglish.pdf

UPDATED TO April 12, 2009


Information for Students in MATH 141 2009 01 10

Item # Due Date Details


P0 DOES NOT COUNT: introduces WeBWorK
WeBWorK P1 18 Jan 09 DOES NOT COUNT; practice for A1
Assignments A1 25 Jan 09
(cf. §1.4.2) P2 01 Feb 09 DOES NOT COUNT; practice for A2
10% A2 08 Feb 09
P3 15 Feb 09 DOES NOT COUNT; practice for A3
A3 22 Feb 09
P4 01 Mar 09 DOES NOT COUNT; practice for A4
A4 15 Mar 09
P5 15 Mar 09 DOES NOT COUNT; practice for A5
A5 22 Mar 09
P6 29 Mar 09 DOES NOT COUNT; practice for A6
A6 05 Apr 09 A1 –A6 count equally, but may have different num-
bers of problems.
Quizzes Q1 19–22 Jan 09 Quizzes Q1 — Q4 count equally, but
(cf. §1.4.4) Q2 09–12 Feb 09 the quizzes may be of different lengths.
20% or 0% Q3 16–19 Mar 09
Q4 30 Mar–02 Apr 09
Final Exam 15–30 Apr 09 Date of exam to be announced by Faculty
70% or 90%
Supplemental 19–20 Aug 09 Only for students who do not obtain standing at the
Exam final. Supplemental exams count in your average
(cf. §1.4.6.1) like taking the course again; exam counts for 100%.

Table 4: Summary of Course Requirements, as of April 12, 2009; (all dates are subject to
change)

first half of J. Stewart, CALCULUS: Early Transcendentals, Sixth Edition, Brooks/Cole


(2008), ISBN 0-495-01166-5, [2]; this edition covers the material for Calculus 3 (MATH 222)
as well, but is not the text-book for that course at the present time. The textbook will be sold
in the McGill Bookstore bundled with its Student Solutions Manual (see below). The ISBN
number for the entire bundle is 0-495-42966-X.

1.5.2 Optional Reference Books


Students are urged to make use of the Student Solution Manual:

• D. Anderson, J. A. Cole, D. Drucker, STUDENT SOLUTIONS MANUAL FOR STEW-


ART’S SINGLE VARIABLE CALCULUS: Early Transcendentals, Sixth Edition, Brooks/Cole
Information for Students in MATH 141 2009 01 11

(2008), ISBN 0-495-01240-8, [3]. This book is also sold “bundled” with the text book;
we expect the Bookstore to stock the bundle numbered ISBN 0-495-42966-X [4].

The publishers of the textbook and Student Solutions Manual also produce

• a “Study Guide”, designed to provide additional help for students who believe they
require it: R. St. Andre, STUDY GUIDE FOR STEWART’S SINGLE VARIABLE
CALCULUS: Early Transcendentals, Sixth Edition, Brooks/Cole (2008), ISBN 0-
495-01239-4, [5]. (The “Study Guide” resembles the Student Solution Manual in ap-
pearance: be sure you know what you are buying.)

• a “Companion” which integrates a review of pre-calculus concepts with the contents


of Math 140, including exercises with solutions: D. Ebersole, D. Schattschneider, A.
Sevilla, K. Somers, A COMPANION TO CALCULUS. Brooks/Cole (1995), ISBN 0-
534-26592-8 [39].

1.5.3 Recommended Video Materials


Use of the following materials is recommended, but is not mandatory17 .

Text-specific DVDs for Stewarts Calculus, early transcendentals, 6th edition [videorecord-
ing]. The publisher of Stewart’s Calculus has produced a series of videodisks, [?]. These will
initially be available for reserve loan at the Schulich Library. There may not be DVD viewing
equipment freely available in the library; the intention is that interested students borrow disks
for viewing on their own equipment at home. Disk 1 covers Chapters 1-6 of the textbook.

Videotapes for Stewart’s Calculus The publisher of Stewart’s Calculus had earlier pro-
duced a series of videotapes, [14] Video Outline for Stewart’s Calculus (Early Transcenden-
tals), Fifth Edition. These will be available for reserve loan at the Schulich Library. There may
not be VCR viewing equipment in the library; the intention is that interested students borrow
a tape for viewing on their own equipment at home.

Larson/Hostetler/Edwards DVD Disks A set of video DVD disks produced for another
calculus book, [28] Calculus Instructional DVD Program, for use with (inter alia) Larson /
Hostetler / Edwards, Calculus of a Single Variable: Early Transcendental Functions, Third
Edition [29] is produced by the Houghton Mifflin Company. A copy has been requested to be
17
No one will check whether you have used any of these aids; a student can obtain a perfect grade in the course
without ever consulting any of them. No audio-visual or calculator aid can replace the systematic use of paper
and pencil as you work your way through problems. But the intelligent use of some of these aids can deepen your
understanding of the subject. However, the most important aid is the Student Solutions Manual to the textbook!
Information for Students in MATH 141 2009 01 12

placed on reserve in the Schulich Library. In Appendix G of these notes there are charts that
indicate the contents of these disks that pertain to MATH 141.

Interactive Video Skillbuilder CD for Stewart’s Calculus: Early Transcendentals, 6th


Edition, (similar to [15]) 18 This CD-ROM is included with certain new copies of the text-
book. It contains, after an enlightening “pep-talk” by the author, a discussion of some of the
worked examples in the text-book, followed by a quiz for each section in the book. Some stu-
dents may find the animations of the examples helpful, although the examples are all worked
in the book. You might wish to try some of the quiz questions using paper and pencil, and
then check your answers with those given on the CD. It is not recommended that you attempt
to enter your answers digitally, as this is a time-consuming process, and uses a different input
method from your WeBWorK assignments, which serve the same purpose.

1.5.4 Other Calculus Textbooks


While students may wish to consult other textbooks, instructors and teaching assistants in
Math 141 will normally refer only to the prescribed edition of the prescribed textbook for the
course. Other books can be very useful, but the onus is on you to ensure that your book covers
the syllabus to at least the required depth; where there are differences of terminology, you are
expected to be familiar with the terminology of the textbook.19
In your previous calculus course(s) you may have learned methods of solving problems that
appear to differ from those you find in the current textbook. Your instructors will be pleased
to discuss any such methods with you personally, to ascertain whether they are appropriate to
the present course. In particular, any methods that depend upon the use of a calculator, or the
plotting of multiple points, or the tabulation of function values, or the inference of a trend from
a graph should be regarded with scepticism.

1.5.5 Website
These notes, and other materials distributed to students in this course, will be accessible at the
following URL:

http://www.math.mcgill.ca/brown/math141b.html

The notes will be in “pdf” (.pdf) form, and can be read using the Adobe Acrobat reader, which
many users have on their computers. This free software may be downloaded from the following
URL:
18
The version of this CD-ROM for the 6th edition is being catalogued by the Library; it may not be available
at the beginning of the term.
19
There should be multiple copies of the textbook on reserve in the Schulich library.
Information for Students in MATH 141 2009 01 13

http://www.adobe.com/prodindex/acrobat/readstep.html 20

The questions on some old examinations will also be available as an appendix to these notes
on the Web.21
Where revisions are made to distributed printed materials — for example these information
sheets — we expect that the last version will be posted on the Web.
The notes and WeBWorK will also be available via a link from the myCourses (WebCT)
URL:

http://mycourses.mcgill.ca

Your grades on assignments and quizzes will be posted on myCourses within about 2 weeks
after they become available.

1.6 Syllabus
Section numbers in the following list refer to the text-book [7]. The syllabus will include all
of Chapters 5, 6, 7, 8, 10, 11 with omissions, as listed below.22

Chapter 5: Integrals. §§5.1 – 5.5. The Midpoint Rule, defined in §5.2, and appearing from
time to time subsequently, is not examination material.

Chapter 6: Applications of Integration. §§6.1 – 6.3; §6.5. (§6.4 is not examination mate-
rial, but Science students are urged to read it.)

Chapter 7: Techniques of Integration. §§7.1 – 7.3; §7.4, excluding the Weierstrass substitu-
tion [1, Exercises 57-61]; §7.5 §7.8. (§7.6, intended for use in conjunction with integral
tables and/or computer algebra systems, is not examination material, but students are
advised to try to solve the problems manually; §7.7 requires the use of a calculator or a
computer, and consequently is not examination material.)
20
At the time of this writing the current version appears to be 8.n.
21
There is no reason to expect the distribution of problems on quizzes or in assignments and examinations from
previous years be related to the frequencies of any types of problems on the examination that you will be writing
at the end of the term.
22
If a textbook section is listed below, you should assume that all material in that section is examination
material even if the instructor has not discussed every topic in his lectures; however, the instructors may give you
information during the term concerning topics that may be considered subsidiary.
Do not assume that a topic is omitted from the syllabus if it has not been tested in a WeBWorK assign-
ment or a quiz, or if it has not appeared on any of the old examinations in the course! Some topics to not lend
themselves to this type of testing; others may have been omitted simply because of lack of space, or oversight.
By the same token, you need not expect every topic in the course to be examined on the final examination.
Information for Students in MATH 141 2009 01 14

Chapter 8: Further Applications of Integration. §8.1, §8.2 only. (§§8.3, 8.4 are not exam-
ination material, but students are urged to read the applications relevant to their course
of study; §8.5 is not examination material.)

Chapter 9: Differential Equations. (No part of this chapter is examination material; how-
ever, students are urged to read §9.4 Exponential Growth and Decay).

Chapter 10: Parametric Equations and Polar Coordinates. §§10.1 – 10.4.


(§§10.5, 10.6 are not examination material.

Chapter 11. Infinite Sequences and Series. §§11.1 – 11.7. (§§11.8–11.12 are not examina-
tion material; however, students are urged to peruse these sections.)

Appendices Appendix G contains material shifted from [22, §5.6]. Students are expected to
know the properties themselves, as they were discussed in MATH 139 and MATH 140.
After the class has studied Chapter 5, the definition of the natural logarithm ln x will
Zx
dt
thenceforth be taken to be that given in the Appendix, as .
t
0

Please do not ask the tutors to provide information as to which textbook sections should be
emphasized. Unless you are informed otherwise by the instructors in the lecture sections or
published notes — printed, or mounted on the Web — you should assume that all materials
listed are included in the syllabus. You are not expected to be able to reproduce proofs of the
theorems in the textbook. However, you could be expected to solve problems in which there
might be unspecific real variables, rather than specific numbers, and which problems might
look like textbook theorems.23

1.7 Preparation and Workload


1.7.1 Prerequisites.
It is your responsibility as a student to verify that you have the necessary prerequisite. It would
be foolish24 to attempt to take the course without it.
Students who obtained only a grade of C in MATH 139 or MATH 140 would be advised
to make a special effort to reinforce their foundations in differential calculus; if weakness
in MATH 139 or MATH 140 was a consequence of poor preparation for that course, it is
23
The intention is that you should be learning how to solve problems, but should not have to memorize whole
proofs from the textbook.
24
and contrary to McGill regulations
Information for Students in MATH 141 2009 01 15

not too late to strengthen those foundations as well.25 The fact that MINERVA may permit
you to register does not relieve you from the responsibility to observe university regulations
concerning prerequisites, and exposes you to the risk of failure in a course for which you
are nor properly prepared; students with an F in MATH 139 or MATH 140 could have their
registration in MATH 141 annulled. The regulations are in place to protect you!

1.7.2 Calculators
The use of calculators is not permitted in either quizzes or the examination in this course.
Students whose previous mathematics courses have been calculator-oriented would be advised
to make particular efforts to avoid the use of a calculator in solving problems in this course,
in order to develop a minimal facility in manual calculation. This means that you are urged to
do all arithmetic by hand. Students who use calculators when they answer their WeBWorK
problems are undermining the usefulness of the programme to themselves: learn to use the
built-in calculation capabilities that are present in WeBWorK.

1.7.3 Self-Supervision
This is not a high-school course, and McGill is not a high school. The monitoring of your
progress before the final examination is largely your own responsibility. Students must not
assume that they will be exposed in lectures and tutorials to detailed model solutions for every
type of Calculus 2 problem. It is essential that you supplement these classes with serious work
on your own, carefully reading the textbook and solving problems therein.
While the tutors and instructors are available to help you, they cannot do so unless and until
you identify the need for help. WeBWorK and quizzes are designed to assist you in doing this.
If you encounter difficulties, take them to the tutors during one of their many office hours: you
may attend the office hours of any tutor in the course, and are not restricted to those of the tutor
of the tutorial in which you are registered.

Time Demands of your Other Courses. Be sure to budget enough time to attend lectures
and tutorials, for private study, and for the solution of many problems. Don’t be tempted to
divert calculus study time to courses which offer instant gratification. While the significance of
the tutorial quizzes in the computation of your grade is minimal, these are important learning
experiences, and can assist you in gauging your progress in the course. This is not a course
that can be crammed for: you must work steadily through the term if you wish to develop the
facilities needed for a strong performance on the final examination.
25
The reality of inflated grading at McGill or at your previous institution must not be overlooked: it could
happen that students who obtained a grade higher than C in the prerequisite course do not have adequate
skills to succeed in MATH 141! The onus is on you to seek help and to take remedial actions where necessary.
Information for Students in MATH 141 2009 01 16

Lecture Times, and Preparation for the Lectures The lecture sections in MATH 141 2009
01 meet at the times that have been made available to us: early in the morning, or late in
the afternoon. While these times may not appeal to you, you should not underestimate the
damage you do to your expectations in the course by missing lectures, either occasionally —
when you find it convenient to divert calculus time to other purposes — or systematically. To
extract maximum benefit from the lectures, you should peruse the scheduled material before
coming to class, trying some of the textbook problems; your instructors invite you to draw
their attention to specific difficulties that you encounter before class in the textbook — it may
be possible to respond to these difficulties during the lecture.

Working Problems on Your Own. An effective way to master the calculus is through work-
ing large numbers of problems from the textbook. Your textbook was selected partly because
of the availability of an excellent Student Solutions Manual [9]; this manual has brief but com-
plete solutions to most of the odd-numbered exercises in the textbook. The skills you acquire
in solving textbook problems could have much more influence on your final grade than either
WeBWorK or the quizzes.

When to do the WeBWorK assignment. I recommend that you defer working WeBWorK
problems until you have tried some of the easier odd-numbered problems in the textbook. For
these you (should) have the Student Solutions Manual to help you check your work. Once
you know that you have the basic concepts mastered, then is a good time to start working
WeBWorK problems. But these should be done first from a printed copy of your assignment
— not worked during real time online.

The real uses of WeBWorK and the quizzes. Students often misunderstand the true signif-
icance of WeBWorK assignments and the quizzes. While both contribute to your grade, they
can help you estimate the quality of your progress in the course. Quizzes are administered un-
der examination conditions, so poor performance or non-performance on quizzes can provide
an indicator of your expectations at the final examination; take proper remedial action if you
are obtaining low grades on quizzes26 . Since WeBWorK is not completed under examination
conditions, the grades you obtain may not be a good indicator of your expectations on the ex-
amination; if you require many attempts before being able to solve a problem on WeBWorK,
you should use that information to direct you to areas requiring extra study: the WeBWorK
grades themselves have little predictive use, unless they are unusually low. However, while
both WeBWorK and the quizzes have a role to play in learning the calculus, neither is as im-
portant as reading your textbook, working problems yourself, and attending and listening at
lectures and tutorials.
26
The worst action is to miss the quizzes, and thereby block out an unwelcome message.
Information for Students in MATH 141 2009 01 17

What to strive for on WeBWorK assignments. Since the practice assignments give you
ample opportunity to experiment, your success rate on the assignments “that count” should be
close to 100%. If you are needing more than 2 attempts to solve a WeBWorK problem, then
you are probably not ready to work the assignment. In order to be able to solve a WeBWorK
problem successfully on the first attempt you will need to check your work, and this is a skill
that you will need on the final examination, and in the advanced studies or the real world where
you may eventually be applying the calculus.

1.7.4 Escape Routes


At any time, even after the last date for dropping the course, students who are experiencing
medical or personal difficulties should not hesitate to consult their advisors or the Student
Affairs office of their faculty. Don’t allow yourself to be overwhelmed by such problems; the
University has resource persons who may be able to help you.

1.7.5 Terminology
Do not be surprised if your instructors and tutors use different terminology from what you
have heard in your previous calculus course, particularly if that course was at a high school.
Sometimes the differences are purely due to different traditions in the professions.

“Negative x” Your instructors and tutors will often read a formula −x as minus x, not as
negative x. To a mathematician the term negative refers to real numbers which are not squares,
i.e. which are less than 0, and −x can be positive if x itself is negative.
However, mathematicians will sometimes refer to the operation of changing a sign as the
replacement of x by “its negative”; this is not entirely consistent with the usual practice, but is
an “abuse of language” that has crept into the professional jargon.

Inverse trigonometric functions A formula like sin−1 x will be read as the inverse sine of
x — never as “sine to the minus 1” or “sine to the negative 1”. However, if we write sinn x,
where n is a positive integer, it will always mean (sin x)n . These conventions apply to any of
the functions sin, cos, tan, cot, sec, csc; they also apply to the hyperbolic functions, which we
have met on general functions, so a formula like f 2 (x) does not have an obvious meaning, and
we will avoid writing it when f is other than a trigonometric or hyperbolic function.

Logarithms Mathematicians these days rarely use logarithms to base 10. If you were taught
to interpret log x as being the logarithm to base 10, you should now forget that — although it
could be the labelling convention of your calculator. Most often, if your instructor speaks of a
logarithm, and writes log x, he will be referring to the base e, i.e. to loge ; that is, he is referring
to the function that calculus books call ln. When a logarithm to some other base is intended,
Information for Students in MATH 141 2009 01 18

it will either be denoted by an explicit subscript, as log2 , or some comment will be made at
the beginning of the discussion, as “all logarithms in this discussion are to the base 2”. Your
instructors try to think like mathematicians even when lecturing to their classes, and so we use
the language and terminology we use when talking to each other.

1.8 Communication with Instructors and TA’s


1. E-mail messages to your instructor or your TA should be sent to the addresses shown in
Table 1 and Table 3. Please show your full name and/or student number, so that we can
clearly identify you.

2. The only messages sent through WeBWorK should be those generated by the Feedback
facility: this means a message that refers to a specific problem on a specific WeBWorK
assignment, generated by clicking on the Feedback button while you are working that
problem, and after you have entered your proposed answer(s) into the answer box(es).27

3. Please do not send instructors messages using the Mail facility of myCourses. This
facility is difficult for instructors to use, since it is not integrated with the other mail
services. We normally disable myCourses mail for that reason. If you had a need to send
a message while you are connected to myCourses, just open another window and send a
message with your regular e-mail client.

4. Keep your e-mail address up to date Both myCourses and WeBWorK contain an e-
mail address where we may assume you can be reached. If you prefer to use another
e-mail address, the most convenient way is to forward your mail from your student
mailbox, leaving the recorded addresses in these two systems unchanged. You can enter
or change a forwarding e-mail address by going to http://webmail.mcgill.ca, and logging
in to your student mailbox at po-box.mcgill.ca.

27
This facility should be used sparingly; you should not expect instant response, so questions sent close to the
due time on the due date will not likely receive a reply before the assignment becomes due.
Information for Students in MATH 141 2009 01 19

1.9 Special Office Hours and Tutorials


The following chart will show any special activities that are scheduled during the term. This
table was last updated on April 04th, 2008 TO BE UPDATED.

Review Tutorial TA/Instructor location Date Time

UPDATED TO April 12, 2009


Information for Students in MATH 141 2009 01 20

2 Sketch of Draft Solutions to Quiz Q1


Release Date: 02 February, 2009

2.1 Instructions to students


1. Show all your work. Marks may not be given for answers not supported by a full solu-
tion. For future reference, the form of your solutions should be similar to those shown
in the textbook or Student Solutions Manual for similar problems.

2. In your folded answer sheet you must enclose this question sheet: it will be returned
with your graded paper. (WITHOUT THIS SHEET YOUR QUIZ WILL BE WORTH
0.) All submissions should carry your name and student number.

3. Time = 20 minutes.

4. No calculators are permitted.

2.1.1 Monday Versions


1. [10 MARKS] Use the Fundamental Theorem of Calculus and the chain rule to find the
derivative of the function Z √x
sin(t)
f (x) = √ dt
3 t5
Solution:

• (This step may not be shown explicitly, but it underlies the successful implemen-
tation of the Chain Rule.) Introduction of an intermediate variable: If the new
Zu
√ 0 d sin(t) du
variable/function is called u = u(x) = x, then f (x) = 5
dt ·
du √ t dx
3

• application of the Fundamental Theorem


Zu
d sin(t) sin(u)
dt = .
du √ t5 u5
3

• completion √ √
sin( x) 1 − 12 sin x
√ 5 · 2 x = 2x3
x
Information for Students in MATH 141 2009 01 21

2. [10 MARKS] Compute


(a)
Z 0
8 sec2 (x) dx
−π/3

Solution: [5 MARKS TOTAL]


• state one antiderivative, e.g., 8 tan x
• indicate that the value of the integral is the net change, 8 tan x]0− π
3
• compute the final answer. Students should know the trigonometric functions
of simple submultiples of π.
(b)
Z 1 √
x(5x2 + 4x − 5) dx
0

Solution: [5 MARKS TOTAL]


• state one antiderivative, here the obvious method is to express as a sum of
fractional powers and to integrate each separately:
Z  
5 3 1 2 7 2 5 2 3
5x 2 + 4x 2 − 5x 2 dx = 5 · x 2 + 4 · x 2 − 5 · x 2 + C
7 5 3
• indicate that the value of the integral is the net change,
" #1
2 7 2 5 2 3
5 · x2 + 4 · x2 − 5 · x2
7 5 3 0

10
• compute the final answer correctly = 7
+ 85 − 10
3

2.1.2 Tuesday Versions


1. [10 MARKS] Use the Fundamental Theorem of Calculus and the chain rule to find the
derivative of the function
Z p
ex
7 + ln6 t
f (x) = dt
5 t

Solution:
Information for Students in MATH 141 2009 01 22

• (This step may not be shown explicitly, but it underlies the successful implemen-
tation of the Chain Rule.) Introduction of an intermediate variable: If the new
variable/function is called u = u(x) = e x , then
Z u p
d 7 + ln6 t du
f 0 (x) = dt ·
du 5 t dx

• application of the Fundamental Theorem


Z u p p
d 7 + ln6 t 7 + ln6 u
dt = .
du 5 t u

• completion q

7 + (ln(e x ))6 7 + x6 x √
x
·e = · e = 7 + x6 .
ex ex
While you may use some judgment about how much simplification you expect, I don’t
believe it would not be appropriate to accept a composition like ln(e x ) not simplified.

2. [10 MARKS] Compute


(a)
Z 0
[8 sec(x) tan(x) + 7 cos(x)] dx
−π/6

Solution: [5 MARKS TOTAL]


• state one antiderivative, e.g., 8 sec x + 7 sin x
• indicate that the value of the integral is the net change in the antiderivative,
e.g., [8 sec x + 7 sin x]0− π
6
• compute the final answer correctly.
!
2 1 9 16
(8 + 0) − 8 · √ − 7 · = − √
3 2 2 3
(b)
Z 4
−3x−1 + 5x + 3
√ dx
1 x

Solution: [5 MARKS TOTAL]


Information for Students in MATH 141 2009 01 23

• state one antiderivative, here the obvious method is to express as a sum of


fractional powers and to integrate each separately:
Z ! !
−3x−1 + 5x + 3 2 − 12 2 3 2 1
√ dx = −3 · − x + 5 · x 2 + 3 · x 2 + C
x 1 3 1

• indicate that the value of the integral is the net change,


" ! #4
2 − 12 2 23 2 12
−3 · − x + 5 · x + 3 · x
1 3 1 1

• compute the final answer correctly


! !
6 10 10 97
= + ·8+6·2 − 6+ +6 =
2 3 3 3

2.1.3 Wednesday Versions


1. [10 MARKS] Use the Fundamental Theorem of Calculus and the chain rule to find the
derivative of the function Z ln x √
f (x) = et 1 + t2 dt
5

Solution:

• (This step may not be shown explicitly, but it underlies the successful implemen-
tation of the Chain Rule.) Introduction of an intermediate variable: If the new
variable/function is called u = u(x) = ln x, then
Z u √
0 d du
f (x) = et 1 + t2 dt ·
du 5 dx

• application of the Fundamental Theorem


Z u √ √
d
et 1 + t2 dt = eu 1 + u2 .
du 5

• completion
p 1 p
eln x 1 + (ln x)2 · = 1 + (ln x)2
x
It is essential that eln x be simplified to x for full marks in this part.
Information for Students in MATH 141 2009 01 24

Z π
4
2. [10 MARKS] Compute f (x) dx, where
− π6
(
4 sin x if x≤0
f (x) = π
5 sec x tan x if 0<x< 2

Solution:

• decompose the interval into subintervals matching the intervals where the 2 parts
of the definition apply:
Z π4 Z 0 Z π4
f (x) dx = f (x) dx + f (x) dx
− π6 − π6 0

• matching the different functions to the appropriate subintervals:


Z 0 Z π4 Z 0 Z π4
f (x) dx + f (x) dx = 4 sin x dx + 5 sec x tan x dx
− π6 0 − π6 0

• shift the constants outside of the integration:


Z 0 Z π4 Z 0 Z π
4
4 sin x dx + 5 sec x tan x dx = 4 sin x dx + 5 sec x tan x dx
− π6 0 − π6 0

• find antiderivatives for both of the 2 integrands, e.g., − cos x and sec x
• indicate that the value of each integral is the net change,
π
4[− cos x]0− π + 5[sec x]04
6

• correctly complete the computations


  √ 
π  π   3  √ √ √
4 − cos 0 + cos +5 sec − sec 0 = 4 −1 +  +5( 2−1) = 2 3+5 2−9
6 4 2

2.1.4 Thursday Versions


1. [10 MARKS] Use the Fundamental Theorem of Calculus and the chain rule to find the
derivative of the function Z 7x √
8 + 9t2
f (x) = dt
3x t
Solution:
Information for Students in MATH 141 2009 01 25

• The integral must be split into two, at a convenient place, each integral with one
fixed and one variable limit; note that the point where the integral is split CANNOT
BE 0, since the integrand is undefined there:
Z 7x √ Z 1 √ Z 7x √
8 + 9t2 8 + 9t2 8 + 9t2
f (x) = dt = dt + dt
3x t 3x t 1 t

• one integral must be reversed so that the dependence on x is in the upper limit:
Z 3x √ Z 7x √
8 + 9t2 8 + 9t2
f (x) = − dt + dt
1 t 1 t

• differentiate each of the integrals separately, using the Fundamental Theorem, and
multiply by the factor of the form dudx
from the Chain Rule (see problems on earlier
versions)
p p
d 8 + 9(3x)2 d(3x) 8 + 9(7x)2 d(7x)
f (x) = − · + ·
dx 3x dx 7x dx
• completion
p p √ p
8 + 9(3x)2 8 + 9(7x)2 − 8 + 81x2 + 8 + 9(49)x2
− ·3+ ·7=
3x 7x x
Z √
3
2. [10 MARKS] Compute f (x) dx, where
0

(
3x if 0≤x≤1
f (x) = 6
1+x2
if x>1

Solution:

• for decomposing the interval into subintervals matching the intervals where the 2
parts of the definition apply:
Z √ Z Z √
3 1 3
f (x) dx = f (x) dx + f (x) dx
0 0 1

• for matching the different functions to the appropriate subintervals:


Z √ Z Z √
3 1 3
6
f (x) dx = 3x dx + dx
0 0 1 1 + x2
Information for Students in MATH 141 2009 01 26

• for shifting constants outside of the integration:


Z Z √ Z Z √
1 3 1 3
6 3 1
3x dx + 2
dx = 2x dx + 6 dx
0 1 1+x 2 0 1 1 + x2

• for finding antiderivatives for both of the 2 integrands, e.g., x2 and arctan x
• for indicating that the value of each integral is the net change,
3 21 √
[x ]0 + 6[arctan x]1 3
2
• for correctly completing the computations
3 21 √ 3  √  3 π π 3 π
3
[x ]0 +6[arctan x]1 = (1−0)+6 arctan 3 − arctan 1 = +6 − = +
2 2 2 3 4 2 2
Information for Students in Lecture Section 1 of MATH 141 2009 01 1001

A Information Specifically for Students in Lecture Section


001
A.1 Timetable for Lecture Section 001 of MATH 141 2009 01
Distribution Date: Monday, January 05th, 2009
(Subject to further correction and change.)
Section numbers refer to the text-book.
MONDAY WEDNESDAY FRIDAY
JANUARY
05 §5.1, §5.2 07 §5.3 09 §5.4
Tutorials begin week of January 12th, 2009
12 §5.4, §5.5 14 §5.5 16 §6.1
Course changes must be completed on MINERVA by Tuesday, Jan. 20, 2009
19 §6.2 Q1 21 §6.2, §6.3 Q1 23 §6.3 Q1
WeBWorK Assignment A1 due Jan. 25, 2009
Deadline for withdrawal with fee refund = Jan. 25, 2009
Verification Period: January 26 – 30, 2009
26 §6.5 A1 28 §6.5, §7.1 30 §7.1
FEBRUARY
02 §7.2 04 §7.2, §7.3 06 §7.3
WeBWorK Assignment A2 due Feb. 08, 2009
09 §7.3, §7.4 Q2 A2 11 §7.4 Q2 13 §7.5, §7.8 Q2
Deadline for web withdrawal (with W) from course via MINERVA = Feb. 15, 2009
16 §7.8 18 §8.1, §8.2 20 §8.2
Study Break: February 22 – 28, 2009
No lectures, no regular office hours, no regular tutorials!
WeBWorK Assignment A3 due Feb. 22, 2009
23 NO LECTURE, NO TU- 25 NO LECTURE, NO TU- 27 NO LECTURE, NO TU-
TORIALS A3 TORIALS TORIALS

Notation: An = Regular WeBWorK Assignment An due about 23:30 hours


on the Sunday preceding this Monday
Qn = Quiz Qn will be administered at the tutorials this week.
X = reserved for eXpansion or review
Information for Students in Lecture Section 1 of MATH 141 2009 01 1002

Section numbers refer to the text-book.


MONDAY WEDNESDAY FRIDAY
MARCH
02 §10.1,§10.2 04 §10.3 06 §10.3, §10.4
WeBWorK Assignment A4 due Mar. 08, 2009
09 §10.4 A4 11 §11.1, §11.2 13 §11.2
16 §11.3 Q3 18 §11.4 Q3 20 §11.4, §11.5 Q3
WeBWorK Assignment A5 due Monday, Mar. 22, 2009
23 §11.5 A5 25 §11.6 27 §11.6
30 §11.7 Q4
APRIL
01 X Q4 03 X Q4
This week’s tutorials are the last.
WeBWorK Assignment A6 due Apr. 05, 2009
06 X A6 08 X
13 NO LECTURE, NO TU- 14
TORIALS

Notation: An = Regular WeBWorK Assignment An due about 23:30 hours


on the Sunday preceding this Monday
Qn = Quiz Qn will be administered at the tutorials this week.
X = reserved for eXpansion or review
Information for Students in Lecture Section 002 of MATH 141 2009 01 2001

B Information Specifically for Students in Lecture Section


002
B.1 Timetable for Lecture Section 002 of MATH 141 2009 01
Distribution Date: Monday, January 05th, 2009
(Subject to further correction and change.)
Section numbers refer to the text-book.
MONDAY WEDNESDAY
JANUARY
05 §5.1, §5.2 07 §5.3
Tutorials begin week of January 12th, 2009
12 §5.4, §5.5 14 §5.5
Course changes must be completed on MINERVA by Tuesday, Jan. 20, 2009
19 §6.1 Q1 21 §6.2 Q1
Verification Period: January 26 – 30, 2009
Deadline for withdrawal with fee refund = Jan. 25, 2009
WeBWorK Assignment A1 due Jan. 25, 2009
26 §6.3 A1 28 §6.5, §7.1
FEBRUARY
02 §7.1, §7.2 04 §7.2, §7.3
WeBWorK Assignment A2 due Feb. 08, 2009
09 §7.3, §7.4 A2 Q2 11 §7.4 Q2
Deadline for web withdrawal (with W) from course via MINERVA = Feb. 15, 2009
16 §7.5, §7.8 18 §8.1, §8.2
Study Break: February 22 – 28, 2009
No lectures, no regular office hours, no regular tutorials!
WeBWorK Assignment A3 due Feb. 22, 2009
23 NO LECTURE, NO TUTORIALS A3 25 NO LECTURE, NO TUTORIALS

Notation: An = Regular WeBWorK Assignment An due about 23:30 hours


on the Sunday preceding this Monday
Qn = Quiz Qn will be administered at the tutorials this week.
X = reserved for eXpansion or review
Information for Students in Lecture Section 002 of MATH 141 2009 01 2002

Section numbers refer to the text-book.


MONDAY WEDNESDAY
MARCH
02 §10.1, §10.2 04 §10.2, §10.3
WeBWorK Assignment A4 due Mar. 08, 2009
09 §10.3, §10.4 A4 11 §10.3, §11.1, §11.2
16 §11.2, §11.3 Q3 18 §11.3, §11.4 Q3
WeBWorK Assignment A5 due Monday, Mar. 22, 2009
23 §11.4, §11.5 A5 25 §11.6
30 §11.7 Q4
APRIL
01 X Q4
This week’s tutorials are the last.
WeBWorK Assignment A6 due Apr. 05, 2009
06 X A6 08 X
13 NO LECTURE, NO TUTORIALS

Notation: An = Regular WeBWorK Assignment An due about 23:30 hours


on the Sunday preceding this Monday
Qn = Quiz Qn will be administered at the tutorials this week.
X = reserved for eXpansion or review
Information for Students in Lecture Section 002 of MATH 141 2009 01 2003

B.2 Supplementary Notes for the Lecture of January 05th, 2009


Release Date: January 05th, 2009,
subject to revision

Textbook Chapter 5. INTEGRALS.


(There will be examples, etc., in these notes that were not discussed specifically at the lecture,
because of time constraints.)

B.2.1 §5.1 Areas and Distances


When, in [1, §2.1], the textbook was motivating the differential calculus, it presented two appli-
cations: “The Tangent Problem”, which was geometric; and “The Velocity Problem”, which
was physical. Now, in motivating the integral calculus, the author presents two analogous
problems: “The Area Problem”, which is geometric; and “The Distance Problem”, which is
physical.

The Area Problem. The textbook discusses approximation of the area under the graph y =
f (x) between x = a and x = b — more precisely, the area between the graph, the x-axis, and
the vertical lines x = a and x = b, as a limit of a sum of areas of narrow vertical rectangles.
The approximation is first motivated with simple functions where the area can be bounded
above and below by easily computable sums, which together converge to the same value as
their number approaches ∞ and their width approaches 0. You should become comfortable
using the “sigma notation”, where, for integers n1 and n2 with n1 ≤ n2 , we write
X
n2
f (i)
i=n1

to mean the sum f (n1 ) + f (n1 + 1) + f (n1 + 2) + . . . + f (n2 ) . The textbook then proposes the
following definition of the area between the graph of a function, two vertical lines, and the
x-axis:

Definition B.1 (Preliminary) Let f be a function which is continuous on the interval [a, b].
The area of the region bounded by the graph of f , the x-axis, and the vertical lines x = a
and x = b (where a ≤ b) is the limit of a sum of the areas of rectangles “hanging” from the
graph, as follows: subdivide the interval [a, b] into smaller intervals by points x1 , x2 , . . . , xn−1 ,
where x1 < x2 < . . . < xn−1 , and, for convenience, we define x0 = a, xn = b; select points xi∗
X
n


(i = 1, . . . , n) such that xi ≤ xi+1 ≤ xi+1 , and consider the sum Rn = f xi∗ (xi − xi−1 ).
i=1
Information for Students in Lecture Section 002 of MATH 141 2009 01 2004

(This definition is “preliminary” in that we haven’t yet argued that such a limit need exist.
This is not the type of limit — of a function of one variable — studied in MATH 140. We also
need to clarify what restrictions hold for the points x1 , x2 , . . ., and how we select the points
x1∗ , x2∗ , . . ..)
Example B.1 (cf. [1, Example 5.1.2, p. 357]) Let a and b be non-negative real numbers, and
consider the area under the parabola y = x2 between the vertical lines x = a, x = b, and above
the line y = 0.
1. Let’s first consider the special case where the left side of the region is along the y=axis,
i.e., where a = 0. From this special case we will be able to derive the general solution.
2. Divide the interval [0, b] into n intervals of equal width b−0 n
; thus the points x1 , x2 , . . . are
b−0
chosen to be xi = · i (i = 1, 2, . . . , n). For the sample points xi∗ , let’s consider the
n
points at the right end of each of the subintervals: so xi∗ = xi (i = 1, 2, . . . , n). Then
X n !2 !
b−0 b−0 b−0
Rn = ·i · ·i− · (i − 1) (1)
i=1
n n n
b
where the second factor is simply the common length of all the subintervals, i.e., .
n
The first factor is squared because we are evaluating the function f (x) = x2 at the point
b−0
· i. In order to evaluate this sum we need to know the sum of the squares of the
n
first n positive integers (=natural numbers). If you didn’t learn this in high school, here
it is:
X n
n(n + 1)(2n + 1)
i2 = . (2)
i=1
6
We won’t be able to prove this formula here: a proof requires use of a tool like Mathe-
matical Induction, which we are omitting from the syllabus. Applying (2) to (1) yields
!
n(n + 1)(2n + 1) 3 1
2+ + 2
3 6 3 n n
Rn = b · 3
=b · .
n 6
b3
As n is permitted to become arbitrarily large positively, Rn → .
3
a3
3. If the region starts at the origin and extends to the line x = a, then the area will be .
3
So, if the region we wish to study starts at the line x = a, and ends at the line x = b,
b3 − a3
where b ≥ a, we need only subtract one of these areas from the other, obtaining .
3
Information for Students in Lecture Section 002 of MATH 141 2009 01 2005

The Distance Problem. Here the textbook considers what appears to be a different type
of problem, and shows that the solution is the same type of sum met in the Area Problem
above. In this case the problem is to determine the distance travelled by a particle moving so
that its velocity at time t is a given function f (t). It will be seen that the distance can again be
interpreted as a limit of a sum — the same sum that would be seen if we attempted to determine
the area in the xt-plane under the graph x = f (t).
dx
If a particle is known to be moving along the x-axis at a velocity of v(t) = x0 (t) = (t)
dt
cm/s, how much distance is traversed between time t = a and time t = b? If, by distance, we
mean displacement or “signed distance”, where movement to the right counts positively, and
to the left negatively28 , then the distance is x(b) − x(a); we shall see that this can be interpreted
d
as the area under the graph y = x(t) between x = a and x = b, which we will be denoting
dt
Zb
d
by x(t) dt . When it is intended to consider all motion as non-negative — the way the
dt
a
odometer of an automobile measures distance, then we would want to find the area under the
graph not of the velocity, v(t) = x0 (t), but of the speed, |v(t)| = |x0 (t)|, and the distance travelled
would be Z b Z b
d
0
|v (t)| dt = x(t) dt .
a a dt
But in practice the word distance is often used with either meaning, so care is required.
I have shown that the area under the graph of the velocity represents the directed dis-
tance travelled by the moving particle. But we expressed the velocity as the derivative of the
displacement of the particle relative to some fixed origin; and the distance travelled can be
expressed as the difference between two values of the displacement. This is a special case of
the Fundamental Theorem of the (Integral) Calculus, which will be introduced below.

5.1 Exercises
[1, Exercise 20, p. 365] Determine a region whose area is equal to the limit
X n !10
2 2i
lim 5+ .
n→∞
i=1
n n
Do not evaluate the limit at this time.
Solution: Take the widths of the approximating rectangles to be constant,
2
4x = ;
n
28
Note that, with this definition, a particle that moves around and then returns to the same point will have
travelled a distance of 0.
Information for Students in Lecture Section 002 of MATH 141 2009 01 2006

then n of these constant widths would span a distance of length 2. The limit can be
Pn
seen to be lim 4x · (5 + i4x)10 . The rectangles could be interpreted, for example,
n→∞ i=1
as “hanging by their upper right hand corners” from the curve y = (5 + x)10 above
11 11
the interval 0 ≤ x ≤ 2. (Later in the course we shall see that the area is (5+2)11 −5 =
1,928,498,618
11
.)

[1, Exercise 21, p. 365] Determine a region whose area is equal to the limit
Xn
π iπ
lim tan .
n→∞
i=1
4n 4n

Do not evaluate the limit at this time.


Solution: You should read the discussion of this problem in your Student Solution Man-
ual. What is different from the preceding example to this one, is that we don’t have any
straightforward algebraic way of evaluating the limit of the sum of rectangles obtained
to approximate this area. Thus there is going to be something new in our theory if we
will be able to determine this area exactly. And, in fact, we will be able to evaluate this
area exactly! The upper boundary of the region in this case could be the graph of the
function y = tan x, and the other boundaries of the region could be, in addition to the
π
x-axis, the vertical lines x = 0 and x = .
4

B.2.2 §5.2 The Definite Integral


The formal definition of the integral involves a number of technical difficulties which I shall
not consider in detail in this course. You should read the definition the textbook gives of the
integral [1, p. 366], but you are not going to be asked to work with it in full generality; in fact
the definition given in the textbook is simpler than the definition that is normally used for the
Riemann Integral. We would need to appeal to this definition if we wished to formally prove
all the properties that the author is going to ascribe to the integral; but we shall not attempt to
provide such proofs.
The usual definition of the integral would permit the widths of the subintervals, here de-
noted by ∆x, to be different: ∆x1 for the first subinterval, ∆x2 for the second, etc., and would
then require that the largest of them should approach zero. This technicality is needed for
general functions, but will not be discussed further in this course.
Read the book and be sure you know the definitions of each of the following terms:

• sample points

• definite integral of f from a to b


Information for Students in Lecture Section 002 of MATH 141 2009 01 2007

• integral sign

• integrand

• limits of integration

• lower limit, upper limit

• Riemann sum
Where we take, as the sample points in the subintervals, global maximum points for the func-
tion on the subintervals, we have what is called the upper Riemann sum; analogously, we may
speak of the lower Riemann sum. To prove the existence of the definite integral we would want
to show that the difference between these sums approaches 0 in the limit. This can be shown
to be the case in particular when the function is continuous everywhere, and it is even true in
certain more general situations. In this course we will normally be taking the functions to be
either continuous or, more generally, “piecewise continuous”; that is, we will permit functions
which can be obtained by “gluing” together functions which are continuous over adjacent in-
tervals. As long as there are only a finite number of such components, it can be shown that the
integral exists; it doesn’t matter if the function is discontinuous at the finite number of loca-
tions were the functions are “glued”. But some of the properties we will be using will apply
only to continuous functions, and we may have to break a problem up into parts in order to
solve it. More about this later.

Evaluating Integrals. Among the integrals discussed in this subsection are several that re-
quire the following formulæ for sums of powers of the natural numbers:
X
n
n(n + 1)
i1 = (3)
i=1
2
X
n
n(n + 1)(2n + 1)
i2 = (4)
i=1
6
Xn
n2 (n + 1)2
i3 = (5)
i=1
4

to which we could add the following trivial result29 :


X
n
i0 = n . (6)
i=1

29
A definition that the product of an empty set of numbers is equal to 1 is consistent with the definition of
multiplication of real numbers.
Information for Students in Lecture Section 002 of MATH 141 2009 01 2008

Formulæ 3, 4, 5 are proved in [1, Appendix E], but you are not expected to have read those
proofs. The proofs given are “by Mathematical Induction”.30

Asking the Right Question. The fact that the formulæ for the sums of powers do not appear to
follow any general pattern is not because there is no pattern, but simply that we are “asking the wrong
question”. If, instead, we had asked for the sums of what are called falling factorials, i.e., products
of an integer with successive integers immediately less than it, we would obtain the following, much
prettier results. You do not need to remember these formulæ:
Xn
n
i0 = (7)
i=1
1
X
n
(n + 1)n
i = (8)
i=1
2
X
n
(n + 1)n(n − 1)
i(i − 1) = (9)
i=1
3
X
n
(n + 1)n(n − 1)(n − 2)
i(i − 1)(i − 2) = (10)
i=1
4
A glance at these formulæ, which are certainly “prettier” than the formulæ for the sums of the powers,
shows that the first one, (7) doesn’t look as though it fits. Here again, that is because we are again
“asking the wrong question”. Let’s formulate the results slightly differently, including the term i = 0 in
each of the sums; only in the case of the 0th powers does this make any difference, since 00 is defined
to be 1:
Xn
n+1
i0 = (11)
i=0
1
X
n
(n + 1)n
i = (12)
i=0
2
X
n
(n + 1)n(n − 1)
i(i − 1) = (13)
i=0
3
X
n
(n + 1)n(n − 1)(n − 2)
i(i − 1)(i − 2) = . (14)
i=0
4
Now we can see much further; we can even conjecture that there is a general result that encompasses
all of these particular cases:
Xn
(n + 1)n(n − 1)(n − 2) · . . . · (n − r + 1)
i(i − 1)(i − 2) · . . . · (i − r + 1) = . (15)
i=0
r
30
Mathematical Induction was not an examination topic in MATH 140 2008 09; in the present course you are
not expected to know how to apply Mathematical Induction, but interested students are urged to read about it in
the textbook [1, pp. 77].
Information for Students in Lecture Section 002 of MATH 141 2009 01 2009

And finally, of what use are these formulæ if we need the sums of the powers of the integers, not
the sums of “falling factorials”. Any power of n can be expressed in terms of “falling factorials”, for
example

n1 = n
n2 = n(n − 1) + n
n3 = n(n − 1)(n − 2) + 3n(n − 1) + n ,

so property (15) can provide all the sums we need.


The purpose of this parenthetical discussion is to illustrate that the main challenge in proofs by
induction is making the right guess, rather than in the details of the proof, which may be routine.

Linearity of the summation operator. The textbook discusses some properties of the “sigma”
notation; these could be called the linearity properties of the operator Σ, and are all special
cases of the following:
k+X̀ k+X̀ k+X̀
(rai + sbi ) = r ai + s bi .
i=k i=k i=k

I may have more to say about the sigma notation after I discuss [1, §5.5], where we shall
encounter properties of the integral that have analogues for sums. For the present let it be
P
k+`
noted that the symbol i in ai is not a “free” variable, in that you cannot assign any values to
i=k
it: it performs a function in the symbol, but that function would be performed equally well if
P
k+` P
k+` P
k+`
we replaced i by any other symbol that is not already in use, e.g., au , aλ , a♥ .
u=k λ=k ♥=k

The Midpoint Rule. The “Midpoint Rule” is an approximation formula for definite inte-
grals. Use of an approximation formula entails a willingness to accept an error in the cal-
culation. Mathematicians normally expect to see an estimate of how good or how bad an
approximation can be before recommending their use. A partial justification of the Midpoint
Rule is contained in [1, §7.7], a section that is to be omitted from the syllabus. For that reason
you are asked to omit this subsection: you will not be expected to know anything about the
Midpoint Rule.

Properties of the Definite Integral – Linearity Properties. The textbook lists many prop-
erties of the Definite Integral, proving some of them.
Information for Students in Lecture Section 002 of MATH 141 2009 01 2010

Zb Zb Zb
2. [ f (x) + g(x)] dx = f (x) dx + g(x) dx
a a a

Zb Zb
3. c f (x) dx = c f (x) dx
a a

Zb Zb Zb
4. [ f (x) − g(x)] dx = f (x) dx − g(x) dx
a a a

for any real numbers a, b, c, and any continuous functions f, g.

Some of these properties can be derived from others, or can be combined into a more general
formula. So, for example we can prove that
Z b Z b Z b
(r · f (x) + s · g(x)) dx = r f (x) dx + s g(x) dx (16)
a a a
Z c Z b Z c
f (x) dx = f (x) dx + f (x) dx (17)
a a b

for any real numbers r, s, a, b, c, and these two equations are equivalent to the properties that
the textbook numbers ##2, 3, 4, etc. [1, p. 387]:
Z a
f (x) dx = 0 (18)
a
Z b Z b Z b
( f (x) ± g(x)) dx = f (x) dx ± g(x) dx (19)
a a a
Z b Z b
c f (x) dx = c f (x) dx (20)
a a

The first property in the textbook list,


Z b
c dx = c(b − a) (21)
a

states, for c ≥ 0 and b ≥ a, that the area of a rectangle of width b − a and height c is c(b − a).
Note that all of these properties hold for constants a, b, c that are positive or or negative!
Here one must be careful in interpreting areas, since, in the definite integral, areas are signed
— they are either positive or negative: we associate the positive sign to areas under a graph
above the x-axis, where the lower limit of the integral is not greater than the upper limit. When
Information for Students in Lecture Section 002 of MATH 141 2009 01 2011

the curve is below the x-axis, or the lower limit of the integral is not greater than the upper
limit, the area is negative. (This is the case for part of [1, Exercise 22, p. 377] which is solved
below: there the portion of the area that was below the x-axis cancelled part of the area above
the x-axis; while the net result we obtained — 21 — was positive, it was not equal to the
total of the magnitudes of the two areas above and below the x-axis, but√was equal to their
difference. The graph√of the integrand crosses the x-axis at the points √ ± 286 − 1. The region
under the interval [1, 6−1] can be shown to have (negative) area −4 6+ 3 ; while the region
√ √
over the interval [ 6 − 1, 4] can be shown to have (positive) area 35 3
+ 4 6.)

Properties of the Definite Integral – Additivity of the Interval. A second type or property
listed states, in principle, that the area under a curve is the sum of the areas under any two parts
into which the curve can be decomposed:
Z c Z b Z c
f (x) dx = f (x) dx + f (x) dx (22)
a a b

for any real numbers a, b, c; here again, the constants are not necessarily positive, so one can
interpret the point c as lying outside of the interval [a, b] when a ≤ b. This property implies
another property listed on [1, p. 373]:
Z b Z a
f (x) dx = − f (x) dx (23)
a b

Properties of the Definite Integral – Comparison Properties. The textbook lists 3 proper-
ties, which are interrelated — each of them can be used to prove the other 2.
Z b
f (x) ≥ 0 for a ≤ x ≤ b ⇒ f (x) dx ≥ 0 (24)
a
Z b Z b
f (x) ≥ g(x) for a ≤ x ≤ b ⇒ f (x) dx ≥ g(x) dx (25)
a a
Z b
m ≤ f (x) ≤ M for a ≤ x ≤ b ⇒ m(b − a) ≤ f (x) dx ≤ M(b − a) . (26)
a
Z b
While the integral f (x) dx has, thus far, been defined only for a function f which is con-
a
tinuous on [a, b], we will eventually permit generalizations to that definition. Under those
generalizations, properties (24), (25), (26) will continue to hold wherever they “make sense”31 .
31
We will even have generalizations permitting infinite values for the integral, and the properties will hold
there, provided we don’t have to work with “values” like ∞ − ∞ or 0 × ∞.
Information for Students in Lecture Section 002 of MATH 141 2009 01 2012

Some worked examples based on problems in an earlier edition of your textbook.

Example B.2 ([7, Exercise 18, p. 391]) “Express the limit as a definite integral on the given
Xn
e xi
interval: lim ∆x, on the interval [1, 5].”
n→∞
i=1
1 + xi
Solution: This problem is not stated in perfect mathematical language, but we know what the
textbook means. We are to consider the interval 1 ≤ x ≤ 5 to be subdivided into n subintervals
e xi
of equal length ∆x, so ∆x = 5−1 n
. Then we are to interpret 1+x i
as f (xi∗ ), the value of a function

at a point xi chosen in the ith subinterval, so

1 + (i − 1)∆x ≤ xi∗ ≤ 1 + i∆x .


ex
Since the function f (x) = is continuous, by virtue of [1, Theorems 7,9, pp. 124–125],
1+x
the limit must exist: Z 5 x
X n
e xi e
lim ∆x = dx .
n→∞
i=1
1 + xi 1 1+ x

(It happens that this integral is one that we will be unable to evaluate exactly, but will only be
able to approximate.)

R1
Example B.3 ([7, Exercise 42, p. 392] “Evaluate x2 cos x dx.”
1
Solution: If in (17)
Z c Z b Z c
f (x) dx = f (x) dx + f (x) dx
a a b
we set c = a and b = a, we obtain
Z a Z a Z a
f (x) dx = f (x) dx + f (x) dx
a a a

from which it follows that Z a


f (x) dx = 0 .
a
The present integral is of this type — the limits are equal.
This result could also have been inferred from (26), by taking b = a, i.e. in
Z b
m ≤ f (x) ≤ M for a ≤ x ≤ b ⇒ m(b − a) ≤ f (x) dx ≤ M(b − a)
Za a
⇒ 0=m·0≤ f (x) d f ≤ M · 0 = 0 ,
a
Information for Students in Lecture Section 002 of MATH 141 2009 01 2013

implying that the integral is equal to 0.


Rb
Later in the course we will have a method for evaluating an integral of the form x2 cos x dx,
a
where a, b are any real numbers. That method32 is not required for this “easy” problem.

Example B.4 ([7, Exercise 54, p. 392]) Use the properties of integrals to verify the inequali-
ties, without evaluating the integral:
Z π2
π π
≤ sin x dx ≤
6 π
6
3
,
Solution: The integrand, sin x, is an increasing function when x is in the 1st quadrant; this
π π
implies that, for ≤ x ≤ ,
6 2
1 π π
= sin ≤ sin x ≤ sin = 1 .
2 6 2
π π π
The length of the interval over which the integral is being evaluated is − = . The value
2 6 3
π π
of the integral is then bounded by the areas of two rectangles on the base , : the lower
6 2
bound is given by the rectangle whose height is the minimum value of the function, the value at
x = π6 ; the upper bound is given by the rectangle whose height is the maximum value, attained
at x = π2 :
Z π2
1 π π
· ≤ sin x dx ≤ 1 · .
2 3 π
6
3

3
The exact value of the integral will eventually be seen to be 2
.

Example B.5 ([7, Exercise 58, p. 392]) “Use (26) to estimate the value of the integral
Z 2
(x3 − 3x + 3) dx .”
0

Solution: On the real line the given function has critical numbers at ±1; of these only x = +1
is in the interval of the integral. By the 2nd Derivative Test x = 1 is a local minimum: the
function value there is 1. At the end-points of the interval of integration, 0 and 2, the function
has values 3 and 5. We conclude that, on the given interval, the function values are bounded
as follows:
1 ≤ x3 − 3x + 3 ≤ 5 .
32
Integration “by parts”
Information for Students in Lecture Section 002 of MATH 141 2009 01 2014

The length of the interval is 2 − 0 = 2. The value of the integral is, therefore, bounded between
2 and 10. (Eventually we shall be able to evaluate this integral
! exactly, and shall be able to
24 3 2
show that its value is − · 2 + 3 · 2 = 4 − 6 + 6 = 4.
4 2

5.2 Exercises
[1, Exercise
R4 22, p. 771] “Use the form of the definition of the integral...to evaluate the integral
2
1
(x + 2x − 5) dx.”
Solution: Here we are finding the area under the graph of the polynomial x2 + 2x − 5, a
continuous function, on the interval 1 ≤ x ≤ 4, i.e., the area of the region bounded by the
graph on top, the lines x = 1 and x = 4 on the two vertical sides, and the x-axis on the
bottom. We divide the interval into n parts of length 4x = 4−1 n
, and the end-points of the
ith subinterval are xi−1 = 1+(i−1)∆x = 1+(i−1)· 3n on the left and xi = 1+i·4x = 1+i· 3n
on the right.
If we choose the sample points to be the right end-points of the subintervals,
Z 4
(x2 + 2x − 5) dx
1
X
n
= lim (xi2 + 2xi − 5)∆x
n→∞
i=1
Xn  !2 ! 
 3i 3i  3
= lim  1 + +2 1+ − 5
n→∞
i=1
n n n
X n ! ! !
3i 9i2 3i 3
= lim 1+2· + 2 +2 1+ −5
n→∞ n n n n
i=1n 
X 6 X n
9 X n X n
6 X n Xn
 3
= lim  1 + i+ 2 2
i +2 1+ i−5 1
n→∞ n i=1 n i=1 n i=1 n
 i=1 i=1 i=1

X n
6X
n
9 X 2
n X n
6X
n Xn
 3
= lim  1 + i+ 2 i +2 1+ i−5 1
n→∞ n i=1 n i=1 n i=1 n
 i=1 i=1
 i=1
 X n
6+6X
n
9 X 2  3
n
= 
lim (1 + 2 − 5) 1+ i+ 2 i
n→∞
i=1
n i=1 n i=1  n
!
12 n(n + 1) 9 n(n + 1)(2n + 1) 3
= lim −2n + · + 2·
n→∞ n 2 n 6 n
!!
9 3 3
= lim −2n + 6(n + 1) + 3n + +
n→∞ 2 2n n
= 21
Information for Students in Lecture Section 002 of MATH 141 2009 01 2015

If we choose the sample points to be the left end-points, we obtain a very similar sum.
We can write the sum, like the preceding, as a sum over the index i ranging from i = 1
to i = n, but where the function is evaluated at the left end-points:
Z 4
(x2 + 2x − 5) dx
1
X
n
2
= lim (xi−1 + 2xi−1 − 5)∆x
n→∞
i=1
Xn 
!2 ! 
 3
3(i − 1) 3(i − 1)
= lim  1 + +2 1+ − 5
n→∞
i=1
n n n
X n ! ! !
3(i − 1) 9(i − 1)2 3(i − 1) 3
= lim 1+2· + 2
+2 1+ −5
n→∞
i=1
n n n n
 n 
X 6 Xn
9 Xn X n
6 Xn Xn  3
= lim  1 + (i − 1) + 2 2
(i − 1) + 2 1+ (i − 1) − 5 1
n→∞
i=1
n i=1 n i=1 i=1
n i=1 i=1
n

At this point we can proceed in a variety of ways. One is to observe that the first term
in the sums of powers is 0, so that we are summing only n − 1 non-zero powers. A more
formal way to to that is to define
j=i−1 (27)
and to rewrite each of the sums as a sum over j, replacing the value i = 1 by j = 0 and
the value i = n by j = n − 1. We then obtain
Z 4
(x2 + 2x − 5) dx
1
 n−1 
X 6 X
n−1
9 X
n−1 X
n−1
6 X
n−1 X
n−1 
 3
= lim  1 + j+ 2 2
j +2 1+ j−5 1
n→∞
j=0
n j=0 n j=0 j=0
n j=0 j=0
n
 
 X
n−1
6+6X
n−1
9 X 2  3
n−1

= lim (1 + 2 − 5) 1+ j+ 2 j 
n→∞
j=0
n j=0 n j=0  n
!
12 (n − 1)(n) 9 (n − 1)n(2n − 1) 3
= lim −2n + · + 2·
n→∞ n 2 n 6 n
!!
9 3 3
= lim −2n + 6(n − 1) + 3n − +
n→∞ 2 2n n
= 21

again. The definition of j in (27) is analogous to the changes of variables that we will be
making in definite integrals in [1, §5.5]: there, as here, we have to change the limits of
the integral to correspond to the new values of the variable of integration. What we have
Information for Students in Lecture Section 002 of MATH 141 2009 01 2016

here is an example of the “finite difference calculus”, where there are results similar to
those that we will be developing in the “infinitesimal calculus”.
R10
[1, Exercise 40, p. 378] “Evaluate the integral by interpreting it in terms of areas: |x−5| dx.”
0
Solution: (While we could evaluate this integral using Riemann sums, the intention is
that the student interpret the area under the curve in terms of familiar geometric objects,
and use known formulæ to determine the value.)
The portion of the integral from 0 to 5 is the area of a right-angled triangle whose hy-
potenuse is on the line y = −(x − 5), with base of length 5, and height 5, so its area is
52 25
= . The portion of the integral from 5 to 10 is the mirror image of the triangle
2 2
described above, this time with hypotenuse along the line y = x − 5; its area is the same
as the previous one, so the value of the integral is 25.
Information for Students in Lecture Section 002 of MATH 141 2009 01 2017

B.3 Supplementary Notes for the Lecture of January 09th, 2008


Release Date: Wednesday, January 09th, 2008, subject to revision

To access WeBWorK, link to either

http://msr04.math.mcgill.ca/webwork/m141w09
or
http://msr05.math.mcgill.ca/webwork/m141w09

according as the last digit of your student number is even or odd. Your
user name is your student number; your first password is also your student
number — change the password after you sign on.
You may also access WeBWorK by clicking on the link shown on WebCT:
I have programmed this link to take you to the proper WeBWorK site.

B.3.1 Summary of the last lecture


1. The objective was to define what we mean by the (signed) area under the graph of a
function; or, equivalently, the (signed) distance travelled by a moving particle, giving its
instantaneous velocity.
Zb
2. For a function f continuous on an interval [a, b] I defined the definite integral f (x) dx
a
P
n
as a kind of limit lim f (xi∗ )∆xi
, called a Riemann sum, and mentioned
max ∆xi → 0 i=1
n→∞
that this definition and introduction was simplified for your first encounter, and some-
what lacking in rigour. The treatment of the textbook, which is restricted to continuous
functions, does not require the possibility that the width ∆xi of the ith interval be pos-
sibly different from the that of other subintervals; thus we will be considering only one
width, and denote it simply by ∆x.

3. I mentioned that it can be shown that the sum must have the same limit for all sub-
divisions of [a, b] into subintervals [a = x0 , x1 ], [x1 , x2 ], . . ., [xi−1 , xi ], [xi , xi+1 ], . . .,
[xn−1 , xn = b], and all choices of xi∗ in the ith subinterval (i = 1, . . . , n).

4. I illustrated computations with Riemann sums using the function f (x) = x2 over the
interval [a, b]; the variation I considered had the rectangles “hanging” from the graph
of f by their upper right corners, but I suggest that students should rework the example
with the rectangles hanging by their upper left corners. In my notes, but not discussed in
Information for Students in Lecture Section 002 of MATH 141 2009 01 2018

class, was the example of a similar computation for the function f (x) = x2 + 2x − 5 over
the interval [1, 4]; for part of the interval f (x) < 0, and the value of the integral includes
a cancellation of “negative” and “positive” contributions to area under the graph of f .

5. I reminded you of the formulæ for summing the 0th, 1st, 2nd, and 3rd powers of the first
n natural numbers, and indicated their use in the preceding computation.

6. I indicated that computation of the integral in this way will normally not be necessary,
as we will be meeting a theorem today which will enable much practical computation
for many functions. However, students are still expected to be able to carry out the
calculation of the value of a definite integral using Riemann sums.

7. I stated basic properties of the definite integral, most of which can be derived from (16),
(25), (22):
Z b Z b Z b
(r · f (x) + s · g(x)) dx = r f (x) dx + s g(x) dx
a a a
Z b Z b
f (x) ≥ g(x) for a ≤ x ≤ b ⇒ f (x) dx ≥ g(x) dx
a a
Z c Z b Z c
f (x) dx = f (x) dx + f (x) dx
a a b

B.3.2 §5.3 The Fundamental Theorem of Calculus


In a number of areas of mathematics there are theorems that have acquired the name “The
Fundamental Theorem of...”. The present section is devoted to such a theorem, also known as
“The Fundamental Theorem of (the) Integral Calculus”, one part of which relates the value of
a definite integral to antiderivatives of its integrand, and provides a method for evaluating such
integrals without the need for computing limits of complicated sums. (The formulation of the
theorem as being divided into two parts is not completely standard.)

Differentiation and Integration as Inverse Processes.

Theorem B.6 (The Fundamental Theorem of Calculus) If f is continuous on [a, b], then
Z x !
d
1. f (t) dt = f (x).
dx a
Z b
0
2. F = f ⇒ f (x) dx = F(b) − F(a) .
a
Information for Students in Lecture Section 002 of MATH 141 2009 01 2019

Definition B.2 We may represent a difference F(b) − F(a) by


[F(x)]ba
or even more briefly by
F(x)]ba
if the latter expression is unambiguous.
I sometimes use a notation which is not standard, but is unambiguous, and write the pre-
ceding difference as
F(x)] x=b
x=a
Example B.7 Following my discussion of [1, §5.2] in these notes there is a solution of [1,
Z4  
Exercise 22, p. 377], in which the integral x2 + 2x − 5 dx is evaluated “from first princi-
1
ples”, proving that its value is 21. Let’s now verify that result using the Fundamental Theorem.
x3
One antiderivative of x2 + 2x − 5 is + x2 − 5x. The value of the integral is, therefore,
3
" 3 #4 ! !
x 2 64 1
+ x − 5x = + 16 − 20 − + 1 − 5 = 21 .
3 1 3 3

Integration of “piecewise continuous” functions. The definition [1, Definition 2, p. 366]


of a definite integral given by the textbook, is more restrictive than necessary. This definition
requires that the integrand be continuous throughout the interval of integration. In fact, the
definition can be weakened to apply to a broader class of functions. While we don’t require
full generality in this course, we do wish to be able to apply the theory to functions that have
isolated “jump” discontinuities; (we can handle removable discontinuities just be “removing”
them, i.e., by extending the function to the appropriate value at the points missing in the origi-
nal definition). If a function f is continuous on an interval [a, c], except for a point b in (a, c)
where lim− f (x) and lim+ f (x) both exist, but are not equal, we will define
x→b x→b
Z c Z b Z c
f (x) dx = f (x) dx + f (x) dx (28)
a a b

that is, we will define the integral to be such as to satisfy the additivity property (22) we saw
earlier in these notes, on page 2011. Note, however, that we cannot apply to the whole interval
Rb
the Fundamental Theorem to evaluate integral f (x) dx if f has a discontinuity at a point c
a
such that a < c < b.
Information for Students in Lecture Section 002 of MATH 141 2009 01 2020

Example B.8 Suppose that (


−1 if x ≤ 0
f (x) =
1 if x > 0
Z 2
Evaluate f (x) dx.
−1
Solution: We cannot apply the Fundamental Theorem to the entire interval [−1, 2], as the
integrand is discontinuous at point 0. So we split the integral at the point x = 0, which
is the point of discontinuity. The two integrals we obtain now satisfy the conditions of the
Fundamental Theorem:
Z 2 Z 0 Z 2
f (x) dx = f (x) dx + f (x) dx
−1 −1 0
Z 0 Z 2
= (−1) dx + 1 dx
−1 0
= [−x]0−1 + [x]20 = (−0 + (−1)) + (2 − 0) = 1.

Example B.9 ([7, Exercise 10, p. 402]) “Use Z Part 1 of the Fundamental Theorem of Calculus
u
1
to find the derivative of the function g(u) = 2
dx.”
3 x+ x
1
Solution: The derivative is the value of the integrand, evaluated at the upper limit of the
x + x2
1
integral, i.e., where x = u: g0 (u) = .
u + u2
u+1 4
(Eventually we will see that g(u) = ln − ln ; students may then verify that the
u 3
Fundamental Theorem is giving us the correct derivative.)

Example B.10 ([7, Exercise 12, p. 402]) “Use


Z Part 1 of the Fundamental Theorem of Calculus
10
to find the derivative of the function F(x) = tan θ dθ.”
Rx x
Solution: First observe that F(x) = − 10 tan θ dθ. Having written the integral in the form to
which the Fundamental Theorem applies, i.e., with the variable in the upper limit, we may
apply that theorem: the derivative is minus the value of the integrand, tan θ, evaluated where
θ = x, i.e., − tan x.
(Eventually we will see that F(x) = ln cos x − ln cos(10); again students may then verify
that the Fundamental Theorem is giving us the correct derivative.)

Example B.11 ([7, Exercise 26, p. 402]) “Use Part 2 of the Fundamental Theorem of Calculus
Z3
to evaluate the integral, or explain why it does not exist: x−5 dx.”
−2
Information for Students in Lecture Section 002 of MATH 141 2009 01 2021

Solution: The integrand is not defined at the point 0 in the interval [−2, 3]. This means that
we cannot even speak of the integral at this time. (Later we will generalize our definition of
integral to permit us to consider certain “improper” integrals where there is an infinite discon-
tinuity. But even that generalization will not apply to this problem, although it is premature to
consider it here. Look at this problem again when we study [7, §7.8].) The theorem is not ap-
plicable because the integrand is not defined at one point in the domain, and the discontinuity
is neither removable nor a jump discontinuity: a removable discontinuity would have no effect
at all, and a jump discontinuity could be accommodated by the method of Example B.8 above,
i.e., by splitting the integral into two parts at the jump discontinuity.
(
x if −π ≤ x ≤ 0
Example B.12 ([7, Exercise 42, p. 403]) Let f (x) = . Use Part 2 of
sin x if 0 < x ≤ π

the Fundamental Theorem of Calculus to evaluate the integral f (x) dx, or explain why it
−π
does not exist.
Solution: The function f is defined piecewise, by gluing together one function defined on
[−π, 0] and another defined on (0, +π]; however, it is continuous, as lim− x = 0 = lim+ sin x,
x→0 x→0
and f is defined at x = 0 and equal to the common value of the two one-sided limits. We know
1
that one antiderivative of x is x2 ; and that one antiderivative of sin x is − cos x. Consider the
( 2
x2
if −π ≤ x ≤ 0
function F(x) = 2 . This function is evidently differentiable at all
1 − cos x if 0 < x ≤ π
points except possibly 0; and F is an antiderivative of f on the interval −π < x < +π, with the
possible exception of the point x = 0. At x = 0 the two one-sided limits of difference quotients
are
x2
F(x) − F(0) 3
−0
lim− = lim−
x→0 x x→0 x
x 1
= lim+ = ·0=0
x→0 3 3
F(x) − F(0) (1 − cos x) − 0
lim+ = lim+
x→0 x x→0 x
1 − (1 − 2 sin2 2x ) − 0
= lim+
x→0 x !
x
sin 2 x
= lim+ x · sin
x→0
2
2
x
sin 2 x
= lim+ x · lim+ sin = 1 · 0 = 0 ,
x→0
2
x→0 2
Information for Students in Lecture Section 002 of MATH 141 2009 01 2022

so the function is also differentiable at x = 0, where the derivative is equal to 0, i.e., to f (0);
thus F is an antiderivative of f on the interval (−π, +π). Thus we can apply the Fundamental
Theorem:
Zπ !
(−π)2 π2
f (x) dx = F(π) − F(−π) = (1 − cos π) − =2− .
2 2
−π

But finding the antiderivative was a complicated computation, and rendered the problem more
difficult than necessary. Instead, one should proceed as follows, applying (22) in these notes,
page 2011:

Zπ Z0 Zπ
f (x) dx = f (x) dx + f (x) dx
−π −π 0
Z0 Zπ
= x dx + sin x dx
−π 0
" #
2 0
x
= + [− cos x]π0
−π2
!
(−π)2
= 0− + (− cos π + cos 0)
2
π2 π2
= − + (−(−1) + 1) = 2 − .
2 2
The lesson to be learned from this example is that there are often advantages to splitting up an
integral, even if it is theoretically possible to evaluate it without doing so.

5.3 Exercises
Z 5x
[1, Exercise 54, p. 389] “Find the derivative of the function y = cos(u2 ) du.”
cos x
Solution: Two observations are necessary:

• The Fundamental Theorem is concerned with an integral whose upper limit is vari-
able; if we wish to apply that theorem here, we shall need to transform the problem
to one where only the upper limit of the integral(s) is variable.
• The Fundamental Theorem is concerned with an integral whose upper limit is the
independent variable under consideration; should we wish to permit the upper limit
to vary in a more complicated way, we will need to apply the Chain Rule.
Information for Students in Lecture Section 002 of MATH 141 2009 01 2023

1. We shall transform the given integral into a sum of two where the lower limit of
each is constant. We do this by splitting the interval of integration, [cos x, 5x] into
two parts at a “convenient” point. It is not even necessary that the point we choose
be inside the interval, since the property we are applying, (17) on page 2010 of
these notes, does not require that fact. I will choose the constant 0 to be the point
where the splitting occurs:
Z 5x Z 0 Z 5x
2 2
cos(u ) du = cos(u ) du + cos(u2 ) du
cos x cos x 0
Z cos x Z 5x
2
= − cos(u ) du + cos(u2 ) du
0 0

Hence
Z 5x Z cos x Z 5x
d 2 d 2 d
cos(u ) du = − cos(u ) du + cos(u2 ) du (29)
dx cos x dx 0 dx 0

2. Each of these derivatives will be computed using the Chain Rule, with the interme-
diate variable being the function appearing as the variable upper limit. In the first
case, if we take the intermediate variable to be, say z = cos x, we have
Z cos x Z z
d 2 d dz
cos(u ) du = cos(u2 ) du ·
dx 0 dz 0 dx
dz
= cos(z2 ) ·
dx
2
= cos(cos x) · (− sin x)

For the second integral we take the intermediate variable to be w = 5x. Here
Z 5x Z w
d 2 d dw
cos(u ) du = − cos(u2 ) du ·
dx 0 dw 0 dx
dw
= cos(w2 ) ·
dx
2
= cos(25x ) · 5

Combining the two results gives


Z 5x Z cos x Z 5x
d 2 d 2 d
cos(u ) du = − cos(u ) du + cos(u2 ) du
dx cos x dx 0 dx 0
= − cos(cos2 x) · (− sin x) + cos(25x2 ) · 5
= cos(cos2 x) · sin x + 5 cos(25x2 )
Information for Students in Lecture Section 002 of MATH 141 2009 01 2024

Z x
1
[1, Exercise 58, p. 389] “Find the interval on which the curve y = dt is concave
0 1 + t + t2
upward.”
1 −1 − 2x
Solution: By the Fundamental Theorem, y0 (x) = 2
; hence y00 (x) = .
1+x+x (1 + x + x2 )2
The denominator of the second derivative is a non-zero square, so it is always positive;
1
the function will be positive whenever −1 − 2x > 0, i.e., whenever x < − : this is where
2
the graph is concave upward.
(Eventually we will be able to evaluate the integral explicitly, showing that the curve is
!
2 2x + 1 π
y = √ arctan √ − .
3 3 6
Students may differentiate to check that I have found the correct second derivative.)
Zb Za
x
[1, Exercise 74, p. 390] Suppose that e dx = 3 e x dx . Express b in terms of a.
0 0
Solution: Substituting the values of the given integrals, from Part 2 of the Fundamental
Theorem, yields

e x ]b0 = 3 e x ]a3 ,
which implies that
 
eb − e0 = 3 ea − e0
⇒ eb = 3ea − 2
⇒ b = ln (3ea − 2) .

Problems Plus
[1, Exercise 12, p. 413] Find
Z x Z sin t √ !
d2
1 + u4 du dt .
dx2 0 1

Solution:
Z x Z sin t √ ! Z x Z sin t √ ! !
d2 d d
1 + u4 du dt = 1 + u4 du dt
dx2 0 1 dx dx 0 1
Z sin x √ !
d 4
= 1 + u du
dx 1
p
= 1 + sin4 x · cos x .
Information for Students in Lecture Section 002 of MATH 141 2009 01 2025

B.4 Supplementary Notes for the Lecture of January 12th, 2009


Release Date: Monday, January 12th, 2009, subject to correction

B.4.1 §5.4 Indefinite Integrals and the “Net Change” Theorem


Indefinite Integrals The traditional symbol for a “general” antiderivative
Z F(x) of a function
f (x) (i.e., some function with the property that F 0 (x) = f (x)) is f (x) dx, which is called the
indefinite integral of f (x). Since two antiderivatives differ by a constant (by a corollary to the
Mean Value Theorem), we usually write statements in the form
Z
f (x) dx = F(x) + C

where F(x) is one specific antiderivative, and C is a constant of integration, intended to range
over all real numbers. Once a particular antiderivative F has been chosen, the particular real
number C that applies in a particular situation has to be determined from additional information
that is usually available in the problem at hand. Much of this course will be concerned with
methods for finding indefinite integrals. While the finding of the indefinite integral may be a
difficult problem, the verification that a function F that is claimed to be an antiderivative of f
is not, since all that needs to be done is to differentiate F and to check whether the derivative
is f . (In principle, if a function F has the property that F 0 = f , then F is an antiderivative of
f : thus it is possible to find an antiderivative by guessing, or by simply copying the answer
from the back of the textbook or from your neighbour’s work. The intention in the course is
that you should normally be expected to be able to show a systematic way of determining an
antiderivative; there will be a very few special situations where you will be presented with an
antiderivative without a convincing way of finding it.) Z
Note the same symbol, a stylized letter S — called the integral sign — is used for
both the indefinite integral and the definite integral; while they are related by the Fundamental
Theorem, they are different operations.
Even though the two operations are different, they share some similar properties. For
example, parallel to property (16) of the definite integral on page 2010 of these notes, we can
also prove that Z Z Z
(r f (x) + sg(x)) dx = r f (x) dx + s g(x) dx , (30)

where r, s are any constants. Equations like the preceding, in which an indefinite integral ap-
pears on both sides of the equal sign, are normally written without any constant of integration.

The “Net Change” Theorem This is simply the author’s name for the second part of the
Fundamental Theorem. It is not a term in standard usage, and I am not likely to use it. As
Information for Students in Lecture Section 002 of MATH 141 2009 01 2026

a beginning in the development of general techniques for determining indefinite integrals, we


can reformulate results that we developed for derivatives. For example, since we know that
d
sin x = cos x ,
dx
we can reformulate this result as
Z
cos x dx = sin x + C .

Some reformulations require minor changes, e.g., division by an appropriate constant. From
the result that
d n
x = n · xn−1 when n , 0
dx
we can conclude that Z
1
xn−1 dx = xn + C when n , 0
n
or, after the substitution of m + 1 for n,
Z
1 m+1
xm dx = x + C when m , −1 .
m+1
Thus Table B.4.1 of antiderivatives on page 2027 of these notes, which I included last semester
in my notes in MATH 140 2008 09, can now be recast in the form of Table B.4.1 on page 2028
below.

Applications While we have been using the 2nd part of the Fundamental Theorem to express
the value of a definite integral in terms of the “net change” in the antiderivative, we can also
apply the result in the opposite way: that the net change can be found by evaluating the integral.
This is the spirit of the motivation that the author called “The Distance Problem” (see above,
page 2005, or the solution below, on page 2030 of [1, Exercise 44, p. 397]]). We will see
another example below in the solution of [1, Exercise 62, p. 398].
Example B.13 ([7, Exercise 10, p. 411]) “Find the general indefinite integral of
Z !
2 1
x +1+ 2 dx .”
x +1
Solution: Break the integrand into two parts: the summand at the end is recognizable as the
derivative of arctan x; the two terms at the beginning are multiples of powers of x, and we have
observed earlier how to integrate them. Thus
Z ! Z   Z
2 1 2 1
x +1+ 2 dx = x + 1 dx + dx
x +1 x2 + 1
1 3 1
= x + x + arctan x + C
3 1
Information for Students in Lecture Section 002 of MATH 141 2009 01 2027

Function One antiderivative


f (x) F(x)
g(x) G(x)
f (x) + g(x) F(x) + G(x)
xn+1
xn (n , −1)
n+1
1
ln |x|
x
ex ex
cos x sin x
sin x − cos x
2
sec x tan x
sec x tan x sec x
csc x cot x − csc x
1
arctan x
1 + x2
1
−arccot x
1 + x2
1
√ arcsin x
1 − x2
1
√ − arccos x
1 − x2

Table 5: Some Antiderivatives


Information for Students in Lecture Section 002 of MATH 141 2009 01 2028

Z Z Z Z
[r f (x) + sg(x)] dx = r f (x) dx + s g(x) dx k dx = kx + C
Z Z
xn+1 1
xn dx = +C (n , −1) dx = ln |x| + C
Z n+1 Z x
ax
e x dx = ex + C a x dx = + C (a > 0)
Z Z ln a
sin x dx = − cos x + C cos x dx = sin x + C
Z Z
sec2 x dx = tan x + C csc2 x dx = − cot x + C
Z Z
sec x tan x dx = sec x + C csc x cot x = − csc x + C
Z Z
1 1
= arctan x + C = −arccot x + C
Z 1 + x2 Z 1 + x2
1 1
√ = arcsin x + C √ dx = − arccos x + C
1 − x2 1 − x2

Table 6: Very Short Table of Indefinite Integrals


Information for Students in Lecture Section 002 of MATH 141 2009 01 2029

where the letter C represents the constant of integration. Even though there are two indefinite
integrals on the right side of the equation, only one constant is needed: if we were to include
two constants, as +C1 + C2 , we would not gain any more freedom.

Example B.14 ([7, Exercise 11, p. 411]) Find the general indefinite integral of
Z 
√ 2
2 − x dx .

Solution: One might be tempted, at first, to consider the possibility


√ that the antiderivative of
the given 2nd power is a multiple of the 3rd power of 2 − x; unfortunately that temptation
will have to be resisted, as the resulting functions will
√ not have the correct derivative. The
1
2
simplest approach is to expand the square: since (2 − x) = 4 − 4x 2 + x,
Z  Z 
√ 2 √ 
2 − x dx = 4 − 4 x + x dx
2 3 1
= 4x − 4 · x 2 + x2 + C
3 2

5.4 Exercises

[1, Exercise 34, p. 397] “Evaluate the integral


Z 9
3x − 2
√ dx.”
1 x

Solution: Simplify the integrand by dividing the denominator into the two summands of
the numerator:
Z 9 Z 9 !
3x − 2 √ 2
√ dx = 3 x − √ dx
1 x 1 x
" #9
2 32 1
= 3 · · x − 2 · 2 · x2
3 1
h 3 √ i9
= 2x 2 − 4 x
1
= (2 · 27 − 4 · 3) − (2 · 1 − 4 · 1) = (54 − 12) − (2 − 4) = 44 .

[1, Exercise 38, p. 397] “Evaluate the integral


Z π
3 sin θ + sin θ · tan2 θ
dθ .”
0 sec2 θ
Information for Students in Lecture Section 002 of MATH 141 2009 01 2030

Solution: When the integrand involves trigonometric functions, one may have to apply a
familiar identity to simplify the integration. There is often more than one way to do this.
In the present example
sin θ + sin θ · tan2 θ
= sin θ · cos2 θ + sin3 θ
sec2 θ
= sin θ · (cos2 θ + sin2 θ) = sin θ

so
Z π Z π
3 sin θ + sin θ · tan2 θ 3
dθ = sin θ dθ
0 sec2 θ 0
π π 1 1
= [− cos θ]03 = − cos + cos 0 = − + 1 = .
3 2 2
Eventually you will know how to evaluate the parts of this integral separately, but the
present solution is faster than evaluating and adding the parts.

Z2
[1, Exercise 44, p. 397] Evaluate the integral | sin x| dx.
0
Solution: The integrand is continuous, so we know the integral exists. However, it is
not convenient to work with an antiderivative
h i of | sin x |. So we split the interval of

integration into the parts [0, π] and π, 2 where the integrand is respectively positive
and negative, and thereby avoid working with the absolute value. For the integrands,
respectively sin x and − sin x, we know antiderivatives − cos x and + cos x, so we may
apply the Fundamental Theorem to each part separately:
Z 3π2 Z π Z 3π2
| sin x| dx = sin x dx + (− sin x) dx
0 0 π

= [− cos x]π0
+ [cos x]π2
= (−(−1) + 1) + (0 − (−1)) = 3.

[1, Exercise 58, p. 398] The velocity function is v(t) = t2 − 2t − 8 for a particle moving along
a line. Find
(a) the displacement; and
(b) the distance travelled by the particle
during the time interval 1 ≤ t ≤ 6.
Solution:
Information for Students in Lecture Section 002 of MATH 141 2009 01 2031

(a) The displacement is simply the difference between initial and final positions of the
moving particle; it is equal to the area under the graph of the velocity function
between the appropriate times.
Z 6
displacement = (t2 − 2t − 8) dt
1
" #6
13 2
= t − t − 8t
3 1
!
1 10
= (72 − 36 − 48) − −1−8 =−
3 3

(b) The distance travelled is equal to the area under the graph of the speed function
between the appropriate times; the speed is the magnitude of the velocity.
Z 6 Z 6
distance travelled = 2
t − 2t − 8 dt = |(t + 2)(t − 4)| dt .
1 1

The function (t + 2)(t − 4) changes sign at t = −2, which is outside the interval of
integration, and again at t = 4, which is inside the interval of integration. We can
break the integral up into two parts at x = 4, and then we can express each of the
parts without using absolute value symbols:
Z 6 Z 4 Z 6
|(t + 2)(t − 4)| dt = |(t + 2)(t − 4)| dt + |(t + 2)(t − 4)| dt
1 1 4
Z 4 Z 6
= − (t + 2)(t − 4) dt + (t + 2)(t − 4) dt
1 4
Z 4 Z 6
2
= − (t − 2t − 8) dt + (t2 − 2t − 8) dt
1 4
" #4 " #6
13 2 13 2
= − t − t − 8t + t − t − 8t
3 3
" # 1 4
44 98
= −[−18] + =
3 3

Note that the factorization of the quadratic was needed in order to determine where the
split the interval of integration, but it did not help in the actually integration operation,
and had to be reversed at that stage.

[1, Exercise 62, p. 398] “Water flows from the bottom of a storage tank at a rate of r(t) =
200 − 4t litres/min, where 0 ≤ t ≤ 50. Find the amount of water that flows from the tank
during the first 10 minutes.”
Information for Students in Lecture Section 002 of MATH 141 2009 01 2032

Solution: Let’s denote by W(t) the amount of water (measured in litres) that has left the
tank by time t. We are told that
d
W(t) = r(t) = 200 − 4t (0 ≤ t ≤ 50).
dt
Then
Z 10
dW
W(10) − W(0) = dt
0 dt
Z 10 h i10
= (200 − 4t) dt = 200t − 2t2
0
0
2
= (200 · 10 − 2 · 10 ) − 0 = 1800 ,

so the total amount of water leaving the tank in the first 10 minutes is 1800 litres.

B.4.2 §5.5 The Substitution Rule


The “Substitution Rule” is a reformulation, in terms of the integral, of the Chain Rule.
Theorem B.15 Let u = g(x) and f (x) be respectively differentiable and continuous on a given
interval; then
Z Z Z
0 0
( f ◦ g)(x) · g (x) dx = f (g(x)) · g (x) dx = f (u) du .

In applying this theorem we usually begin with a “complicated” integral, whose form we try
to interpret like the left side of the above equation, and try to find an appropriate “substitution”
of the form u = g(x) which will transform the integral into one whose integrand is one that we
are able to integrate. In practice one works with the differentials dx and du in a “mechanical”
way that can be justified by the theorem.
Proof: Let F and g be differentiable. Then
d
F 0 (g(x)) · g0 (x) = F(g(x)) .
dx
Integrating with respect to x, we have
Z Z
0 0 d
F (g(x)) · g (x) dx = F(g(x)) dx = F(g(x)) + C.
dx
If we define u = g(x), f = F 0 , then
Z Z Z
0 d
f (g(x)) · g (x) dx = F(g(x)) dx = F(u) + C = f (u) du .
dx
Information for Students in Lecture Section 002 of MATH 141 2009 01 2033


In practice this “substitution” is often applied by “mechanically”, substituting functions
and differentials, and such operations can be shown to be fully justifiable. The general idea in
looking for substitutions is to try to reduce the complication of the original indefinite integral.
This is a subjective term, and different users may find a variety of distinct substitutions which
they will find helpful in evaluating an indefinite integral.

Finding the appropriate substitution is one step in solving a problem. In the first prob-
lems in the list of exercises the author suggests a substitution which will be helpful; eventually
students are expected to find an appropriate substitution on their own — often there are several
possible choices. You should be experimenting with different substitutions and getting to know
the types of problems each of them is useful in solving.

Example B.16 Earlier, in Example B.14 on page 2029 of these Z notes, I considered [7, Exercise
 √ 2
11, p. 411], which was concerned with the indefinite integral of 2 − x dx. We evaluated
this integral by expanding the square and then integrating
Z  the powers of x separately. Could
√ 10000
we use the same methods for the indefinite integral 2− x dx?
Solution: While we could expand the integrand in this case too, the result would have 10,001
terms, each of which would have to be integrated. The result we would obtain would not
be very useful. Consider the following alternative approach, that
Z could also have been used
when the exponent was 2. We have here an integral of the form ( f ◦ g)(x) · g0 (x) dx, where
√ √ 1
f (x) = x10000 , and g(x) = 2 − x. Define u = g(x) = 2 − x, so that du = − √ dx, so
2 x

dx = −2 x du = 2(u − 2)du .

Z  Z
√ 10000
2− x dx = 2 u10000 (u − 2) du
Z  
= 2 u10001 − 2u10000 du
2 4
= u10002 − u10001 + C
10002 10001
2 √ 4 √
= (2 − x)10002 − (2 − x)10001 + C
10002 √ 10001 !
√ 10001 2 x 4
= (2 − x) − − +C
10002 (10001)(10002)
Information for Students in Lecture Section 002 of MATH 141 2009 01 2034

R
Example B.17 [1, Example 6, p. 403] “Calculate tan x dx.”
Solution: This problem does not, at first, appear to be a candidate for a substitution; but, by
sin x
expressing the tangent as , the textbook shows that it can be simplified by treating cos x
cos x
as the new variable. That is, by defining u = cos x, which implies that du = − sin x dx, we can
evaluate the integral as follows:
Z Z
sin x dx
tan x dx =
Z cos x
d(cos x)
= −
cos x
Z Z
du
which has the form − , which we should recognize as − d(ln |u|). Thus one sub-
u R
stitution that is indicated is u = cos x: the integral becomes − d(ln |u|) = − ln |u| + C =
− ln | cos x| + C. The integral could also be expressed as ln | sec x| + C or as ln |k sec x|, where
C or k are constants of integration.
Information for Students in Lecture Section 002 of MATH 141 2009 01 2035

B.5 Supplementary Notes for the Lecture of January 14th, 2009


Release Date: Wednesday, January 13th, 2009 (subject to revision)

B.5.1 §5.5 The Substitution Rule (conclusion)


Review The Substitution Rule (cf., p. 2032) states that
Let u = g(x) and f (x) be respectively differentiable and continuous on a given
interval; then
Z Z Z
0 0
( f ◦ g)(x) · g (x) dx = f (g(x)) · g (x) dx = f (u) du
R
In Example B.17 on page 2034 I used a substitution to evaluate tan x dx. Three other indefi-
nite integrals can be evaluated in similar ways; perhaps you should remember33 them together:

Z
tan x dx = ln | sec x | + C = − ln | cos x | + C
Z
cot x dx = − ln | csc x | + C = ln | sin x | + C
Z (31)
tanh x dx = ln cosh x + C
Z
coth x dx = ln | sinh x | + C

(Why are absolute signs missing in one of these cases?)


Z
Example B.18 ([7, Exercise 2, p. 420]) “Evaluate the integral x(4 + x2 )10 dx by making the
substitution u = 4 + x2 .”
Solution: First we should observe that we could solve this problem without any substitution
at all. We could expand the 10th power of the binomial into a polynomial of degree 20,
multiply each of the terms by an additional x, and then integrate the resulting sum of 11 terms
one by one. The resulting polynomial would be the correct solution. But what would you
do it the exponent were not 10, but 10000? Applying the method of substitution, we set
du
u = g(x) = 4 + x2 , f (u) = u10 , so = g0 (x) = 2x. Then
dx
Z   Z  10 1 d  
2 10
x 4+x dx = 4 + x2 · 4 + x2 dx
2 dx
33
This doesn’t mean to memorize them — just to remember the “trick” needed to evaluate them.
Information for Students in Lecture Section 002 of MATH 141 2009 01 2036

Z
1 d  11 
= 4 + x2 dx
2 dx
1  11
= 4 + x2 + C .
22
In practice we often operate mechanically with differentials; from u = 4 + x2 we have du =
2x dx, so
Z   Z Z
2 10 10
x 4+x dx = xu dx = u10 · x dx
Z
1 1
= u10 · du = u11 + C .
2 22
But it would be bad form to stop here, since our answer has been expressed in terms of u, not
the original variable x; so we continue
1  11
= 4 + x2 + C .
22
Example B.19 ([7, Exercise 30, p. 421]) Evaluate the indefinite integral
Z
ax + b
√ dx .
ax2 + 2bx + c
Solution: Later in the term we will consider the integration of any function of the form
kx + `
√ ,
ax2 + 2bx + c
but this problem concerns a special case which is easy to handle. (Note that the textbook
should have insisted that not all of a, b, c can be 0, as then the ratio is not defined.)
Again, we begin by attempting to simplify the integrand. It would appear that the quadratic
function whose square root appears in the denominator is the feature we should attempt to
simplify. One way to do this is to define u = ax2 + 2bx + c. This implies that

du = 2(ax + b) dx,

so we obtain
Z Z
ax + b 1 du
√ dx = √
ax2 + 2bx + c 2 u
1
= u2 + C
1
= (ax2 + 2bx + c) 2 + C .
1
The problem could also be solved by the substitution v = (ax2 + 2bx + c) 2 .
Information for Students in Lecture Section 002 of MATH 141 2009 01 2037

Example B.20 ([7, Exercise 38, p. 421], slightly changed) Evaluate the indefinite integral
Z √
7
x3 + 1 · x5 dx .

Solution: The method I propose to use here is based on the fact that

d(xn ) = nxn−1 dx

for any integer n. When an integrand is expressible in terms of a power of the variable, short by
just 1, then this type of substitution is often a good way to begin simplifying it, although further
steps might be needed. So, in this case, I begin with u = x3 , which implies that du = 3x2 dx,
i.e., that x2 dx = 13 du. I then obtain
Z √ Z √
7
5 1 7
x3 + 1 · x dx = u + 1 · u du
3
after which I consider further simplification. At√this point I would like to simplify the 7th root.
7
I can do this by either setting v = u+1, or w = u + 1. In the first case I would obtain dv = du,
while, in the second, u + 1 = w7 , so du = 7w6 dw.
Z √ Z
1 7 1 √7
u + 1 · u du = v · (v − 1) dv
3 3Z
1  8 1

= v 7 − v 7 dv
3
" #
1 7 157 7 87
= v − v +C
3 15 8
" #
1 7 15 7 8
= (u + 1) 7 − (u + 1) 7 + C
3 15 8
" #
1 7 3 15 7 3 8
= (x + 1) 7 − (x + 1) 7 + C
3 15 8
Z p Z  
1 1
w · w7 − 1 · 7w6 dw
7
or (u + 1) · u du =
3 3Z
7  
= w14 − w7 dw
3
" #
7 1 15 1 8
= w − w +C
3 15 8
7 15 7 8 8
= (u + 1) 7 − (u + 1) 7 (u + 1) 7 + C
45 24
7 3 15 7 8 8
= (x + 1) 7 − (x3 + 1) 7 (x3 + 1) 7 + C .
45 24
Information for Students in Lecture Section 002 of MATH 141 2009 01 2038

Definite Integrals Thus far I have been applying the Substitution Rule to indefinite integrals.
Substitution may be applied to a definite integral
Z b
f (g(x)) g0 (x) dx
a

in two ways:

• Apply the Substitution Rule to the corresponding indefinite integral,


Z
f (g(x))g0 (x) dx ;

then apply the second part of the Fundamental Theorem to the resulting antiderivative;
or

• A variant of the Substitution Rule can be formulated specifically for definite integrals.
Using the same functions as described earlier, it is
Z b Z g(b)
0
f (g(x))g (x) dx = f (u) du .
a g(a)

That is, change the limits of the new integral to the values that u has when x = a and
x = b.

These two methods are equivalent — use whichever you prefer.

Symmetry The textbook reviews the definitions of even and odd functions, and considers
their definite integrals over a finite interval centred at the origin. The author shows that the
definite integral of an even function f over the interval −a ≤ x ≤ +a will be twice the integral
over the interval 0 ≤ x ≤ a; and the integral of an odd function over the same interval will be
0, because the integral to the left of 0 will cancel the integral to the right of 0. The proofs are
simple applications of the Substitution Rule.
Z π6
Example B.21 Evaluate the definite integral tan2 θ dθ.
− π6
Solution: Normally we will evaluate an integral of this type by replacing the integrand, tan2 θ,
by sec2 θ − 1. In this case we can also use the symmetry of the integrand and the interval of
integration to further simplify the calculations:
Z π
6
Z π
6  
2
tan θ dθ = sec2 θ − 1 dθ
− π6 − π6
Information for Students in Lecture Section 002 of MATH 141 2009 01 2039

Z π
6  
= 2 sec2 θ − 1 dθ
0
since the integrand is an even function (Prove this!)
π
= 2 [tan θ − θ]06
!
1 π
= 2 √ − .
3 6

5.5 Exercises
Z
tan−1 x
[1, Exercise 28, p. 407] “Evaluate the indefinite integral dx.”
1 + x2
Solution: In looking for a substitution, our general intention is to try to simplify the
integrand. In the present integrand, we might be expected to consider the arctangent
factor the most complicated, so I will try to simplify by the substitution u = tan−1 x.
1
This implies that du = dx, which is also present in the integral. The integral
1 + x2
becomes Z Z
tan−1 x 1 2 (tan−1 x)2
dx = u du = u + C = +C.
1 + x2 2 2
We can verify our work by differentiating the function we claim to be an antiderivative:
!
d 1  −1 2 1 −1 1 tan−1 x
tan x = · 2 tan x · = .
dx 2 2 1 + x2 1 + x2
Z
[1, Exercise 24, p. 407] Evaluate the indefinite integral (1 + tan θ)5 sec2 θ dθ .

Solution: In this case one should observe that there is a factor sec2 θ, which we recognize
as the derivative of tan θ. A first simplification would be obtained by the substitution

u = tan θ

since then du = sec2 θ dθ, or dθ = cos2 θ du. The integral transforms to


Z Z
5 2
(1 + tan θ) sec θ dθ = (1 + u)5 sec2 θ cos2 θ du
Z
= (1 + u)5 du .

One can observe that the integral here will be a constant multiple of (1 + u)6 , and then
replace u by tan θ. If you don’t make the observation, a second substitution would be in
Information for Students in Lecture Section 002 of MATH 141 2009 01 2040

order. I would observe that, in the last mentioned integral, the “most complicated” factor
is 1 + u; so a substitution v = 1 + u could be attempted. This yields

Z dv = 0
Z + du
⇒ (1 + u)5 du = v5 dv
1 6
= ·v +C
6
1
= · (1 + u)6 + C
6
1
= · (1 + tan θ)6 + C .
6
Z
sin x
[1, Exercise 36, p. 407] “Evaluate the indefinite integral dx.”
1 + cos2 x
Solution: First we see the cosine in the denominator; then we see in the numerator
sin x dx,Z which is −d(cos x). This suggests a substitution u = cos x. The integral be-
−du
comes du = − arctan u + C = − arctan(cos x) + C.
1 + u2
Z
x
[1, Exercise 42, p. 407] “Evaluate the indefinite integral dx.”
1 + x4
Solution: (I am going to begin the solution by making a poor choice for a substitution,
a choice that I will then “fix” by following it with a second substitution. Then I will
observe what would have been a better first choice.) Examining the integral we see that
the powers of x present are x1 in the numerator, and x4 in the denominator. A first idea
might be to try u = x4 . That would yield
Z Z
x 1 1
4
dx = √ du
1+x 4 u(1 + u)
which looks more√complicated than before. However, we could then undertake a second
substitution, v = u, which would correspond to an original substitution of v = x2 ; this
would produce
Z Z
x 1 1
4
dx = dv
1+x 2 1 + v2
1 1  
= arctan v + C = arctan x2 + C
2 2
An experienced student would have been able to see the substitution v = x2 imme-
diately: a substitution u = x2 is indicated when the entire integrand can be expressed in
simple terms of x2 , with the exception of one extra single power of x that is “left over”.
Information for Students in Lecture Section 002 of MATH 141 2009 01 2041

Z π
6
[1, Exercise 57, p. 407] Evaluate the definite integral tan3 θ dθ.
− π6

Solution: Eventually you will be able to integrate the cube of the tangent; but, at this
point, you may not be able to do that. However, you can observe that the integrand is an
odd function of θ. (Why?) Hence the integral from − π6 to 0 will be equal in magnitude
but opposite in sign to the integral from 0 to π6 ; so the given integral is 0.
Z π6
[1, Exercise 57, p. 407] “Evaluate the definite integral tan3 θ dθ.”
− π6

Solution: Eventually you will be able to integrate the cube of the tangent; but, at this
point, you may not be able to do that. However, you can observe that the integrand is an
odd function of θ. (Why?) Hence the integral from − π6 to 0 will be equal in magnitude
but opposite in sign to the integral from 0 to π6 ; so the given integral is 0.
Z 2 √
[1, Exercise 65, p. 407] “Evaluate the definite integral x x − 1 dx.”
1
Solution: This integral can be simplified by defining u = x − 1, so du = dx.34 Then
Z √ Z

x x − 1 dx = (u + 1) u du
Z  
3 1
= u 2 + u 2 dx
2 52 2 23
= u + u +C
5 3
2 5 2 3
= (x − 1) 2 + (x − 1) 2 + C
5 3
Taking the differences of the values of this antiderivative (i.e., with any specific value
for C, e.g., with C = 0) at the limits yields
" #2
2 5 2 3
(x − 1) 2 + (x − 1) 2
5 3
!1 !
2 5 2 3 2 5 2 3
= (2 − 1) 2 + (2 − 1) 2 − (1 − 1) 2 + (1 − 1) 2
5 3 5 3
!
2 2 16
= + − (0 + 0) =
5 3 15
34

An even better substitution would be v = x − 1, which would imply that v2 = x − 1, so dx = 2v dv, and the
R1
integral would be equal to (v2 + 1)v · 2v dv etc.
0
Information for Students in Lecture Section 002 of MATH 141 2009 01 2042

Alternatively, the Definite Integral version of the Substitution Rule could have been used,
to obtain
" #2−1 ! !
2 52 2 32 2 52 2 32 2 52 2 23
u + u = 1 + 1 − 0 + 0 etc.
5 3 1−1 5 3 5 3

The Logarithm Defined as an Integral (material in §5.6 of the 5th Edition) 35 While it is
theoretical, the discussion below is an essential part of the course; its purpose is to substantiate
some of the discussions in [1, §§1.5, 1.6] by replacing the weakest parts of the definitions given
at that point in the textbook. This is one of several possible ways of treating exponentials and
logarithms “properly”; the treatment in [1, §§1.5, 1.6] was necessarily inadequate, as there
was a perceived urgency to make logarithms and exponentials available to students who didn’t
have the background for a more substantial treatment.

The Natural Logarithm Previously we had “defined” the logarithm to be the inverse of
the exponential function, whose definition was, at best, intuitive. Now we sketch very briefly a
more rigorous definition of both functions, beginning with the logarithm. This theory has been
delayed until now because parts of the proofs require the concept of substitution in a definite
integral.
Henceforth we take the following as our primary definition:

Definition B.3 Let x be any positive real number. Define


Z x
1
ln x = dt
1 t

1
i.e., it is the area under the right branch of the hyperbola y = , between t = 1 and t = x.
t

Which Definition is “Correct”? The definitions in [1, §§1.5, 1.6] were all we had avail-
able at that time to follow the objective of the textbook to introduce the exponential and log-
arithm functions “early”. We had to rely largely on intuition in deriving properties of these
functions. Now that we have the integral available, we can return and replace those inadequate
definitions by some that are more rigorous. Even the present definitions have some issues,
since we haven’t rigorously proved all the properties that we are using for the definite integral.
While most mathematicians today would probably follow the present development — of intro-
ducing the logarithm first, as a definite integral, and the exponential as its inverse — there are
other possible ways of defining the functions. Yet another way will be available to you if you
follow the theory of infinite series beyond what we will be doing in [1, Chapter 11].
35
The textbook material has been moved in the 6th edition to [1, Appendix G, pp. A48-A55]
Information for Students in Lecture Section 002 of MATH 141 2009 01 2043

Properties of the Logarithm From the first part of the Fundamental Theorem we imme-
diately obtain the fact that
Theorem B.22
d 1
ln x = . (32)
dx x
We can also prove the following, using basic properties of the integral:
Theorem B.23 Let x, y be positive real variables. Then
1. ln(xy) = ln x + ln y
2. ln 1 = 0
1
3. ln = − ln x
x
d 1
4. ln |x| =
dx x
5. lim ln x = ∞
x→∞

6. lim+ ln x = −∞
x→0

While I do not suggest you memorize the proofs, except possibly that of the first part below,
they provide simple exercises on various aspects of the integral.
Proof:
1. This is a simple application of properties of the definite integral, and in the Substitution
Rule.
Z xy
1
ln xy = dt
1 t
Z x Z xy
1 1
= dt + dt
1 t x t
by decomposing the interval [1, xy] into [1, x] and [x, xy]
Z x Z y
1 1
= dt + x du
1 t 1 xu
using the substitution t = xu
Z x Z y
1 1
= dt + du = ln x + ln y
1 t 1 u

Z1
1
2. ln 1 = dt = 0 since the integration is over an interval of length 0.
t
1
Information for Students in Lecture Section 002 of MATH 141 2009 01 2044

1 1
3. Using a substitution u = , where dt = − 2 du, we obtain
t u
Z 1x
1 1
ln = dt
x 1 t
Z x !
1
= u − 2 du
1 u
Z x
1
= − du = − ln x
1 u

4. Recall that (
d +1 if x>0
|x| = .
dx −1 if x<0
It is convenient to write these values in the form



 |x|
d  if x,0
|x| = 
 x .
dx 
 undefined if x=0
Hence, by the Chain Rule and the Fundamental Theorem,
d 1 |x| 1
(ln |x|) = · =
dx |x| x x
when x , 0.
5. We will reconsider this proof when we study “harmonic series” in [1, §11.2, Example 7,
Z2n
1
pp. 691-692]. Consider the Riemann sum obtained for the definite integral dt. We
t
1
will hang the rectangles by their upper right-hand corners, so that the Riemann sum is
clearly less than the integral. Then the area under the curve is greater than the sum
1 1 1 1
+ + + ... + n .
2 3 4 2
Let’s collect these terms into groups ending with the reciprocal of each power of 2. We
obtain
1 1
=
2 2
1 1 1 1 1
+ > + =
3 4 4 4 2
1 1 1 1 1 1 1 1 1
+ + + > + + + =
5 6 7 8 8 8 8 8 2
... ...
1 1 1 n−1 1 1
+ + . . . + > 2 · =
2n−1 2n−1 + 1 2n 2n 2
Information for Students in Lecture Section 002 of MATH 141 2009 01 2045

We see that the area of the rectangles under the curve is n2 . As we allow n → ∞ this
lower bound for the area approaches ∞, implying that the full area surely → ∞.
1
6. Let’s make the substitution y = , so that, as x → 0+ , y → +∞. Then
x
1
lim+ ln x = lim ln = − lim ln y = −∞ .
x→0 y→+∞ y y→+∞

7. I presented the preceding proof because it is classical. The definition of the logarithm
as a definition integral permits a much simpler proof of the same result. Since 1t ≤ 1 for
Zx Zx
1
t ≥ 1, dt ≥ 1 dt = x − 1 → ∞ as x → ∞.
t
1 1

The construction began with a formal definition for the (natural) logarithm. At this point we
can define the (natural) exponential function as the inverse of the logarithm. We begin by
calling the function exp(x); only after we prove that it behaves the way we expect a power to
behave, do we revert to the familiar notation.

The Natural Exponential Function We could now justify the various equations that we
used intuitively in the “early transcendentals” treatment in [1, §§1.5, 1.6]. As mentioned, we
begin by showing that ln, as defined above, is invertible. Then we temporarily call the inverse
function, i.e., ln−1 x, exp(x). Thus we have,
Theorem B.24
exp x = y ⇔
ln y = x (33)
exp(ln x) x = (34)
ln(exp x) x = (35)
exp(x + y) =
(exp x) · (exp y) (36)
exp x
exp(x − y) = (37)
exp y
(exp x)y = exp(xy) (38)
following which, knowing that the exponent rules are satisfied, we define
Definition B.4
e = exp 1 (39)
and change our notation by recognizing that
exp x = e x . (40)
We also show that
Information for Students in Lecture Section 002 of MATH 141 2009 01 2046

Theorem B.25
d
(exp x) = exp x . (41)
dx
Using standard properties of the definite integral we can also show that

Theorem B.26

lim exp x = 0 (42)


x→−∞
lim exp x = ∞ (43)
x→∞

General Exponential Functions The preceding constructions have been for the natural
logarithm and the natural exponential. We may now extend these definitions to exponentials
to any positive base a , 1.

Definition B.5 For any positive real number a and any real number x we define

a x = e x ln a . (44)

This leads to the exponent rules for general (positive) bases:

a x+y = a x · ay (45)
ax
a x−y = y (46)
a
x y
(a ) = a xy (47)
(ab) x = a x · b x (48)

and to the derivative property


d x
(a ) = a x · (ln a) (49)
dx

General Logarithmic Functions Finally the inverse function of a x is named loga , and
its familiar properties are developed, e.g.,

Theorem B.27

loga x = y ⇔ ay = x (50)
d  1
loga x = (51)
dx x ln a
Information for Students in Lecture Section 002 of MATH 141 2009 01 2047

The Number e Expressed as a Limit


 1

Theorem B.28 Let a > 0. Then lim (1 + ax) x = ea
x→0

Proof:
 1
  1 
lim (1 + ax) x = lim eln(1+ax) x
x→0 x→0
since exponential and logarithm are inverses
 1 
= lim e x ·ln(1+ax)
x→0
by the exponent rules

ln(1+ax)
  ln(1+ax)−ln(1+0·x) 
= lim e x = lim e x
x→0 x→0
ln(1 + ax) − ln(1 + 0 · x)
lim
= e x→0 x
by the continuity of the exponential function

d
ln(1 + ax)
= e dx x=0

by definition of the derivative at x = 0



a

= e 1 + ax x=0 by the Chain Rule
a
= e

B.5.2 5 Review
True-False Quiz Students tend to avoid the True-False questions because these are unlikely
to appear on quizzes or examinations. It’s true that I would not wish to have a simple True-
False question in any test, since there would be a 50% chance of a correct guess. However,
the True-False questions in your textbook are much more difficult than that, as they ask “De-
termine whether the statement is true of false. If it is true, explain why. If it is false, explain
why or give an example that disproves the statement.” This requirement of a proof or a coun-
terexample makes these problems very challenging. The odd-numbered problems are solved
in your Student Solutions Manual [3]. I consider some of the even-numbered problems below.
[1, True-False Exercise 2, p. 409] If f and g are continuous on [a, b], then
Z b Z b Z b
[ f (x) · g(x)] dx = f (x) dx · g(x) dx
a a a

Solution: This statement is FALSE. That is, it is not true for all constants a, b and for all
functions f, g continuous on [a, b]. There will, of course, be functions and intervals for
Information for Students in Lecture Section 002 of MATH 141 2009 01 2048

which the statement is true, but the generalized statement, quantified for all functions
and intervals is not true. While you would have know to suspect this after we study
[1, §7.1], but you should be suspecting it already, and be able to construct a simple
counterexample. Here is one:
Define f (x) = g(x) = x. Then
Z b Z b #b
x2 b2 − a2
f (x) dx = g(x) dx = = .
a a 2 a 2
Z b Z b #b
2 x3 b3 − a3
[ f (x) · g(x)] dx = x dx = =
a a 3 a 3
2 2 2 2 3 3
b −a b −a b −a
While the polynomials · and look different, that does not consti-
2 2 3
tute a counterexample, as it could happen that we have two algebraic functions that are
equal by virtue of some identity that we don’t happen to recognize at the moment. So,
in order to complete the counterexample, we need to find specific values of a and b that
will entail that Z b Z b Z b
f (x) dx · g(x) dx , [ f (x) · g(x)] dx ,
a a a
i.e., that
b2 − a2 b2 − a2 b3 − a3
· , .
2 2 3
4
One such example is a = 0, and b having any value except 0, :
3
b2 b2 b4 b3
· = , ;
2 2 4 3
for example, take b = 1.
But the counterexample given is far from the simplest! For example, take f (x) = g(x) =
1 for all x. Then the left side of the claimed equation is equal to b − a, while the right
side is (b − a)2 , which will be different from b − a except where b − a = 1 or b = a.

[1, True-False Exercise 4, p. 409] This is very similar to [1, True-False Exercise 2, p. 409];
you should have no trouble constructing a counterexample.

[1, True-False Exercise 8, p. 409] If f and g are differentiable, and f (x) ≥ g(x) for a < x < b,
then f 0 (x) ≥ g0 (x) for a < x < b.
Solution: This statement is false. Think geometrically. The condition that f (x) ≥ g(x)
tells us that the graph of f is above the graph of g; the condition that f 0 (x) ≥ g0 (x) tells us
Information for Students in Lecture Section 002 of MATH 141 2009 01 2049

that the graph of f is steeper than the graph of g. So we can construct a counterexample
by finding, for a convenient function g, a function that is larger, but whose graph is not
so steep. For example, take g(x) = 0 for all x; now all we need is a positive function f
whose graph is sloping downward; for example, f (x) = e−x will work, with any interval
a ≤ x ≤ b. A simpler counterexample is f (x) = a + b − x, g(x) = 0.

[1, True-False Exercise 14, p. 409] All continuous functions have antiderivatives.
Solution: This problem is interesting, as it follows [1, True-False Exercise 13, p. 409],
which states that “All continuous functions have derivatives.” That preceding statement
is false, and you should be able to give counterexamples (e.g., |x| is continuous, but lacks
a derivative at x = 0). The present statement is TRUE, by virtue of the first part of the
Fundamental Theorem. If f is continuous on an interval [a, b], then
Z x
d
f (x) = f (t) dt .
dx a

Exercises
 !9 !9 !9  n 9 
1  1 2 3 
[1, Exercise 70, p. 411] Evaluate lim  + + + ... + 
n→∞ n n n n n
Solution: The form of this limit should suggest that it is a Riemann sum. There will
be more than one way to interpret this limit as such, but I will choose the interpretation
that is, in some sense, obvious. First, we see a factor 1n that is tempting to interpret as a
common subinterval length ∆x; so let’s do that: consider the number of subintervals to
be n, and the length of the interval to be 1 — it’s convenient to take a = 0 and b = 1,
 i 9
but these are not the only possible choices. Then we can interpret as f (xi∗ ); the
n
most convenient choice is to think of f (x) = x9 , and xi∗ chosen as the right end-point of
the subinterval [xi−1 , xi ]. Thus we see that the limit can be interpreted as the limit of an
upper Riemann sum associated with the integral
Z 1
x9 dx
0

#1
x10 1
and its value is the value of the integral, i.e., = .
10 0 10
Information for Students in Lecture Section 002 of MATH 141 2009 01 2050

B.6 Supplementary Notes for the Lecture of January 19th, 2009


Release Date: Monday, January 19th, 2009 (subject to revision)
Textbook Chapter 6. APPLICATIONS OF INTEGRATION.

B.6.1 §6.1 Areas between Curves


If, for a ≤ x ≤ b, continuous functions f and g have the property that f (x) ≥ g(x) — i.e., if
the graph of f is not lower than the graph of g for that interval, then the area bounded by the
graphs and the vertical lines x = a and x = b is given by the integral
Z b
( f (x) − g(x)) dx .
a

Before working some examples, I make several observations:


• When g is the 0 function — i.e., when the lower
R b boundary of the region is the x-axis —
this formula reduces to the definite integral a f (x) dx.
• We can find the areas of certain regions by decomposing them into parts that can be
described as above. Sometimes there is more than one “natural” way to decompose the
region, but all decompositions should yield the same area.
• An analogous formula can be applied if we consider the “region” bounded by graphs of
the form x = f (y), x = g(y), for a ≤ y ≤ b. In that case we say that we are integrating
with respect to y or integrating along the y-axis.
• Often we wish to find the area between two graphs determined by points where they
intersect. In these cases the vertical sides of the region have zero length.
• In solving problems it is useful to make a sketch showing an “element of area” — a
thin rectangle whose width is shown as dx or ∆x in the case of integration with respect
to x, and analogously for the case of integration with respect to y. It is difficult for me
to include such sketches in these notes, but they will be shown on the chalkboard. The
sketch is not a formal part of the solution, but you are likely to find it helps you formulate
your solution.
• Where two graphs cross in several places, so that the area between them is in several
pieces, be sure you understand what you intend if you write an integral that extends
over several pieces: if the curves interchange positions, with the upper one becoming
the lower, then the signs of the areas will change, and some of the areas will cancel. If
this is not what you intend, you must either write the integrand with absolute signs, as
| f (x)−g(x)|, or, equivalently, find the area of each of the pieces, and add their magnitudes
so as to prevent cancellation.
Information for Students in Lecture Section 002 of MATH 141 2009 01 2051

Example B.29 ([7, Exercise 8, p. 442]; see Figure 1 on page 2051) To find the area of the

1.0

0.75

0.5

0.25

0.0
−1.0 −0.5 0.0 0.5 1.0
x

Figure 1: The region(s) bounded by y = x2 and y = x4

region bounded by the curves y = x2 and y = x4 .


Solution: Note that the text-book uses the word “region” in a general way. Not all authors use
the word in this way. For this problem there are two connected regions bounded by the curves,
and the intention of the textbook was that you find the areas of both of them together.
To determine where the curves intersect, we solve their equations, obtaining (x, y) = (0, 0)
and (x, y) = (±1, 1). Note that it is not enough to guess the coordinates of these points of
intersection from the graph: you must determine the coordinates by rigorous solution of the
equations!
Information for Students in Lecture Section 002 of MATH 141 2009 01 2052

First solution: Integration with respect to x.


Z 1  " #1
2 4 x3 x5 2 2 4
2 x −x dx = 2 − = − = .
0 3 5 0 3 5 15

Second solution: Integration with respect to y. I find the area of the region in the first quad-
√ √
rant. The equations of the curves are, respectively, x = y and x = 4 y. The area is
Z 1  " #1
1 1 4 54 2 32 4 2 2
y −y4 2 dy = ·y − ·y = − = .
0 5 3 0 5 3 15

Now double this area. Note that the order of the curves in this integral is the reverse
of that used when we integrated with respect to x, because we now order them by their
relative distance from the y-axis.

While the limits of the two integrals look as though they are the same, the limits in the first
case refer to the extreme values of x, while the second refer to the extreme values of y.

Example B.30 ([7, Exercise 22, p. 442]) (see Figure 2 on page 2053) To find the area of the
region bounded by the curves y = sin x and y = sin 2x between the vertical lines x = 0 and
x = π2 .
Solution: To determine the points of intersection of the curves, we solve y = sin x with y =
sin 2x, and solve sin x − sin 2x = 0. Since sin 2x = 2(sin x) · (cos x), the intersections must
satisfy sin x · (1 − 2 cos x) = 0: either

2 cos x = 1

or we must be unable to divide by sin x, because

sin x = 0 .

In the interval 0 ≤ x ≤ π2 , we must therefore have either x = π3 (to make cos x = 12 ) or x = 0


 √ 
(to make sin x = 0). This yields the points of intersection (0, 0), and π
, 3 .
3 2
The
regions to be considered
 √  therefore consists of a lune-shaped region
 √ with one end at the origin
and the other at π3 , 23 ; and a second region which begins at π3 , 23 and ends at the vertical
line x = π2 . We will find the areas of each of the regions and add them, since that appears to
be what the textbook expects. (The wording of the question is not totally unambiguous; some
readers might be justified in assuming that the author intended signed areas to be used, rather
than the absolute values, as I am taking.)
Information for Students in Lecture Section 002 of MATH 141 2009 01 2053

1.0

0.75

0.5

0.25

0.0

0.0 0.25 0.5 0.75 1.0 1.25 1.5

π
Figure 2: The region(s) bounded by y = sin x, y = sin 2x between x = 0 and x = 2

For the left-most region the curve y = sin 2x is above y = sin x, so the area will be
Z π " # π3
3 1
(sin 2x − sin x) dx = − cos 2x + cos x
0 2
" ! # "0 #
1 1 1 1 1
= − · − + − − +1 =
2 2 2 2 4
For the right region the orders of the curves are reversed, and the area is
Z π " # π2
2 1
(sin x − sin 2x) dx = − cos x + cos 2x
π
3
2 π
3
Information for Students in Lecture Section 002 of MATH 141 2009 01 2054

" # " !#
1 1 1 1 1
= −0 + · (−1) − − + − =
2 2 2 2 4
1 1
Thus the total area is 4
+ 4
= 21 .

6.1 Exercises
[1, Exercise 18, p. 420] (see Figure 3 on page 2054) To find the area of the region bounded

10.0

7.5

5.0

2.5

0.0

−3 −2 −1 0 1 2 3

−2.5

Figure 3: The region(s) bounded by y = 8 − x2 , y = x2 between x = ±3

by the curves y = 8 − x2 and y = x2 between the vertical lines x = −3 and x = 3.


Solution: Let’s first determine where the curves intersect. Solving the equations shows
that the intersections occur when x = ±2, y = 4, i.e., in the points (±2, 4). But we are
asked to find the area from x = −3 to x = 3. Thus this region has three parts:
Information for Students in Lecture Section 002 of MATH 141 2009 01 2055

• −3 ≤ x ≤ −2, where 8 − x2 ≤ x2 ;
• −2 ≤ x ≤ 2, where 8 − x2 ≥ x2 ;
• 2 ≤ x ≤ 3, where 8 − x2 ≤ x2 .
A naive attack gives the following:
Z 3
Area = |(8 − x2 ) − x2 | dx
−3
Z −2 Z 2 Z 3
2 2 2 2
= |(8 − x ) − x | dx + |(8 − x ) − x | dx + |(8 − x2 ) − x2 | dx
−3 −2 2
Z −2   Z 2 
= − (8 − x2 ) − x2 dx + (8 − x2 ) − x2 dx
−3 −2
Z 3 
− (8 − x2 ) − x2 dx
2
Z −2 Z 2 Z 3
2 2
= − (8 − 2x ) dx + (8 − 2x ) dx − (8 − 2x2 ) dx
−3 −2 2
" #−2 " #2 " #3
2 3 2 3 2 3
= − 8x − x + 8x − x + 8x − x
3 −3 3 −2 3 2
" ! # " ! !#
16 16 16
= − −16 + − (−24 + 18) + 16 − − −16 +
3 3 3
" !#
16
− (24 − 18) − 16 −
3
92
=
3
This attack is naive in that we have not taken advantage of symmetry. Since the integrand
is an even function, and the interval of integration is symmetric about x = 0, we can find
the integral from 0 to 3 and double it: that means evaluating 2 integrals instead of 3.
The areas could also be found by integrating with respect to y, where we would have to
sum three pieces (or two if we make use of symmetry). For example, the middle region
could be described as being doublep the following region: bounded by the y-axis, the

curve x = y, and the curve x = 8 − y. This approach is cumbersome, as the region

has to be broken up further into two pieces: one bounded by the y-axis, x = y, and
p
y = 4, and the other bounded by they-axis, y = 4, and x = y − 8.
[1, Exercise 32, p. 420] “Evaluate the integral and interpret it as the area of a region:
Z 4 √

x + 2 − x dx .”
0
Information for Students in Lecture Section 002 of MATH 141 2009 01 2056

Solution: (see Figure 2 on page 2056) We can interpret this as the area of the region

0
−2 −1 0 1 2 3 4


Figure 4: The region(s) bounded by y = x + 2, y = x between x = 0 and x = 4

bounded by the curve y = x + 2, the line y = x, and the vertical lines x = 0 and x = 4.
If we square the former equation, we obtain y2 = x+2, which can be seen to be a parabola
that is symmetric about the x-axis, with its vertex on the line x = −2 and opening to the
right. Squaring the equation caused the new equation to include the lower branch of this
parabola; the original equation represents only the upper branch. The upper branch of
the parabola and the line y = x meet both in the origin and in the point (2, 2). (The other
point of intersection of the parabola and the line does not lie on the upper branch of the
parabola, and so is extraneous to this discussion.)
Information for Students in Lecture Section 002 of MATH 141 2009 01 2057

The simplest way of evaluating this integral is to break it up into two parts:
Z 4 √ Z 2√  Z 4 √ 

x + 2 − x dx = x + 2 − x dx + x − x + 2 dx
0 0 2
 3 2  3 4
 (x + 2) 2 x2   (x + 2) 2 x2 
=  3
−  + − 3
+ 
2
2 2 2
" # "0 #2 " # " #
2 32 2 32 2 32 2 32
= ·4 −2 − ·2 −0 + − ·6 +8 − − ·4 +2
3 3 3 3
16 2 3 2 3 16
= − 2 − · 2 2 − · 6 2 + 18 + −2
3 3 3 3
44 4 √ √
= − 2−4 6
3 3

[1, Exercise 50, p. 421] 1. “Find the number a such that the line x = a bisects the area
1
under the curve y = 2 , 1 ≤ x ≤ 4
x
2. “Find the number b such that the line y = b bisects the area in part (a).”
Solution:

1. We solve for a the equation


Z a Z 4
1 1
dx = dx
1 x2 a x2

to obtain a = 85 . It follows that the area of half of the region is the common value
h ia
of the two integrals, 1x = 83 .
1
2. The right side of the region has an irregular boundary: the upper part has equation
y = x12 , i.e., x = √1y ; the lower part is on the line x = 4. The two parts of the
 
1
boundary meet in the point 4, 16 . Thus the area of the rectangle bounded by the
1 3
lines x = 1, x = 4, y = 0, y = 16 is 16 . As this is less than half the total, we know
1
that b > 16 . This fact is important, as it tells us that we can represent the upper half
of the area — the portion above the line y = b, by the integral
Z 1 !
1
√ − 1 dy ,
b y

where the −1 in the integrand represents the lower boundary of the region — now
viewed as being “under” x = √1y and “over” x = 1. Setting this integral equal to 38
Information for Students in Lecture Section 002 of MATH 141 2009 01 2058

h √ i1
and solving, we obtain 2 y − y = 38 , which evaluates to
b

√ 3
1−2 b+b=
8
√ 3
⇔ ( b − 1)2 =
r8
√ 3
⇔ b=1±
8
r
11 3
⇔ b= ±
8 2
One of the values we obtain is greater than 1, which contradicts our assumption
that 0 ≤ b ≤ 1. The other gives the bisecting line
r
11 3
y= − ≈ 0.150255129.
8 2

The value we rejected has a geometric significance: it is the height of a horizontal


line at which the area bounded by the curve y = x12 , the line x = 1 and that line is
equal to 38 .
1
And what would have happened if we failed to observe that b ≥ 16 ? We would
have obtained an equation for b that could not be solved; it would be equivalent to
requiring that 85 − 3b = 16 5 1
when 0 ≤ b ≤ 16 , which are contradictory statements.

The area which I found to equal 1 − 2 b + b by integrating with respect to y could
1
also be evaluated
 by
 integrating with respect to x: here the line y = b meets y = x2
in the point √1b , b , so the area is
Z √1 !
b 1 √ √
2
− b dx = b − 2 b + 1 = ( b − 1)2 ,
1 x
3
which we must again equate to 8
and solve.
Information for Students in Lecture Section 002 of MATH 141 2009 01 2059

B.7 Supplementary Notes for the Lecture of January 21st, 2009


Release Date: Wednesday, January 21st, 2009, subject to further revision

B.7.1 §6.2 Volumes


Just as we defined the area of a region as the limit of a sum of narrow rectangles, we can
define volumes as limits of sums of thin elements; these elements can be assembled in various
ways. In this course you will see volumes expressed either as sums of thin slices with planar
sides (called laminæ), and — for solids with rotational symmetry (called solids of revolution)
— as sums of thin shells with cylindrical sides. Some problems will lend themselves only
to one type of dissection; where a problem can be approached in more than one way, it is
instructive to try it in several ways, in order to verify your answer and also to gain experience
in choosing the method that is more efficient for different types of problems. In this section we
will be considering dissections into thin slices; where the slice has the shape of a disk with a
concentric disk cut from its centre, the author uses the term washer36 .

Example B.31 1. (see Figure 1 on page 2051) ([7, Exercise 2, p. 452]) “Find the volume

7
2.5

6
2
5

1.5 4

3
1

2
0.5
1

0 0
0 0.20.40.60.8 1 0 0.5 1 1.5 2

1. 4.

Figure 5: Regions for Example B.31

of the solid obtained by rotating the region bounded by the curves y = e x , y = 0, x = 0,


x = 1 about the x-axis.” We will first solve the problem as stated, and then consider some
related problems obtained by changing some of the data.
36
Students whose native language is not English sometimes cannot understand why the author will use the
name of a household appliance here; this is another meaning of the English word washer, which refers to a thin
disk with a hole in the middle
Information for Students in Lecture Section 002 of MATH 141 2009 01 2060

Solution: (Remember to come back to this problem when we study [1, §6.3], to solve it
using the methods of that section.)
We will decompose the solid into laminæ that are thin disks. The “washers” will be
obtained by rotating about the x-axis the element that we would have used for the area
if we had found the area by integrating with respect to x. For the disk whose faces are
centred at points (x, 0) and (x + ∆x, 0), the volume obtained by hanging the element from
its upper left corner on the curve is πy2 ∆x = πe2x ∆x. By the same reasoning that led us
to express areas as definite integrals, we have
Z 1
Volume = πe2x dx
0
π 2x 1 π
= ·e = · (e2 − 1)
2 0 2

2. Now let us change the problem, asking that the solid be rotated about the y-axis. Here
the description of the cross sections will depend on the height of the cross section. We
will be expressing everything in terms of y, not x, and integrating “with respect to y”.
For y ≤ 1 (the height where the curve cuts the y-axis), the cross sections are disks of
radius 1, and the volume of that part of the solid is
Z 1
π12 dy = πy]10 = π.
0

For y ≥ 1 the cross sections are “washers”, with outer radius 1 and inner radius x; to
express this in terms of y we need to rewrite the equation of the curve y = e x in an
equivalent way that expresses x in terms of y, as x = ln y. The washers at height y have
volume
π(12 − x2 )∆y = π(1 − (ln y)2 )∆y
so this part of the solid of revolution has volume
Z e
π (1 − (ln y)2 ) dy .
1

where the upper limit of integration is the ordinate of the point where the line x = 1
meets the curve y = e x . You are not quite ready to complete this integration.
R1 You can
make a substitution like x = ln y, under which the integral transforms to π 0 (1−x2 )e x dx,
and you know how to integrate part of this integral, as
Z 1
π 1e x dx = πe x ]10 = π(e − 1) ,
0
Information for Students in Lecture Section 002 of MATH 141 2009 01 2061

R1
but you are not ready
R to integrate π 0
x2 e x dx. We will, in [1, §7.1], develop a method
to determine that x2 e x dx = (x2 − 2x + 2)e x + C (which is obvious by differentiation).
With that we know that
Volume = π[e x − (x2 − 2x + 2)e x ]10 = π[(−x2 + 2x − 1)e x ]10 = π .

3. As another variant on this problem, consider the following: “Find the volume of the
solid obtained by rotating the region bounded by the curves y = e x , y = 0, x = 0, x = 1
about the line y = −3.” The analysis is similar to what we did originally, except that the
laminæ now have a hole of radius 3 in the middle, and the outer radius is 3 units larger.
This leads to an integral
Z 1   Z 1  
x 2 2
Volume = π (e + 3) − 3 dx = π e2x + 6e x dx
0 0
" #1
1
= π e2x + 6e x
2
" !0 !#
1 2 1 π 2 
= π e + 6e − +6 = e + 12e − 13
2 2 2

4. Finally, suppose that the right boundary of the region generating the solid by revolution
changed from x = 1 to y = e2 (x − 1). This new right boundary also passes through (1, 0),
but meets the curve in the point (x, y) = 2, e2 . The volume will be
Z 1 Z 2   2 
2x x 2 2
π (e ) − e (x − 1) dx
πe dx +
0 1
Z 2 Z 2
π 2 
2x 4
= e −1 +π e dx − πe (x − 1)2 dx
2 1 1
  " 2x #2 " #2
π 2 e 4 1 3
= e −1 +π − πe (x − 1)
2 2 1 3 1
! !
e4 − 1 e4 e4 1
= π −π =π −
2 3 6 2
Example B.32 ([7, Exercise 8, p. 452]) Find the volume of the solid obtained by rotating the
region bounded by the curves y = sec x, y = 1, x = −1, x = 1 about the x-axis.
Solution: The lines x = ±1 intersect y = sec x in the respective points (1, ± sec 1). The area of
the washer centred on the x-axis between cross sections x = X and x = X+4X is approximately
π(sec2 X − 12 ) · 4X. The volume of revolution will be
Z 1  
π sec2 x − 1 · dx = π[tan x − x]1−1 = 2π(tan 1 − 1).
−1
Information for Students in Lecture Section 002 of MATH 141 2009 01 2062

(Had the integral involved arctan 1, you would have been expected to simplify it further; but
you cannot evaluate tan 1 without calculators or techniques that you will not meet until Calcu-
lus 3.)

Example B.33 ([7, Exercise 56, p. 454]) Here is a problem where the solid is not generated
by revolving a plane region about an axis. “Find the volume of the solid S: the base of S is the
parabolic region {(x, y)|x2 ≤ y ≤ 1} ; cross-sections perpendicular to the y-axis are equilateral
triangles.”

Solution: The cross-section of the base at level y has ends with coordinates (± y, y), so the
√ √ √ √ √
length of the base is 2 y, and the area of the triangular cross-section is 12 ·2 y· 3 y = 3·y.
Integrating along the y axis we find that
Z 1√  √ 1
 3 2 
Area = 3y dy =  y 
0 2 0
√ √
3 2 3
= (1 − 02 ) =
2 2
Example B.34 ([7, Exercise 15, p. 458]) (Where this problem appeared in the cited textbook,
students were asked to solve it using the method of “shells”. Let’s see if we can solve it using
the method of “washers”.) To find the volume of the solid of revolution generated by revolving
the region bounded by y = x2 , y = 0, x = 1, x = 2 about the axis x = 1.
Solution: The washers generated by elements of area parallel to the x-axis will have two kinds
of descriptions, depending on whether their cross sections are above or below the point where
x = 1 meets y = x2 , i.e. (1, 1): below y = 1 the cross sections are rectangles of width 2 − 1 = 1;
above y = 1 the cross sections are rectangles obtained from such a rectangle as below y = 1
by cutting away a rectangle of length x2 starting at the left end. But, in evaluating the limit
of the sum of the volumes of these washers, we shall be integrating with respect to y, so we
need to express the dimensions and position of the rectangular element in terms of y. The

equation of the right branch of the parabola y = x2 is x = y. The hole in the washer has

radius x − 1 = y − 1; the washer will have area
√ √
π(12 − ( y − 1)2 ) = π(2 y − y) .

The line x = 2 meets the parabola in the point (2, 4), so we shall integrate for y ranging from
0 to 4: from 0 to 1 using the constant integrand π12 , and from 1 to 4 using the integrand

π(2 y − y). The volume of revolution is
Z 1 Z 4 " #4
2 √ 1 4 32 y2
π1 dy + π(2 y − y) dy = πy 0 + π y −
0 1 3 2 1
" # " #
4 4 1 17π
= π+π ·8−8 −π ·1− =
3 3 2 6
Information for Students in Lecture Section 002 of MATH 141 2009 01 2063

which is the same result given in the textbook for the solution that could be obtained in the
next section using the method of cylindrical shells.

6.2 Exercises

[1, Exercise 12, p. 430] Find the volume of the solid obtained by rotating the region bounded
by the curves y = e−x , y = 1, x = 2 about the line y = 2.
Solution: Through the point (x, 2) on the line y = 2 the cross section will be an annulus
(ring) with outer dimension 2−e−x and inner dimension 2−1 = 1; the annulus is bounded
by concentric circles centred at the point (x, 2). The volume will be
Z 2   
−2 2 2
π 2−e − π1 dx
0
Z 2 
= π 3 − 4e−x + e−2x dx .
0

The first summand of the integrand has antiderivative 3x; the second summand can be
integrated by using a substitution u = −x, which leads to an antiderivative +4e−x ; the
last summand can be integrated by using a substitution u = −2x, which leads to an
antiderivative − 21 · e−2x . Putting these three components together we find the value of the
integral to be
" #2 ! !
−x 1 −2x 4 1 1
π 3x + 4e − e = π 6+ 2 − 4 −π 0+4−
2 0 e 2e 2
!
5 4 1
= π + 2− 4 .
2 e 2e

[1, Exercise 39, p. 431] The textbook asks you to use a Computer Algebra System to find the
volume of the solid generated by revolving about the line y = −1 the region bounded by
y = sin2 x, y = 0, for 0 ≤ x ≤ π. No Computer Algebra System is needed, although we
haven’t yet seen how to integrate this. Here is the full solution. The cross sections are
washers with centre on the line y = −1, outer radius ending on the curve y = sin2 x, and
inner radius ending on the line y = 0 (the x-axis). The volume is thus
Z 
Z !2 
π  π 
 1 − cos 2x
π 2 2 2
(sin x − (−1)) − 1 dx = π  + 1 − 1 dx
0 0 2
Z π 
π
= (3 − cos 2x)2 − 4 dx
4 0
Z
π π 
= 5 − 6 cos 2x + cos2 2x dx
4 0
Information for Students in Lecture Section 002 of MATH 141 2009 01 2064

Z !
π π 1 + cos 4x
= 5 − 6 cos 2x + dx
4 0 2
Z
π π
= (11 − 12 cos 2x + cos 4x) dx
8 0
" #π
π sin 4x π 11π2
= 11x − 6 sin 2x + = · 11π =
8 4 0 8 8

[1, Exercise 44, p. 431] Describe the solid whose volume is represented by the integral
Z π
2
π [(1 + cos x)2 − 1] dx .
0

Solution: The dx tells us that the integration is along the x-axis. That is, the planes of
the washers are perpendicular to the x-axis. The integrand is the difference of 2 squares,
multiplied by π. We may interpret this as the area of a washer whose outer radius is
1 + cos x, and whose inner radius is 1. If we interpret 1 + cos x as cos x − (−1), we
can interpret this as the radius of a disk whose centre is at the point (x, y) = (x, −1),
generated by a radius extending from that centre to the point (x, cos x) on the graph of
the cosine function. The subtracted term −π12 can be interpreted as the area of a disk
whose centre is at the same point, but its radius extends from that point (x, −1) to the
point (x, 0) above it on the x-axis. Thus the integral represents the solid of revolution
about the line y = −1 of the region bounded by the graph of the cosine function, and the
x-axis, between x = 0 and x = π2 . (This integral is not difficult to evaluate. We will see
in [1, Chapter 7] that, if we replace cos2 x by 12 (1 + cos 2x), the value of the integral is
" #π
x sin 2x 2 π2
π 2 sin x + + = 2π + .
2 4 0 4

This is not the only way of interpreting the integral. We could also reason that it rep-
resents the volume of a region rotated about the x-axis, bounded by the y-axis, the line
y = 1, and the graph of y = cos x + 1.)

[1, Exercise 49, p. 431] Find the volume of a right circular cone with height h and base radius
r.
Solution: This is a standard problem that every student should be able to work.
A cone is a surface generated by joining to all points curve in a plane a fixed point (called
the apex outside of the plane. A cone is circular if the base curve is a circle. It is right
circular if the apex is located on the normal to the plane of the curve through the centre
of that base circle.
Information for Students in Lecture Section 002 of MATH 141 2009 01 2065

It is convenient to set up coordinate axes so that the cone is generated by a right angled
x y
triangle with height h and base r, i.e., with hypotenuse along the line + = 1, which
r h
triangle is to be rotated about the y-axis. The horizontal rectangular elements of area,
with height dy and length x are rotated about the y-axis to generatedisk-shaped laminæ.
y
Expressing x as a function of y for the hypotenuse, we have x = r 1 − , so the cross-
 h
2 y 2
sectional area at height y is πr 1 − . Hence the volume must be
h
Z h  y 2
πr2 1 − dy .
0 h
We know how to evaluate an integral of this type by expanding the square. But it is
y 1
easier to make a change of variable: u = 1 − , so du = − dy, dy = −h du and
h h
Z 0 " 3 #0
2 2 2 u 1
Volume = − πr u h du = πr h = (πr2 h) .
1 3 1 3

(cf. [9, p. 47]). If you remember this as 13 π × the area of the base, you will know this
as a special case of a general theorem. In fact, it is not hard to show that the area is
not affected by the fact that the cone is a right cone: even if the apex were moved to a
location not over the centre of the circular base, the area would not change. Moreover,
it can be shown that even the fact that the base is circular is not relevant! The volume of
any “cone” is as shown:
1
× π × Area of base × height .
3
For example, the case of a square base is discussed in [1, Example 8, p. 429].

[1, Exercise 55, p. 432] Find the volume of the solid S which is a tetrahedron with three mu-
tually perpendicular faces and three mutually perpendicular edges with lengths 3 cm, 4
cm, and 5 cm.
Solution: I find it convenient to locate the three perpendicular sides along the coordi-
nate axes, with one vertex at the origin. So I locate the vertices of the tetrahedron at
(0, 0, 0), (3, 0, 0), (0, 4, 0), (0, 0, 5). This tetrahedron is just a pyramid or cone on a trian-
gular base;
 thus we know by the theory of [1, Example 8, p. 429] that the volume will
be 13 × 12 × 3 × 4 × 5 = 10. But I will pretend we don’t know that.
Consider cross sections perpendicular to the z-axis. These are triangles whose x-dimension
at height z will be 35 (5 − z) (by similar triangles in the xz-plane); and whose y=dimension
Information for Students in Lecture Section 002 of MATH 141 2009 01 2066

at height z will be 45 (5 − z), (again by similar triangles). The area of the triangle will be
1
2
(5 − z)2 , so the volume will be
Z " #5
5
1 12 2 6 1 3 2  
· · (5 − z) dz = − (5 − z) = − 0 − 53 = 10 .
0 2 25 25 3 0 25
Information for Students in Lecture Section 002 of MATH 141 2009 01 2067

B.8 Supplementary Notes for the Lecture of January 26th, 2008


Release Date: Monday, January 26th, 2008, subject to further revision

Some announcements:

Information Session for Freshmen Students Entering U1 in the 2008-2009 Academic


Year. Unfortunately the Faculties of Arts and Science scheduled their information session at
a time which clashed with our lecture. If you missed the lecture, you should conscientiously
invest time in understanding the contents of [1, §6.3] and its relations to [1, §6.2]. This is
one portion of the syllabus that requires extensive time spent in solving textbook problems: as
usual, you should never rely only on working problems on WeBWorK.

Coverage of course topics in lectures, assignments, and quizzes. The course content
really is defined by the description in the course outline, in terms of sections of [1]. Lectures,
assignments, and quizzes all have a degree of spontaneity which can mean that some topics
in the course may appear more than once, and some may not appear at all. Multiple appear-
ances should not necessarily be interpreted as indicating importance, although there could be a
subconscious element active here. Appearance or non-appearance of topics on WeBWorK as-
signments should certainly not be taken as indicating the importance of the topic, since some
topics are not amenable to WeBWorK treatment, and we don’t have suitable problems for
them in our WeBWorK collection yet. The worked examples in the body of the textbook are
worth reading: cover the proof first, and try to solve the problem yourself. I usually avoid
those examples in the lectures so that I can widen your exposure to examples, not because the
examples are bad — in fact, they are often the “standard” examples, and you should read them.

Is it possible that there are coding errors in WeBWorK? I had an instance last year
where a student observed that she had applied the same method on two similar problems, on on
a practice WeBWorK assignment, and one on the assignment that counts. What I discovered
to have happened is that the WeBWorK system had, in the case of the practice assignment,
given her full marks for a solution that was wrong. This can happen sometimes because of the
way in which WeBWorK evaluates your submitted answer. But, in this case, it was caused
by the programmer’s oversight in permitting students to enter their solutions in decimals, and
by his demanding answers correct only to a specific decimal place. In this case there was no
good reason to permit students to enter decimal answers, and it would have been appropriate
to require a very high level of accuracy, so that students would enter their solution only by
writing vulgar functions and not attempting to evaluate square roots or π. We don’t have
the programming power available to fine tune all of the WeBWorK problems, so you may
find some that permit a solution with lower accuracy than is appropriate. (In the case of this
Information for Students in Lecture Section 002 of MATH 141 2009 01 2068

particular student, the randomized data assigned to her problem by WeBWorK was such that
the error — it was a real error — would be very small; but, such an error on an examination
problem requiring a full solution might have earned no marks at all.)
WeBWorK evaluates some functions that are input by testing them at points on a grid;
sometimes that testing happens not to discover errors in the submission — your function, and
the correct answer could possibly coincide on all the points where WeBWorK performs its
comparisons, and still differ elsewhere; so it can, indeed, happen that WeBWorK will give you
full marks for a function where your description is defective. This is a very rare phenomenon:
if you have doubts whether you deserved the full marks that WeBWorK gave you, bring your
work to a TA or to your instructor for checking.

I received full marks for my solution to a problem on the quiz; doesn’t that mean that
my solution was perfect? If your quiz is graded carefully, full marks are usually associated
with a solution that is essentially correct. Sometimes there are reasons (other than careless
grading) why your work will be graded unusually generously, even though there could be some
aspects that could still be improved substantially. If you have any doubts about the correctness
of a solution, bring it to your instructors or a TA and ask them to check it. (Even if the grading
of a question was incorrectly high, no one is planning to lower your grade.)
For model solutions to the types of problems you meet in this course you are referred to
the Student Solutions Manual [3].

B.8.1 §6.3 Volumes by Cylindrical Shells


The integrand when using cylindrical shells The volume of a right circular cylinder (i.e.,
with a disk as base, axis perpendicular to the base) is — as you can easily prove using washers
or otherwise — the product of the area its base and its height. If we consider a hollowed out
cylinder of radius r2 , in which an inner cylinder of radius r1 is removed, then the volume will
be, if the height is h,  
πr22 h − πr12 h = π r22 − r12 h .
Let’s assume that r2 = r1 + 4r, and expand this product:
   
π r22 − r12 h = π (r1 + 4r)2 − r12 h
 
= π r12 + 24r · r1 + (4r)2 − r12 h
= 2πh4r · r1 + πh(4r)2
Both of these products approach zero as we allow 4r → 0. If we wish to determine the volume
of a solid of revolution by decomposing it into cylindrical shells about the axis of revolution,
it would appear that we should add elements of volume
2πh4r · r1 + πh(4r)2
Information for Students in Lecture Section 002 of MATH 141 2009 01 2069

and then permit the number of shells to approach infinity, and the width ∆r → 0. It can be
shown that, in any such limiting process, the sum of the terms of type πh(4r)2 approaches
0; that is, not only do the individual terms πh(∆r)2 approach 0, as we permit 4r → 0, but
even the sum of these terms, increasing arbitrarily in number as 4r → 0, also approaches 0.
Thus the
R volume sought can be viewed as a Riemann sum, leading to a definite integral of the
form 2πr1 h dr. Thus, to find the volume of a solid that we can decompose into elements
which are cylindrical shells, we need only consider an integral related to terms of the first type.
The integrand can be interpreted as the product of 2πr1 and h, i.e., as the area of a rectangle
obtained by “cutting open” the inner surface of the cylindrical shell and “unrolling” it; then the
4r can be interpreted as the thickness of a thin rectangular lamina based on that rectangle. Of
course, when you attempt to do that, you find that there will be a small error in that product,
but, as I have mentioned, it can be shown that the totality of these errors approaches 0 as we
replace the sum of volumes of elements by the definite integral. All that remains to be done is
to express h in terms of the radius, and to determine the appropriate limits for integration.
If you choose to approach these problems by substituting in formulæ, you are urged to
remember how to generalize to situations where the axis of circular symmetry is parallel to but
different from one of the coordinate axes. I do not recommend memorizing these formulæ.

6.3 Exercises

[1, Exercise 14, p. 436] (see Figure 6 on page 2070) “Use the method of cylindrical shells to
find the volume of the solid obtained by rotating the region bounded by the given curves
about the x-axis. Sketch the region and a typical shell: x + y = 3, x = 4 − (y − 1)2 .”
Solution: The parabola meets the line in the points (0, 3) and (3, 0), on the coordinate
axes.

1. First we follow the instructions, using the method of cylindrical shells. The ele-
ments of area that generate the shells will be narrow horizontal rectangles; at height
y the rectangle has width (4 − (y − 1)2 ) − (3 − y), obtained by expressing the equa-
tions in the form x =function of y. The circumference of the base of the generated
cylinder is then 2πy, and the volume is
Z 3 
2π (4 − (y − 1)2 ) − (3 − y) y dy
0
Z 3 
= 2π 3y2 − y3 dy
0
" #3
3 y4 27π
= 2π y − = .
4 0 2
Information for Students in Lecture Section 002 of MATH 141 2009 01 2070

0
-2 -1 0 1 2 3 4

-1

Figure 6: The region(s) bounded by x + y = 3 and x = 4 − (y − 1)2

2. Now let us compute the volume using washers. The equation of the line may be
rewritten as y = 3 − x; but the√ parabola now splits into 2 curves — the√ upper
branch has equation y = 1 + 4 − x, and the lower has equation y = 1 − 4 − x.
The description of the element of area that generates the washer will change at
x = 3.√ To the left of x = 3 the element of area at horizontal position x has height
(1 + 4 − x) − (3 − x). We need to compute the area of the annulus the element
generates. For that purpose the length of the element is not enough, as we need to
know the distance√from the axis about which it is revolving. The outer radius of
the washer is 1 + 4 − x, and the inner radius is 3 − x, so the area of the annulus is
Information for Students in Lecture Section 002 of MATH 141 2009 01 2071

the difference between the areas of two disks:



π(1 + 4 − x)2 − π(3 − x)2

and the volume of the solid generated up to x = 3 is


Z 3 √ 
π (1 + 4 − x)2 − π(3 − x)2 dx .
0

The elements
√ of area to the right of x = 3 have inner radius 1 − 4 − x, and outer
radius 1 + 4 − x, so the area of the annular cross-section is
 √ 2  √ 2 √
π 1 − 4 − x − π 1 + 4 − x = 4π 4 − x ,
R4 √
and the volume is 4π 3
4 − x dx. The total volume by the method of washers is
Z 3 √  Z 4√
2 2
π (1 + 4 − x) − π(3 − x) dx + 4π 4 − x dx
0 3
Z 3 √  Z 4√
= π −(x − 1)(x − 4) + 2 4 − x dx + 4π 4 − x dx
0 3
Z 1 Z 0
2
√ √
= π (u − 3u + 2 u) (−1) du + π 4 u(−1) du
4 1
using the substitution u = 4 − x, du = −x, x = 4 − u
27π
= as before.
2
[1, Exercise 20, p. 437] “Use the method of cylindrical shells to find the volume generated
by rotating the region bounded by the given curves about the specified axis. Sketch the
region and a typical shell: y = x2 , x = y2 , about y = −1.”
Solution:

1. At height y the cylinder is generated by an element of area whose horizontal di-



mension is y − y2 , and whose vertical dimension is ∆y; the circumference of the
circle generate by a point at one end of this element under revolution is 2π(y−(−1))
(since the radius is the distance between a point (x, y) and the point (x, −1) below
it), so the volume is
Z 1 

y − y2 2π(y + 1) dy
0
Z 1 
3 √
= 2π y 2 − y3 + y − y2 dy
0
Information for Students in Lecture Section 002 of MATH 141 2009 01 2072

" #1
2 52 1 4 2 23 1 3
= 2π y − y + y − y
5 4 3 3 0
29
= π.
30
2. It wasn’t asked for in the problem, but let’s find the same volume
√ by the√method of
washers. The external radius of the vertical washer at x is x − (−1) = x + 1; the
internal radius of that washer is x2 − (−1) = x2 + 1. The area of the cross section
will be π times the difference of the squares of the radii, i.e.,
 √ 
π ( x + 1)2 − (x2 + 1)2 .
29
The integral from x = 0 to x = 1 is again equal to 30
π.
[1, Exercise 29, p. 437] The following integral represents the volume of a solid; describe the
solid: Z 3
2πx5 dx .
0

Solution: Since the problem appears in this section, the textbook expects you to interpret
the integral as the result of application of the method of cylindrical shells. If we interpret
2πx5 · ∆x as (2πx) · x4 · ∆x, we see that it is the volume of the solid generated by rotating
about the y-axis the region bounded by the x-axis and the curve y = x4 from the point
(0, 0) to the line x = 3.
But the author should not have used the definite article the, since the integral can be
interpreted in other ways. For example, we can interpret it as resulting from application
 √ 2
of the method of washers also. This time we interpret 2πx5 · ∆x as π 2x5 · ∆x: it
is the area of the solid obtained
√ by rotating about the x-axis the region bounded by that
axis and the curve y = 2x5 between x = 0 and x = 3.
π
Z4
[1, Exercise 32, p. 437] The integral 2π(π − x)(cos x − sin x) dx represents the volume of a
0
solid. Describe the solid.
Solution: The same integral could easily represent more than one solid. One interpre-
tation is the following: The solid is a solid of revolution around the vertical line x = π,
generated by revolving the region bounded by the y-axis  and  the graphs y = cos x and
y = sin x up to the point where they intersect, (x, y) = π4 , π4 . But the region could be de-
formed vertically without changing the volume. For example, the region could be taken
to be bounded by the lines y = 0, x = 0,
π ! π π 
2
−x+x −x−x 1
y = cos x − sin x = sin · cos 2 = √ cos − x
2 2 2 4
Information for Students in Lecture Section 002 of MATH 141 2009 01 2073

from x = 0 to x = π4 .

[1, Exercise 43, p. 437] Use cylindrical shells to find the volume of a sphere of radius r.
Solution: Here is a case where the solid is prescribed, but not the way of generating it. A
sphere can be generated by the rotation of a half-disk around its diameter. I chose to take
2 2 2
the diameter
p as the y-axis, and the equation of its boundary as x +y = r , more precisely,
as x = + r2 − y2 where −r ≤ y ≤ r. The length of a vertical rectangular √ element of
area a√distance of x from 2 2
√ the axis is the distance between the points (x, r − x ) and
2 2 2 2
(x, − r − x ), i.e., 2 r − x . This is the height of the cylindrical generated when the
element is rotated about the axis. The base of the shell is at a distance of x from the
axis, so it generates a circular band (annulus) of radius 2x, hence of circumference 2πx.
Here I have not been precise about whether this is the inner or outer circumference of
the annulus, since, in the limit, these distinctions have no effect on the calculations. The
thickness of the annulus is represented by the differential, dx, in the integral, and we
obtain a volume of
Z r √ 4π h 2 i
3 r 4πr3
2πx(2 r2 − x2 ) dx = − (r − x2 ) 2 = . (52)
0 3 0 3
Of course, this volume could also be evaluated by the method of washers, see [1, Exam-
ple 1, pp. 423-424]. If you found the number of variables in equation (52), you could
amplify the notation, and write
Z x=r √ 4π h 2 i
3 x=r 4πr3
2πx(2 r2 − x2 ) dx = − (r − x2 ) 2 = for all r . (53)
x=0 3 x=0 3

[1, Exercise 44, p. 437] Use cylindrical shells to find the volume of the solid torus generated
by rotating a disk of radius r around a line located a distance R from its centre.
Solution: We can take the disk to be bounded by (x − R)2 + y2 = r2 , and the axis of
rotational symmetry of the torus (doughnut)
p to be the y-axis. The boundary of the disk
ispthe graphs of 2 functions, y = ± r2 − (x − R)2 , so the height of the element of area is
2 r2 − (x − R)2 . The volume is
Z R+r p Z r √
2 2
2πx · 2 r − (x − R) dx = 2π (u + R)2 r2 − u2 du
R−r −r

Z under
r √
the substitution uZ = x − R
r √
= 4π 2 2
u r − u du + 4πR r2 − u2 du
−r −r

Here the first integral can be seen to be the area under the graph of an odd function from
−r to the symmetrically located value +r, so the volume is 0; it may also be integrated
Information for Students in Lecture Section 002 of MATH 141 2009 01 2074

" #r
4π 2 2 32
as · (r − u ) = 0. The second integral is 2πR times the area under the curve
3√ −r
y = r2 − u2 from u = −r to u = r, which can be seen to be the area of a half disk of
πr2
radius r, which we know to be . Thus the volume is 2π2 Rr2 .
2

B.8.2 §6.4 Work


This section has been omitted from the syllabus because it involves physical concepts that
some students from outside of the Faculties of Science and Engineering might not be prepared
for. If you are a Science or Engineering student, you are urged to peruse the section and try
the problems. Your instructors and TA’s will be happy to help you with any difficulties.
Information for Students in Lecture Section 002 of MATH 141 2009 01 2075

B.9 Supplementary Notes for the Lecture of January 26th, 2009


Release Date: Monday, January 26th, 2009, subject to further revision

Review of preceding two lectures We have studied how to use a definite integral to deter-
mine volumes of solids, by expressing the solid as the limit of a union of thin layers: in [1,
§6.2] as the union of thin laminæ with planar sides and, in [1, §6.3], for solids of revolution, as
the union of thin circular cylindrical shells. Where the solid is obtained by revolving a plane
region about a line in that plane the solid can be called a solid of revolution about that line, and
the methods of both sections are, in principle, applicable. I do not recommend attacking these
problems by substitution in formulæ, as there are many variants to be considered, and the blind
use of formulæ often leads to disaster when the wrong formula is applied, or the right formula
is applied incorrectly. I list some formulæ below just to summarize the nature of the results
we have found, with the intention that, in each case, you decompose the solid in one of the
methods that is applicable and set up the integral by carefully examining the decomposition.

1. When a solid is decomposed into thin laminæ between parallel planes which are per-
Zb
pendicular to the x-axis: the volume is expressible in the form A(x) dx, where the
a
decomposition ranges between values x = a and x = b, and A(u) is an expression for the
cross-sectional area at x = u. (If the decomposition is along the y-axis, then the limits of
the integral will be the limiting values of y, and we will usually express the integral in
terms of y; of course, the x or y that appears in the integral is irrelevant, since these are
bound or “dummy” variables, that are part of our notation, and don’t actually affect the
numerical value of the integral.)

2. When a decomposition as in item 1 above has circular symmetry as a solid of revolution


about a line x = c of a region in the xy-plane, the laminæ can be interpreted as being
generated by revolving a thin rectangle, and will consist of disks, possibly with a hole
in the middle — the textbook calls them “washers”. The integrand will be an expression
of the form π (r2 (x))2 − π (r1 (x))2 , where r2 and r1 are the outer and inner radius of the
disk, determined by the distances of the two ends of the thin rectangle from the point of
intersection of the extended rectangle with the axis of rotational symmetry of the solid.
Here we need to be prepared to work with either x or y, and possibly to consider rotation
about a line parallel to but distinct from the coordinate axes.

3. In the case of decomposition into cylindrical shells of a solid with rotational symmetry
about an axis, the cylindrical shells can also be interpreted as being generated by rotating
a thin rectangle about the axis — but here the long dimension of the rectangle is parallel
to the axis. The integrand may also be interpreted as the area of a “flattened” cylindrical
Information for Students in Lecture Section 002 of MATH 141 2009 01 2076

shell, obtained by slitting the shell open and unrolling it; one dimension will be the
circumference of the circle generated by revolving any point of the cylinder around the
axis of symmetry — this factor will be of the form 2πx when we are at distance x from
the axis of rotational symmetry; the other dimension will be the height of the cylindrical
shell. Again, we need to adjust this formula according to the orientation of the axis of
rotational symmetry, and according as the axis is or is not a coordinate axis.

B.9.1 §6.5 Average value of a function


In this section the textbook defines what is meant by the term average of a continuous function
or a function “pieced” together from continuous functions over an interval a ≤ x ≤ b. The
definition is a generalization of the definition familiar to you of the average of a finite number
of numbers y1 , y2 , . . . , yn , i.e.,
Pn
yi
i=1
average = .
n
If, in the finite case just mentioned, we treat each of yi as the height of a rectangle of unit width,
situated so that its base is placed on the x-axis with its corners at (i − 1, 0) and (i, 0), and with
X n
height yi , then the sum yi is the area under the graph of the function f defined by
i=1



 y1 if 0 ≤ x ≤ 1



 y2 if 1 < x ≤ 2
f (x) = 



 ...

 yn if n − 1 < x ≤ n

For any function that is piecewise continuous on the interval a ≤ x ≤ b, we define


Z b
1
average of f over [a, b] = f (x) dx ,
b−a a
Rb
f (x) dx
= aR b
a
1 dx

and so our definition is consistent with the earlier definition when the function is defined at a
finite number of points x1 , . . . , xn : we are simply extending the definition of that function by
extending the value at any integer point to the entire interval of unit length to the left of the
point.
Information for Students in Lecture Section 002 of MATH 141 2009 01 2077

The Mean Value Theorem for Integrals If f Ris continuous on [a, b], the Mean Value The-
x
orem may be applied to the function g(x) = a f (t) dt, which we know from the Funda-
mental Theorem to be differentiable. It asserts the existence of a point c in (a, b) such that
g(b) − g(a)
= g0 (c), i.e., such that
b−a
Z b
f (t) dt = g(b) − g(a) = f (c) · (b − a) ,
a
Rb Rb
a
f (x) dx a
f (x) dx
or f (c) = R b = .
1 dx b−a
a
[1, Exercise 23, p. 445].
As mentioned in connection with the Mean Value Theorem in Math 140, the spirit of this
theorem is in the existence of the point c, not in the specific values that c takes. The theorem
is not constructive: it proves the existence without telling you how to find the point. (A
constructive proof of a theorem proves existence by providing a way of finding the point; the
proof we have given for the present theorem is not constructive.)

Example B.35 [1, Exercise


Z 13, p. 445] The textbook asks you to prove that, if f is any contin-
3
uous function for which f (x) dx = 8, then f takes on the value 4 somewhere in the interval
1
1 ≤ x ≤ 3. The MVT for Integrals tells you that there is a point c such that 1 ≤ x ≤ 3 and
Z 3
1
f (x) dx = f (c)
3−1 1

and the left side of this equation is exactly 82 = 4.


What if we were to ask the same question with 4 replaced by another real number, e.g.,
4.001? The answer would now be negative: as a counterexample37 take the constant function
f (x) = 4. This function is continuous and has the desired area over the interval 1 ≤ x ≤ 3; but
it never assumes the value 4.001.

Average velocity Suppose that the position at time t of a particle moving along the x-axis is
f (t). In [1, §3.7, p. 221] the textbook has defined the Average Velocity of the particle over a
time interval a ≤ t ≤ b to be
∆x f (b) − f (a)
= .
∆t b−a
37
an example to disprove a general statement: since the statement refers to all functions with a given property,
we can disprove it by exhibiting just one example.
Information for Students in Lecture Section 002 of MATH 141 2009 01 2078

You may recall being told by your instructor that the use of the word average there was also
a generalization of the traditional meaning recalled above for the average of a finite set of
numbers. You can now see that, since
Rb d
f (b) − f (a) dt
f (t) dt
= a ,
b−a b−a
that use of the word is consistent with the use we have defined here. In other words, the Average
Velocity is, in fact, the average of the velocities. So the earlier use of the word “average” was,
though premature, consistent with the generalization that was planned. Previously “Average
Velocity” was a two-word name for a concept, and you would not be justified in treating the
first word as a modifier of the second; now you may interpret that as a conventional use of
language, where average is an adjective. When mathematicians name concepts we try to make
the nomenclature intuitive, and consistent with earlier usage.

6.5 Exercises

[1, Exercise 4, p. 445] Find the average value of the function g(x) = x2 1 + x3 on the interval
[0, 2].
Solution:
Z 2 √
1
average = x2 1 + x3 dx
2−0 0

We apply the substitution u = x3 , so du = 3x2 dx and


Z 23 √
1 1
average = 1 + u · du
2−0 0 3
#8
1 2 3 1h 3 3
i
= · (1 + u) 2 = 9 2 − 1 2
6 3 0 9
26
=
9

(We could also use the substitution u = 1 + x3 .) Had the problem asked us to “sketch
a rectangle whose area is the same as the area under the graph of g,” we could take a
rectangle based on the interval 0 ≤ x ≤ 2, with height 269 .)

Example B.36 (taken from a problem book for students in Russian technical universities) [38,
Problem 1647, p. 126] Suppose that a trough has a parabolic cross-section with equation of
the form y = K x2 , and measures 1 metre across the top and is 1.5 metres deep. Determine the
average depth.
Information for Students in Lecture Section 002 of MATH 141 2009 01 2079

 
1 3 3
Solution: From the data we know the cross section passes through the point ,
2 2
, so 2
=
 2
K 23 , and K = 6. The depth of the trough at position x is 32 − 6x2 , so
R 1  
3
2
− 6x2 dx
− 12 2
average depth =  
1
2
− − 12
Z 12 !
3 2
= 2 − 6x dx
0 2
by symmetry, since the integrand is an even function
" # 12
3x
= 2 − 2x3 = 1
2 0

so the average depth is 1 metre, i.e., two-thirds of the way to the bottom of the trough.

6 Review

[1, Exercise 32, p. 447] “Let R1 be the region bounded by y = x2 , y = 0, and x = b, where
b > 0. Let R2 be the region bounded by y = x2 , x = 0, and y = b2 , to the right of the line
x = 0.

1. “Is there a value of b such that R1 and R2 have the same area?
2. “Is there a value of b such that R1 sweeps out the same volume when rotated about
the x-axis and the y-axis?
3. “Is there a value of b such that R1 and R2 sweep out the same volume when rotated
about the x-axis?
4. “Is there a value of b such that R1 and R2 sweep out the same volume when rotated
about the y-axis?”

Solution: Note that the statement of the problem in the textbook is ambiguous, as the
description of R2 applies to one region to the right of the y-axis and another to the left;
for that reason I have added the italicized words. Now the two regions combine to form
the rectangle {(x, y)| 0 ≤ x ≤ b, 0 ≤ y ≤ b2 }.

1. We are asked to investigate solutions of the equation


Z b Z b
2 2 2 b3 2b3
x dx = (b − x ) dx ⇔ = ,
0 0 3 3
Information for Students in Lecture Section 002 of MATH 141 2009 01 2080

which has no positive solution. (We could have evaluated either or both of these
areas by integration along the y-axis, writing the equation of the right branch of the

parabola as x = y. With both integrations along that axis the equation would be
Z b2 Z b2 
√ √ 
(b − y) dy = y dy .
0 0

2. I will find the volume about the x-axis using washers, and the volume about the
y-axis using the same elements of area, using cylinders. Equating the two areas I
obtain the equation
Z b   Z b " 5 #b " 4 #b
2 2 2 x x
π x dx = 2πx · x dx ⇔ π = 2π
0 0 5 0 4 0

which is equivalent to b4 (2b − 5) = 0 and has one positive solution, b = 52 . (This


problem could be solved by evaluating either of the integrals in the “other” way.)
3. We have determined the volume swept out by rotating R1 about the x-axis to be
πb5
5
. If we compute the volume obtained by rotating R2 about the x-axis, we find it
to be
Z b      " #b
2 2 2 2 4 x5 4π 5
π b − x dx = π b x − = b using washers, and
0 5 0 5
Z b2
√ 2 5 ib2 4π 5
2πy · y dy = 2π · y 2 = b using cylindrical shells.
0 5 0 5
Equating the volume obtained by rotating R1 about the same axis to this, we obtain
πb5 5
5
= 4πb
5
, which has no positive solution.
4. The volume obtained by rotating R1 about the y-axis has been determined above to
be π2 b4 . The volume obtained by rotating R2 about the y-axis is
Z b2 Z b2 " #b2
√ 2 y2 π 4
π( y) dy = π y dy = π = b
0 0 2 0 2
using washers, or
Z b   " #b
2 2 b2 x 2 x 4 π 4
2πx b − x dx = 2π − = b
0 2 4 0 2
using shells. Here we see that the two volumes of revolution are equal for all values
of the parameter b.
Information for Students in Lecture Section 002 of MATH 141 2009 01 2081

Textbook Chapter 7. TECHNIQUES OF INTEGRATION.

B.9.2 §7.1 Integration by Parts


Earlier we developed the Substitution Rule for evaluating integrals, from the Chain Rule for
differentiation. Now we will develop another procedure for evaluation of integrals, called
Integration by Parts, based on the Product Rule for differentiation. As with the Substitution
Rule, this rule will be applicable to both definite and indefinite integrals. If does not affect the
(“independent”) variable and so there will be no change to limits in definite integrals.
Starting from the Product Rule,
d d d
[ f (x) · g(x)] = f (x) · g(x) + f (x) · g(x) ,
dx dx dx
we integrate all 3 members with respect to x:
Z Z " # Z " #
d d d
[ f (x) · g(x)] dx = f (x) · g(x) dx + f (x) · g(x) dx
dx dx dx
and observe that the integral on the left is simply f (x) · g(x) + C. Moving the terms around
gives the Rule of Integration by Parts:
Z " # Z " #
d d
f (x) · g(x) = f (x) · g(x) dx + f (x) · g(x) dx
dx dx
Z " # ⇔ Z " #
d d
f (x) · g(x) dx = f (x) · g(x) − f (x) · g(x) dx ,
dx dx
Z " # Z " #
d d
f (x) · g(x) dx = f (x) · g(x) − f (x) · g(x) dx ,
dx dx
Z Z
or, compactly, f dg = f g − g d f .

Traditionally we often name the functions u and v, or variations of these symbols38 — since
the solution of a specific problem may require multiple applications of integration by parts.
These equations are always true for differentiable functions, but we shall be applying them
38
like u1 , v1 , U, V, ũ, ṽ, . . .
Information for Students in Lecture Section 002 of MATH 141 2009 01 2082

when they tend to replace a difficult integral by one that is “easier” to evaluate. Usually the
applications will be such that the integrand admits a “natural” factorization into two factors,
where either one of them becomes “simpler” under differentiation, or one becomes “simpler”
under integration.
Example B.37 Sometimes the factorization of the integral is less than obvious. Consider the
problem of integrating ln x (cf. [1, Example 2, p. 454]). The “factorization” we choose is
u = ln x, dv = dx . (54)
By (54),
dx
du = , and
x
v = x
where we have chose one convenient antiderivative.
Z Z
dx
ln x dx = (ln x)x − x·
x
= (ln x)x − x + C .
This is a derivation you should remember, as we often need an antiderivative of a logarithm.39

Example B.38 Similar to the preceding example is the integration of arctan x (cf. [1, Example
5, p. 456]). Here again the function does not admit a well defined factorization, but we can try
(54) and obtain Z Z
x
arctan x dx = x · arctan x − dx .
1 + x2
Now apply a substitution to the remaining integral, either
v = x2 ⇒ dv = 2x dx or
w = 1 + x2 ⇒ dw = 2x dx
Hence
Z Z
1
arctan x dx = x · arctan x − dw
w
= x · arctan x − ln |w| + C
= x · arctan x − ln |1 + x2 | + C .
39
Perhaps one should remember it in the “more general” form
Z
ln |x| dx = x ln |x| − x + C .
Information for Students in Lecture Section 002 of MATH 141 2009 01 2083

Of course, the absolute signs are not needed here because the argument of the logarithm func-
tion is evidently non-negative.
There are several “standard” situations where we need to use integration by parts. Suppose
we need to integrate a function of one of the following forms, where P(x) is some polynomial:
P(x) · sin x, P(x) · cos x, P(x) · e x , P(x) · cosh x, P(x) · sinh x.
In each of these cases the derivative of the polynomial is “simpler” (here meaning “of lower
degree”), while the integral of the other factor is “not more complicated”. Repeated applica-
tions cause the polynomial to disappear, leaving only an integral involving the second factor.
Here again one should remember the derivation, but not memorize the formulæ, since they can
be easily reconstructed, and there are too many variations to memorize.
The rule of integration by parts need not be used in isolation: it may be necessary to
precede or follow its use by substitutions, and several applications of integration by parts could
be needed to complete the solution to a problem.

Two applications of Integration by Parts? We saw in connection with substitutions that


we might need to use the procedure more than once. Can that occur with Integration by Parts?
It can always occur, but, if not done carefully, the second iteration will reverse the action of the
first, and return us to the integral that we began with. Where the purpose of using Integration
by Parts is to truly simplify the integrand, then it is unlikely you will make this tactical error.
But consider the first example in the next paragraph.

Solving an equation to evaluate an indefinite integral Sometimes the application of inte-


gration by parts does not appear to make any progress, but a second or more applications may
eventually produce a constraint on the integral, which enables one to evaluate it. This idea will
be extended later. Z
Example B.39 To evaluate e x cos x dx.
Solution: Set u = e x , dv = cos x, so that du = e x dx, v = sin x. Then
Z Z
e cos x dx = e sin x − e x sin x dx .
x x

The new integral is of similar difficulty to the old one. We apply integration by parts again:
U = e x , dV = sin x, dU = e x dx, V = − cos x:
Z Z
e cos x dx = e sin x − e x sin x dx
x x

Z !
x x x
= e sin x − −e cos x + e cos dx
Z
= e sin x + e cos x − e x cos dx
x x
Information for Students in Lecture Section 002 of MATH 141 2009 01 2084

Could this be an instance of the pitfall that was described in the preceding paragraph? Fortu-
nately not. The same integral appears on both sides of the equation, but with different coeffi-
cients. If we move the integral from the right side to the left, we obtain
Z
2 e x cos x dx = e x sin x + e x cos x dx + C (55)

which implies that Z


1 x
e x cos x dx = (e sin x + e x cos x) + C (56)
2
Several comments are appropriate:
1. If we had taken the second application of Integration by Parts as U = sin x, dV = e x dx,
dU = cos x dx, V = e x , then we would have obtained
Z Z
e cos x dx = e sin x − e x sin x dx
x x

Z !
x x x
= e sin x − e sin x − e cos dx
Z Z
x x x
= e sin x − e sin x + e cos dx = e x cos x dx ,

which is a tautology. The statement obtained would not be incorrect, but we would have
wasted our time to obtain an end result that could have been stated immediately, and
which does not get us any closer to solving the problem at hand.
2. Why did the constant of integration appear in equations (55), (56) but not in the preced-
ing equation? The preceding equation was of the form
Z Z
f (x) dx = g(x) + h(x) dx .

When both sides of an equation contain an indefinite integral, one understands that each
side is a set of functions which differ by a constant; no more generality is achieved if we
add the notational comment that one may add a constant to one side. But, when one side
would consist of a single function — here it is e x sin x + e x cos x — the inclusion of a
constant changes the meaning from one specific function to all functions obtained from
it by adding any real number.
3. When we integrate the dv term, we appear to be selecting a specific antiderivative. Is
this restrictive? Should we be including a constant of integration? Try to convince
yourself that he selection of a specific antiderivative is not at all restrictive; that is, if you
do include a constant of integration, the changes to the formula will cancel each other
out. For that reason you should always choose the simplest antiderivative of v that is
convenient.
Information for Students in Lecture Section 002 of MATH 141 2009 01 2085

B.10 Supplementary Notes for the Lecture of February 02nd, 2009


Release Date: Monday, February 02nd, 2009,
subject to further revision

B.10.1 §7.1 Integration by Parts (conclusion)


Recapitulation In the last lecture I introduced “Integration by Parts” — an integration tech-
nique related to the Product Rule of differentiation. I discussed the routine types of applications
we will see, as well as application to the integration of ln x and arctan x. I ended with an appli-
cation which required two successive applications of the technique, followed by the solution
of an equation.

Reduction Formulæ Since integration by parts is helpful when the integrand is a product of
two functions, one of which does not become “more complicated” under differentiation, and
the other of which does not become “more complicated” under integration, this method should
be able to assist in the integration of the product of a polynomial and a sine, cosine, exponen-
tial, or hyperbolic function. But what happens if we have to integrate other products of these
functions? In some cases the differentiations and integrations do not produce major changes
of simplicity, but they can lead to information that enables us to determine the integral, in the
way in which the preceding example was solved. This method is particularly important when
we wish to obtain an algorithm for evaluating certain general classes of integrals. Sometime
we will need to use the methods of the preceding paragraph in this connection, and sometimes
not.
Z
Example B.40 Find a general formula for evaluating I(n) = xn e−x dx where n is a positive
integer.
Solution: Let u = xn , dv = e−x dx, so du = nxn−1 dx, v = −e−x . Then
Z Z
x e dx = −x e + n xn−1 e−x dx .
n −x n −x
(57)

or I(n) = −xn e−x + n · I(n − 1) . For any specific value of n, this


Z formula may be used to evaluate
the integral recursively. For example, if we need to know x5 e−x dx, we have
Z
x5 e−x dx

= I(5) = −x5 e−x + 5 · I(4)


= −x5 e−x + 5(−x4 e−x + 4 · I(3))
= −x5 e−x + 5(−x4 e−x + 4(−x3 e−x + 3 · I(2)))
Information for Students in Lecture Section 002 of MATH 141 2009 01 2086

= −x5 e−x + 5(−x4 e−x + 4(−x3 e−x + 3(−x2 e−x + 2 · I(1))))


= −x5 e−x + 5(−x4 e−x + 4(−x3 e−x + 3(−x2 e−x + 2(−x1 e−x + 1 · I(0)))))
= −x5 e−x − 5x4 e−x − 5 · 4x3 e−x − 5 · 4 · 3x2 e−x − 5 · 4 · 3 · 2xe−x − 5!e−x + C
 
= − x5 + 5x4 + 5 · 4x3 + 5 · 4 · 3x2 + 5 · 4 · 3 · 2x1 + 5 · 4 · 3 · 2 · 1x0 e−x + C

Example B.41 [1, Exercise 50, p. 458] Let n be an integer greater than 1. Find a procedure
— i.e., a reduction formula — that can be used to evaluate secn x.
Solution:
Z Z  
n
sec x dx = secn−2 x · sec2 x dx

Applying integration by parts with u = secn−2 x, dv = sec2 x dx, we set

du = (n − 2) secn−3 x · sec x tan x dx = (n − 2) secn−2 x · tan x dx, v = tan x .

We obtain
Z Z
n n−2
sec x dx = sec x · tan x − (n − 2) tan2 x · secn−2 x dx
Z
n−2
= sec x · tan x − (n − 2) (sec2 x − 1) · secn−2 x dx
Z  
n−2
= sec x · tan x − (n − 2) secn x − secn−2 x dx

which may be solved for the desired indefinite integral. First we move all copies of the same
integral to one side of the equation:
Z Z
(n − 1) sec x dx = sec x · tan x dx + (n − 2) secn−2 x dx .
n n−2

Dividing by n − 1 yields the reduction formula


Z Z
n 1 n−2 n−2
sec x dx = sec x · tan x + secn−2 x dx . (58)
n−1 n−1
Since we know the integrals of the 0th and 2nd powers of the secant we can now find the
integral of any even positive power. For the odd positive powers we may reduce the problem
to the integration of sec x, which has not been achieved yet.

Example
Z B.42 ([7, Exercise 36, p. 480]) Follow a substitution by integration by parts to inte-
2
grate x5 e x dx.
Information for Students in Lecture Section 002 of MATH 141 2009 01 2087

Solution: The obvious substitution is u = x2 , so du = 2x dx. Then


Z Z
5 x2 1
x e dx = u2 eu du
2
to which we apply integration by parts with
U = u2 , dV = eu du, dU = 2u du, V = eu
Z !
1 2 u
= u e − 2 u du
2
Z
1 2 u
= u e − ueu du
2
to which we apply integration by parts with
u = u, dv = eu du, du = du, v = eu
Z !
1 2 u u u
= u e − ue − e du
2
1 2 u
= u e − (ueu − eu ) + C
2 !
1 2
= u − u + 1 eu + C
2
!
1 4 2
= x − x + 1 ex + C
2
2
Your solution is incomplete unless you express the integral in terms of the original variable.

7.1 Exercises
Z
[1, Exercise 10, p. 457] “Evaluate the integral arcsin x dx.”

Solution: Since you probably don’t know an antiderivative of the inverse sine, but do
know its derivative, we can try integration by parts with u = arcsin x, dv = dx, so
1
du = √ dx and v = x. (I say try because not every attempt to apply one of the
1 − x2
integration rules will be successful: the rules are valid, and do convert the given integral
into another; but if you are unable to evaluate the new integral, you haven’t fully solved
the problem, and sometimes you may have made it even more difficult to solve.)
Z Z
x
arcsin x dx = x arcsin x − √ dx
1 − x2
to which we apply the substitution w = 1 − x2 , so dw = −2x dx
Z !
1 1
= x arcsin x − √ − dw
w 2
Information for Students in Lecture Section 002 of MATH 141 2009 01 2088

1 √
= x arcsin x + ·2 w+C
2√
= x arcsin x + 1 − x2 + C

Other substitutions that could have been used are w = x2 , w = 1 − x2 .
Z 1
r3
[1, Exercise 30, p. 458] Several methods suggest themselves for evaluating √ dr.
0 4 + r2
One solution using integration by parts can be based on
r √
u = r2 , dv = √ dr ⇒ du = 2r dr, v = 4 + r2 .
4 + r2
Hence
Z 1 h √ i1 Z 1 √
r3 s 2
√ dr = r 4 + r − 2 r 4 + r2 dr
0
0 4 + r2 0
h √ i1 " #1
s 2
1 2˙ 2 32
= r 4 + r − 2 · (4 + r )
0 2 3 0
" 2 √ # 1
r −8
= · r2 + 4
3 0

16 − 7 5
=
3
which is approximately 0.115841, where the second integral was found by observation
again.
Some students may have difficulty observing the integral of dv, but a substitution could
make that phase easier.

For a solution that does not use integration by parts, try the substitution u = 4 + r2 .
Then u · du = r dr, and r2 = u2 − 4.
Z Z √
1 5
r3
√ dr = (u2 − 4) du
0 4 + r2 2
" # √5
u3
= − 4u
3
√2
16 − 7 5
=
3
as before.
Information for Students in Lecture Section 002 of MATH 141 2009 01 2089

[1, Exercise 44, p. 458] 1. Prove the reduction formula


Z Z
n 1 n−1 n−1
cos x dx = cos x · sin x + cosn−2 x dx . (59)
n n
R
2. Use Part 1 to evaluate cos2 x dx .
R
3. Use Parts 1, 2 to evaluate cos4 x ds.
Solution: First observe that the textbook has overlooked the restriction that n ≥ 1; the
equation is not meaningful when n = 0. We do know that, when n = 1,
Z
1
cos1 x dx = sin x + 0. (60)
1
1. For n ≥ 2, we may take u = cosn−1 x, dv = cos x; then
du = (n − 1) cosn−2 x · (− sin x) dx
Z
by the Chain Rule, and v = sin x. Let’s denote cosn x dx by In . Then a first
application of Integration by Parts yields
Z Z
cos x dx = cos x · sin x + (n − 1) cosn−2 x · sin2 x dx
n n−1

Z
= cos x · sin x + (n − 1) cosn−2 x · (1 − cos2 x) dx
n−1

= cosn−1 x · sin x + (n − 1) (In−2 − In ) .


Collecting all terms in In to the left side of the equation, and dividing by n − 1, we
obtain
1 n−1
In = · cosn−1 x · sin x + In−2
n n
for n ≥ 2, as desired.
2. When n = 2, the reduction formula reduces to
Z
1 1
I2 = cos x · sin x + dx
2 2
1 x
= cos x · sin x + + C.
2 2
3. When n = 4, a second application gives
1 3
I4 = cos3 x · sin x + I2
4 4 !
1 3 3 1 x
= cos x · sin x + cos x · sin x + +C
4 4 2 2
1 3 3
= · cos3 x sin x + · cos x sin x + · x + C
4 8 8
Information for Students in Lecture Section 002 of MATH 141 2009 01 2090

B.10.2 §7.2 Trigonometric Integrals


This section is concerned with integrating functions that can be expressed simply in terms of
trigonometric functions. The techniques rely on heavy use of familiar trigonometric identities.
In particular, integration of the following types of functions is considered:
• products of non-negative powers of sin x and cos x
• products of non-negative powers of tan x and sec x
The integration of other functions that can be reduced to functions of these two types is also
considered. Strategies are developed for products of these types. However, students may well
be able to use techniques already seen to integrate certain integrals of these types in ways other
than those suggested here. To reiterate: YOU MAY BE ABLE TO INTEGRATE CERTAIN
FUNCTIONS OF THE TYPES LISTED BY USING OTHER METHODS. The objective in
all of these procedures is to “simplify” the integration; where the function is a product of
trigonometric functions, this “simplification” is usually measured by a reduction in the total
degree of the product, i.e., in the total number of trigonometric factors. Since the total is finite,
repeated applications of such procedures will eventually result in successful integration.
R R
Two ways of integrating sin2 x dx and cos2 x dx: One way of evaluating these integrals
is to use one of the double angle formulæ from trigonometry:
sin 2θ = 2 sin θ · cos θ (61)
cos 2θ = cos2 θ − sin2 θ = 2 cos2 θ − 1 = 1 − 2 sin2 θ (62)
which follow from the formulæ for the sine and cosine of a sum. Two formulæ involving cos 2θ
may be rewritten as
1
sin2 θ = (1 − cos 2θ) (63)
2
1
cos2 θ = (1 + cos 2θ) (64)
2
From these we obtain
Z Z
2 1
sin x dx = (1 − cos 2x) dx
2
!
1 1
= x − sin 2x + C
2 2
Z Z
1
cos2 x dx = (1 + cos 2x) dx
2
!
1 1
= x + sin 2x + C
2 2
Information for Students in Lecture Section 002 of MATH 141 2009 01 2091

R
As we saw earlier in connection with cos2 x dx, another way to evaluate these integrals is
through integration by parts. See [1, Exercises 43(a), 44(a)(b) p. 458] which describe how
to use the reduction formula in [1, Example 6, p. 457] and an analogue for cosines for these
purposes.
Z
Strategy for evaluating sin m x · cos n x dx

0. This is a recursive procedure: if the first 2 steps do not lead to a substitution producing an
integral that may be evaluated immediately, the last 2 steps will lead to a simplification
in the integrand, after which the procedure is begun again.
1. If n is odd, use the identity cos2 x = 1 − sin2 x to convert all but one of the cosine factors
into a function of sines. Then apply the substitution u = sin x with du = cos x dx.
2. If m is odd, proceed analogously to the preceding: use the same identity to convert all but
one of the sine factors into a function of cosines. Then apply the substitution u = cos x,
with du = − sin x dx.
3. If both m and n are even, use the identities cos2 x = 12 (1+cos 2x) and sin2 x = 21 (1−cos 2x)
to express the integrand as a sum of products of sines and cosines of 2x. The degrees
of these terms will be less than the degree of the integrand we started with, so that we
have simplified the problem, and can repeat the procedure until we have completed the
integration.
4. When both m and n are odd, other identities may also be used to simplify the integrand;
for example, we can use sin 2x = 2 sin x · cos x combined with the two double angle
formulæ mentioned immediately above.
Z
Example B.43 To determine sin4 x · cos2 x dx.
Solution:
Z Z !2
4 2 1 − cos 2x 1 + cos 2x
sin x · cos x dx = · dx
2 2
Z !
1 − cos 2x − cos2 2x + cos3 2x
= dx
8
Z
1 1 1 + cos 4x
= ·x− sin 2x − dx
8 16
Z  16
1  d
+ 1 − sin2 2x · sin(2x) dx
16 dx
etc. (Use the substitution u = sin 2x to evaluate the last integral.)
Information for Students in Lecture Section 002 of MATH 141 2009 01 2092

B.11 Supplementary Notes for the Lecture of February 04th, 2009


Release Date: Wednesday, February 04th, 2009,
subject to further revision

B.11.1 §7.2 Trigonometric Integrals (conclusion)


Z
Strategy for evaluating tan m x·sec n x dx. Earlier I discussed an algorithm for integrating
products of non-negative integer powers of sines and cosines. I now consider another important
class of products of trigonometric functions.
1. The basis of the solution I propose for most of these cases is to use the facts that

d(tan x) = sec2 x dx
d(sec x) = sec x tan x dx
sec2 x = tan2 x + 1

2. If n is even and n ≥ 2, then we can replace the factor secn x by


  n−2 d   n−2 d
sec2 x 2
tan x = tan2 x + 1 2 · tan x
dx dx
which, when multiplied by tanm x — where m has any value — yields an integral that is
simplified by the substitution u = tan x.
Z
3. If n = 0 we have tanm x dx:
R
(a) When m = 0, the solution is dx = x + C.
R
(b) When m = 1, the function integrates as tan x = ln | sec x| + C = − ln | cos x| + C.
(c) When m ≥ 2, one may detach 2 powers of the tangent from the others, replacing
them by sec2 x − 1, thereby reducing the problem
Z to the integration of a lower
power of the tangent, and an integral of the form tanm−2 x · sec2 x dx, which can
be integrated following a substitution u = tan x.

Henceforth we may assume that n is odd.

4. If m is odd, we may detach one power of tan x from tanm x and one power of sec x from
secn x, and write the integrand as
  m−1 d
tan2 x 2
· secn−1 x · sec x
dx
Information for Students in Lecture Section 002 of MATH 141 2009 01 2093

which is equal to
  m−1 d
sec2 x − 1 2 · secn−1 x · sec x
dx
and can be integrated after a substitution u = sec x.

5. In the only remaining cases m is even and n is odd. One method would be to transform
the entire integrand into powers of sec x and then to use the reduction formula [1, Exer-
cise 50, p. 458] to reduce everything to the problem of integrating sec x. Other reductions
are possible: for example, into a function expressible in terms of cos x, with one factor
cos x left over — this would permit a substitution u = cos x that converts the problem to
the type we shall meet in [1, §7.3]; alternatively, one may express the integrand as a sum
of powers of tan x, and develop a reduction formula for them (cf. [1, endpapers, item 75
p. 9.]).
Z
The integral tan x dx. As observed above,
Z
tan x dx = − ln | cos x| + C = ln | sec x| + C .

Z
The integral sec x dx. The textbook observes that
Z
sec x dx = ln | sec x + tan x| + C .

Do not forget the absolute value signs when you quote these results, although you may expect
that in many of the problems you consider, where sec x + tan x is positive, omission of the signs
may not produce any visible error.
While it is possible to prove the validity of the preceding equation simply by differentiation
of the alleged antiderivative, a more direct proof requires some ingenuity.40
40
One way to derive this result is to observe that
1 cos x
sec x = =
cos x 1 − sin2 x
cos x
=
(1 − sin x)(1 + sin x)
1  cos x cos x 
= +
2 1 − sin x 1 + sin x
using ideas of partial fractions that we shall be meeting in [1, §7.4]. (See [33, pp. 505-506 ].)
Information for Students in Lecture Section 002 of MATH 141 2009 01 2094

Z
Strategy for evaluating cot m x · csc n x dx. Analogues of the preceding techniques can
simplify integrals of these types.

“Other” trigonometric identities. By adding and subtracting the expansions of sin(A ± B)


and cos(A ± B), one may obtain the following identities
1
sin A · cos B = (sin(A − B) + sin(A + B)) (65)
2
1
sin A · sin B = (cos(A − B) − cos(A + B)) (66)
2
1
cos A · cos B = (cos(A − B) + cos(A + B)) (67)
2
These identities permit the derivation of another class of useful identities. If we replace A − B
V +U V −U
and A + B respectively by U and V — equivalently, if we replace A by and B by ,
2 2
we obtain
V +U V −U
sin U + sin V = sin · cos (68)
2 2
−V + U V +U
sin U − sin V = sin · cos (69)
2 2
V +U V −U
cos U + cos V = cos · cos (70)
2 2
V +U V −U
cos U − cos V = sin · sin (71)
2 2
Z
Example B.44 Integrate (sin 50x · cos 12x) dx.
Solution: (One possible solution)
Z Z
1
(sin 50x · cos 12x) dx = (sin 38x + sin 62x) dx
2
1 1 1 1
= − · · cos 38x − · · cos 62x + C
2 38 2 62
Example B.45 ([7, Exercise 62, p. 488]) Find the volume obtained by rotating the region
bounded by the curves y = cos x, x = 0, y = 0, x = π2 about the axis y = 1.
Solution: I give a solution, by washers. (The problem can be solved by cylindrical shells also,
but that method is much more difficult.)
Z π2  
Volume = 12 − (1 − cos x)2 dx
0
Information for Students in Lecture Section 002 of MATH 141 2009 01 2095

Z π
2
= (2 cos x − cos2 x) dx
0
" # π2
x sin 2x
= 2 sin x − −
2 4 0
 π 
= π 2− .
4

7.2 Exercises
π
Z3
[1, Exercise 30, p. 466] To evaluate tan5 x sec6 x dx.
0
Solution: This integral can be evaluated in at least two different ways.
1. Since the exponent of the secant is even, we can use a substitution u = tan x, so
du = sec2 x dx.
π
Z3 Z tan π
3  2
5 6
tan x sec x dx = u5 u2 + 1 du
tan 0
0
Z √
3  2
= u5 u2 + 1 du
0
Z √
3  2
= u9 + 2u7 + u5 du
0
" 10 8 6
# √3
u u u
= + +
10 4 6 0
243 81 27 981
= + + =
10 4 6 20
2. Alternatively, since the exponent of the tangent is odd, we can use a substitution
v = sec x, so dv = sec x · tan x dx:
π
Z3 Z 2
5 6
tan x sec x dx = (v2 − 1)2 v5 dv
1
0
" #2
v10 v8 v6
= − +
10 4 6 1
981
=
20
Information for Students in Lecture Section 002 of MATH 141 2009 01 2096

Z
dx
[1, Exercise 48, p. 466] To evaluate .
cos x − 1
Solution: This integral is not of any of the forms shown in the chapter, so some ingenuity
is needed. I give more than one solution.

1. First solution, using double angle formula. When you see cos x − 1, that should
suggest the identity cos 2θ = 1 − 2 sin2 θ. Applying that identity here, with θ = 2x ,
yields
Z Z
dx dx
=
cos x − 1 −2 sin2 2x
Z
1 x x
= − csc2 dx = cot + C
2 2 2

2. Second solution, using the identity sin2 x + cos2 x = 1. The idea here resembles
the rationalization of fractions involving square roots, seen earlier.
Z Z
dx 1 cos x + 1
= · dx
cos x − 1 Z cos x − 1 cos x + 1
cos x + 1
= 2
dx
Z − sin x Z
cos x
= − 2
dx − csc2 x dx
sin x
The first integral may be evaluated in several ways, for example by using the sub-
stitution u = sin x, so du = − cos x dx. That integral becomes
Z Z
cos x du
2
dx =
sin x u2
1
= − + C1 = − csc x + C1 .
u
The second integral can be seen immediately to be − cot x + C2 . The two together
give us Z
dx
= csc x + cot x + C .
cos x − 1
But, are the two answers equal? This can be seen through trigonometric identities. For
example
1 + cos x 2 cos2 2x x
csc x + cot x = = x x = cot
sin x 2 sin 2 · cos 2 2
Information for Students in Lecture Section 002 of MATH 141 2009 01 2097

B.11.2 §7.3 Trigonometric Substitution


In this section we describe a type of substitutions which simplify certain commonly met inte-
grals. We approach these substitutions in the reverse direction from that used earlier. Whereas
earlier we investigated substitutions of the form u = g(x), this time we will usually formulate
our substitutions first in the form x = h(u); that is, we will look for a substitution that will
simplify the integrand, and then try to implement it. We usually proceed “mechanically” in
these problems; but, in principle, we are postulating the existence of an inverse function, and
should be checking that there really is an inverse whenever we use the method. I will try to
go through some of those steps in the first examples we consider, but, in practice, we often
become careless and don’t check everything unless something indicates a problem. This is
unwise; the reason that it “works” is that we usually confine the substitutions to certain well
understood pairs of functions/inverses, where all of the snags have already been worked out.
Z 2 √3
x3
Example B.46 [1, Exercise 4, p. 472] To evaluate √ dx.
0 16 − x2
Solution: √A first look at the integrand suggests that the complication comes from the ex-
pression
√ 16 − x2 in the denominator. We could try to simplify by giving this a new name,
u = 16 − x2 . That leads to
x
du = − √ dx ⇒ x dx = −u du
16 − x2
and so the integral becomes
Z 2 " #2
2 u3 40
(u − 16) du = − 16u = .
4 3 4 3
But we would like to illustrate the notion of trigonometric
√ substitution here (in a prob-
2
lem where it isn’t really needed!) The component 16 − x suggests that we might wish
to interpret the problem geometrically, with this component arising from an application of
Pythagoras’s Theorem to a right-angled triangle; equivalently, from the identity that

sin2 θ + cos2 θ = 1 .

To do this, we can first divide out the factor 16, which, when it leaves the square root, will
reappear outside as 4: r r
√ x2  x 2
2
16 − x = 4 1 − =4 1− .
16 4
This suggests a substitution
x
= sin v or, equivalently, x = 4 sin v
4
Information for Students in Lecture Section 002 of MATH 141 2009 01 2098

x
that will make into a sine or a cosine — either one will work. When we express the substi-
4
tution that way we are really working with the inverse — to conform with our earlier theory
about substitutions we should be expressing the new variable in terms of the old; so we should
be starting with
x
v = arcsin .
4

√ x 3
The range of values of x that interest us is 0 ≤ x ≤ 2 3, equivalently 0 ≤ ≤ , and
4 2
we know that the inverse sine function is defined over this domain. So, beginning with the
substitution above, we obtain
1 1 dx
dv =  2 · dx = √ ,
1 − 4x 4 16 − x2

and the integral transforms as follows:


Z 2 √3 3 Z arcsin 2 √4 3
x
√ dx = 64 sin3 v dv
0 16 − x 2 0
arcsin 4
Z π3
= 64 sin3 v dv
0

which we proceed to evaluate using the methods of the preceding section:


Z π3 Z π3  
3
64 sin v dv = 64 sin v 1 − cos2 v dv
0 0
" # π3
1 40
= 64 − cos v + cos3 v =
3 0 3
In practice this method works smoothly, but one must occasionally be careful about the
evaluation of the inverse function, remembering our original definitions of the restricted inter-
val
√ where the inverse was taken.
√ The method is indicated whenever we see expressions like
1 − x or, more generally, a − x2 , since we can transform the latter into the former by di-
2 2

vision by a positive real number. The inverse cosine could be used instead of the inverse sine,
and the results would be no more difficult.
Z √ 2
x −4
Example B.47 ([7, Exercise 8, p. 494]) Evaluate dx.
x4
Solution: We need a substitution that will convert x2 to 4 times the square of a secant.41 One
way to achieve this is to make u have the property that
x = 2 sec u ;
41
Alternatively, we could make x2 four times the square of a hyperbolic cosine.
Information for Students in Lecture Section 002 of MATH 141 2009 01 2099

so the actual substitution will be


x
u = arcsec ,
2
which implies that
dx = 2 sec u tan u du .
√ √
Under the substitution what happens to x2 − 4? It becomes 4 tan2 u, i.e., 2| tan u|. Do we
need the absolute signs? Recall that the inverse secant takes its values in the two intervals
0 ≤ u < π2 and π ≤ u ≤ 3π
2
. In these two intervals the tangent is always positive, so the absolute
signs may be dropped.
Z √ 2 Z
x −4 2 tan u
4
dx = 4u
· 2 sec u · tan u du
x Z16 sec
1
= sin2 u · cos u du
4
1
= sin3 u + C
12
We can’t leave the answer in this form, as it must be expressed in terms of the original variable
x. Since

sin u = tan u · cos u


tan u
=
sec u
1

2
x2 − 4
= x ,
2
3
1 1 (x2 − 4) 2
sin3 u = ·
12 12 x3
Hence Z √ 3
x2 − 4 (x2 − 4) 2
dx = +C.
x4 12x3
Information for Students in Lecture Section 002 of MATH 141 2009 01 2100

B.12 Supplementary Notes for the Lecture of February 09th, 2009


Release Date: Monday, February 09th, 2009

B.12.1 §7.3 Trigonometric Substitution (conclusion)


Table of trigonometric substitutions I can expand the table of substitutions of this type
given in the textbook:
Expression Inverse Substitution Substitution Identity

a2 − x2 θ = arcsin ax  x = a sin θ  1 − sin2 θ = cos2 θ
(−a ≤ x ≤ +a) − π2 ≤ θ ≤ π2

a2 − x2 θ = arccos ax x = a cos θ 1 − cos2 θ = sin2 θ
(−a ≤ x ≤ +a) (0 ≤ θ ≤ π)

a2 + x2 θ = arctan ax  x = a tan θ  1 + tan2 θ = sec2 θ
(−∞ < x < +∞) − π2 ≤ θ ≤ π2

x 2 − a2 θ = arcsec ax  x = a sec θ sec2 θ − 1 = tan2 θ
(−∞ < x ≤ −a or π ≤ θ < 3π
2 
or
π
a ≤ x < ∞) 0≤θ< 2
I have shown both sine and cosine versions of the first substitution, and could similarly have
produced a cotangent version of the tangent substitution, and a cosecant version of the secant
substitution; in practice the latter two variants are not used frequently, and usually offer no
advantages over the substitutions shown.

7.3 Exercises
Z
1
[1, Exercise 24, p. 472] Evaluate √ dt.
t2 − 6t + 13
Solution: I shall complete the square of the quadratic polynomial in the denominator
in order that, after a first substitution, this integral will be of a type that we recognize.
Since  !2 

 t − 3 
t2 − 6t + 13 = (t − 3)2 + 4 = 4  + 1 ,
2
t−3
a first substitution u = , which implies that dt = 2 du, could be applied:
2
Z Z
dt du
√ = √ .
t2 − 6t + 13 u2 + 1
Information for Students in Lecture Section 002 of MATH 141 2009 01 2101

Now I take u = tan θ, i.e., θ = arctan u. Thus − π2 < θ < + π2 .


Z Z
du sec2 θ
√ = dθ
u2 + 1 | sec θ|
Z
= | sec θ| dθ
Z
π π
= sec θ dθ since − <θ<
2 2
= ln | sec θ + tan θ| + C
  12
= ln ± 1 + tan θ + tan θ + C
2

π
where the + sign is taken since |θ| <
 2
 1
= ln 1 + u2 + u + C
2


 !2  12 !
 t − 3  t − 3
= ln 1 +  + +C
2 2
 1
= ln t2 − 6t + 13 2 + (t − 3) + (C − ln 2)

And I could rename the constant with a single letter, e.g., K = C − ln 2.

Hyperbolic substitutions It is possible to achieve the same sorts of simplifications by using


inverse hyperbolic functions. Since we have spent little time in becoming comfortable with
the hyperbolic functions, I will not discuss these substitutions in general, but may apply them
in specific cases.

Example B.48 This is [1, Exercise 31, p. 472].

1. Use trigonometric substitution to show that


Z  √ 
dx
√ = ln x + x2 + a2 + C
x2 + a2

2. Use hyperbolic substitution x = a sinh t to show that


Z  
dx −1 x
√ = sinh +C
x 2 + a2 a

Solution:
Information for Students in Lecture Section 002 of MATH 141 2009 01 2102

1. We can simplify the surd by making x = a tan u. But our theory of substitutions requires
that we express u as a function of x; so we confine ourselves, for example, to − π2 < x < π2 ,
and define u = arctan ax on that interval. Then

dx 1 a dx
du = · = 2
1+ x2 a x + a2
a2
a dx
=
sec2 u
1
⇒ dx = sec2 u du
Z a
Z
dx sec2 u du
√ =
x2 + a2 | sec u|
Z
= sec u du = ln | sec u + tan u| + C
since the secant is positive in the given interval
r 
x x 2
= ln + + 1 + C
a a
 √

= ln x + x2 + a2 − ln a + C

= ln x + x2 + a2 + C 0

where we absorb the subtracted logarithm of a constant into a new constant of integra-
tion.

2. The hyperbolic function sinh is invertible, since its derivative is positive always. If we
wish to have x = a sinh u, we can define u = sinh−1 ax . Taking differentials gives
r
p x2 √
2
dx = a cosh u du = a 1 + sinh u du = a + 1 du = x2 + a2 du
a2
Hence
Z Z
dx
√ = du = u + C
x 2 + a2
x
= arcsinh + C .
a

Example B.49 I have avoided computing the area of a disk until now. It is trivial with a
trigonometric substitution. If the disk is the set of points (x, y) such that x2 + y2 ≤ R2 , then it is
Information for Students in Lecture Section 002 of MATH 141 2009 01 2103


the region bounded by the 2 graphs y = ± R2 − x2 . Its area is
Z R√  √  Z R √
2 2 2
R − x − − R − x dx =2 2 R2 − x2 dx
−R −R
Z R √
= 4 R2 − x2 dx ,
0

since the integrand is even, and the interval of integration is symmetric around 0. The sub-
stitution u = cos−1 Rx for 0 ≤ x ≤ R implies that cos u = Rx or x = R cos u, where the
interval of integration is now from u = cos−1 0 = π2 to u = cos−1 1 = 0; dx = −R sin u du;

R2 − x2 = R| sin u| = R sin u, since the sine is positive in this interval. The integral transforms
to
Z R√ Z 0
4 2 2
R − x dx = 4 |R sin u|(−R sin u) du
π
0 2
Z 0
= −4R 2
sin2 u du
π
2
π
since the sine is positive for 0 ≤ u ≤ 2
" #0
2 u 2u
= −4R − sin
2 4 π
 π  2

2
= −4R 0 − = πR2
4
If we had not used the evenness of the integrand to reduce the original problem to integrating
over the interval 0 ≤ x ≤ R, there would have been a serious difficulty. That is because the
inverse cosine function takes its values between 0 and π.
If we had naively carried through the substitution over the entire interval −R ≤ x ≤ R,
we would have obtained
Z Rp Z −π
2
4 R2 − x2 dx = 4 |R sin u|(−R sin u) du
π
−R 2
Z 0 Z − π2
= 4 |R sin u|(−R sin u) du + 4 |R sin u|(−R sin u) du
π
2 0
Z 0 Z − π2
= 4 (R sin u)(−R sin u) du + 4 (−R sin u)(−R sin u) du
π
2 0
Z 0 Z − π2
2 2 2
= −4R sin u du + 4R sin2 u du
π
2 0
Z 0 Z π
2
= −4R2 sin2 u du + 4R2 sin2 u du .
− π2 0
Information for Students in Lecture Section 002 of MATH 141 2009 01 2104

But note that the integrand is an even function, so the two integrals would cancel, and
the answer would be 0. This is clearly incorrect, but what went wrong? The error was
in attempting to replace x by R cos u: u is not uniquely defined for − π2 ≤ x ≤ π2 ! We
could, though, have used a substitution u = sin−1 Rx over the full interval, and the correct
answer would have been obtained. We would have defined u = arcsin Rx , so sin u = Rx ,
cos u du = R1 · dx.
Z R p Z π
2
4 R2 − x2 dx = 4 (R cos u)R cos u du
−R − π2
Z π
2 1 + cos 2u
2
= 4R du
2
− π2
" #π
2 sin wu 2
= 2R u +
2 −π
 π   2π 
2
= 2R + 0 − − + 0 = πR2 .
2 2
Z
x2
Example B.50 ([7, Exercise 26, p. 494]) Evaluate √ dx.
4x − x2
Solution: Completion of the square yields
 !2 
2 2 2
 x − 2  2 2
4x − x = −(x − 4x) = 4 − (x − 4x + 4) = 4 − (x − 2) = 2 1 −  .
2

Hence
Z Z
x2 x2
√ dx = q  2 dx
4x − x2 2 1 − x−2
2
Z
(2u + 2)2
= √ 2 du
2 1 − u2
x−2
under substitution u =
Z  2

= 4 sin2 v + 2 sin v + 1 dv
x−2
under substitution v = arcsin u = arcsin
Z 2
1 − cos 2v
= 4 dv − 8 cos v + 4v
2
= 6v − sin 2v − 8 cos v + C
= 6v − 2(4 + sin v) cos v + C
Information for Students in Lecture Section 002 of MATH 141 2009 01 2105

Now the arcsine function takes values between − π2 and π2 , in which interval the cosine is posi-
tive; hence p √ 1√
cos v = + 1 − sin2 v = 1 − u2 = 4x − x2 .
2
We conclude that
Z
x2 x−2 x+6 √
√ dx = 6 arcsin − · 4x − x2 + C .
4x − x 2 2 2
Z
1
Example B.51 ([7, Exercise 28, p. 494]) Evaluate  5 dx.
5 − 4x − x2 2
Solution: As in the preceding example, I shall complete the square of the quadratic polynomial
in the denominator in order that, after a first substitution, this integral will be of a type that we
recognize. Since
 !2 
2 2 2
 x + 2 
5 − 4x − x = −(x + 4x − 5) = 9 − (x + 2) = 9 1 −  ,
3
x+2
a first substitution u = , which implies that dx = 3 du, could be applied:
3
Z Z
1 1 du
 dx = .
5 − 4x − x2 2
5
81 5
(1 − u2 ) 2
Now we can apply a second substitution u = sin φ — actually it is φ = arcsin u – where
du
dφ =  1 . We obtain
1 − u2 2
Z Z
1 du 1 dφ
=
81 5
(1 − u2 ) 2 81 cos4 φ
Z
1
= sec4 φ dφ
81 Z
1  
= tan2 φ + 1 sec2 φ dφ
81
" #
1 1 3
= tan φ + tan φ + C .
81 3
The function φ was defined to be an arcsine, so its values are in the interval − π2 ≤ φ ≤ π2 , in
which the cosine is positive. Hence
sin φ sin φ u
tan φ = = q = √
cos φ 1 − u2
+ 1 − sin2 φ
Information for Students in Lecture Section 002 of MATH 141 2009 01 2106

x+2
3 x+2
= q  2 = √
1− x+2 9 − 4x − x2
3

" #
1 1 3 (x + 2)3 x+2
and tan φ + tan φ =  + 1
81 3 3
243 5 − 4x − x2 2 81 5 − 4x − x2 2

B.12.2 §7.4 Integration of Rational Functions by Partial Fractions (Introduction, to be


continued)
In this section we shall see that an entire class of functions can be integrated by a systematic
algebraic decomposition procedure, followed by specific methods for the various components
into which we decompose the functions.

An example to illustrate the general procedure Before describing the general procedure
let us consider some examples that will make it easier to comprehend.
Z
1
[1, Exercise 14, p. 482] Evaluate the integral dx.
(x + a)(x + b)
Solution: There will be two quite different solutions, depending on whether a = b.

Case a = b: This can be integrated directly by observation, or by using the substitution


u = x + a. Z
1 1
2
dx = − +C.
(x + a) x+a
Case a , b: We try to decompose the integrand into the sum of fractions whose denom-
inators are, respectively, x + a, and x + b. The degrees of the numerators must be
less than these polynomials of degree 1, so they have to have degree 0, i.e., they
have to be constants. So suppose that
1 α β
dx = + .
(x + a)(x + b) x+a x+b
It can be shown algebraically that such a decomposition is always possible; all that
is missing is to know the values of the constants α and β. The last equation, after
multiplication of both sides by (x + a)(x + b), yields

1 = α · (x + b) + β · (x + a) .

Here are 2 ways to find α and β:


Information for Students in Lecture Section 002 of MATH 141 2009 01 2107

1. Express both sides of the equation as polynomials in x, and equate the coeffi-
cients of corresponding powers of x:
degree 0: 1 = α · b + β · a
degree 1: 0 = α + β
Solving these equations gives
1
α=
b−a
1
β=
a−b
2. This equation must be true for all values of x. Give x “convenient” values to
obtain equations in α and β, and solve those equations. Two “convenient” val-
ues are x = −a and x = −b. They give, respectively, the following equations:
1
1 = α · (−a + b) + β · 0 ⇒ α =
b−a
1
1 = β · (0) + β · (−b + a) ⇒ β =
a−b
We may now complete the integration:
Z Z !
1 1 1 1
dx = − +
(x + a)(x + b) a−b x+a x+b
1
= (− ln(x + a) + ln(x + b)) + C
a−b
1 |x + b|
= ln +C
a − b |x + a|

1 x + b
= ln +C
a − b x + a
This family of functions could be written in other ways. For example, since C · (a −
b) = ln eC·(a−b) , we could call eC·(a−b) K, and write the family as
!
1 x + b 1 x + b
ln + ln K =
ln K ·
a−b x + a a−b x + a

where K serves as the constant of integration.


Information for Students in Lecture Section 002 of MATH 141 2009 01 2108

B.13 Supplementary Notes for the Lecture of February 11th, 2009


Release Date: Wednesday, February 11th, 2009

B.13.1 §7.4 Integration of Rational Functions by Partial Fractions (conclusion)


Polynomials Recall that a polynomial [1, p. 28] is a function of the form

P(x) = an xn + an−1 xn−1 + · · · + a1 x1 + a0 x0

where a0 , . . . , an are real numbers, called the coefficients of the polynomial. Except for the
zero polynomial, which is the constant function 0, all polynomials will have a largest integer
m such that am , 0; m is called the degree of the (non-zero) polynomial, and am is called the
leading coefficient; a0 is called the constant term. We usually write x0 simply as 1, and x1
simply as x.

Rational Functions A rational function is a ratio of polynomials of the form


A(x)
B(x)
where A and B are polynomial functions. The function will have as discontinuities the roots of
B, if there are any.

The goal of partial fraction decompositions In a partial fraction decomposition we express


a ratio of polynomials as a sum of “partial” fractions — fractions that have special properties.
In these “partial” fractions the denominator polynomials will always be powers of one polyno-
mial that is irreducible, i.e., it cannot be factored further. It is a theorem of algebra that

Theorem B.52 If a non-zero real polynomial is irreducible then it must be of one of the fol-
lowing forms:
1. a non-zero constant

2. a polynomial of degree 1, of the form ax + b, where a , 0.

3. a polynomial of degree 2 without real roots, i.e., of the form ax2 + bx + c where a , 0
and b2 < 4ac.
The only type of ratio to which we apply this decomposition is one where the degree of the
numerator is strictly less than the degree of the denominator; if the rational function that we
start with does not have this property, then we will have to preprocess it to obtain a ratio of this
type; after the procedure of partial fraction decomposition is applied, the resulting “partial”
Information for Students in Lecture Section 002 of MATH 141 2009 01 2109

fractions will have the same property — that the degree of their numerator will always be
less than that of the denominator. We will then show that we are able to integrate all partial
A(x)
fractions of these types, and so we will be able to integrate all rational polynomials .
B(x)
The procedure we shall develop depends on having such a factorization of the denominator
of the given ratio into irreducible polynomials. In this course we shall not be concerned with
finding the factorization itself, beyond knowing
• that P(a) = 0 ⇒ x − a divides P(x) — the so-called “Factor Theorem”;

• how to factorize quadratic polynomials, both by using the quadratic formula, and by
“completing the square”;

• that, for any positive integer n,

an − bn = (a − b)(an−1 + an−2 b + . . . + abn−2 + bn−1 )

• that, for any positive integer n,

a2n+1 + b2n+1 = (a + b)(a2n − a2n−1 b + . . . − ab2n−1 + b2n ).

Sometimes a factor x − a may divide P(x), so that P(x) = (x − a)Q(x), and again x − a may
also divide Q(x), and possibly divide even more quotients. We speak of the multiplicity of the
factor x − a of P(x), and say that a is a root of P(x) of that multiplicity.

“Long division” of polynomials. You should know from high school how to divide a poly-
nomial B(x) into a polynomial A(x) and obtain a quotient polynomial and a remainder poly-
nomial. The procedure is similar to long division of integers, and you will be reminded very
briefly of it at the lecture. (You will see an example of long division of polynomials in the
left margin of [1, p. 474], but the way you learned to write such a calculation may be slightly
different from that used in the textbook.)

A(x)
The first step. The procedure for integrating begins with the division of B(x) into A(x),
B(x)
obtaining a quotient and a remainder:

A(x) = Q(x) · B(x) + R(x)

where the remainder has degree less than that of the divisor B(x). This is an essential step;
if it is omitted, it may cause the subsequent steps to fail. However, it is not necessary if the
degree of B is greater than the degree of A, since, in that case, the quotient will be 0 and the
remainder will be A(x).
Information for Students in Lecture Section 002 of MATH 141 2009 01 2110

An example to illustrate the general procedure.


Z
1
[1, Exercise 14, p. 482] Evaluate the integral dx.
(x + a)(x + b)
Solution: I discussed an example of this type at the last lecture; the solution, shown on
pages 2106 through 2107 of these notes, may be written as
Z Z !
1 1 1 1
dx = − +
(x + a)(x + b) a−b x+a x+b
1
= (− ln(x + a) + ln(x + b)) + C
a−b
1 |x + b|
= ln +C
a − b |x + a|

1 x + b
= ln +C
a − b x + a

Suppose that, instead of the given integral, we wished to integrate


Z 3
x + (a + b)x2 + abx + 1
dx .
(x + a)(x + b)
This function cannot be expanded into partial fractions until it is arranged that the degree
of the numerator — presently 3 — be less than the degree of the denominator — 2. If
we divide the denominator into the numerator, we find that

x3 + (a + b)x2 + abx + 1 = x · (x + a)(x + b) + 1 .


Z !
1
Hence the integral may be expressed as x+ dx, and its value will be
(x + a)(x + b)

x2 1 x + b
+ ln +C
2 a − b x + a
What would have happened if we had attempted to expand this function into partial frac-
tions? No such decomposition can exist. Using the first method (equating coefficients of
corresponding powers) we would have obtained 4 equations which would overdetermine
the constants α and β, and which would be inconsistent — there would be no solution.
But, if we used the second method, and didn’t take enough equations, we might not no-
tice that there was an inconsistency, and the alleged partial fraction would be incorrect.
Information for Students in Lecture Section 002 of MATH 141 2009 01 2111

The general procedure. The general procedure has many facets, and some will not be ex-
plicitly discussed in the lecture; you are expected to work many problems in this section of the
textbook, not only problems discussed in the lecture or appearing on WeBWorK or the quizzes.
Remember the steps we need to follow:

1. The first step in any of these problems is to ensure that the degree of the
numerator must be less than that of the denominator. If it is not, you must
divide the denominator into the numerator, obtaining a quotient and a re-
mainder. The quotient integrates as a polynomial, and we are left with the
ratio of the remainder to the denominator, for which the methods we are
discussing will enable a complete solution.
2. Factorize the denominator into a product of polynomials of degrees 1 and 2
which are irreducible, i.e., which do not factorize further into lower degree
polynomials. The quadratic factors will be those having no real roots. Group
factors which are exactly the same together, so that your denominator is a
product of powers of distinct, irreducible polynomials.
3. Then the fraction must be decomposed into partial fractions using methods
we have illustrated before, and continue in this lecture.
Z
x2 − 2x − 1
Example B.53 [1, Exercise 28, p. 482] Evaluate the integral dx.
(x − 1)2 (x2 + 1)
Solution: The denominator has two distinct irreducible factors: x − 1, of degree 1 and multi-
plicity 2, and x2 + 1, an irreducible quadratic factor of degree 2 and multiplicity 1. The degree
of the numerator is less than 4, which is the degree of the denominator, so there is no need for
any long division. One type of partial fraction decomposition is of the form

x2 − 2x − 1 αx + β γx + δ
2 2
= 2
+ 2
(x − 1) (x + 1) (x − 1) x +1
in which we take the most general numerator in each case, of degree less than the degree of the
denominator. This is not the most useful type of partial fraction decomposition, but we will
carry this step out and then improve on it. Multiplying both sides by (x − 1)2 (x2 + 1), we obtain
a polynomial equation

x2 − 2x − 1 = (αx + β)(x2 + 1) + (γx + δ)(x2 − 2x + 1)


⇔ x2 − 2x − 1 = (α + γ)x3 + (β − 2γ + δ)x2 + (α + γ − 2δ)x + (β + δ)

Equating coefficients of corresponding powers of x yields equations corresponding to the terms


of degrees 3, 2, 1, 0:

α+γ = 0
Information for Students in Lecture Section 002 of MATH 141 2009 01 2112

β − 2γ + δ = 1
α + γ − 2δ = −2
β + δ = −1

which we proceed to solve, obtaining

(α, β, γ, δ) = (1, −2, −1, 1) ,

so the decomposition is

x2 − 2x − 1 x−2 −x + 1
2 2
= 2
+ 2 .
(x − 1) (x + 1) (x − 1) x +1
We shall see that we will be able to integrate the second summand immediately, by breaking it
into two parts. We can integrate the first summand if we first apply the substitution u = x − 1:
Z Z Z !
x−2 u−1 1 1
dx = du = − du
(x − 1)2 u2 u u2
1
= ln |u| + + C
u
1
= ln |x − 1| + +C
x−1
In practice we anticipate the results of this substitution by refining the partial fraction decom-
position: in place of a summand of the form

an−1 xn−1 + an−2 xn−2 + . . . + a0


(x − a)n
we repeatedly divide x − a into the numerator, so that we can express the numerator as a sum
of powers of x − a; then we decompose the fraction and divide excess powers of x − a, to obtain
a decomposition of the form
bn bn−1 b1
n
+ n−1
... +
(x − a) (x − a) x−a
in which we can integrate each of the summands at sight. This decomposition can be accom-
plished as a second phase of partial fraction decomposition, or immediately, by assuming a
decomposition of the form

x2 − 2x − 1 ζ η γx + δ
2 2
= 2
+ + 2
(x − 1) (x + 1) (x − 1) x−1 x +1
Information for Students in Lecture Section 002 of MATH 141 2009 01 2113

which leads to the polynomial identity


x2 − 2x − 1 = ζ(x2 + 1) + η(x − 1)(x2 + 1) + (γx + δ)(x − 1)2
⇔ x2 − 2x − 1 = (η + γ)x3 + (ζ − η − 2γ + δ)x2 + (η + γ − 2δ)x + (ζ − η + δ)
which we proceed to solve, obtaining
(ζ, η, γ, δ) = (−1, 1, −1, 1) ,
Z
x2 − 2x − 1
dx
(x − 1)2 (x2 + 1)
Z Z Z Z
−1 1 −x 1
= dx + dx + dx + dx
(x − 1)2 (x − 1)1 x2 + 1 x2 + 1
1 1
= + ln |x − 1| − ln |x2 + 1| + arctan x + C
x−1 2
1 1
= + ln |x − 1| − ln(x2 + 1) + arctan x + C
x−1 2
1 |x − 1|
= + ln √ + arctan x + C
x−1 x2 + 1

7.4 Exercises
Z
1
[1, Example 19, p. 482] Evaluate the integral dx .
(x + 5)2 (x − 1)
Solution: Since the degree of the numerator is 0, and that of the denominator is 2 + 1 =
3 > 0, we can skip the long division step. We have to decompose the fraction into a sum
of partial fractions, which we may take to be of the form
1 A B C
2
= 2
+ 1
+ .
(x + 5) (x − 1) (x + 5) (x + 5) (x − 1)1
Multiplying both sides by (x + 5)2 (x − 1), we obtain
1 = A(x − 1) + B(x − 1)(x + 5) + C(x + 5)2 . (72)
i.e.,
0x2 + 0x1 + 1x0 = A(x − 1) + B(x2 + 4x − 5) + C(x2 + 10x + 25) . (73)
One way to obtain the values of A, B, C is simply to equate coefficients of like powers of
x:
0 = B+C
0 = A + 4B + 10C
1 = −A − 5B + 25C
Information for Students in Lecture Section 002 of MATH 141 2009 01 2114

 
which we may solve to show that (A, B, C) = − 61 , − 36
1 1
, 36 .
Another method of solution is to assign to x “convenient” values of x, and thereby obtain
more convenient equations to solve. Two values which re “convenient” are x = 1 and
x = −5, and they yield from equation (72) the following equations:

x = 1 ⇒ 1 = 36C

and
x = −5 ⇒ 1 = −6A
implying that A = − 61
and C = 361
. To obtain the value of B we would need a third
equation. This could be obtained by equating coefficients, as in the earlier method, or by
simply choosing another value; e.g.,
2 16
x = −1 ⇒ 1 = −2A − 8B + 16C = − 8B +
6 36
which yields the same values as before. Integration is straightforward:
Z Z  1 
1  − 16 1
− 36 36  
2
dx =  2
+ +  dx
(x + 5) (x − 1) (x + 5) x+5 x−1
1 1 1 1
= · − ln |x + 5| + ln |x − 1| + K .
6 x + 5 36 36
This indefinite integral could be further simplified, e.g., by combining the two logarith-
mic terms into the logarithm of a quotient of polynomials of degree 1.

Repeated irreducible quadratic factors.


x4 + 1
Example B.54 [7, Exercise 38, p. 504] To integrate .
x(x2 + 1)2
Solution: What distinguishes this example from those studied earlier is the multiplicity of the
irreducible quadratic factor x2 + 1 in the denominator; this is the first case we have seen where
that multiplicity exceeds 1. we proceed analogously to the last phase of the preceding example,
always taking the numerator to be the most general polynomial whose degree is less than the
degree of the irreducible factor in the denominator, but now taking separate summands for the
powers of that irreducible factor. Thus we assume a decomposition of the form
x4 + 1 α βx + γ δx + η
2 2
= +  2
+ 2
x(x + 1) x x2 + 1 x +1
and multiply through by the denominator on the left, to obtain the polynomial identity
 2  
x4 + 1 = α x2 + 1 + (βx + γ)x + (δx + η) x2 + 1 x
Information for Students in Lecture Section 002 of MATH 141 2009 01 2115

in which the identification of coefficients of like powers of x leads to the 5 equations

α+δ = 1
η = 0
2α + β + δ = 0
γ+η = 0
α = 1

implying that

(α, β, γ, δ, η) = (1, −2, 0, 0, 0) .


x4 + 1 1 −2x
= +  .
2
x(x + 1) 2 x x2 + 1 2
Z Z Z
x4 + 1 1 −2x
dx = dx +  dx
2
x(x + 1) 2 x x +1 2
2

1
= ln |x| + 2 +C
x +1
Z
1 constant
 n dx. Should one of the partial fractions be of the form  , where n is
x2 + 1 n
x2 +1
an integer greater than 1, we can begin the integration by a substitution u = arctan x, which
1
implies that du = 2 dx, and
x +1
Z Z
1 1
 n dx = du
x2 + 1 sec2n−2 u
Z
= cos2n−2 u du
Z !n−1
1 + cos 2u
= du ,
2

which, in principle, we know how to integrate.


Z
1
Example B.55 To evaluate the indefinite integral,  dx.
x2 + 1 2
dx
Let x = tan u, i.e., u = arctan x. Then du = .
1 + x2
Z Z
1
 dx = sec2 u du
x2 +1 2
Information for Students in Lecture Section 002 of MATH 141 2009 01 2116

Z
= cos2 u du
Z
1 + cos 2u
= du
2
!
1 u sin 2u
= + +C
2 2 4
arctan x sin u · cos u
= + +C
2 2
arctan x tan u
= + +C
2 2 sec2 u
arctan x x
= + 2
 +C
2 2 tan u + 1
arctan x x
= + 2
 +C
2 2 x +1
The presence of the arctangent function in the integral indicates that this could not have been
evaluated without some step equivalent to this use of the arctangent function in a substitution.

Rationalizing Substitutions. In some integrals in which the integrand is not originally a


rational function, it can be transformed into a rational integrand by an appropriate substitution.

Example B.56 ([7, Exercise 40, p. 504]) “Make Z a substitution to express the integrand as a
1
rational function, and then evaluate the integral √ dx.”
x − √x + 2
Solution: One substitution that suggests itself is u = x + 2. Under this substitution, u2 =
x + 2, dx = 2u du,
Z Z
1 2u
√ dx = du
x− x+2 (u − 2)(u + 1)
Z !
2 2 1
= + du
3 u−2 u+1
2
= (2 ln |u − 2| + ln |u + 1|) + C
3
2  √ 2 √ 
= ln
x + 2 − 2 x + 2 + 1 + C
3
4 √ 2  √ 
= ln x + 2 − 2 + ln x + 2 + 1 + C ,
3 3

where I removed the absolute signs from the second term because x + 2 + 1 must be positive.
Information for Students in Lecture Section 002 of MATH 141 2009 01 2117

B.14 Supplementary Notes for the Lecture of February 16th, 2009


Release Date: Monday, February 16th, 2009

B.14.1 §7.5 Strategy for Integration


The textbook suggests a strategy for solving integration problems:
1. Simplify the integrand if possible.
2. Look for an obvious substitution.
3. Classify the integrand according to its form.
4. Try again. The preceding instructions are vague, depend on your experience and your
intuition, and are occasionally not appropriate, as sometimes the best way to attach a
problem will be obscured by these methods. So be prepared to try again. As for experi-
ence, you need to work many problems to acquire it.

Table of integration formulæ Many of these formulæ are just recasts of familiar differenti-
ation formulæ that you already know. But you should remember the integrals of tan x, cot x,
sec x, csc x, even though you may know how to derive some of them.

Can We Integrate All Continuous Functions? Read this subsection. For most of the func-
tions you will meet in this course it will be possible to integrate them. If that is not the case,
it could be that we are asking you indirectly to do something else than to integrate. You are
not expected to be able to detect which functions cannot be integrated in terms of elementary
functions: this is a difficult problem even for a trained mathematician. “You may be assured,
though, that the integrals in the...exercises are all elementary functions.”

7.5 Exercises
Z
x ln x
[1, Exercise 58, p. 489] Evaluate √ dx.
x2 − 1
Solution: A first impression is that the most complicated part of the integrand is the
logarithm, and I would like to dispose of it. One way to do that would be through a
substitution, but that could well render the denominator rather complicated. I propose to
try integration by parts, and to assign u and v so that the ln x term is part of u. But how
much of the integrand should be taken for u? I observe that if I take only the ln x term,
it leaves a function that is easy to integrate, so it is an ideal first step:
dx
u = ln x ⇒ du =
x
Information for Students in Lecture Section 002 of MATH 141 2009 01 2118

x
dv = √ dx
x2 − 1
But what is v? If you don’t see, use the substitution U = x2 :
Z Z
x 1 1
√ dx = √ dU .
2
x −1 2 U −1
You should be able to evaluate this last integral by sight; but, if you can’t, try the substi-
tution V = U − 1:
Z Z √ √
1 1 1 1
v= √ dU = √ dV = V + C = x2 − 1 + C .
2 U −1 2 V
Hence Z Z √ 2
x ln x √ x −1
2
dx = (ln x) x − 1 − dx .

x2 − 1 x
The problem isn’t solved yet, but at least the logarithm is gone. Now I propose to try a
trigonometric substitution to simplify the square root:
w = arcsecx ⇒ x2 − 1 = tan2 w, x dx = sec2 w tan w dw :

Z √ Z
x2 − 1 | tan w|
dx = · sec w tan w dw
x Z sec w
= tan2 w dw
Z
= (sec2 w − 1) dw
= tan w − w + C

= sec2 w − 1 − arcsec x + C

= x2 − 1 − arcsec x + C
from which we may conclude that
Z √ √
x ln x
√ dx = (ln x) x2 − 1 − x2 − 1 + arcsec x + C1
x2 − 1
This was not the only way to solve this problem, and it may not have been the best! But
I have written down the way I solved it first. One shorter, but less intuitive way would
√ x
be to use the substitution u = x2 − 1, which implies that du = √ dx. Then
x2 − 1
Z Z √ Z
x ln x 1
√ dx = ln u2 + 1 du = ln(u2 + 1) du .
2
x −1 2
Information for Students in Lecture Section 002 of MATH 141 2009 01 2119

Does this look simpler to you? It can be integrated by parts by taking u = ln(u2 + 1),
du
dv = du, so du = 2 2
u +1
Z Z
1 2 1 2 u2
ln(u + 1) du = u ln(u + 1) − du .
2 2 u2 + 1
The point is that the last integrand is rational, and we know how to integrate all such
functions!
Another strategy would be to use the same trigonometric inverse substitution immedi-
ately: w = arcsec x ⇒ dx = sec w tan w dw ⇒
Z Z  
x ln x
√ du = sec2 w ln sec w dw .
x2 − 1
In the last integral you should recognize ln sec w as being an antiderivative of tan w. This
suggests using integration by parts with ũ = ln sec w and dṽ = sec2 w dw: dũ = tan w dw,
ṽ = tan w.
Z   Z
sec w ln sec w dw = (ln sec w)(tan w) − tan2 w dw
2

Z
= (ln sec w)(tan w) − (sec2 w − 1) dw
= (ln sec w)(tan w) − tan w + w + C
√ √
= (ln x) x2 − 1 − x2 − 1 + arcsec x + C

In several places I have casually suppressed absolute value signs; these steps can be
justified, and are consequences of the way in which we defined the inverse functions.
Z 2π √
Exercise B.1 To integrate 1 + sin x dx.
0

Solution: This is an interesting integral which, at first glance, does not appear to fit into any of
the families of integrals we have been studying. However, it can easily be seen that, except for
the determination of a sign, the integral is not very difficult to evaluate. But the sign question
is delicate, and even the previous edition of a well known text book overlooked√this difficulty.
In the Student Solution Manual to that textbook the answer was given to be −2 1 − sin t + C,
and the following hint was given for integration:

“Multiply numerator and denominator of the integrand by 1 − sin x.”
The hint is a good one, and does, indeed, lead to one way of solving the problem. Unfortu-
nately, the answer that was given in that textbook was correct only within certain intervals.
Information for Students in Lecture Section 002 of MATH 141 2009 01 2120

Solution following the suggestion


√ s
Z √ Z √ Z
1 − sin x 1 − sin2 x
1 + sin x dx = 1 + sin x · √ dx = dx
1 − sin x 1 − sin x
Z r Z
cos2 x | cos x|
= dx = √ dx
1 − sin x 1 − sin x
Had the absolute signs not been present, we could make the substitution u = sin x, subject to
which we would have du = cos x · dx
Z Z
cos x 1
√ dx = √ du
1 − sin x 1−u

= −2 1 − u + C

= −2 1 − sin x + C

Unfortunately, this function has derivative 1 + sin x only when cos x is non-negative, i.e.,
only when 4n−1 · π ≤ x ≤ 4n+1 · π, where n is any integer. For example, the value of the definite
Z 2π2 √ 2
h √ i2π
integral 1 + sin x dx is certainly not equal to −2 1 − sin x = 0 , since the integrand
0
0 √
is positive for most x, so the area under the curve y = 1 + sin x must be positive; the correct
value is
h √ i π2 h √ i 3π2 h √ i2π
−2 1 − sin x + 2 1 − sin x π + −2 1 − sin x 3π
0 2 2
h √ i 3π2 h √ i π2
= 2 1 − sin x π + −2 1 − sin x −π
√ √ 2 √ 2

= 2 2+2 2=4 2,0

Other ways to find the indefinite integral While we cannot remove the sign difficulties
in this problem, we can show that the problem does, in fact, lend itself to a more systematic
integration — i.e., the hint given above is not really necessary. One way to see this is to
remember that we have a trigonometric identity that expresses 1 + cos θ as a square. But, as
the trigonometric function given here is a sine, and not a cosine, one must first arrange for the
presence of a cosine. One way is as follows:
Z √ Z r π 
1 + sin x dx = 1 + cos − x dx
2
Z r π x
= 2 cos2 − dx
4 2
√ Z  π x 
= 2 cos − dx
4 2
Information for Students in Lecture Section 002 of MATH 141 2009 01 2121

Another approach, suggested by a student in this course several years ago, is to observe that
r
√ x x x x
1 + sin x = sin2 + cos2 + 2 sin · cos
2 2 2 2

x x
= cos + sin .
2 2
To integrate this we need to determine the sign of the  xfunction inside the absolute signs. This
√ π
can be done by observing that it is equal to 2 sin + , essentially the same function as
2 4
determined just above.
A more systematic approach would have been to attempt to simplify the original integral
by the substitution u = sin x, which would imply that du = cos x dx, so
Z √ Z √
1+u
1 + sin x dx = du
cos x
Z √ Z
1+u 1
= ± √ du = ± √ du
1 − u2 1−u
√ √
= ∓2 1 − u + C = ∓2 1 − sin x + C
where the sign still depends upon the interval in which x is located.

What, then, is one antiderivative of 1 + sin x? We have found that, if we confine
ourselves to one interval of the form 4n+1 π < x < 4n+3 π, any antiderivative has the form
√ 2 2
−2 1 − sin x + C; and, if we confine ourselves to one interval of the form 4n+3 π < x <
4n+5
√ 2

2
π, any antiderivative has the form 2 1 − sin x + C. By choosing the constants to make
the function continuous (indeed, differentiable) we can patch such subfunctions together to
form an antiderivative which is valid over an extended domain. The function f defined by the
following table is one such antiderivative:
x f (x)
··· √ · ·√·
−3 −1
2
π ≤ x < 2 π −4√ 2 + 2√ 1 − sin x
−1
2
π ≤ x < 12 π 0 √2 − 2 √1 − sin x
1
2
π ≤ x < 23 π 0 √2 + 2 √1 − sin x
3 5
2
π ≤ x < 2π 4 √2 − 2 √1 − sin x
5 7
2
π ≤ x < 2π 4 √2 + 2 √1 − sin x
7 9
2
π ≤ x < 2π 8 √2 − 2 √1 − sin x
9 11
2
π ≤ x < 2 π 8 √2 + 2 √1 − sin x
11
2
π ≤ x < 13
2
π 12 2 − 2 1 − sin x
··· ···
Information for Students in Lecture Section 002 of MATH 141 2009 01 2122

√ Z 2π
We can verify that the value of the definite integral 1 + sin x dx is [ f (x)]2π
0 = f (2π) −
 √ √   √ √ 0 √
f (0) = 4 2 − 2 1 − sin 2π − 0 2 − 2 1 − sin 0 = 4 2.

B.14.2 §7.6 Integration Using Tables and Computer Algebra Systems (OMIT)
This section “is not examination material, but students are to try to solve the problems manu-
ally.”

B.14.3 §7.7 Approximate Integration (OMIT)


This section is not part of the syllabus of this course.

B.14.4 §7.8 Improper Integrals


Piecewise continuous integrands Consider a function f that is continuous everywhere in an
interval [a, b], including continuity from the right at b and from the left at a. For such functions
we have developed the theory of the definite integral, and the Fundamental Theorem applies.
Now suppose that there is a point c such that a < c < b, where f has a jump discontinuity:
lim f (x) and lim+ f (x) both exist, but are different. It is possible to define the integral of f as
x→c− x→c
follows: Z Z Z
b c b
f (x) dx = f (x) dx + f (x) dx ,
a a c
and it can be shown that the familiar properties of the definite integral hold here, with the
exception of the Fundamental Theorem. We can proceed in the usual way with such integrals,
until we need to actually evaluate them; then we must split them up at the jump discontinuity
before we attempt to apply the Fundamental Theorem. More generally, this definition “works”
for functions with many jump discontinuities.42 43

Other types of generalizations In this section we wish to consider other types of general-
izations related to a condition of the original definition of the definite integral that fails to be
satisfied. We will follow the terminology of the textbook, calling two types of “impropriety”
Type 1 and Type44 2, but students should be aware that these terms are not in universal use. The
42
We will not explore here what limits — if any — exist on the number of discontinuities.
43
We have already seen that there may be other situations where the splitting of an integral into pieces for
different parts of the domain may simplify the integration; but, where there is a jump discontinuity, the splitting
is not by choice, it is by our definition of the meaning of the generalized symbol.
44
Note that the terms Type 1 and Type 2 refer to a type of “impropriety” — not to a type of integral. One
improper integral could contain multiple instances of Type 2, and as many as 2 instances of an impropriety of
Type 1.
Information for Students in Lecture Section 002 of MATH 141 2009 01 2123

general spirit of these definitions, and of the preceding generalization to jump discontinuities,
is that the familiar properties of integrals proved for continuous functions should hold for these
broader classes wherever they make sense. We shall still have to keep away from any situation
that might lead us to attempt to, for example, add +∞ to −∞, and any other undefinable oper-
ations. Remember that, in this theory, you must rely on the definition, and not attempt to write
down what “makes sense”. The restrictions in some of these definitions are needed to avoid
paradoxes elsewhere.

Type 1: Infinite Intervals Our original definition of the definite integral was given for a
finite interval. If we wish to speak of an integral where either or both limits are infinite, we
need to define what these are to mean. The definition we give is one that is consistent with the
definition for finite intervals, and preserves those properties of the integral that are meaningful
when the limits before infinite. I repeat the boxed definition on [1, p. 509]:
Z ∞ Z t
Definition B.6 1. f (x) dx = lim f (x) dx provided the integral on the right exists
a t→∞ a
for all t ≥ a, and the limit exists as a finite number. The “improper” integral on the left
is then said to converge or to be convergent. If the limit does not exist, the improper
integral is divergent.

2. An analogous definition holds when the upper limit is −∞, or when the lower limit is
either of ±∞. Read the textbook.

3. When both limits of the definite integral are infinite, we define the value to be the sum
of two integrals obtained by splitting the domain. It can be shown that it doesn’t matter
where the line is split, the following definition will always give the same value:
Z ∞ Z a Z ∞
f (x) dx = f (x) dx + f (x) dx
−∞ −∞ a

R∞
, TWO limits have to be de-
4. It is ESSENTIAL to understand that, in the definition of −∞ Z t
termined independently. IT IS NOT CORRECT TO CONSIDER ONLY lim f (x) dx.
t→∞ −t
In this case one has to add two finite numbers; if either of the limits does not exist as a fi-
nite number — and that includes being infinite — the sum is not finite, and the improper
integral does not exist.

Type 2: Discontinuous Integrands I have observed above how to cope with a finite, jump
discontinuity in the integrand. Here we are interested in other types of discontinuity, in partic-
ular, discontinuities where the function has a vertical asymptote. If the discontinuity occurs at
Information for Students in Lecture Section 002 of MATH 141 2009 01 2124

the left end-point of the interval, then the value of the “improper” integral is defined by
Z b Z b
f (x) dx = lim+ f (x) dx .
a t→a t
If the discontinuity is at the right end-point b, then the value of the “improper” integral is
defined by Z b Z t
f (x) dx = lim− f (x) dx .
a t→b a
And, if the discontinuity occurs at a point c between a and b, then the definition is based on
splitting the integral into two parts:
Z b Z c Z b
f (x) dx = f (x) dx + f (x) dx ,
a a c
where both of the summands on the right are defined to be limits as above, and the two limits
have to be evaluated independently. It is important to understand that in this case it is not
acceptable that the two limits be linked so that one may affect the other.
Z 2
1
Example B.57 The improper integral dx does not converge, since at least one of the
0 x−1
limits Z t Z 2
1 1
lim− dx , lim+ dx
t→1 0 x−1 t→1 t x−1
does not exist; indeed, neither of them exists! We say that this improper integral diverges.
Without this severe definition we might find that some of the properties we wish the integral
to possess might not be present.
Z ∞
1
Example B.58 ([1, Example 4, p. 511]) “For what values of p is the integral dx con-
1 xp
vergent?”
Solution:
Z ∞ Z t
1 1
dx = lim dx
1 xp t→∞ 1 x p



 lim [ln x]t1 when p = 1

 t→∞
=  1 h it


 lim x1−p when p , 1
 t→∞ 1− p 1



 lim [ln t − 0] = ∞ when p = 1


 t→∞ " #


 1 lim 1 − 1 = 1
 when p > 1
=   1 − p t→∞ t p−1 p − 1


 h i


 1
 lim t1−p − 1 = ∞
 1 − p t→∞ when p < 1
Information for Students in Lecture Section 002 of MATH 141 2009 01 2125

This result will be needed in Chapter 11 in connection with the “p-series test” [1, p. 700]. In
1
the special case p = 1, it states that the area under the hyperbola y = from x = 1 indefinitely
x
to the right is unbounded. By symmetry, that region should have the same area as the area
between x = 0 and x = 1 under the same curve, and over the line y = 1. That area is the value
of the improper integral
Z 1 Z 1
1 1
dx = lim+ dx
0 x t→0 t x
= lim+ [ln x]1t
t→0
= − lim+ ln t = +∞
t→0

Example B.59 Consider the improper integrals


Z +∞ Z +∞
1 x
2
dx , 2
dx .
−∞ x + 1 −∞ x + 1

In both of these cases we must split the interval of integration at a finite point, and consider
two improper integrals of Type 1 independently. In the first case an antiderivative is arctan x,
so
Z +∞ Z 0 Z +∞
1 1 1
2
dx = 2
dx + dx
−∞ x + 1 −∞ x + 1 0 x2 + 1
Z 0 Z b
1 1
= lim dx + lim dx
a→−∞ a x2 + 1 b→+∞ 0 x2 + 1
= lim (0 − arctan a) + lim (arctan b − 0)
a→−∞ b→+∞
 π π
= − − + = π,
2 2
and the improper integral is convergent. (We could have split the interval at any convenient
point other than 0.)
But, in the second case, we have
Z +∞ Z 0 Z +∞
x x x
2
dx = 2
dx + 2
dx
−∞ x + 1 −∞ x + 1 0 x +1
Z 0 Z b
x x
= lim dx + lim dx
a→−∞ a x2 + 1 b→+∞ 0 x2 + 1
! !
ln(x2 + 1) ln(x2 + 1)
= lim 0 − + lim −0 .
a→−∞ 2 b→+∞ 2
Information for Students in Lecture Section 002 of MATH 141 2009 01 2126

Here we find that each of the limits is infinite: the integral is said to be divergent. In as-
signing a meaning to a doubly infinite integral we do not permit +∞ and −∞ to be ap-
proached at the same rate. That is, it is not permitted to interpret this integral as being equal
Za
x
to lim 2
dx. (Here the finite, symmetric integral is 0, so the limit is 0. But we do not
a→∞ x +1
−a
give that meaning to the original, improper integral, which we insist diverges.

7.8 Exercises
Z∞
dz
[1, Exercise 18, p. 515] Determine whether the integral is convergent or di-
z2 + 3z + 2
0
vergent. Evaluate it if it is convergent.
Solution: Using standard methods of Partial Fractions we can show that
Z Z !
dz 1 1
= − dz
z2 + 3z + 2 z+1 z+2
= ln|z + 1| − ln |z + 2|

z + 1
= ln .
z + 2
Hence
Z∞ " #
dz z + 1 a
= lim ln
z2 + 3z + 2 a→∞ z + 2 0
0
 
 1 + 1a 1 
= lim  ln − ln 
a→∞ 1 + a2 2
1
= ln 1 − ln = ln 2
2
Z∞
dz
Note that we could not have expressed as the difference of the improper
z2 + 3z + 2
0
Z∞ Z∞
dz dz
integrals and , as both of these improper integrals are divergent and the
z+1 z+2
0 0
difference of their “values” is not defined.
Information for Students in Lecture Section 002 of MATH 141 2009 01 2127

Extended Definition of the Integral The original definition in the textbook — in terms of
Riemann sums — was for continuous functions f , where the interval of integration was a finite
interval [a.b]. This definition has been extended in several different ways.
1. First extension: to functions with a finite number of finite jump discontinuities. The
same notation, viz. Z b
f (x) dx
a
is used; but it now means the sum of integrals over the various (disjoint) subintervals
where the function is continuous. Remember that continuity over a closed interval [a, b]
means
• continuity at every point x such that a < x < b,
• continuity from the right at x = a; and
• continuity from the left at x = b. (When a = b the integral is defined to be 0, with
no continuity restrictions.)
2. Type 1 Improper Integral with one limit of integration equal to +∞. We define
Z +∞ Z b
f (x) dx = lim f (x) dx ; and
a b→+∞ a
Z a Z a
f (x) dx = lim f (x) dx .
+∞ b→+∞ b

3. Type 1 Improper Integral with one limit of integration equal to −∞. We define
Z b Z b
f (x) dx = lim f (x) dx ; and
−∞ a→−∞ a
Z −∞ Z a
f (x) dx = lim f (x) dx .
b a→−∞ b

4. Type 1 Improper Integral with one limit of integration equal to −∞ and one limit
of integration equal to +∞. We define
Z +∞ Z a Z +∞
f (x) dx = f (x) dx + f (x) dx and,
−∞ −∞ a
Z −∞ Z −∞ Z a
f (x) dx = f (x) dx + f (x) dx ,
+∞ a +∞

where a is any convenient point chosen to break the line into two semi-infinite rays. In
practice one often chooses a = 0, but that is not required. It is not permitted to compute
the two limits together — each of them must exist as a finite limit, independent of the
other.
Information for Students in Lecture Section 002 of MATH 141 2009 01 2128

5. Type 2 Improper Integral where end-point a is a point of infinite discontinuity. Here


we “excise” the point a and define
Z b Z b
f (x) dx = lim+ f (x) dx .
a c→a c

6. Type 2 Improper Integral where end-point b is a point of infinite discontinuity. Here


we “excise” the point b and define
Z b Z c
f (x) dx = lim− f (x) dx .
a c→b a

7. Type 2 Improper Integral where a point c such that a < c < b is a point of infinite
discontinuity. Here we “excise” the point c and define
Z b Z c Z b
f (x) dx = f (x) dx + f (x) dx .
a a c

This means that the integral is the sum of two separate limits, where the “bad” point
has been excised at one end of each of the smaller intervals of integration. The two
limits must be computed separately, and both of them must exist (as finite limits) for the
integral to be said to converge. It is not permitted to compute the two limits at the same
time in a symmetric way.
With these extended definitions we can show that the familiar rules for definite integrals are
still operational, where they make sense. Because the definitions for improper integrals in-
volve limits, we use the terminology of convergence and divergence, the same terminology we
could use in connection with other limits, and which we will see again when we study infinite
sequences and series.

Comparison Test for Improper Integrals The definitions I have sketched concern limits of
the values of certain integrals. Where we are unable to evaluate certain integrals directly, we
can still justify a comparison theorem similar to that we saw in connection with finite integrals:
Theorem B.60 (Comparison “Test” for Improper Integrals) Suppose that f and g are con-
tinuous functions such that, on the interval a ≤ x ≤ ∞, 0 ≤ g(x) ≤ f (x). Then
Z∞ Z∞
f (x) dx converges ⇒ g(x) dx converges
a a
Z∞ Z∞
g(x) dx diverges ⇒ f (x) dx diverges
a a
Information for Students in Lecture Section 002 of MATH 141 2009 01 2129

Z e3
dx
Example B.61 Evaluate the definite integral √ . Show that the integral is a conver-
1 x ln x
gent, improper integral, and find its value.
Solution: The integrand is not defined at x = 1, since one factor in the denominator is expressed
in terms of ln x. For x > 1 the substitution u = ln x is valid. Hence
Z e3 Z e3
dx dx
√ = lim+ √
1 x ln x t→1 t x ln x
Z 3
1
= lim+ √ du
t→1 ln t u
h √ i3
= lim+ 2 u
t→1 ln t
h √ √ i √ h√ i √
= lim+ 2 3 − 2 ln t = 2 3 − 2 lim+ ln t = 2 3
t→1 t→1
√ √ √
since, as t → 1+ , ln t → 0, so ln t → 0 = 0, by the continuity of the function t from the
right at t = 1.

7.8 Exercises

[7, Exercise
Z ∞ 52, p. 516] “Use the Comparison Theorem to determine whether the integral
x
√ dx is convergent or divergent.”
1 1 + x6
Solution: At first glance, this integral suggests a substitution u = x2 . While that would
simplify its form, it would not enable us to integrate it immediately, and it is not nec-
essary, since we can prove the convergence without this step. A simpler attack is to
√ 1 1
observe that 1 + x6 > x6 , so, for positive x, 1 + x6 > x3 , √ < 3 . Hence we can
Z t " #t 1 + x6 x
x 1 1
consider the limit lim 3
dx = lim − = 1 − lim 1 = 1. The convergence of the
t→∞ 1 x t→∞ x 1 t→∞ t
larger, improper integral implies the convergence of the given one.

[1, Exercise 75, p. 517] “Show that


Z ∞ Z ∞
2 −x2 1 2
xe dx = e−x dx .”
0 2 0

Solution: This is an interesting question, because we cannot express an antiderivative


2
of e−x in terms of elementary functions. So, at first glance, one wonders how it will be
possible to work with these integrals. Before doing that, I must prove that the integrals
Information for Students in Lecture Section 002 of MATH 141 2009 01 2130

are convergent — otherwise we don’t have any right to include them as numbers in an
equation.
I note that

x ≥ 1 ⇒ −x2 ≤ −x (multiplying the inequality by a negative number)


2
⇒ e−x ≤ e−x (exponential function is increasing)
Z t Z t
−x2 1 1
⇒ for t ≥ 1 e dx ≤ e−x dx = − t
1 1 e e
1
Hence, as t → ∞, the integral on the right approaches , i.e., the improper integral
e
Z ∞
1
e−x dx = .
1 e
Z ∞
2
By the [1, Comparison Theorem, p. 514], the improper integral e−x dx is also con-
1
1
vergent, and is less than . But we were considering the integral from 0, not from 1!
e
The integral from 0 to 1 can be bounded in another way, since the reasoning given above
is valid only for x ≥ 1. For example,
2
−x2 < 0 ⇒ e−x < e0 = 1
Z 1 Z 1
−x2
⇒ e dx < 1 dx = 1
0 0

Hence Z Z Z
∞ 1 ∞
−x2 −x2 1 2
e dx = e dx + . e−x dx < 1 +
0 0 1 e
This is not the exact value of the integral; in fact, it can be shown that
Z ∞ √
−x2 π
e dx = ,
0 2
but you are not expected to know this fact, nor how to prove it.
2
Now to prove the desired equality. Let us apply integration by parts with dv = xe−x dx
2
and u = x, so v = − 12 e−x , and du = dx:
Z t #t Z
2 −x2 1 −x2 1 t −x2
x e dx = − xe + e dx
0 2 0 2 0
Z
t 1 t −x2
= + e dx.
2et2 2 0
Information for Students in Lecture Section 002 of MATH 141 2009 01 2131

By l’Hospital’s Rule,
t 1
lim 2
= lim = 0.
t→∞ e t t→∞ 2tet2

Hence
Z ∞ Z t
2 −x2 2
xe dx = lim x2 e−x dx
0 t→∞ 0
Z t
1 2
= lim e−x dx
2 t→∞ 0
Z
1 ∞ −x2
= e dx .
2 0
Information for Students in Lecture Section 002 of MATH 141 2009 01 2132

B.15 Supplementary Notes for the Lecture of February 18th, 2009


Release Date: Wednesday, February 18th, 2009

Textbook Chapter 8. FURTHER APPLICATIONS OF INTEGRATION.

B.15.1 §8.1 Arc Length


Just as with the earlier concepts of area, volume, and average, we are faced first with adopting
a definition that appears to have the properties that we associate with the concept, and, at the
same time, is workable in practice. The length of an arc will be defined to be the limit — if
there is a limit — of the sum of the lengths of the sides of an approximating polygon formed
by choosing points closer and closer together on the curve, and joining them by line segments.
Note that we haven’t even defined what we mean in general by a curve, so the definition we
give will apply at first only to the graph of a function.
Suppose that we wish to find the length of the arc of the graph of y = f (x) between the
points (a, f (a)) and (b, f (b)). We can subdivide the interval [a, b] on the x-axis by intermediate
vertices, so that we have a sequence a = x0 , x1 , x2 , . . . , xn = b of points on the x-axis. If we
define
∆xi−1 = xi − xi−1 , and
∆ f (xi−1 ) = ∆yi−1 = yi − yi−1 = f (xi ) − f (xi−1 )
then the distance between successive points Pi−1 = (xi−1 , f (xi−1 )) and Pi = (xi , f (xi )) is
p p
|Pi−1 Pi | = (xi − xi−1 )2 + (yi − yi−1 )2 = (∆xi )2 + (∆yi )2 ,

which square root can be expressed as either of the following:


s s
!2 !2
∆ f (xi ) ∆xi
1+ · ∆xi = 1 + · ∆yi .
∆xi ∆yi
Note that the orientation of the increments in x and y is not relevant, as the increments appear
in these formulæ only as magnitudes (or squares). When we pass to the limit, as the “mesh” of
points selected on the x-axis become closer and closer together, the first of these expressions,
gives rise in the limit to the following integral representing the length of the arc:
Z bp
1 + ( f 0 (x))2 dx.
a

If the curve is given by an equation in the form x = g(y), then we find the arc length from the
point (g(k), k) to (g(`), `) to be Z `p
1 + (g0 (y))2 dy.
k
Information for Students in Lecture Section 002 of MATH 141 2009 01 2133

When the function whose graph is y = f (x) is invertible, both formulæ are applicable, and they
give the same length.45
Evaluation of these integrals can often require an approximation method, as the integrands
tend to be of types for which a function expressible in terms of elementary functions is un-
available. For that reason the problems that one meets in calculus books are often confined to
a small set of functions for which antiderivatives can be found.

The Arc Length Function. If we fix a point on a curve, we can then define a function
that expresses distance along the curve from the fixed point. This distance is expressed as an
integral with a variable upper limit, and is signed, so that, in effect, we have parameterized the
curve with a variable — usually denoted by the symbol s — uniquely denoting the position of
a point on a path along the curve. This practice differs from that employed when we evaluate
the length of the arc between two points, where only the magnitude is of interest. I have written
“...on a path along the curve” rather than “...on the curve”, because we shall be generalizing
these ideas in [1, Chapter 10], to consider curves that are not the graphs of functions; in those
generalizations a curve may cross itself, and the same point could be traversed more than once
by a point whose path we are studying: in that case it is the length of the path that will be
given by the arc length function.

Example B.62 Circumference of a circle. What is the circumference of the circle x2 +y2 = R2
(where R > 0)?
Solution: Since the equation given is not the graph of a function (because the curve crosses
some vertical lines more than once), let’s find the length of the upper arc
r from x = −R to
√  x 2
x = +R, and double it. This is given by the function f (x) = R2 − x2 = R 1 − .
R
x
0 R
y = −q  2
1 − Rx
2 1
1 + y0 =  2
1 − Rx
Z R
1
Circumference = 2 q  2 dx
−R
1 − Rx
Z R
1
= 4 q  2 dx
0
1 − Rx
since the integrand is even, and the interval symmetric around 0
45
Passage between the two forms can be seen to result from the change of variable given by y = f (x).
Information for Students in Lecture Section 002 of MATH 141 2009 01 2134

Z t
1
= 4 lim− q  2 dx
t→R 0 x
1− R
since the integral has a Type 2 impropriety at x = R
Z Rt
1
= 4R lim− √ du
t→R 0 1 − u2
x
under the substitution u =
Z a R
1
= 4R lim− √ du
a→1 0 1 − u2
t
under the substitution a =
R
Z arcsin a
cos v
= 4R lim− dv
a→1 arcsin 0 | cos v|
under the substitution v = arcsin u
Z arcsin a
= 4R lim− 1 dv
a→1 0
since cos v is non-negative for − π2 ≤ v ≤ π
2
= 4R lim− [v]arcsin
0
a
a→1
= 4R arcsin 1 by continuity of arcsin
π
= 4R · = 2πR .
2
Of course, we didn’t need to apply this last substitution v = arcsin u because we know two
1
antiderivatives of √ :
1 − u2
Z Rt
1 t
4R lim− √ du = 4R lim− [arcsin u]0R
t→R 0 1 − u2 t→R
 t 
= 4R lim− arcsin − 0
t→R R
= 4R [arcsin 1 − 0] by continuity of arcsin
π 
= 4R − 0 = 2πR .
2
x2 ln x
Example B.63 ([7, Exercise 8, p. 552]) Find the length of the arc y = − , (2 ≤ x ≤ 4).
2 4
Solution: Note the way in which the information is presented: we need a description of the
underlying curve, here given by an equation, and specifications of the portion of the curve
whose length is to be determined, here given by an interval 2 ≤ x ≤ 4 or in the alternative
Information for Students in Lecture Section 002 of MATH 141 2009 01 2135

notation x ∈ [2, 4]. Only the absolute value of the length is of interest, so we need not be
careful about which of the end-points 2, 4 is in which limit of the integral; alternatively, it is
the absolute value of the integral that we seek.

x2 ln x dy 1
y= − ⇒ = x−
2 4 dx 4x s
!
p 1 1
⇒ 0 2
1 + (y ) = 1 + x − + 2
2 16x2
r
p 1 1 1
⇒ 0 2
1 + (y ) = x + + 2 = x +
2 16x2 4x
!
1 1
Hence the arc length between x = 2 and x = 4 (where the function x + is equal to x +
4x 4x
is
Z 4 Z 4 !
x + 1 dx = x+
1
dx
4x 4x
2 2
" 2 #4
x ln x
= +
2 4 2
2 ln 2 ln 2 ln 2 √4
= 8+ −2− =6+ = 6 + ln 2 .
4 4 4

Example B.64 ([7, Exercise 12, p. 552]) Find the length of the curve y = ln x for 1 ≤ x ≤ 3.
Solution: Let’s attack this problem by integrating along either axis.

Integrating along the x-axis:


dy 1
y = ln x ⇒ =
dx
s x
!2 r
dy 1
⇒ 1+ = 1+
dx x2

Hence Z √ r
3
1
arc length = 1+ dx .
1 x2

To complete the integration, one approach is to try the substitution u = x2 + 1 to
simplify the integral. Then

x dx = u du
Information for Students in Lecture Section 002 of MATH 141 2009 01 2136

Z √
3
r Z 2
1 u2
1 + 2 dx = √ 2
du
1 x 2 u −1
Z 2 1 1 
 
= √ 1 +
2
− 2  du
2 u−1 u+1
 r 2
 u − 1 
= u + ln 
u + 1  √2
s√
√ 1 2−1
= 2 − 2 + ln √ − ln √
3 2+1
√ ln 3 1
= 2− 2− − ln √
2 2+1
√ ln 3 √
= 2− 2− + ln( 2 + 1) , etc.
2

Integrating along the y-axis: Here the curve can be rewritten as x = ey , whose length is to
be found for 0 ≤ y ≤ ln23 .
Z ln 3
2 √
arc length = 1 + e2y dy
0
Z 2
v2
= √ dv
2 v2 − 1

under substitution v = 1 + e2y
Z 2 1 1 
 
= 
√ 1 +
2
− 2  dv
2 v−1 v+1
" #2
1 v − 1
= v + ln
2 v + 1 √2

√ ln 3 2−1
= 2− 2− − ln √
2 2+1
√ √ 
√ ln 3  2 − 1 2 − 1 
= 2− 2− − ln  √ · √ 
2 2+1 2−1
√ ln 3 √ 2
= 2− 2− − ln 2 − 1
√ 2 √ √ 
= 2 − 2 − ln 3 3 − 2 6 .

Example B.65 ([7, Exercise 14, p. 552]) Find the length of the curve y2 = 4x, 0 ≤ y ≤ 2.
Information for Students in Lecture Section 002 of MATH 141 2009 01 2137

y2 dx
Solution: Since x = ,
4 dy
= 2y , and the arc length is the integral
Z r Z π
2
y2 4
1 + dy = sec v · 2 sec2 v dv
0 4 0
under the substitution y = 2 tan v, i.e., v = arctan 2y
π
= [tan v · sec v + ln | sec v + tan v|]04
√ √
= 2 + ln 2 + 1 .
R
(I have applied the reduction formula for sec3 v dv determined in [1, p. 458, Exercise 50,
§7.1].)

8.1 Exercises
π
[1, Exercise 12, p. 530] Find the length of the curve y = ln(cos x) for 0 ≤ x ≤ .
3
q
Solution: y0 = − tan x ⇒ 1 + (y0 )2 = | sec x|.
Z π
3
Arc length = | sec x| dx
0
Z π
3
= sec x dx
0
π
= [ln | sec x + tan x|]03
√ √
= ln |2 + 3| − ln |1 + 0| = ln(2 + 3) .

B.15.2 §8.2 Area of a Surface of Revolution


I develop a formula for the area of a surface of revolution by observing that an element of arc
of length ∆s will, when rotated about an axis whose distance from the element is r, generate
an element of surface whose area is approximately 2πr · ∆s. Remembering that
s s
!2 !2
∆ f (x) ∆x
∆s = 1 + · ∆x = 1 + · ∆y .
∆x ∆y

we can integrate with respect to either x or y as the conditions of the problem demand (provided
the derivative exists).
Information for Students in Lecture Section 002 of MATH 141 2009 01 2138

8.2 Exercises

[1, Exercise 12, p. 537] Find the area of the surface obtained by rotating the curve x = 1 + 2y2
(1 ≤ y ≤ 2) about the x-axis.
Solution:

dx
x = 1 + 2y2 ⇒ = 4y
dy
s
!2
dx p
⇒ 1+ = 1 + (4y)2
dy
Z 2 p
⇒ Arc length = 2π y 1 + (4y)2 dy
1
π   3 2
= 1 + 16y2 2
24 1
π  √ √ 
= · 65 65 − 17 17
24

[1, Exercise
√ 22, p. 537] ...Find the area of the surface obtained by rotating the curve y =
2
x + 1 (0 ≤ x ≤ 3) about the x-axis.
Solution:

y= x2 + 1
x
⇒ y0 = √
x2 + 1
q √
1 + 2x2
⇒ 1 + (y0 )2 = √
1 + x2
Z 3 √ √
1 + 2x2
⇒ Area of revolution = 2π 1 + x2 · √ dx
0 1 + x2
Z 3√
= 2π 1 + 2x2 dx
0
Z √
arctan 3 2

= √ sec3 θ dθ
2 0

under the substitution θ = arctan(x 2)
Z arctan 3 √2 √
π π
= √ sec θ dθ + √ [tan θ · sec θ]arctan
0
3 2
2 0 2
by the reduction formula [1, Exercise 50, p. 458]
Information for Students in Lecture Section 002 of MATH 141 2009 01 2139

π √
√ [tan θ · sec θ + ln | sec θ + tan θ|]arctan
= 0
3 2

"2 #
√ 1 √ √
= π 3 19 + √ ln( 19 + 3 2) .
2
Note that the textbook suggested the use of either a table of integrals or a computer
algebra system, but that neither was needed, as the solution of this problem is well
within the abilities of a student in this course (if she has time to do the calculations).
[1, Exercise 24, p. 537] Find the area of the surface obtained by rotating the curve y = ln(x+1)
(0 ≤ x ≤ 1) about the y-axis.
Solution:
dx
y = ln(x + 1) ⇒ dy =
x+1 s
q
1
⇒ 1 + (y0 )2 = 1+
(x + 1)2
Z s
1
1
Area of surface = 2π x dx1+
0 (x + 1)2
Z 2 r
1
= 2π (u − 1) 1 + 2 du
1 u
under the substitution u = x +√1
Z 2√ Z 2
2
1 + u2
= 2π 1 + u du − 2π du
1 1 u
Z 2 √ Z arctan 2
2
1 + u du = sec3 θ dθ
π
1 4

under the substitution θ = arctan u


1
= [tan θ · sec θ + ln | sec θ + tan θ|]arctan
π
2
2 4

by the reduction formula [1, Exercise 50, p. 458]


1 h √ √  √ √ i
= 2 5 + ln( 5 + 2) − 2 + ln( 2 + 1)
√ 2
Z 2 2
Z arctan 2
1+u
du = (sec θ · tan θ + csc θ) dθ
1 u π
4

under the substitution θ = arctan u


= [sec θ + ln | csc θ − cot θ|]arctan π
2
4
 √  
 √  5 − 1   √ √ 
=  5 + ln   − 2 + ln( 2 − 1) 
2
Information for Students in Lecture Section 002 of MATH 141 2009 01 2140

etc.

Here again the integral does not require special software or tables, just patience.

B.15.3 §8.3 Applications to Physics and Engineering (OMIT)


Omit this section. (But read it if you are majoring in Physics or Engineering!)

B.15.4 §8.4 Applications to Economics and Biology (OMIT)


Omit this section. (But read it if you are majoring into economics or biology.)

B.15.5 §8.5 Probability (OMIT)


Omit this section.

Textbook Chapter 9. DIFFERENTIAL EQUATIONS. (OMIT)


No parts of this chapter are included in the syllabus.
Information for Students in Lecture Section 002 of MATH 141 2009 01 2141

B.16 Supplementary Notes for the Lecture of March 02nd, 2009


Release Date: Tuesday, March 03rd, 2009

Textbook Chapter 10. PARAMETRIC EQUATIONS AND POLAR


COORDINATES.

B.16.1 §10.1 Curves Defined by Parametric Equations


A parameter is just a variable. When we call a variable by this term we usually are thinking
of a function or set of functions involving the variable as representing a family of objects. In
this first contact, the family will be a set of points. We will be taking the variable to be a
real variable, and so it is natural to consider not only the family of points on the graphs of a
function, but also the way in which the points are generated by the assignment of real numbers
to the parameter as representing a point moving along the curve. We can name the parameter
with any available symbol. Often we use the letter t; and a common use for this representation
is to treat t as time, so that the curve can be thought of as the trajectory of (i.e., the path
traced out by) a moving point. If we adopt this point of view, and if the parameter values are
chosen from an interval a ≤ t ≤ b, then we can speak of the curve (x(t), y(t)), and can think
of (x(a), y(a)) and (x(b), y(b)) as, respectively, the initial and terminal points. Curves given
parametrically in this way need not be graphs of functions: a curve may cross vertical lines
more than once, and may even cross itself, possibly more than once.

Graphing Devices This subsection may be omitted, as, in this course, we shall not be con-
cerned with the use of graphing devices.

Can the graph of a function, given non-parametrically, be expressed in parametric form?


The curve y = f (x) can be expressed in parametric form as, for example, x = t, y = f (t). But
there are infinitely other ways of expressing it parametrically, for example

x = t3 , y = f (t3 ),

but not necessarily


x = t2 , y = f (t2 ),
since the latter would include only the points with non-negative abscissæ.

Parametric vs. nonparametric representation of a curve When we represent a curve in


parametric form, the parametrization sometimes contains information beyond what is available
Information for Students in Lecture Section 002 of MATH 141 2009 01 2142

in a non-parametric representation. Often we can see a dynamic way of actually tracing out
the curve by allowing the parameter to range through its domain. So, for example, the curve

x = cos t
y = sin t

can be thought of as being traced out by a point that moves around the unit circle centred at
the origin, starting at the point (1, 0) at time t = 0, counterclockwise at a rate of 1 radian per
unit time as t increases (and clockwise at a rate of 1 radian per unit time as t decreases). If we
compare the curve with

x = cos 2t
y = sin 2t

we see that both trace out the same circle, covering the curve infinitely often, but the second
curve moves twice as fast. If we wanted to eliminate the multiple covering, we could include
the inequalities 0 ≤ t < 0 in the first case, or 0 ≤ t < π2 in the second.
The standard way to transform from parametric to non-parametric equations is to eliminate
the parameter “between” the equations, which can be interpreted as solving one equation for
the parameter, and substituting that value into the second equation. However, this operation
sometimes “loses information”.

“Cartesian” representation of curves Where the textbook speaks of a Cartesian represen-


tation, I prefer to speak of a non-parametric representation. All the representations for curves
considered in these two sections are Cartesian, since they all refer to the system of represen-
tation associated with René Descartes. In the following two sections we will be considering
curves represented in another way — in the so called polar representation.

Example B.66 [7, Exercise 13, p. 656] asks you to find a Cartesian equation of the curve
x = cos2 θ, y = sin2 θ, − π2 < θ < π2 . We can eliminate θ by solving one equation for θ and
substituting into the second; or, more elegantly, by adding the two equations together, obtaining
an equation that does not involve θ:
x+y=1
But this equation does not convey all of the information we started with. One type of infor-
mation that has been lost is the fact that both x and y, being squares, are non-negative. Thus
it is not the whole line x + y = 1 that is equivalent, but only the line segment joining the
points (0, 1) and (1, 0). This feature is essential, since the doubly-infinite line is not the curve
represented by the parametric equations. Another type of information that is contained in the
parametrization is the way in which this line segment is traversed. The point is moving back
Information for Students in Lecture Section 002 of MATH 141 2009 01 2143

and forth between the point (1, 0) and the point (0, 1), covering the open segment in a parame-
ter interval of length π4 . Distance is not covered at a constant speed — the point moves fastest
in the middle of the segment; but, so far, we are not interested in how fast the point is moving.
There is more than one correct way to describe this curve, but giving the equation x + y = 1
is not enough.

Example B.67 [7, Exercise 17, p. 656]


(a) Eliminate the parameter to find a Cartesian equation of the curve.

(b) Sketch the curve and indicate with an arrow the direction in which the curve is traced as
the parameter increases:

x = cosh t y = sinh t
For every value of t a point with these coordinates can be seen to lie on the curve x2 − y2 = 1,
which is a hyperbola with two branches, one opening to the right, and passing through the point
(1, 0), the other opening to the left, and passing through the point (−1, 0); the curves are both
asymptotic to the lines y = ±x. But, since the hyperbolic cosine is positive, the parametrization
applies only to the right branch of the hyperbola: the curve comes in from −∞ from below,
passes through (1, 0) at t = 0, and the moves off along the upper half as t → +∞. So one way
to describe the curve non-parametrically is

x2 − y2 = 1
x ≥ 0.

This curve may be parameterized in other ways. For example, we could represent it as

x = sec t
y = tan t

(cf. [7, Exercise 14, p. 656]) but this time we need to restrict the values the parameter may
take, for example by
π π
− <θ< .
2 2

The Cycloid In this subsection the textbook describes the construction of this interesting
curve. You are not expected to remember specific properties of this curve, nor its history.
However, you should be able to work with this curve if the parametric equations are given
to you, in the same way as you would be expected to be able to work with any reasonable
curve given parametrically. The particular parametrization developed in the textbook is x =
r(θ − sin θ), y = r(1 − cos θ).)
Information for Students in Lecture Section 002 of MATH 141 2009 01 2144

Families of Parametric Curves Here the author considers a family of curves given para-
metrically, in which the various members of the family all have a similar equation obtained
by assigning different values to a variable in the parametric equations. That variable is also
called a parameter, and the family of curves could be called a parametric family of curves
represented by parametric equations. Of course, the parameter that represents the family of
curves is not the same as the parameter that represents the family of points on any specific
curve. For example, x2 + y2 = a2 could be considered a parametric family of curves (circles)
given in non-parametric form; we could represent the same family in parametric form by

x = a cos t
y = a sin t

where the parameter t represents position on a specific curve, and the parameter a represents
the different curves in the family. We could also interpret the equation by switching the roles
of the parameters: the curve
x = a cos t y = a sin t (74)
with t constant and a variable represents the points on the line through the origin inclined to
the positive x-axis by an angle of t radians. This point of view will become important in [1,
§10.3].
Here again, you are expected to be able to work with reasonable families of parametric
curves, but not to know specific properties of those families (with the exception of the obvious
parametric equations for familiar curves, like the circle).
Example B.68 Equation of a line in the plane. The line in the plane through the point
(x0 , y0 ) and with slope m has non-parametric equation y = y0 + m(x − x0 ). It can be represented
parametrically in infinitely many ways. If we choose to relate the parameter to distance along
the line, one can show that the following equations represent the line:

x = x0 + t
y = y0 + mt
m
Check that the line segment joining any two points on this line has slope = m. (Take two
1
points with coordinates (x0 + a, y0 + ma) and (x0 + b, y0 + mb).)

B.16.2 §10.2 Calculus with Parametric Curves


Tangents If a curve is given parametrically by (x, y) = (x(t), y(t)), then (subject to certain
conditions) we can differentiate y with respect to x by passing through an intermediate variable
t. Recalling that
dx dt dx
· = = 1,
dt dx dx
Information for Students in Lecture Section 002 of MATH 141 2009 01 2145

we have
dy dy dt
= ·
dx dt dx
dy 1
= ·
dt dx
dt
dy
so we can determine the first derivative in terms of the derivatives of the functions f and
dx
d2 y
g. This calculation can be extended to the second derivative 2 , although the expressions are
dx
not as pretty:
!
d2 y d dy
=
dx2 dx dx
!
d dy dt
= ·
dt dx dx
 dy 
 
d  dt  1
=  ·
dt  dx  dx
 dt dt 
 d y dx d2 x dy 
2
 2 · − · 
 dt dt dt2 dt  1
=  !2  · dx
 dx 
  dt
dt
d2 y dx d2 x dy
2
· − 2 ·
= dt dt dt dt
!3
dx
dt
Rather than substituting in this formula, memorized, you are advised to be able to carry out
this computation for a specific parameterized curve. It enables us to study the concavity of
a parameterized curve, and to apply the 2nd derivative test if necessary in an optimization
problem.
Example B.69 (cf. [1, Exercise 6, p. 636]) (see Figure 7 on page 2146) Let’s investigate the
curve given parametrically by

x = cos θ + sin 2θ (75)


y = sin θ + cos 2θ (76)
Information for Students in Lecture Section 002 of MATH 141 2009 01 2146

0.5

-1.5 -1 -0.5 0 0.5 1 1.5


0

-0.5

-1

-1.5

-2

Figure 7: The curve x = cos θ + sin 2θ, y = sin θ + cos 2θ

Differentiating the parameterizing functions, we obtain


dx
= − sin θ + 2 cos 2θ

= −4 sin2 θ − sin θ + 2 !
1 1
= −4 sin2 θ + sin θ −
4 2
 !2 
 1 33 
= −4  sin θ + − 
8 64
Information for Students in Lecture Section 002 of MATH 141 2009 01 2147

 √  √ 
 1 + 33   1 − 33 
= −4 sin θ +  sin θ + 
8 8
dy
= cos θ − 2 sin 2θ
dθ !
1
= −4 cos θ sin θ −
4

The textbook asks us to find an equation for the tangent


! to the curve at the point with parameter
dx dy
value θ = 0. We find that, when θ = 0, , = (2, 1), so the slope of the tangent at the
dθ dθ
point (x(0), y(0)) = (1, 1) is 12 , and an equation for the tangent is

1
y = 1 + (x − 1)
2
or x − 2y + 1 = 0.
We can now use the same curve to illustrate some of the other theory of this section. For
dy
example, we can determine where the curve is horizontal, by solving the equation = 0,
dx
dy
which implies that = 0. We find that this happens when

1
cos θ = 0 or sin θ = ,
4
which we could proceed to solve. We can also determine where the curve is vertical, by
dx
determining where = 0; this happens when


−1 ± 33
sin θ =
8
All of these equations could be solved. Since the curve is expressed entirely in terms of
sin θ, cos θ, sin 2θ, cos 2θ, and these functions are periodic, repeating themselves after θ passes
through an interval of length 2π, we can see the whole curve by confining θ to the interval
π 3π 1
0 ≤ θ ≤ 2π. In that interval the cosine vanishes when θ = , , and sin θ = when
2 2 4
1 1
θ = arcsin and π − arcsin . These are the 4 points where the curve is horizontal; one of
4 4
those point is the origin, because the curve passes through the origin several times, one of those
times with a horizontal tangent. The curve has the shape of a “3-leafed rose” or a “trefoil” (3-
leafed clover) centred at the origin, with one petal centred on the y-axis. We will see other
curves with this shape when we study polar coordinates in [1, §10.3].
Information for Students in Lecture Section 002 of MATH 141 2009 01 2148

For the same curve given parametrically by equations (75), (76), let’s consider the follow-
ing question, similar to [1, Exercise 25, p. 636]: “Show that the curve..has (several)...tangents
at (x, y) = (0, 0), and find their equations.”
Solution: If we set the coordinates equal to zero and solve, we can simplify the resulting
equations, to obtain:
cos θ · (1 + 2 sin θ) = 0
(sin θ − 1) · (1 + 2 sin θ) = 0 .
The curve passes through the origin whenever both of the equations are satisfied. This means
that either
1
sin θ = − ,
2
46
or both of the following equations must hold:
cos θ = 0
sin θ = 1 .
π
For θ between 0 and 2π, these last equations are satisfied when θ = ; the first equation is
2
7π 11π
satisfied when θ = , . The curve, which has the shape of a 3-leafed clover or a “3-leafed
6 6
rose”, passes through the origin 3 times, and we can find the slopes of the tangents in the usual
dy
way, by taking the ratio dθ .
dx

Areas. Consider the arc of the curve x = f (t), y = g(t) determined by points with parameter
value t between α ≤ t ≤ β. Since
Z β Z f (β)
0
g(t) · f (t) dt = y · dx ,
α f (α)

the first integral represents the area between the arc and the x-axis. Remember, though, that
the curve need no longer be the graph of a function, so it could cross vertical lines more than
once. This means that the area could be “folded”, and there might be portions that could be
counted negatively and canceling the portions that you are interested in. For that reason you
should use this integral only where you are clear about the shape of the region whose area you
are finding, and there might be situations where the region should be broken up into parts and
the areas of the parts found separately.
46
Note that the second equation implies the first, but the first does not imply the second; we could thus have
shown only the second equation.
Information for Students in Lecture Section 002 of MATH 141 2009 01 2149

Arc Length. In a similar way to the preceding, we can argue that the length of the arc
α ≤ t ≤ β of the curve x = u(t), y = v(t) between α ≤ t ≤ β is given by the integral
s
Z β !2 !2
dx dy
+ dt .
α dt dt

Note that, when we consider the parametrization

x=t y = f (t) (a ≤ x ≤ b)

of the graph of the function y = f (x) between the points (a, f (a)) and (b, f (b)), this reduces
to the formula we derived earlier. Here again, be careful that you are finding the length of the
curve that you intend. In this case there cannot be any cancellation, since the integrand is a
square root, which cannot be negative. If you obtain a negative length, it could simply be a
consequence of the direction in which you have parameterized the curve, which is harmless;
or of an error you have made in you calculations, which is serious.

10.2 Exercises

[1, Exercise 54, p. 637] “Find the total length of the astroid x = a cos3 t, y = a sin3 t.”
Solution: This curve is generated over an interval of length 2π: we can take 0 ≤ t ≤ 2π.
the curve looks like a deformed circle, that has been pinched towards the centre at the
points away from where it crosses the coordinate axes.
s
Z 2π !2 !2
dx dy
Total length = + dt
0 dt dt
Z 2π r
  2
= −3a cos2 · sin t 2 + 3a sin2 t · cos t dt
0
Z 2π
= 3|a| · | cos t · sin t| dt
0

By symmetry we can find the length of one quarter of the curve:


Z π
2
Total length = 4 3|a| · | cos t · sin t| dt
0
Z π
2
= 4|a| 3 cos t · sin t dt
0
h iπ
= 6|a| sin2 t 2 = 6|a| .
0
Information for Students in Lecture Section 002 of MATH 141 2009 01 2150

[1, Exercise 48, p. 637] Find the length of the loop of the curve x = 3t − t3 , y = 3t2 .
Solution: Where does this loop begin and end? We need to find all parameter values
t1 , t2 where t1 , t2 but

3t1 − t13 = 3t2 − t23


3t12 = 3t22

The equations can be simplified to

(t1 − t2 )(3 − t12 − t1 t2 − t22 ) = 0


(t1 − t2 )(t1 + t2 ) = 0

When we know that t1 , t2 , the equations further simplify to the system

3 − t12 − t1 t2 − t22 = 0
t1 + t2 = 0

which imply that t1 = −t√2 = ± 3. Thus these two curves intersect in the points that
have parameter values ± 3.
Since
s
!2 !2 q
dx dy 
+ = 3 − 3t2 2 + (6t)2
dt dt
= 3(1 + t2 ) ,

the length of the arc of the loop is


Z √
3 h i √3 √
2 3
√ 3(1 + t ) dt = 2 3t + t = 12 3 ,
0
− 3

since the integrand is an odd function, and the limits of integration are symmetrically
located around t = 0.
Information for Students in Lecture Section 002 of MATH 141 2009 01 2151

B.17 Supplementary Notes for the Lecture of March 04th, 2009


Release Date: Wednesday, March 04th, 2009

B.17.1 §10.2 Calculus with Parametric Curves (conclusion)


Surface Area. We can also adapt, in the obvious ways, our previous formulæ for area of
surfaces of revolution.

Example B.70 Let’s determine a formula for the surface area of a doughnut. Suppose that the
doughnut is generated by the curve

x = R + r cos θ
y = 0 + r sin θ

where 0 ≤ θ ≤ 2π, where this circle, centred at the point (x, y) = (R, 0), is revolved around the
y-axis. Assume R ≥ r.
s
Z 2π !2 !2
dx dy
Area of revolution = 2π(R + r cos θ) + dθ
0 dθ dθ
Z 2π √
= 2π(R + r cos θ) r2 dθ
0
= 2πr [Rθ + r sin θ]2π 2
0 = 2πr[R · 2π − R · 0] = 4π rR.

10.2 Exercises (conclusion)

[1, Exercise 34, p. 637] “Find the area of the region enclosed by the astroid x = a cos3 θ,
y = a sin3 θ.” This is the same curve considered above in [1, Exercise 54, p. 637]. Let’s
consider the arch in the first quadrant. The area will be 4 times the area in the first
quadrant, which is
Z a Z π Z π
2 dx(θ) 2  
y dx = y(θ) · dθ = a sin θ · −3a cos θ · (− sin θ) .
3 2
0 0 dθ 0

You know one way to evaluate an integral of this type — by replacing sin2 θ and cos2 θ
1 − cos 2θ 1 + cos 2θ
respectively by and . A variant of that method is as follows:
2 2
Z π Z π2
2  

3 2
a sin θ · −3a cos θ · (− sin θ) = 3a 2
a sin4 θ · cos2 θ
0 0
Information for Students in Lecture Section 002 of MATH 141 2009 01 2152

Z π !2
2 1 − cos 2θ sin 2θ
2
= 3a ·
0 2 2
2 Z π2  
3a
= sin2 2θ − sin2 2θ · cos 2θ dθ
8 0
Z π2 !
3a2 1 − cos 4θ 2
= − sin 2θ · cos 2θ dθ
8 0 2
" #π
3a2 θ sin 4θ sin3 2θ 2
= − −
8 2 8 6 0
2
3πa
= ,
32
3πa2
so the area of the interior of the astroid is .
8

Laboratory Project: Bézier Curves Omit this subsection.

Volumes. The textbook appears to say little about this, but here again the earlier formulæ can
be adapted, to determine, for example, the volume of revolution generated by a given curve
about a given axis.

B.17.2 §10.3 Polar Coordinates


Polar Curves. In the polar coordinate system we locate points in the plane by taking a special
point, called the pole, a special half-line or ray which emanates from the pole, called the polar
axis and a direction for measuring positive angles — usually taken to be the counter-clockwise
direction. Any point can be located if we know its distance from the pole, usually denoted by
r, and the angle that the line joining them makes with the polar axis in the positive direction,
usually denoted by θ. However, the angle is not unique, since angles of θ and θ + 2nπ will
give the same point for any integer n. So here we have one of the essential differences between
polar and cartesian coordinates:

Theorem B.71 The polar coordinates of a point are never unique.

In the case of the pole itself, the angle θ is totally undetermined: once we know that r = 0, any
angle θ will give the same point.

Convention permitting negative r. It is convenient to broaden the multiplicity of coordi-


nates by permitting the distance from the pole to be negative. We do this by agreeing that (r, θ)
and (−r, θ + π) represent the same point. This convention permits for continuous representation
Information for Students in Lecture Section 002 of MATH 141 2009 01 2153

of certain curves, but causes complications at various stages: occasionally additional care is
required.

Relations between polar and cartesian coordinates. In theory we can set up polar and
cartesian systems independently in the plane, placing the pole at any convenient place. In
practice we often place the pole at the origin of a cartesian system, with the polar axis along the
positive x-axis. When the author of the textbook suggests that you are to consider two systems
at the same time, and gives no other information, this is what he expects you to do.47 When
the polar and cartesian systems are placed in this “standard” way, the following relationships
hold:

x = r · cos θ
y = r · sin θ
x + y2
2
= r2
y
= tan θ
x
Note that, while it is possible to transform from polar to cartesian coordinates without ambi-
guity, it is not always possible to move painlessly in the other direction; this is because of the
non-uniqueness of polar coordinates, about which much more will be said. If you are given an
equation in cartesian coordinates, you can transform it to polar by substituting the appropriate
formulæ for x and y and simplifying — try [1, Problems 21-26, p. 648]; if you are given the
coordinates of a point in polar coordinates, you can transform to cartesian in the same way —
try [1, Exercises 3-4, p. 647].

Consequences of Non-uniqueness of Polar Coordinates. This is a difficult topic, and will


require considerable practice before you will become comfortable with it! Some of the prob-
lems you will have are related to understanding what is meant by an equation for a curve:
you need to understand that any curve can be represented in multiple ways, even when we use
cartesian coordinates. But, when the representation of the points themselves is not unique, the
results can be confusing.
We saw with parametric equations that the same point on a curve can appear more than once
on a “curve”. In that context there was a “natural” way of tracing out the curve, by following
increasing values of the parameter. When we come to study polar coordinates, the situation is
much more complicated, because there is no “natural” way of following the generation of the
curve, and no one set of coordinates for a point has preferential status with respect to others.
Polar curves can be expressed by any relationship between r and θ, although more often than
47
However, there is an important application, involving conic sections — ellipses, parabolæ, hyperbolæ, where
we place the origin at a different point; this topic is not on the syllabus of the present course, but you may read
about it in [1, §10.6].
Information for Students in Lecture Section 002 of MATH 141 2009 01 2154

not the equation will be in the form r = f (θ); however, you will see some equations in the form
π
θ = f (r) — for example, the equation θ = represents the line through the pole inclined at an
4
π
angle of to the polar axis.
4

Symmetry When we studied symmetry in the context of cartesian coordinates, we could


have considered [1, pp. 19-20, 308]
• reflective symmetry in a vertical line, particularly in the y-axis;
• reflective symmetry in a horizontal line, particularly in the x-axis;
• rotational symmetry about a point, particularly about an angle of π around the origin
(which is equivalent to reflective symmetry in both the x-axis and the y-axis); and
• periodicity of a graph under a horizontal shift.
These are not the only types of symmetries that a graph can possess, so the study in the textbook
is selective, and we haven’t investigated thoroughly what principles were involved.
Similarly, when we consider symmetry in the context of polar coordinates, we will not
attempt a thorough study, but will consider types of symmetries which the polar system is
particularly able to accommodate. Here are the types of symmetry that the textbook mentions:
• reflective symmetry in the polar axis — exhibited by the invariance of the equation of a
curve under the substitution θ 7→ −θ.48
• rotational symmetry under a rotation through an angle of π around the pole — exhibited
by the invariance of the equation under the transformation r 7→ −r or under θ 7→ θ + π
π
• reflective symmetry under reflection in the line θ = , exhibited by invariance under the
2
transformation θ 7→ π − θ.
Again, these are not the full range of symmetries that can occur in the plane. Because of
the non-uniqueness of coordinates, curves can have the symmetries listed without exhibiting
invariance under the transformations listed. For example, the curve θ = π is the line that
extends the polar axis. It certainly has symmetry in the polar axis, but the equation is not
unchanged when we replace θ by −θ. You are not expected to be an expert in the subject of
symmetries.

Graphing Polar Curves with Graphing Devices. Omit this section — this is a device-free
course.
48
This description is incomplete. See the discussion below on page 2164 of Example B.74 ([7, Exercise 37, p.
678]).
Information for Students in Lecture Section 002 of MATH 141 2009 01 2155

Where is the point with polar coordinates (r, θ)? The ambiguity in polar coordinates is not
in locating a point with given coordinates — it is only that all points possess multiple sets of
coordinates. If you are given the coordinates (r, θ), you may locate the point by:
• First, locating the ray obtained by turning the polar axis (the distinguished ray that em-
anates from the pole, relative to which we refer all coordinate angles) through an angle
of θ in the positive direction; and

• then

– if r ≥ 0, proceeding along that ray a distance of r; or


– if r ≤ 0, proceeding along the extension of the polar axis beyond the pole a distance
of −r.
 
Example B.72 ([7, Exercise 2, p. 677]) Plot the point whose polar coordinates are −1, − π2 .
Then find two other pairs of polar coordinates of this point, one with r > 0, and one with r < 0.
Solution: The second coordinate tells us that the point is on the line inclined to the polar axis
at an angle of − π2 radians measured in the positive direction: this takes us to the ray which is
obtained by turning the polar axis in the clockwise direction through a right angle. But then
the negative first polar coordinate tells us to proceed along the opposite ray for 1 unit. If the
polar system is superimposed on a cartesian system in the usual way, the point is the unit point
on the positive y-axis. This point has the following other sets of polar coordinates:
 
with positive r: 1, π2 + 2nπ
   
with negative r: −1, −π 2
+ 2mπ or −1, 3π
2
+ 2`π
where n, m, and ` are any integers.

Example B.73 ([7, Exercise 12, p. 677]) “Sketch the region in the plane consisting of points
whose polar coordinates satisfy the conditions −1 ≤ r ≤ 1, π4 ≤ θ ≤ 3π4
.”
Solution: Let’s separate the portion of the region described by positive r, zero r, and negative
r:
positive r. 0 < r ≤ 1, π4 ≤ θ ≤ 3π
4
describes a sector of the unit disk centred at the pole — all
points in the region bounded by two perpendicular radii bisecting the first and second
quadrants. The points on the radii and the bounding circle are included.
π 3π
zero r. The region described by r = 0, 4
≤θ≤ 4
consists of the pole alone: the θ coordinate
is irrelevant for the pole.

negative r. −1 ≤ r < 0, π4 ≤ θ ≤ 3π4


describes the region antipodal to the one described for
positive r — between radii at angles π + π4 and π + 3π
4
.
Information for Students in Lecture Section 002 of MATH 141 2009 01 2156

10.3 Exercises

[1, Exercise 14, p. 648] “Find a formula for the distance between the points with polar coor-
dinates (r1 , θ1 ) and (r2 , θ2 ).”
Solution: One way to solve this problem is to transform to cartesian coordinates, then
apply the usual distance formula, after which cancellations will occur. A more direct
way is to apply the Law of Cosines to a triangle whose vertices are the pole and the two
given points. That is easy if we assume that r1 and r2 are non-negative, so they represent
the lengths of the sides of the triangle. To solve the problem in this way we need to
consider several cases. The result is the same: the distance is
q
r12 + r22 − 2r1 r2 cos(θ1 − θ2 ) .

[1, Exercise 18, p. 648] Identify the curve r = 2 cos θ + 2 sin θ.


Solution: The textbook suggests finding a cartesian equation first. This is not always
easy, but is not impractical in the present problem. If we multiply the given equation by
r, we obtain the equation
r2 − 2r cos θ − 2r sin θ = 0
which we see is equivalent to

x2 + y2 − 2x − 2y = 0

i.e., to
(x − 1)2 + (y − 2)2 = 2

which is a circle with radius 2 centred at the point (1, 1). However, we must note that,
in multiplying an equation by a factor — equivalently in multiplying the two equations

r = 2 cos θ + 2 sin θ (77)


r = 0 (78)

we were, in the second equation, possibly permitting new points to be included in the
“curve”. We must carefully analyze whether the equation r = 0 did that. But we know
that r = 0 represents only the pole! And the pole was already on the given curve: it
appears there as !

(r, θ) = 0, .
4
So multiplying by r = 0 does not introduce any new points.49
49
But what would happen if you were to multiply an equation like r = 1 by r = 0?
Information for Students in Lecture Section 002 of MATH 141 2009 01 2157

[1, Exercise 30, p. 648] Describe the curve with the equation r2 − 3r + 2 = 0.
Solution: The equation can be rewritten as

(r − 1)(r − 2) = 0 ,

which is satisfied by all points whose r-coordinate is either 1 or 2, i.e., by all points on
either of two concentric circles around the pole.
[1, Exercise 33, p. 648] Describe the curve with the equation r = 2(1 − sin θ) (θ ≥ 0).
Solution: (see Figure 8 on page 2157) Under the transformation θ 7→ π − θ the equation

-2 -1 0 1 2
0

-1

-2

-3

-4

Figure 8: The cardioid with equation r = 2(1 − sin θ)

is unchanged, so the curve has symmetry in the y-axis. This is a “heart-shaped” curve,
Information for Students in Lecture Section 002 of MATH 141 2009 01 2158

called a cardioid. The textbook describes another example in [1, Example 9, pp. 645-
646], where the author shows, as we could show here, that the curve is tangent to the
y-axis. We call the point of tangency (here, the pole) a cusp. (The condition θ ≥ 0 does
not restrict the curve in any way, since the function 2(1 − sin θ) is periodic with period
2π: any part of the curve that might require a negative value θ0 of θ to represent it, can
also be represented by the θ0 + 2πn, where n is any integer; taking n sufficiently large
will make this value positive.)
[1, Exercise 34, p. 648] Describe the curve with equation r = 1 − 3 cos θ. (see Figure 9 on
page 2158)

-4 -3 -2 -1 0
0

-1

-2

Figure 9: The lima¸con r = 1 − 3 cos θ,

Solution: It is not practical to represent this curve in cartesian coordinates. We note


Information for Students in Lecture Section 002 of MATH 141 2009 01 2159

that, when θ is replaced by −θ, the equation is unchanged. This tells us that the curve is
symmetric about the polar axis. Try tracing it out by starting with the point with θ = 0. If
we superimpose a cartesian system in the usual way, that point is the point with cartesian
coordinates (−2, 0). The curve passes through the pole first when θ = arccos 13 , about
70 degrees. It crosses the y-axis at the point with cartesian coordinates (x, y) = (0, 1),
and then moves to its maximum distance of 1 + 3 = 4 from the pole when θ = π. Then
it returns, crossing the negative y-axis at (x, y) = (0, −1), and passing again through the
pole. The curve is one of the family called limaçon’s, see them sketched in [1, Example
11, p. 647].
Information for Students in Lecture Section 002 of MATH 141 2009 01 2160

B.18 Supplementary Notes for the Lecture of March 09th, 2009


Release Date: Monday, March 09th, 2009

B.18.1 §10.3 Polar Coordinates (continued)


Tangents to Polar Curves I discuss how to determine the tangents to curves of the form
r = f (θ). The discussion is based on the observation that, if we superimpose polar and cartesian
systems in the most usual way, we can express x and y in terms of θ by

x = f (θ) · cos θ
y = f (θ) · sin θ

and thereby interpret θ as a parameter in the parametric representation of a curve. We find that
dy dr
dy · sin θ + r · cos θ
= dθ = dθ . (79)
dx dx dr
· cos θ − r · sin θ
dθ dθ
In the case of the cardioid r = 2(1 − sin θ), we find that
dy 2 sin θ − 1
=
dx 4 cos θ
when cos θ , 0. When θ = 0, at the unit point on the initial ray, the tangent has slope − 14 . As θ
increases to π6 the tangent becomes vertical; then, as θ > π6 and θ → π2 − , the tangent approaches
the vertical, with positive slope. Similarly, as θ → π2 + , the tangent approaches the vertical, with
negative slope. We say that this curve has a cusp at the pole.

[1, Exercise 35, p. 648] Describe the curve with equation r = θ, with θ ≥ 0.
Solution: This curve is a spiral, turning around the pole. (see Figure 10 on page 2161)
If, however, we were to ask about the curve with equation r = θ, with θ ≤ 0, it is also a
spiral around the pole. (see Figure 11 on page 2162) The superposition of the two curves
is shown in Figure 12 on page 2163.
Information for Students in Lecture Section 002 of MATH 141 2009 01 2161

30

20

10

-30 -20 -10 0 10 20 30


0

-10

-20

-30

Figure 10: The spiral with equation r = θ, (θ ≥ 0)


Information for Students in Lecture Section 002 of MATH 141 2009 01 2162

30

20

10

-30 -20 -10 0 10 20 30


0

-10

-20

-30

Figure 11: The spiral with equation r = θ, (θ ≤ 0)


Information for Students in Lecture Section 002 of MATH 141 2009 01 2163

30

20

10

-30 -20 -10 0 10 20 30


0

-10

-20

-30

Figure 12: The full spiral with equation r = θ, −∞ < θ < +∞


Information for Students in Lecture Section 002 of MATH 141 2009 01 2164

Example B.74 ([7, Exercises 37, 40, p. 678]) Describe the curves with equations r = sin 2θ,
r = sin 5θ.
Solution: First let’s consider the curve r = sin 2θ. (see Figure 13 on page 2164) While the

0.6

0.4

0.2

0
-0.6 -0.4 -0.2 0 0.2 0.4 0.6

-0.2

-0.4

-0.6

Figure 13: The “4-leafed rose” with equation r = sin 2θ,

equation does not remain unchanged when we replace θ by −θ, it changes to r = − sin 2θ,
which contains the same points, since it can be rewritten as −r = sin 2(θ + π), which can be
obtained from the original equation by the transformation
(r, θ) 7→ (−r, θ + π) .
Thus this curve is symmetric about the polar axis. It is also symmetric about the y-axis, since
the replacement of θ by π2 − θ leaves the equation unchanged. The curve is a “4-leafed rose”,
where each petal is tangent to the cartesian axes, if located in the usual way.
Information for Students in Lecture Section 002 of MATH 141 2009 01 2165

But, when the multiplier of θ is an odd integer, the situation changes. (see Figure 14 on
page 2165) The curve r = sin 5θ is again a “rose”, passing through the pole every 36 degrees.

0.8

0.4

0
-0.8 -0.4 0 0.4 0.8

-0.4

-0.8

Figure 14: The “5-leafed rose” with equation r = sin 5θ,

It is not symmetric about the y-axis, nor about the x-axis. (It is symmetric under reflections in
other axes, and also under rotation through certain angles around the pole.)

Example B.75 ([7, Exercises 42, p. 678]) Sketch the curve with the polar equation r2 = sin 2θ.
Note that there are no points on this curve for π2 < θ < π2 since, in that interval, sin 2θ < 0. The
entire curve is traced out for 0 ≤ θ ≤ π2 : every value of θ gives rise to two points on the graph.
(see Figure 15 on page 2166)

Example B.76 Where is the curve r = sin 2θ horizontal?


Information for Students in Lecture Section 002 of MATH 141 2009 01 2166

0.8
0.3

0.2
0.4

0.1

0 0
-0.8 -0.4 0 0.4 0.8 -1 -0.5 0 0.5 1

-0.1

-0.4
-0.2

-0.3
-0.8

Figure 15: The lemniscates r2 = sin 2θ, r2 = cos 2θ

dy
Solution: We set = 0 in (79):
dx
dr
· sin θ + r · cos θ = 0

⇔ 2 cos 2θ · sin θ + sin 2θ · cos θ = 0
⇔ 2(1 − 2 sin2 θ) · sin θ + 2 sin θ · cos2 θ = 0
⇔ 2 sin θ(2 − 3 sin2 θ) = 0
so the tangents will be horizontal when
r
2
sin θ = 0, ± ,
3
i.e., when r r
2 2
θ = 0, ± arcsin , π ± arcsin .
3 3

B.18.2 §10.4 Areas and Lengths in Polar Coordinates


Areas To find the area bounded by a curve given in polar coordinates we express the area as
the limit of a sum of narrow triangles whose bases are along the bounding curve, and whose
upper vertex is at the pole. It was shown in the lecture that the area subtended by the arc of the
curve r = f (θ) between θ = a and θ = b is then
Z b
1
· ( f (θ))2 dθ .
a 2
Information for Students in Lecture Section 002 of MATH 141 2009 01 2167

(This could be shown in two different ways: by treating the element of area as a sector of a cir-
cle, or as a narrow, isosceles triangle.) As with all other formulæ involving polar coordinates,
one must use this formula with care. Be sure that you know precisely what the region looks
like; in case of doubt, break the region up into parts, and find the areas of the parts separately.
The limits must be chosen carefully, to be sure that, for example, you are not computing more
1
area than
 you intend. For example, the curve r = cos θ is a circle of radius 2 centred at the
point 21 , 0 , and passing through the pole. The curve is swept out as θ ranges from 0 to π, so
the area of the disk is
Z Z
1 π 2 1 π π
cos θ dθ = (1 + cos 2θ) dθ = .
2 0 4 0 4
Compare this with the area of the disk r = 1, centred at the pole, where the curve is swept out
as θ ranges from 0 to 2π; Z
1 2π
Area = 1 dθ = π .
2 0
In that case, if we were to stop at π, we would obtain only the area of the upper half-disk.
Rather than attempting to memorize rules about limits for integrals, I suggest you carefully
analyze each problem individually.

Finding the intersections of curves in polar coordinates The textbook [1, p. 651-652]
discusses the difficulties that of finding intersections which are a consequence of the multiple
sets of coordinates for points. Some textbooks suggest that finding the intersections can only
be done visually, and that is not true. In fact, intersections can be found algebraically, but one
must be careful and thorough. I attach below a discussion I prepared for a class some years
ago, where another textbook had made a false statement that I felt obliged to correct; the error
is continued in the current edition of the same textbook.
Example B.77 In an example in another textbook [31, Example 8, p. 579], [30, Example 8, p.
635] the objective is to find the points where the curves

r = 1 + sin θ (80)
r2 = 4 sin θ (81)

intersect. It was stated in the textbook solution that only one of the points of intersection
can be found algebraically, and that the others can be found only “when the equations are
graphed”. We show here all intersection points can be found algebraically! We never resort to
calculations on a sketch: all procedures can be justified theoretically — the sketch serves only
to help visualize a situation that can be adequately described verbally and/or with mathematical
formulæ.
(cf. Figure 16, page 2168) Had the curves been given in cartesian coordinates, we could
Information for Students in Lecture Section 002 of MATH 141 2009 01 2168

–1 –0.5 0 0.5 1

–1

–2

Figure 16: Intersecting polar curves r = 1 + sin θ, r2 = 4 sin θ

have found all intersections by solving the equations simultaneously. Why can’t we solve the
polar equations in the same way? The difficulty derives from the fact that any point has in-
finitely many different polar representations. More precisely, a point that can be represented
by polar coordinates (r, θ) also has coordinates ((−1)n r, θ + nπ), where n is any integer — pos-
itive or negative; moreover, the pole can be represented by (0, θ), where θ is any real number.
To determine the points of intersection, one must consider the possibility that the same point
appears with different coordinates.
Solve the given equations algebraically: By eliminating sin θ between the two equations, we
obtain r2 = 4(r−1), which implies that (r−2)2 = 0, so r = 2, and sin θ = 2−1 = 1. Hence

π π
θ = +2mπ, where m is any integer, and the points of intersection are 2, + 2mπ : but,
2 2
by the convention describedabove,
 these are representations of the same point, whose
π
“simplest” representation is 2, .
2
Transform the equations in all possible ways and solve again: Apply to one of the equa-
tions the transformation
(r, θ) 7→ (−r, θ + π) (82)
Information for Students in Lecture Section 002 of MATH 141 2009 01 2169

and solve it with the original form of the other equation. Repeat this process until the
equations transform to a pair already solved. Equation (80) transforms to

−r = 1 + sin(θ + π) (83)

which is equivalent to
r = −1 + sin θ (84)

which√equation we solve with (81). Eliminating sin θ yields r = 2 ± 2 2, so sin θ =
3 ± 2 2. √The upper sign is inadmissible, as a sine cannot exceed 1 in magnitude. Hence
r = 2 − 2 2 and √
sin θ = 3 − 2 2 . (85)
 √   √ 
The solutions to (85) are θ = sin−1 3 − 2 2 + 2mπ and θ = π − sin−1 3 − 2 2 + 2mπ;
we may take m = 0, as all other values of m give the same two points:
 √  √   √  √ 
2 − 2 2, sin−1 3 − 2 2 and 2 − 2 2, π − sin−1 3 − 2 2 .

As the first coordinate in these cases is negative, we could equally well represent the
points as  √  √  
2 2 − 2, sin−1 3 − 2 2 + π
and  √  √ 
2 2 − 2, − sin−1 3 − 2 2 .
A second application of (82), to (83), restores the original equation; hence there are no
other intersection points, except possibly the pole.
!

Check whether the pole is on both curves: On (80) the pole appears as 0, , etc.; on (81)
2
it appears as (0, 0), etc. Thus the pole is also a point of intersection. The reason we did
not find it when we solved pairs of equations is that it appears on the two curves only
with different sets of coordinates, no two related by (82).

Example B.78 [7, Exercise 29, p., 683] “Find the area of the region that lies inside both
curves: r = sin θ, r = cos θ.”
Solution: (see Figure 17 on page 2170) The curves are circles through the pole. To see this,
multiply the first by r = 0 (which, of course, could be bringing the pole into the curve). The
resulting equation is r2 = r sin θ, which, in cartesian
 coordinates, would be x2 + y2 = y, a
circle with centre (in cartesian coordinates) 0, 2 and radius 12 . The operation of multiplying
1

by r = 0 did not, however, alter the curve, since the pole was already on the curve, with polar
coordinates (r, θ) = (0, 0). In the same way we can  show that the second curve is a circle of
the same radius centred at the point (r, θ) = 12 , 0 ; polar coordinates for the centre of the first
 
circle are, for example, 12 , π4 .
Information for Students in Lecture Section 002 of MATH 141 2009 01 2170

1.0

0.75

0.5

0.25

0.0
−0.5 −0.25 0.0 0.25 0.5 0.75 1.0

−0.25

−0.5

Figure 17: Curves r = sin θ, r = cos θ

Where do the curves meet? (This was [7, Exercise 37, p. 683], which should have pre-
ceded the present problem in the exercises!) We can begin this investigation by solving their
equations, which imply that tan θ = 1, so θ = π4 + nπ, where n is any integer; that implies that
r = ± √12 : the + sign is associated with the cases where n is even, the − sign with those where
   
n is odd. But (r, θ) = √12 , π4 + 2mπ and (r, θ) = − √12 , π4 + (2m + 1)π are all the same point —
 
having cartesian coordinates 12 , 12 .
But the curves also meet at the pole, even though this did not show up when we solved the
equations. This is because the pole appears on the first curve with θ = nπ, and on the second
curve with θ = (n + 21 )π. The only practical way to determine whether curves intersect at the
pole is to examine each of them separately for that point.
Information for Students in Lecture Section 002 of MATH 141 2009 01 2171

Could it be that the curves meet in any other points? While a sketch does not suggest that,
we should never rely on a sketch! If we transform the first equation under the transformation
(r, θ) 7→ (−r, θ + π), we find that the equation does not change (except for a sign change on
both sides); the same applies to the second equation. Thus, in the present example, there are no
intersections other than the pole in which a point will appear on the two curves with different
coordinates. We can comfortably proceed to the integration part of the problem, confident that
we have not missed any points of intersection.
We can see that the region whose area is sought is symmetric about the line θ = π4 , so it
suffices to find half of it and to double it. The lower half of the area is subtended at the pole by
the arc 0 ≤ θ ≤ π4 of the circle r = sin θ; hence
Z π
1 4 2
Area = 2· sin θ dθ
2 0
Z π
1 4
= (1 − cos 2θ) dθ
2 0
" #π
1 sin 2θ 4 π 1
= θ− = − .
2 2 0 8 4

(You could have solved this problem using Cartesian coordinates.)

10.4 Exercises

[1, Exercise 22, p. 653] “Find the area enclosed by the loop of the strophoid r = 2 cos θ −
sec θ.”
Solution: (see Figure 18 on page 2172) (The textbook did not expect a rigorous proof
that the curve crosses itself only at the pole.) First we observe that the curve is entirely
traced out when θ passes through an interval of length 2π; so, without limiting generality,
we may confine ourselves to such an interval, say − π2 < θ < π2 , and thereby remove the
some possible ambiguity of coordinates. We must exclude the points where sec θ is
undefined. For convenience, let’s confine θ to the union of intervals
π π π 3π
− <θ< and <θ< .
2 2 2 2
Consider a point on the second branch, say with θ = φ + π, where − π2 < φ < π2 . We find
that, for this point,

r = 2 cos(φ + π) − sec(φ + π)
= −2 cos φ + sec φ
= −(2 cos φ − sec φ)
Information for Students in Lecture Section 002 of MATH 141 2009 01 2172

0.6

0.4

0.2

0
-0.4 -0.2 0 0.2 0.4 0.6 0.8 1

-0.2

-0.4

-0.6

Figure 18: The strophoid r = 2 cos θ − sec θ

So we see that the point is the same as the point (−r, φ). Thus, in order to see the whole
curve, it suffices to investigate angles θ between − π2 and π2 ; some of the points will,
however, appear with negative r-values.
We observe that the replacement of θ by −θ does not change the equation: this tells us
that the curve is symmetric about the polar axis and its extension.
For the problem to be meaningful, there should be just one loop. The curve passes
through the pole when 0 = 2 cos θ−sec θ, equivalently, when cos θ = ± √12 . In the interval
to which we have confined θ, this will occur when θ = ± π4 . If we follow the curve as θ
comes from − π2 , we find that r is negative, and the point is in the 2nd quadrant. It passes
through the pole first when θ = − π4 , and is then in the fourth quadrant, striking the polar
Information for Students in Lecture Section 002 of MATH 141 2009 01 2173

axis when θ = 0, at the point (r, θ) = (1, 0) and it then moves into the first quadrant,
following the mirror image of the portion in the fourth quadrant, passing through the
pole again when θ = π4 , and moving into the 3rd quadrant. If we consider the cartesian
coordinates of the curve in parametric form, we have

x = 2 cos2 θ − 1
y = sin 2θ − tan θ

and see that, as cos θ → 0, x → −1, and y → ±∞, so the curve is asymptotic to the
vertical line x = −1. Our present problem is to determine the area of the loop from
θ = − π4 to θ = π4 , or, by symmetry,
!Z π Z π
 
1 4
2
4
2 (2 cos θ − sec θ) dθ = 4 cos2 θ + sec2 θ − 4 dθ
2 0 0
Z π
4  
= 2 cos 2θ + sec2 θ − 2 dθ
0
π
= [sin 2θ + tan θ − 2θ]04
 π π
= 1+1− −0=2− .
2 2

[1, Exercise 26, p. 653] Find the area of the region that lies inside the curve

r = 2 + sin θ (86)

and outside the curve


r = 3 sin θ . (87)

Solution: (see Figure 19 on page 2174) The first step is to determine where the curves
intersect. We can start by solving equations (86) and (87) algebraically. We find that
r = 3 and θ = π2 +2nπ,where n is any integer. The second curve is a circle with centre at
the point (r, θ) = 32 , π2 . The first curve appears to be some sort of oval, and touches the
 
circle at its topmost point, which we shall call A, with coordinates (r, θ) = 3, π2 . Could
there be any other points of intersection? There don’t appear to be any, from the sketch,
but we can resolve this question algebraically. The pole does not lie on first curve, since
that would entail that 0 = 2 + sin θ, i.e., that the sine of an angle exceeds 1 in magnitude.
Since that is not possible, the pole cannot lie on the curve. If there were to be any other
intersections, they would have to have different sets of coordinates on the 2 curves. We
investigate what happens to (87) under the transformation

(r, θ) 7→ (−r, θ + π)
Information for Students in Lecture Section 002 of MATH 141 2009 01 2174

0
-2 -1 0 1 2

-1

Figure 19: Curves r = 2 + sin θ, r = 3 sin θ

and find that the equation does not change significantly: it becomes −r = sin(θ + π) =
− sin θ, which is evidently equivalent to the original equation. Equation (86) changes to
−r = 2 + sin(θ + π) = 2 − sin θ, or

r = −2 + sin θ. (88)

When we solve this equation with (87), we find that  r = −3, sin θ = −1: in the interval

0 ≤ θ ≤ 2π we find only the point (r, θ) = −3, 2 , which is the same point as (r, θ) =
 
3, π2 , so we don’t find any new intersections. Thus there are no other intersections than
A.
A direct way to solve this problem is to find the area of the region bounded by the outer
Information for Students in Lecture Section 002 of MATH 141 2009 01 2175

curve, and then subtract from it the area of the disk inside. The outer curve is traced out
as θ ranges from 0 to 2π, so the area of the region it bounds will be
Z Z
1 2π 1 2π
2
(2 + sin θ) dθ = (4 + 4 sin θ + sin2 θ) dθ
2 0 2 0
Z !
1 2π 1 − cos 2θ
= 4 + 4 sin θ + dθ
2 0 2
" #2π
1 9θ 1 9π
= − 4 cos θ − sin 2θ =
2 2 4 0 2
The inner curve bounds an area of
Z Z
1 π 2 9 π
9 sin θ dθ = (1 − cos 2θ) dθ
2 0 4 0
" #π
9 1 9
= θ − sin 2θ = · π .
4 2 0 4
What would have happened if we had taken the upper limit of integration to be 2π? We
would have found twice the area, because the entire circle is swept out as θ ranges over
an interval of length π.
Thus we see that the area of the region between the curves is
9π 9π 9π
− = .
2 4 4
Could we find the area by taking the difference of two squares under the integral sign,
as is suggested in [1, Example 2 and Figures 5 and 6, p. 651]? We could do this for the
portion of the area above the polar axis and its extension, where we would obtain
Z
1 π 
(2 + sin θ)2 − (3 sin θ)2 dθ
2 0
Z
1 π
= (4 + 4 sin θ − 4(1 − cos 2θ)) dθ
2 0
1
= [−4 cos θ + 2 sin 2θ]π0 = (2 + 0) − (−2 + 0) = 4
2
For the remainder of the area only curve (86) and the extended polar axis serve as a
boundary:
Z " #2π
1 2π 2 1 9θ 1
(2 + sin θ) dθ = − 4 cos θ − sin 2θ
2 π 2 2 4 π
" !#
1 9π 9π
= (9π − 4 − 0) − − (−4) − 0 = −4
2 2 2
Information for Students in Lecture Section 002 of MATH 141 2009 01 2176

which, when added to the area of the upper portion, gives the same area as before.
What would have happened if we had subtracted the square of 3 sin θ over the full range
of the integration? We would have been subtracting the area of the inside disk twice!
And what would have happened if we had taken (88) as the equation of the outer curve?
If we used it only to find the area of the (larger) region bounded, we would obtain the
correct value there. But, if we had taken the difference of squares under the integral sign,
we would obtain values for regions that do not correspond to the one whose are we are
trying to find.
Before finding an area in polar coordinates, (or, indeed, in cartesian coordinates as well),
you are urged to make a sketch and study the element of area that you are summing in
the limit, to ensure that the integral represents the area that you are seeking.

Example B.79 Another example of finding the intersections of curves.

Find the intersections of the lima¸con r = 1 − 2 cos θ and the circle r = 1. [29]

Solution: (see Figure 20 on page 2177) When we  solve the given equations, we find the inter-
π
section points (r, θ) for which cos θ = 0, i.e., 1, ± 2 . But a glance at the graphs shows that
there appears to be a third point of intersection. Under the transformation (r, θ) → (−r, θ + π),
the given equation for the circle r = 1 does not change in a significant way. However, the
equation of the lima¸con changes to r = −1 − 2 cos θ; when we solve this latter equation with
the equation r = 1, we find that cos θ = −1, so the point (r, θ) = (1, π) is another point of
intersection: i.e., (1, π) on the circle, (−1, 0) on the original equation for the limaçon.
To finish this investigation we should investigate whether the pole is on both curves, since
that may not be detected by this method of transforming the equations. We note, however, that
the curve r = 1 cannot possibly contain the pole, since all of its points are 1 unit from the
pole. Thus the investigation is complete: there are three point of intersection, and we have
now found them all.

Example B.80 A problem from a recent examination:

[12 MARKS] Use polar coordinates — no other method will be accepted — to


1
find the area of the region bounded by the curve r = 2 and the line r = , and
cos θ
containing the pole.

Solution: (see Figure 21 on page 2178) The first step is to determine where the two curves
intersect. While the points of intersection can all be found algebraically, that procedure is
tedious, so it is best to make a sketch first to see whether the elaborate procedure is required.
You should be able to see immediately that the curve r = 2 is a circle of radius 2 centred at
the pole. But what about r = sec θ? Since the secant is never less than 1 in magnitude, you
Information for Students in Lecture Section 002 of MATH 141 2009 01 2177

1.5

0.5

0
-3 -2 -1 0 1
-0.5

-1

-1.5

Figure 20: Intersections of the limaçon r = 1 − 2 cos θ with the circle r = 1

know this curve does not pass through the pole. You also know that, as θ ranges between − π2
and π2 , the function ranges from −∞ to +∞ monotonely, so the curve will be closest to the pole
when θ = 0 — when it is 1 unit from the pole. As θ increases to + π2 , r increases steadily —
monotonely — the curve crossing the circle r = 2 and moving a greater and greater distance
from the pole. What happens when r < 0? A point with coordinates (−a, θ), where a > 0 will
still be a distance a from the pole! (It’s located on the ray obtained by turning the polar axis
through an angle of θ + π.) So, as r → −∞, the curve again crosses each of the circles centred
at the pole and moves a greater and distance from the pole. This is all you need to know in
order to solve the problem, but you should be able to see that this is a very simple curve: just
1
multiply both sides of r = by cos θ, and you see that the equation is r cos θ = 1, or, in
cos θ
Information for Students in Lecture Section 002 of MATH 141 2009 01 2178

0
-2 -1 0 1 2

-1

-2

Figure 21: Intersections of the curve r = sec θ with the circle r = 1

cartesian coordinates, x = 1 — the curve is a straight line perpendicular to the polar axis!
We have seen that there will be just 2 points of intersection of the curves. Solving their
equations by eliminating r we obtain cos θ = 12 , so θ = ± π3 : the points of intersection are
 π
(r, θ) = 2, ± .
3
We saw that the element of area in the form of a thin triangle with apex at the pole (or a thin
sector of a disk with centre at the pole) has area of the form 12 r2 dθ. In this problem such an
element would have to be described in two different ways:
Information for Students in Lecture Section 002 of MATH 141 2009 01 2179

π π
− ≤ θ ≤ : The area of this triangle is
3 3
Z π √
1 3 1 1 π
dθ = [tan θ] 3
− π = 3.
2 − π3 cos2 θ 2 3

π 5π
≤θ≤ : The area of this portion of the disk is
3 3
Z 5π3
1 1 5π 8π
22 dθ = [4θ] π3 = .
2 π3 2 3 3

√ 8π
Thus the area of the region determined by the two curves, and containing the pole, is 3+ .
3
2
This problem could also have been solved by subtracting from the area of the disk, π2 , the
area given by
Z π
1 3 2 1 π 4π √
(2 − sec2 θ) dθ = [4θ − tan θ]−3 π = − 3.
2 − π3 2 3 3
Information for Students in Lecture Section 002 of MATH 141 2009 01 2180

B.19 Supplementary Notes for the Lecture of March 11th, 2009


Release Date: Wednesday, March 11th, 2009

B.19.1 §10.4 Areas and Lengths in Polar Coordinates (conclusion)


Arc Length To develop a formula for the arc length of a curve given in polar coordinates as
r = f (θ), we can apply the theory of curves given in parametric form to the curve

x = f (θ) · cos θ
y = f (θ) · sin θ

We find that !2 !2 !2
dx dy dr
+ = + r2
dθ dθ dθ
so the length is given by the integral
s
Z b !2
dr
L= + r2 dθ
a dθ

where the limits θ = a and θ = b need to be determined from the parametrization of the curve.
As usual, you should be careful to determine the appropriate values of θ to define the portion
of the curve that interest you.

10.4 Exercises (continued)

[1, Exercise 46, p. 654] Find the exact length of the arc of the polar curve r = e2θ from θ = 0
to θ = 2π.
Solution:
s
Z 2π !2
dr
Length = + r2 dθ
0 dθ
Z 2π q
 
= 2e2θ 2 + e2θ r2 dθ
0
√ Z 2π
= 5 e2θ dθ
0
√ √  4π 
h i
5 2θ 2π 5 e − 1
= e =
2 0 2
Information for Students in Lecture Section 002 of MATH 141 2009 01 2181

[1, Exercise 54, p. 654] Graph the curve r = cos2 2θ , and find its length.
Solution: The equation can be simplified to read
 
1 + cos 2 · 2θ
r=
2
which we recognize to be a cardioid. As θ ranges over the interval 0 ≤ θ ≤ 2π the entire
curve is traced out once.
s
Z 2π !2
dr
Length = + r2 dθ
0 dθ
Z
1 2π p
= (− sin θ)2 + (1 + cos θ)2 dθ
2 0
Z
1 2π p
= 2(1 + cos θ) dθ
2 0
Z r 
1 2π 2
θ
= 2 2 cos dθ
2 0 2
Z 2π  
= cos θ dθ
2
0
 θ π  θ 2π
= 2 sin − 2 sin
2 0 2 π
= 2 − (−2) = 4 .

B.19.2 §10.5 Conic Sections


Omit this section.

Textbook Chapter 11. INFINITE SEQUENCES AND SERIES.

B.19.3 §11.1 Sequences


A sequence is an ordered set of objects, usually labelled with either the non-negative integers
0, 1, . . . , n, . . . or the positive integers 1, 2, . . . , n, . . . . In this chapter we shall be interested in
sequences of real numbers or of real functions. Technically, such a sequence is a function that
Information for Students in Lecture Section 002 of MATH 141 2009 01 2182

maps either the non-negative integers or the positive integers on to the set of real numbers (or
to the set of real functions). We usually denote a sequence by a single letter, e.g. a, and show
the labelling by either a parenthesized value, as a(n), or a subscripted values, as an ; when we
wish to talk about a sequence with specific terms, we may just list the terms with a general
term that shows the pattern we are describing, if there is one, as in

1, 1, 2, 3, 5, 8, 13, 21, 24, 55, 89, . . .

which is the Fibonacci sequence, in which every entry past the second is the sum of the two
entries immediately preceding it. Unless the general term is described unambiguously (as here,
where Fn+2 = Fn+1 + Fn for all n ≥ 3), there will be more than one way to generalize a pattern
that appears to hold between a few terms at the beginning of the sequence.

Notation When a sequence consists of terms a0 , a1 , a2 , . . . we may speak of the sequence


{an }, or perhaps the sequence {an }n=0,1,... , or sometimes simply the sequence an . Other notations
are also possible, and should be understandable from the context.

The limit of a sequence We define lim an = L in a way analogous to the definition of


n→∞
lim f (x) = L; another way of writing the same definition is
x→∞

an → L as n → ∞.

The precise definition is to be found as [1, Definition 2, p. 677], but is not on the syllabus —
you are not expected to be able to work with the precise definition of the limit of a sequence.
Note that we usually do not write the braces { and } when we speak of the limit of a sequence.
Since you bring to this course some understanding of limits of functions, we will occasion-
ally appeal to the following

Theorem B.81 [1, Theorem 3, p. 678] If f is a function, and {an } is such that f (n) = an for all
n, and if lim f (x) = L, then lim an = L.
x→∞ n→∞

We also generalize our definition of the limit of a sequence to permit limits to be ±∞; the
definition is again analogous to the corresponding definition for the meaning of lim f (x) =
x→∞
±∞.
A sequence that has limit L is said to converge to L; if there is no limit, the sequence is
said to diverge. We do not permit L to be ±∞ in this usage, so a sequence whose limit is ±∞
is said to diverge.
Information for Students in Lecture Section 002 of MATH 141 2009 01 2183

Limit Laws We can prove limit laws for sequences analogous to the limit laws we saw in
the previous course for functions. I shall not give the details in these notes. Remember the
usual restriction that we cannot divide by 0, so there must be a restriction on the quotient law.
The limit laws can be extended to infinite limits whenever the operations can be justified; so,
we can think of ∞ + ∞ = ∞, but we cannot attach a meaning to ∞ + (−∞), nor of 0 · ∞, as it is
not possible to assign to these expressions a meaning that will be consistent with the algebraic
operations on real numbers.

Increasing and Decreasing Sequences We defined the concepts of increasing and decreas-
ing in connection with real functions of a real variable (cf. [1, p. 20]. The identical defini-
tions apply for sequences, where we consider the domain of the function to be the positive
or non-negative integers. When a sequence is either increasing or decreasing we say that it is
monotonic or monotone. Sometimes we find it convenient to work with a slightly weaker prop-
erty than increasing, and may speak of a sequence as being non-decreasing, which permits the
function to remain constant for a while.
A function is bounded above if there exists a number M which is greater than all values of
the function. For example, the sine function is bounded above, since sin x ≤ 1 for all x in its
domain. We could also have observed that sin x < 10000, and conclude that such a statement
justifies the conclusion that the function is bounded above: we don’t care how high above the
graph of the function the bounding line appears, only that such a line exists. But the function
tan x is not bounded above. In the same way we can define bounded below and bounded —
meaning bounded both above and below. An important theorem we shall need in this chapter
is

Theorem B.82 1. A sequence that is bounded above and monotonely increasing (or even
monotonely non-decreasing) is convergent.

2. A sequence that is bounded below and monotonely decreasing (or even monotonely non-
increasing) is convergent.

(Note that the theorem stated here is stronger than [1, Monotonic Sequence Theorem, p., 683].)

Example B.83 [1, Exercise 36, p. 685] Determine whether the sequence an = ln(n + 1) − ln n
converges.
Solution: We might be tempted to analyze a difference by using the difference law for limits.
But we find that each of the terms approaches +∞, and we cannot give a meaning to ∞ − ∞.
So that approach will not work.
However, if we observe that the difference of logarithms is
!
n+1 1
ln = ln 1 +
n n
Information for Students in Lecture Section 002 of MATH 141 2009 01 2184

we can reason as follows. As n → ∞, n1 → 0, 1 + 1n → 1 + 0 = 1. By continuity of the


logarithm, the sequence will approach ln 1 = 0. (I am using Theorem B.81 above.)

Sequences of powers For a fixed real number a we know the behavior of a function a x as
x → ∞, and how it depends on a; this permits us to study the behavior of an when the exponent
is restricted to integer values:

lim an = +∞ if a > 1
n→∞
lim an =1 if a = 1
n→∞
lim an =0 if −1 < a < 1
n→∞
lim an does not exist if a ≤ −1
n→∞

(Note that, when a limit is ±∞, as in the first case above, we still say that the limit does not
exist!)
n!
Example B.84 Consider the sequence {an }, where an = n . Does it converge? If so, to what
n
value?
Solution: Consider the limit of the ratio of an+1 to an . It is

an+1 (n + 1)! nn 1 1
= · =  n →
an n! (n + 1)n+1 1 + 1n e

as n → ∞. Thus, for sufficiently large n, every term is less than half of the one before it, which
1
is positive. Compared to the nth term, the (n + 10)th will be less than 1000 of the size; the
n + 20th will be less than one-millionth the size, etc. The sequence is thus approaching 0.

Example B.85 ([7, Exercise 57, p. 711]) Determine whether the sequence is increasing, de-
creasing, or not monotonic. Is the sequence bounded?

an = cos
2
Solution: The values of this sequence are (starting at n = 0) 1, 0, −1, 0, 1, 0, −1, 0, . . .. The
sequence is neither increasing nor decreasing, so it is not monotonic, and the [1, Monotonic
Sequence Theorem, p. 683] is not applicable. The sequence is bounded. It does not converge
— but that is not a consequence of the fact that the sequence does not satisfy the conditions of
cos nπ2
the theorem. (Compare with the sequence which converges to 0.)
n
Information for Students in Lecture Section 002 of MATH 141 2009 01 2185

11.1 Exercises
) (
ln n
[1, Exercise 32, p. 685] Determine whether the sequence converges.
ln 2n
Solution:
ln n ln n + ln 2 − ln 2
=
ln 2n ln n + ln 2
ln 2
= 1−
ln 2n
The subtracted fraction has a constant numerator, but the denominator is increasing, so
the fraction is decreasing as n increases; hence 1 minus the fraction is increasing; in
fact, we can see that the subtracted fraction is approaching 0, so the given fraction is
approaching 1.
[1, Exercise 52, p. 685] Investigate the convergence or divergence of the sequence
1 · 3 · 5 · . . . · (2n − 1)
an =
n!
Solution:
1 · 3 · 5 · . . . · (2n − 1)
an =
n!
2 · 4 · . . . · (2n − 2)
≥ 1·
n!
2n−1
=
n
We can determine the limit of this ratio by L’Hospital’s Rule:
2n−1 2n−1 · ln 2 ln 2 n
lim = lim = lim · 2 = +∞
n→∞ n n→∞ 1 n→∞ 2
since it is a non-zero multiple of the sequence of powers of a constant whose magnitude
exceeds 1; thus the original sequence diverges.

Example B.86 For each of the following sequences, determine whether it converges; if it does,
find the limit.
sinh x
1. an =
cosh x
r
3 + sin n
2. an =
n
Information for Students in Lecture Section 002 of MATH 141 2009 01 2186

Solution:

1.
sinh x e x − e−x
lim = lim x
n→∞ cosh x n→∞ e + e−x
1 − e−2x
= lim
n→∞ 1 + e−2x
 
lim 1 − e−2x
n→∞
=  
lim 1 + e−2x
n→∞
1
= =1
1

2. Consider the fraction under the radical sign: the numerator is bounded, while the de-
nominator approaches infinity, so the fraction approaches 0. By a familiar result about
the composition of continuous functions, the square root approaches 0.

B.19.4 §11.2 Series


The sum of the terms in a sequence. Having defined what we mean by convergence of a
sequence {an } — analogous to the existence, for a function a(x), of lim a(x) — we proceed
x→∞
to apply that concept to generalize the concept of addition. To a mathematician, the operation
of addition is a binary operation on numbers, that is, it maps two real numbers on to one real
number. This operation has a number of properties that mathematicians take as primitive:

• it is commutative: x + y = y + x for all real numbers x and y;

• it is associative: (x + y) + z = (x + y) + z for all real numbers x, y, z;

• it has an “additive identity”, which we call zero and denote by 0, with the property that
x + 0 = x for every real number x;

and other properties that we will not discuss here. The second property permits us to talk of
more general sums than of just 2 numbers. If we have three real numbers x, y, z, then the
second property permits us to define x + y + z to be the common value of (x + y) + z and
(x + y) + z; we can generalize this idea to the sum of x1 + x2 + . . . + xn , although we will not
consider the details of this definition in this calculus course.50 But the definition we obtain in
this way applies only to a finite sequence of numbers. In this section we consider a broader
50
This is a topic that would likely appear in a first Abstract Algebra course, like MATH 235.
Information for Students in Lecture Section 002 of MATH 141 2009 01 2187

definition, to permit us to speak of the sum of all the terms in a sequence. In this context we
no longer write the sequence with commas, as

a0 , a1 , . . . , an , . . . ,

but, instead, place plus signs between the terms, writing

a0 + a1 + . . . + an + . . . ,

or, more compactly, as


X

an ,
n=0

and we speak of a series, rather than a sequence. Note that the word series is both singular
and plural in English: there is no word “serie” in English51 . Our definition proceeds from the
partial sums, which we define to be the terms in the sequence

a0 , a0 + a1 , a0 + a1 + a2 , ... , a0 + a1 + . . . + an , ...

or, more compactly, as

X
0 X
1 X
m
an , an , ... , an , ... ,
n=0 n=0 n=0

all of which are finite sums, and therefore well defined. (Note that we had to use a different
letter — m — for the general term in this sequence, because the letter n was “busy”52 .) We say
that
X

an = L
n=0

if L is the limit of the sequence of partial sums of the series. The series is then said to converge,
or to be convergent; if it does not converge, it diverges or is divergent. Here, as with series
and functions, we generalize to write that a sum = ∞ or = −∞, but still describe such series as
divergent. We will often consider series which we will know to be convergent, without being
able to specify the precise value of the limit.

“Geometric” series. A geometric series is one, each of whose terms after the first is a con-
stant multiple of its predecessor. We will often represent such series as

a + ar + ar2 + . . . + arn + . . .
51
But the singular word for “series” in French is série, and the plural is séries.
52
Technically, we call n a bound variable in this case.
Information for Students in Lecture Section 002 of MATH 141 2009 01 2188

where a , 0 and the “common ratio” is the ratio of each term to its predecessor, here denoted
by r. By “standard” methods, usually seen in high school, one can prove that the value of the
partial sum of m + 1 terms is

Xm 

 a (1 − rm )
 if r , 1
a + ar + . . . + arm = arn = 
 1 − r .

 (m + 1)a if r = 1
n=0

This ratio can be seen to have the following limit properties:


 a

 m   


=
1−r
if −1 < r < 1
X   
 = +∞
a + ar + . . . + arm + . . . = lim  arn  = 

if r ≥ 1 and a > 0 .
m→∞ 

 = −∞ if r ≥ 1 and a < 0
n=0 

 diverges if r ≤ −1 and a , 0

Divergence of the “harmonic” series. The harmonic series is


1 1 1
1+ + + ... + + ... .
2 3 n
In the next section we will see a proof, using an improper integral, that this series diverges.
Here we shall see a simpler proof without integrals. We need only observe the following
inequalities:
1 1
2
≥ 2

1 1 1 1 2 1
3
+ 4
> 4
+ 4
= 4
= 2

1
5
+ 16 + 17 + 1
8
> 1
8
+ 18 + 18 + 1
8
= 4
8
= 1
2

...
1 1 1 1 n−1 1 1
+ + . . . + + > 2 · =
2n−1 + 1 2n−1 + 2 2n − 1 2n 2n 2
Thus we can make the partial sums as large as we like by proceeding out sufficiently far in the
series. It follows that
X∞
1
= +∞ ,
n=1
n
so the harmonic series diverges.
Information for Students in Lecture Section 002 of MATH 141 2009 01 2189

B.20 Supplementary Notes for the Lecture of March 16th, 2009


Release Date: Monday, March 16th, 2009

B.20.1 §11.2 Series (conclusion)


“Necessary” and “sufficient” conditions. Suppose that A and B are sentences that may be
true or false. If A cannot be true except when B is true, we say that the truth of A entails the
truth of B, or that
A implies B
and write this symbolically as
A ⇒ B.
We call A a sufficient condition for B. An example of such an implication where x and y are
members of the “universe” of real numbers is
x = y2 − 2y + 4 ⇒ x ≥ 3
since y2 − 2y + 4 = (y − 1)2 + 3, and a square cannot be negative.
We can interpret the statement
A⇒B
in another way, by observing that, when A is true, B cannot be false. We can say that B is a
necessary condition for A, since A cannot be true unless B is true.
Mathematicians usually search for conditions that are both necessary and sufficient, since
these can characterize a situation. But often we have to be satisfied with conditions that are
either of one type or the other.

A “necessary” condition for convergence of a series The implication that interests use here
is in the “universe” of series of real numbers. It states that
X∞
an is convergent ⇒ lim an = 0 .
n→∞
n=0
This — once it has been proved — is a test that can be applied to a series to see whether
the series is convergent. It can never prove that a series is convergent! What it can prove,
sometimes, is that a certain series is not convergent, since no series that “fails the test” can
be convergent. Many textbooks call this test the “nth term test”; your textbook calls it “‘The’
test for divergence”. This is not a standard term, and you might wish to avoid using it, since
a listener who has not read Stewart’s books might not know what you are referring to.53 In
practice you should “internalize” this test and always apply it, since you might otherwise
waste time trying to prove that a divergent series is convergent.
53
You will meet other tests that could be given such a name, although this is the most likely one to distinguish
in this way. Stewart’s name for the test may be no more objectionable than the name in general use: it is not good
Information for Students in Lecture Section 002 of MATH 141 2009 01 2190

“Telescoping” series. Sometimes the partial sums can be interpreted as sums where there
is heavy cancellation of intermediate terms, leaving only a few at each end. An example is
X∞ X∞ !
1 1 1 1
= − where the partial sum of the first N terms is simply 1 − .
n=1
n(n + 1) n=1
n n + 1 N + 1
(cf. [1, Example 6, p. 691]).

Changes of variable in a sum. We have already seen the concept of changing a variable in
connection with definite integrals. We can carry out similar changes in sums — both finite and
infinite, although we will not investigate all the details. So, for example, we can change the
X m X
m
name of the index of summation, and write ar in place of the sum an , by a “change of
r=0 n=0
X
m
variable” given by r = n. More generally, we could define s = n + 4, say, and change an
n=0
X
m+4
into a s−4 . Notice how the “limits of summation” have to be changed in the same way that
s=4
we changed the limits of integration in a definite integral.

Operations on series. Convergent series may be added term by term, or multiplied by a


constant:
X
∞ X
∞ X

(an + bn ) = an + bn
n=0 n=0 n=0
X
∞ X

can = c an
n=0 n=0

But note well: while you may add convergent series term by term, you may not rearrange a
series; we shall see later that rearrangement of the terms can result in a series having a different
sum, or even in a convergent series being rendered divergent.

11.2 Exercises
2 X

[1, Exercise 36, p. 695] Determine whether the series is convergent or diver-
n=1
+ 4n + 3 n2
gent by expressing the partial sum as a telescoping sum (as in [1, Example 6, p. 691]).
If it is convergent, find its sum.
form to use the symbol n in the name of a test, since the test does not depend on giving that particular name to
the variable that indexes the members of the sequence; my objections to the name Stewart assigns are that (1) it is
not universally accepted; and (2) the definite article suggests uniqueness, and this is not the only test that exists.
Information for Students in Lecture Section 002 of MATH 141 2009 01 2191

Solution: Since

2 2 1 1
= = − ,
n2 + 4n + 3 (n + 1)(n + 3) n + 1 n + 3
the Nth partial sum is equal to

X
N XN !
2 1 1
= −
n=1
n2 + 4n + 3 n=1
n+1 n+3
X
N
1 X 1
N
= −
n=1
n + 1 n=1 n + 3
XN
1 X 1
N+2
= −
n=1
n + 1 m=3 m + 1
setting m = n + 2 in the second sum
X
N
1 X
N+2
1
= −
n=1
n + 1 n=3 n + 1
replacing the variable m by n
X2
1 XN
1 XN
1 X
N+2
1
= + − −
n=1
n + 1 n=3 n + 1 n=3 n + 1 n=N+1 n + 1
X
2
1 X
N+2
1
= −
n=1
n + 1 n=N+1 n + 1
1 1 1 1
= + − − .
2 3 N+2 N+3
As N → ∞, this partial sum approaches 12 + 13 = 56 . (In this type of problem we don’t
expect students to be write the proof out formally in this way; I have included this
solution so that you can see how the problem can be solved “properly”, without the use
of “dots”.
X

n
[1, Exercise 40, p. 695] Determine whether the series ln is convergent or divergent
n=1
n + 1
by expressing sn is a telescoping sum (as in [1, Example 6, p. 691]). If it is convergent,
find its sum.
Solution: In a similar way to that shown above in the solution to [1, Exercise 36, p. 695],
we can show that the partial sum of the first N terms of the series is ln 1 − ln N + 1. As
N → ∞ the partial sum approaches −∞: the series diverges.
Information for Students in Lecture Section 002 of MATH 141 2009 01 2192

[1, Exercise 44, p. 695] Express the “repeating decimal expansion” 6.254 as a ratio of inte-
gers.
Solution: By a “repeating decimal expansion”, we intend that the digits under the hori-
zontal line are to be repeated as a subsequence of the expansion indefinitely; thus
6.254 = 6.25454545454... ,
which we can interpret as the sum of a series
2 54 54 54
6+ + + + + ...
10 1000 100000 10000000
which can be seen as a constant added to a geometric series:
!  !2 !3 
2 54  1 1 1 
6+ + 1 + + + + . . .
10 1000 100 100 100
whose sum is
2 54 1 1 3 344
6+ + · 1
=6+ + = .
10 1000 1 − 100 5 55 55
(In this way we can show that any “repeating decimal” is a rational number.)
X

(x + 3)n
[1, Exercise 50, p. 695] Find the values of x for which the series converges. Find
n=0
2n
the sum of the series for those values of x.
X∞ !n
(x + 3)
Solution: The given series can be rewritten as : it can be viewed as a
n=0
2
x+3
geometric series with initial term 1, and common ratio . We know that a geometric
2
series converges if and only if its common ratio is less than 1 in magnitude. Thus the
x + 3
given series converges iff < 1. Solving this inequality for x we obtain
2

x + 3 < 1 ⇔ −1 < x + 3 < +1
2 2
⇔ −2 < x + 3 < 2
⇔ −5 < x < −1 .
Thus the given series converges for and only for x ∈ (−5, −1). For x in this interval the
sum of the series is
first term 1 2
= x+3
=− .
1 − common ratio 1 − 2 1+x
Information for Students in Lecture Section 002 of MATH 141 2009 01 2193

B.20.2 §11.3 The Integral Test and Estimates of Sums


Thus far we have met only one “test” that can be applied to series to investigate whether the
series converges. That is the test Stewart calls “The Test for Divergence”, and it gives negative
information: it states that, if lim an does not exist, or if the limit does exist but its value is
n→∞
X

not 0, then the series an is divergent; this test cannot be used to confirm a suspicion that a
n=0
series does converge. This is the only test we shall have that can be applied to “general” series,
where the signs of the terms do not follow a specific pattern.

X

The Integral Test. The idea of this test is to interpret the terms of a series an as a sum
n=1
of areas. We can do this by interpreting an as the area of a rectangle whose width is 1, and
whose height is an . The test will be restricted to sequences where the terms are monotonely
decreasing. Suppose that we know of a function f defined on the interval [1, ∞), whose graph
passes through the left upper end-points of the rectangles. Then the area under the curve will
be less than the sum of the areas of the rectangles. If we can show that the area under the curve
is infinite, we will be able to conclude that the sum of the series is divergent. In a similar way,
if we can pass the graph of a function f through the right upper endpoints of the rectangles,
then the area under the curve will exceed the sum of the areas of the rectangles: if the area
under the curve is convergent, then the same can be said about the series. In these ways we
can shown that the series converges precisely when the integral representing the area under
the curve converges! We call this result

Theorem B.87 (The Integral Test) If f is a continuous, positive, decreasing54 function de-
fined on the interval [1, ∞), and if the sequence {an } has the property that an = f (n). Then

X
∞ Z∞
an is convergent ⇔ f (x) dx is convergent .
n=1
1

The convergence or divergence of both the series and the improper integral do not depend on
the lower limits of the sum and the definite integral. (However, when, below, we investigate
bounds for the actual value of the sum of the series, then our bounds will depend on the specific
values we choose for the limits of sum and integral.)
54
Actually, it suffices for the function to be non-increasing: we can permit the function values to stay at the
same level so long as they eventually decrease again and eventually approach 0.
Information for Students in Lecture Section 002 of MATH 141 2009 01 2194

B.20.3 Sketch of Solutions to Problems on the Final Examination in MATH 141 2005
01
(not for discussion at the lecture)

The best way to prepare for the examination in MATH 141 2009 01 is to study the textbook,
working problems, and verifying your solutions using the Student Solutions Manual: for sev-
eral reasons I don’t recommend your studying exclusively from old examinations. However
the old examinations do have some uses, and I am including one of them here, with solutions,
as a timely reminder that less than 1 month remains between now and the final examination in
this course. I am hoping to discuss last April’s examination briefly in a final lecture; there will
not be time for me to discuss the following examination in the lectures.
1. SHOW ALL YOUR WORK!
Z 3
(a) [4 MARKS] Evaluate |x − 1| dx .
0
Solution:
Z 3 Z 1 Z 3
|x − 1| dx = |x − 1| dx + |x − 1| dx
0 0 1
Z 1 Z 3
= (1 − x) dx + (x − 1) dx
0 1
" #1 " 2 #3
x2 x
= x− + −x
2 0 2 1
! !!
1 3 1 5
= −0 + − − =
2 2 2 2
Z 5√
d
(b) [3 MARKS] Evaluate 4 + t2 dt .
dx x
Z 5√ Z x√ √
d 2
d
Solution: 4 + t dt = − 4 + t2 dt = − 4 + x2 .
dx x dx 5
Z x2
d
(c) [3 MARKS] Evaluate sec t dt .
dx 2π
Solution: Let u = x2 . By the Chain Rule
Z x2 Z u
d d du
sec t dt = sec t dt · = sec u · 2x = sec(x2 ) · 2x .
dx 2π du 2π dx
Information for Students in Lecture Section 002 of MATH 141 2009 01 2195

Z  √3 
(d) [3 MARKS] Evaluate x5 x3 + 1 dx .
 1
Solution: Let u = x3 + 1 3 , so u3 = x3 + 1, 3u2 du = 3x2 dx.

Z Z    37   43
3 3
 √3   7
u u4 x + 1 x + 1
x5 x3 + 1 dx = u3 − 1 ·u·u2 du = − +C = − +C
7 4 7 4

2. SHOW ALL YOUR WORK!


For each of the following series you are expected to apply one or more tests for conver-
gence or divergence and determine whether the series is convergent. In each case you
must answer 3 questions:

• Name the test(s) that you are using.


• Explain why the test(s) you have chosen is/are applicable to the given series.
• Use the test(s) to conclude whether or not the series is convergent.
X

2 − cos n
(a) [4 MARKS]
n=2
n
1 2 − cos n
Solution: This is a positive series. Since 0 < ≤ , for all n, the terms
n n
are bounded below by the terms of the harmonic series, a positive series known
to diverge. We may apply the Comparison Test to such pairs of series, and may
conclude that the given series also diverges.
X∞
n(−3)n
(b) [4 MARKS]
n=0
4n
Solution: As formulated in your textbook, we may apply the Ratio Test to the
sequence of absolute values of ratios of terms to their predecessors. Here
(n+1)(−3)n+1 !
4n+1 1 3 3
lim n(−3)n = lim 1 + · = < 1,
n→∞ n→∞ n 4 4
n 4

from which we may conclude that the given series is (absolutely) convergent.
X ∞
1
(c) [4 MARKS]
n=2
n ln n
Solution: Define f (x) = x ln1 x . Then f is a positive, continuous function, taking
on the values of the given sequence at the positive integer points. By the Integral
Information for Students in Lecture Section 002 of MATH 141 2009 01 2196

Test, the series and the following improper integral will either both converge or
both diverge.
Z ∞ Z b
dx dx
= lim = lim [ln ln x]b2 = lim ln ln b − ln ln 2
2 x ln x b→∞ 2 x ln x b→∞ b→∞

which approaches +∞ as n → ∞. Hence the series diverges (to ∞).

3. BRIEF SOLUTIONS Express the value of each of the following as a definite integral
or a sum, product, or quotient of several definite integrals, but do not evaluate the inte-
gral(s). It is not enough to quote a general formula: your integrals must have integrand
and limits specific to the given problems:

(a) [6 MARKS] The area of the region bounded by the parabola y = x2 , the x-axis, and
the tangent to the parabola at the point (1, 1).  
Solution: The tangent to the parabola at the point x, x2 has slope 2x; at (1, 1) the
slope is 2. The equation of the tangent
  is y = 1 + 2(x − 1) = 2x − 1, which line
intercepts the x-axis in the point 12 , 0 .
If we evaluate the area by integration with respect to x, we have
Z 1 Z 1  " 3 # 12 " #1
2
2 2 x (x − 1)3 1 1
x dx + x − (2x − 1) dx = + = +
0 1
2
3 0 3 1 24 24
2

1
=
12
We can also integrate with respect to y:
Z 1 ! " #1
√ y+1 2 32 y2 y 2 1 1 1
− y+ dy = − y + + =− + + = .
0 2 3 4 2 0 3 4 2 12

(Note that you were not expected to evaluate the integrals: I did so in order to use
the opportunity of having 2 methods in order to verify my work.)
(b) [3 MARKS] The volume of the solid obtained p by rotating about the line y = 4 the
region bounded by x = 0 and the curve x = sin y (0 ≤ y ≤ π).
Solution: This problem is from your textbook [7, Exercise 25, p. 459], and is solved
in the Student Solution Manual, [9, p. 269]; it is one of the problems for which the
CD-Roms accompanying the textbook provide extra help.
Integrating by the method
Z π of Cylindrical Shells centred on the line y = 4, we obtain
p
the volume to be 2π (4 − y) sin y dy.
0
Information for Students in Lecture Section 002 of MATH 141 2009 01 2197

(c) [3 MARKS] The area of the surface obtained by revolving about the y-axis the
curve y = e x , 1 ≤ y ≤ 2.
Z ln 2 √
Solution: Integrating with respect to x gives the integral 2πx 1 + e2x dx. Inte-
Z 2 s 0
1
grating with respect to y gives the integral 2π(ln y) 1 + 2 dy.
1 y
2x
(d) [2 MARKS] The average value of the function over the interval 0 ≤ x ≤
(1 + x2 )2
2. R 2 2x
R 2 0 (1+x2 )2
dx
Solution: The answer expected was either 12 0 (1+x 2x
2 )2 dx or R2 . The inte-
0
1 dx
gral can be evaluated by using substitutions like u = x2 , or u = x2 + 1.
4. SHOW ALL YOUR WORK!
[12 MARKS] Evaluate the indefinite integral
Z
2x3 + 3x2 + 3
dx .
x2 + x − 12

Solution: Since the degree of the numerator of the rational function which is the inte-
grand is not less than the degree of the denominator, the first step is to divide denominator
into numerator, obtaining
2x3 + 3x2 + 3 23x + 15
2
= (2x + 1) + 2 .
x + x − 12 x + x − 12
R
The polynomial quotient may be integrated to (2x + 1) dx = x2 + x + C. The excess
must be expanded using the method of Partial Fractions. Assuming an expansion of the
form
23x + 15 A B
= + ,
(x + 4)(x − 3) x + 4 x − 3
we multiply both sides by (x+4)(x−3) to obtain the identity 23x+15 = A(x−3)+B(x+4).
We may obtain 2 equations for A, B by assigning to x the successive values x = 3 and
x = 4, obtaining B = 12 and A = 11. Hence
Z Z !
2x3 + 3x2 + 3 11 12
dx = 2x + 1 + +
x2 + x − 12 x+4 x−3
= x2 + x + 11 ln |x + 4| − 12 ln |x − 3| + C ,

which I then checked by differentiation.


Information for Students in Lecture Section 002 of MATH 141 2009 01 2198

5. SHOW ALL YOUR WORK!

(a) [9 MARKS] Use integration by parts to


prove that, for integers
m ≥ 2, Z Z
m 1 m−1 m−1
cos x dx = cos x · sin x + cosm−2 x dx
m m
Solution: (The instructions asked that the problem be solved using Integration by
Parts. Otherwise, the reduction formula could also have been proved by differenti-
ating both sides and showing that the same derivative was obtained.)
Let u = cosm−1 x, dv = cos(x) · dx, so du = (m − 1) cosm−2 x · (− sin x) dx and
v = sin x. Then
Z Z
cos x dx = cos x · sin x + sin2 x(m − 1) cosm−2 x dx
m m−1

Z  
m−1
= cos x · sin x + (m − 1) cosm−2 x − cosm x dx
Z m Z
⇒m x dx = cos x · sin x + (m − 1) cosm−2 x dx
m−1

Z Z
m 1 m−1 m−1
⇒ cos x dx = cos x · sin x + cosm−2 x dx .
m m

(b) [3 MARKS] Showing all your work, use the formula you have proved to evaluate
Z π2
cos6 x dx.
0
Solution:
Z
6 1 5 5
cos x dx = cos x · sin x + cos4 x dx
6 6
Z !
1 5 5 1 3 3 2
= cos x · sin x + cos x · sin x + cos x dx
6 6 4 4
! Z !
1 5 5 3 5 1 1
= cos x + cos x sin x + cos x · sin x + dx
6 24 8 2 2
!
1 5 5 3 5 5
= cos x + cos x + cos x sin x + x + C .
6 24 16 16

π 5π 5π
Evaluating between 0 and we obtain 0 + = .
2 32 32

6. SHOW ALL YOUR WORK!


Information for Students in Lecture Section 002 of MATH 141 2009 01 2199

Consider the curve C defined by

x = 2 cos t − cos 2t
y = 2 sin t − sin 2t .

(a) [8 MARKS] Determine the points where the arc of the curve given by
π 7π
≤t≤
4 4
has a vertical tangent.
Solution:
dx
= −2 sin t + 2 sin 2t
dt
dy
= 2 cos t − 2 cos 2t
dt
dy dx dy
Tangents are vertical where dx
is infinite, i.e., where dt
= 0 but dt
, 0.
dx
= 0 ⇔ −2 sin t + 4 sin t · cos t = 0
dt !
1
⇔ sin t cos t − =0
2
1
⇔ sin t = 0 or cos t =
2
π 5π
⇔ t = π, ,
3 3
π 7π
for t restricted to lie in the interval ≤ t ≤ . We check the values of dy
dt
at these
4 4
three points, and find that the respective values are −4, 2, 2, none of which is 0.
Hence the tangents will be vertical at the following points:

t=π (x, y) = (−3, 0)


 √ 
π  3 3 
t= (x, y) =  , 
3 2 2
 √ 
5π  3 − 3 
t= (x, y) =  , − 
3 2 2

(b) [4 MARKS] Determine the length of the arc of the curve given by

0 ≤ t ≤ 2π .
Information for Students in Lecture Section 002 of MATH 141 2009 01 2200

Solution:
!2 !2
dx dy
+ = (−2 sin t + 2 sin 2t)2 + (2 cos t − 2 cos 2t)2
dt dt
= 4 + 4 − 8(sin t · sin 2t + cos t · cos 2t)
= 8 − 8 cos(2t − t) = 8(1 − cos t) .
The length of the arc is
r
√ Z 2π √ √ Z 2π t
8 1 − cos t dt = 8 2 sin2 dt
0 0 2
Z 2π
= 4 sin t dt
2
0
Z 2π 
t
= 4 sin dt
0 2
 t 2π
= −8 cos = −8(−1 − 1) = 16 .
2 0
7. SHOW ALL YOUR WORK!

(a) [5 MARKS] Determine whether the following integral is convergent; if it is con-


vergent, determine its value: Z 1
dx

−1 1 − x2
Solution: The integral is improper because the integrand is not defined — becomes
infinite — at both ends of the interval of integration. According to the definition,
we must integrate between points away from ±1, and allow the limit to be taken
independently. The safest way to do that is to split the improper integral into two,
and then to evaluate the two of them separately.
Z 1 Z 0 Z b
dx dx dx
√ = lim+ √ + lim− √
−1 1 − x2 a→−1 a 1 − x2 b→+1 0 1 − x2
= lim+ (arcsin 0 − arcsin a) + lim− (arcsin b − arcsin 0)
a→−1 b→+1
 π π
= 0− − + − 0 = π.
2 2
(b) [5 MARKS] Determine whether the following series is conditionally convergent,
absolutely convergent, or divergent.
X

n!
(−1)n
n=1
nn
Information for Students in Lecture Section 002 of MATH 141 2009 01 2201

Solution: I will apply the Ratio Test to the sequence of absolute values of the ratios
(−1)n+1 (n+1)! !−n
(n+1)n+1 (n + 1)! nn 1 1
of terms to their predecessors: n!
= n+1
· = 1+ →
(−1) n (n + 1)
nn
n! n e
as n → ∞. As this limit is less than 1, the given series is absolutely convergent,
hence divergent.
(c) [3 MARKS] Determine whether the sequence an = ln(n + 1) − ln n is convergent;
if it is convergent, carefully determine its limit.
Solution: This is not a question about telescoping series! It is a problem about
sequences!
!  
1
ln(n + 1) − ln n = ln 1 + . As n → ∞, 1 + 1n → 1, so ln 1 + n1 → ln 1 = 0 (by
n
continuity of the logarithm function from the right at the point 1).

8. SHOW ALL YOUR WORK!


[12 MARKS] Find the area of the region bounded by the curves

r = 4 + 4 sin θ
r sin θ = 3

which does not contain the pole.


Solution: The problem is to determine the area cut off from the cardioid r = 4 + 4 sin θ
by the line y = 3. First we must determine where the curves cross. Solving the polar
equations by eliminating sin θ between them we obtain r2 − 4r − 12 = 0, equivalent
to (r − 6)(r + 2) = 0. The values r = 6, −2 yield corresponding values sin θ = 12 , − 32 .
Of these the second is impossible, as a sine cannot be less than 1. We conclude that
sin θ = 12 and
 the values of θ in the interval
 0 ≤ θ ≤ 2π are θ = π6 , 5π
6
: the curves intersect
π 5π
in (r, θ) = 6, 6 and (r, θ) = 6, 6 . A naive way to solve the problem is to integrate
directly,
Z 5π
 
1 6
Area = (4 + 4 sin θ)2 − (3 csc θ)2 dθ
2 π
6
Z 5π6 ! !
1 1 − cos 2θ 2
= 16 + 2 sin θ + 1 − 9 csc θ dθ
2 π6 2
1 5π
= [24θ − 32 cos θ − 4 sin 2θ + 9 cot θ] π6
2 √
6

= 8π + 12 3 .
Information for Students in Lecture Section 002 of MATH 141 2009 01 2202

Another way to find the area is to observe that the triangle with base √ on y = 3√between
1
the points of intersection and its third vertex at the pole has area 2 (6 3)·3 = 9 3. Then
Z 5π6
1
one can simply subtract this area from (4 + 4 sin θ)2 dθ.
2 π6
Information for Students in Lecture Section 002 of MATH 141 2009 01 2203

B.21 Supplementary Notes for the Lecture of March 18th, 2009


Release Date: Wednesday, March 18th, 2009
(subject to revision)

Review of the preceding lecture


X

• Theorem: ai converges ⇒ lim an = 0
n→∞
i=0

• The contrapositive of this theorem is “The” Test for Divergence: If lim an either does
n→∞
X

not exist, or does exist but does not equal 0, then the series ai diverges.
i=0

• The harmonic series diverges — proved by grouping successive terms.

• “telescoping” series

• evaluating a “repeating” decimal expansion as a rational number

• The Integral Test — requiring the association with a positive series of a continuous,
decreasing, positive function f taking, at the positive integer points, values equal to the
terms of the series

• Appended to my online notes for the last lecture is the Final Examination in MATH 141
2005 01, with a sketch of solutions. I do not intend to discuss this examination in the
lectures, but do plan to discuss the Final Examination in MATH 141 2008 01.

B.21.1 §11.3 The Integral Test and Estimates of Sums (conclusion)


Tests of Positive Series for convergence. Most of the tests we shall meet require a specific
arrangement of signs for the terms. In fact, all but one of them will require that all the signs
are the same. We call such series “positive” series, thinking of all the signs as being +; but
similar results hold when all signs are −. The first test of positive series will be discussed in
this section. I do not think of this as the most elementary of the tests, and would have preferred
to discuss the material of the next sections first; but I will grudgingly follow the order of topics
in the textbook.
Information for Students in Lecture Section 002 of MATH 141 2009 01 2204

“Ultimate” satisfaction of properties imposed on the terms of a series. In the statement


given for the a sum beginning with the 1st term, and compared it with the integral whose
lower bound was x = 1. Both of these lower bounds can be replaced by any integers you
like, provided the series and function are defined whenever you refer to them. Changing these
lower bounds does not change the truth of the logical equivalence. It could, however, change
the value of the sum of the series or the definite integral.

X 1
Application: the “p-series”. The “p-series” are the series , where p is a positive
np
constant. The integral test shows that
X 1
is convergent if and only if p > 1.
np
The case p = 1 is the harmonic series, which we have already shown to be divergent, using
a different proof. (This application is sometimes called the “p-series test”.) Since the tests
we will meet in [1, §11.4] involve comparing a given series with series that we know to be
convergent and divergent, the p-series are particularly important as they provide us with a
family of series that can be used for comparison purposes.

Estimating the Sum of a Series In the proof of the Integral Test we compared the sum of a
series of decreasing positive terms with the area under the graph of a function that is positive
and decreasing. This comparison can be refined, and gives rise to inequalities that bound the
value of the sum from below and above.
P
By associating with a decreasing, positive series an a decreasing positive function f , we
X
N
are able to “trap” the value of the partial sums an between the definite integrals giving the
n=1
areas for regions respectively contained in and containing the region represented by the partial
sum, when interpreted as the area of a region formed by rectangles of width 1 and respective
heights 1, 2, . . ., N. From the inequalities

X
N Z N
an ≤ a1 + f (x) dx
n=1 1

X
N Z N+1
an ≥ f (x) dx
n=1 1

we
Z ∞are able to relate the convergence of the infinite sum to that of the improper integral
f (x) dx.
1
Information for Students in Lecture Section 002 of MATH 141 2009 01 2205

Let us, extending the notation used in the enunciation of the theorem, define
X

Rn = am ;
m=n+1

that is, Rn is the sum of the “tail” or Remainder of the series, starting with the (n + 1)st term.
Then Z ∞ Z ∞
f (x) dx ≤ Rn ≤ f (x) dx ,
n+1 n
or, alternatively, Z Z
∞ ∞
f (x) dx ≤ Rn ≤ an+1 + f (x) dx ,
n+1 n+1
or Z ∞
0 ≤ Rn − f (x) dx ≤ an+1 .
n+1
In particular, when n = 0, this gives the inequalities
X∞ Z ∞
0≤ an − f (x) dx ≤ a1 .
n=1 1

While, for the purposes of testing convergence, it is sufficient to demand “ultimate” sat-
isfaction of the conditions of the Integral Test, this is not sufficient if we wish to determine
bounds for the sum. In that case the estimations discussed above depend on the comparison of
the area under a curve and the sum of the areas of the step function that represents the series;
these comparisons are valid only when the condition the the function be decreasing is satisfied.
It is possible to refine this type of bounding of partial sums and remainders, as in [1, Examples
5,6, pp. 701-702], but we will not go further in this course.
1 X

Example B.88 We proved earlier that the telescoping series converges, and we
n=1
n(n + 1)
found the sum. The integral test could be used to prove that it converges, but the test would
not give the exact value of the sum, although it would provide bounds which compare the sum
to the improper integral
Z ∞
1
dx = lim ((ln a − ln 1) − (ln(a + 1) − ln 2))
1 x(x + 1) a→∞
 a 
= lim ln + ln 2
a→∞ a+1
= ln 1 + ln 2 = 0 + ln 2 = ln 2 ,
so the sum of the series is bounded between ln 2 = 0.6931471806 and ln 2+ 12 = 1.1931471806,
which is a weaker result than the fact we already know that the sum is equal to 1.
Information for Students in Lecture Section 002 of MATH 141 2009 01 2206

11.3 Exercises
X

1
[1, Exercise 6, p. 703] Use the Integral Test to determine whether the series √ is
n=1 n+4
convergent or divergent.
Solution: We begin by verifying that the Integral Test is applicable to this series:

• Since the terms are reciprocals of square roots, they are positive.
1
• The function f (x) = √ is decreasing. This can be shown either by showing
x+1
3
that f 0 (x) = − 12 (x + 1)− 2 < 0, or by the following reasoning:
x is an increasing function of x
⇒ x√+ 1 is an increasing function of x

⇒ x + 1 is an increasing function of x since preserves order
1
⇒ √ is a decreasing function of x.
x+1
Z∞
dx
The Integral Test tells us that the series will converge iff the improper integral √
x+1
1
converges. But
Z∞ Za h √ ia
dx dx
√ = lim √ = lim 2 x + 1 = +∞ .
a→∞ a→∞ 1
x+1 x+1
1 1

so the series must also diverge.


X

1
[1, Exercise 20, p. 704] To investigate the convergence of .
n=1
n2 − 4n + 5
Solution: We know that
Z Z
1 1
2
dx = dx = arctan(x − 2) + C ,
x − 4x + 5 (x − 2)2 + 1
Z ∞
1
and might be tempted to compare the integral 2
dx with the series. This,
1 x − 4x + 5
however, is not directly possible from the Integral Test, because the function f (x) =
1
2
is decreasing only for x ≥ 2: from x = 1 to x = 2 the function is increasing.
x − 4x + 5
Information for Students in Lecture Section 002 of MATH 141 2009 01 2207

X

1
Thus we can apply the integral test to the series , provided we first verify
n=2
x2
− 4x + 5
that the necessary conditions are satisfied by the function f whose values at the positive
integers coincide with the terms of the series:
1. Is f decreasing?
d −(2x − 4)
f (x) = 
dx x2 − 4x + 5
−2(x − 2)
=  .
(x − 2)2 + 1 2
The numerator is negative for x > 2; the denominator is a non-zero square, always
positive. Thus the ratio is negative for x > 2, and so f is decreasing for x > 2.
2. Is f continuous? f is a rational function, and [1, Theorem 5(b), p. 122] all rational
functions are continuous on their domain. (The domain of f is all of R.)
Bounding the sum for n ≥ 2 from below yields
X ∞ Z ∞
1 dx

n=2
n2 − 4n + 5 2 x2 − 4x + 5
= lim arctan(a − 2) − arctan 0
a→∞
π π
= −0= .
2 2
Bounding the sum for n ≥ 3 from above yields
X∞ Z ∞
1 dx

n=3
n2 − 4n + 5 2 x2 − 4x + 5
π
= ,
2
which implies that
X
∞ Z ∞
1 1 dx π
≤ + = 1 + .
n=2
n2 − 4n + 5 22 − 4(2) + 5 2 x2 − 4x + 5 2

X

1 1+π
Thus the series converges, and its sum is bounded between and
n=1
n2 − 4n + 5 2
3+π
. (If we were to add more of the terms at the beginning of the series manually, we
2
could determine much “tighter” bounds for the sum of the series.)
Information for Students in Lecture Section 002 of MATH 141 2009 01 2208

X

1
[1, Exercise 28, p. 704] Find the values of p for which the series is con-
n=3
n ln n[ln(ln n)] p
vergent.
Solution: The denominators of the fractions being summed are products of n, ln n, and
a power of ln ln n, all of which are increasing functions of n. Hence we may apply the
1
Integral Test with the function f (x) = , confident that it is decreasing
x(ln x) ([ln(ln x)] p )
and continuous. When x ≥ 3, ln x > 1.098 > 1, so ln ln x > 0, and f (x) > 0. When
p = 1,
Za
dx
= [ln ln ln x]a3 = ln ln ln a − ln ln ln 3 → ∞
x ln x[ln(ln x)]1
3

as a → ∞; hence, for the series to converge, we must certainly have p > 1; (any positive
power of p less than 1 will yield an integral which is larger than the integral we obtain
when p = 1). But, when p > 1,
Za " #a
dx 1 1 1 1
p
=− · p−1

x ln x[ln(ln x)] p − 1 (ln ln ln x) 3 p − 1 ln ln ln 3
3

as a → ∞; since the improper integral is convergent for 0 < p < 1, the positive series is
also convergent. Thus the given series converges for 0 < p < 1, and diverges for p ≥ 1
— the same values for the exponent as for p-series.

B.21.2 §11.4 The Comparison Tests


We continue to study series in a “reverse-logical” order. That is, as we proceed, we will be
encountering properties that are more and more basic. In this section we will meet another
theorem that — like the Integral Test — applies only to series whose terms all have the same
sign, the so-called “positive series”, or “series of positive terms”. Finally, in [1, §11.6] we will
see a theorem [1, Theorem 3, p., 715] that will justify our continued investigation of series
with positive terms. When we have covered all the sections on the syllabus, I hope to return to
the various tests.
The comparison tests are designed so that we can infer the convergence or divergence of
a given positive series by comparison of its terms with those of another series that we know
to be convergent or divergent. So, in order to use this test, we need to have available as large
as possible a family of positive series whose convergence or divergence are known. Let’s
remember for which series we know such facts; these include

• positive geometric series of common ratio less than 1 (convergent)


Information for Students in Lecture Section 002 of MATH 141 2009 01 2209

• positive geometric series of common ration 1 or more (divergent)

• harmonic series (divergent)

• p-series (convergent if and only if p > 1)

• positive series whose terms do not approach 0 (divergent)

• series obtained from a given series by deleting any finite number of terms at the begin-
ning (have the same convergence properties as the original series)

• series obtained from a given series by multiplying by a positive constant (have the same
convergence properties as the original series)
X
Theorem B.89 (Comparison Test) Let an be a series with positive terms.
X
1. If bn is a series of positive terms such that bn ≤ an for all n, then
X X
an converges ⇒ bn converges .
X
2. If bn is a series of positive terms such that an ≤ bn for all n, then
X X
an diverges ⇒ bn diverges .

3. In either of the preceding results, the words “for all n” may be replaced by “for all n
sufficiently large”, which means, in mathematical jargon, that all we require is that the
inequality holds after some integer, which could be as large as you like; if the inequality
fails a finite number of times, even an enormously large finite number of times, that is
sufficient for the purposes of the hypotheses of the theorem.
P
Prior to applying this test we may first adjust the series an that we are using for comparison
purposes by

• deleting or otherwise changing a finite number of terms at the beginning;

• multiplying by a positive constant


Information for Students in Lecture Section 002 of MATH 141 2009 01 2210

B.21.3 Sketch of Solutions to Problems on the Final Examination in MATH 141 2006
01
(not for discussion at the lecture)

The best way to prepare for the examination in MATH 141 2009 01 is to study the textbook,
working problems, and verifying your solutions using the Student Solutions Manual: for sev-
eral reasons I don’t recommend your studying exclusively from old examinations. However
the old examinations do have some uses, and I am including a second one of them here, with
solutions. I am hoping to discuss last April’s examination briefly in a final lecture; there will
not be time for me to discuss the following examination in the lectures.

1. SHOW ALL YOUR WORK!


For each of the following series you are expected to apply one or more tests for conver-
gence or divergence and determine whether the series is convergent. In each case you
must answer 3 questions:

• Name the test(s) that you are using.


• Explain why the test(s) you have chosen is/are applicable to the given series.
• Use the test(s) to conclude whether or not the series is convergent.
X

1
(a) [4 MARKS]
n=1
(tanh n)2 + 1
n −n −2n
Solution: As n → ∞, tanh n = een −e +e−n
1−e
= 1+e 1
−2n → 1 = 1 , so the terms summed in

this series have limit 121+1 = 12 , 0. By the “Test for Divergence” this series cannot
converge.
X∞
2
(b) [4 MARKS] n2n e−n
n=1
Solution:
pn n2
nth root of nth term = n2n e−n2 = n2 e−n = n .
e

By l’Hospital’s Rule ( ∞ case) the limit of this nth root as n → ∞ is

n2 2n 2
lim = lim = lim = 0 < 1.
n→∞ en n→∞ en n→∞ en

By the Root Test, the series is convergent.


Information for Students in Lecture Section 002 of MATH 141 2009 01 2211

X

n2 − 85n + 12
(c) [4 MARKS]
n=1
n(n + 6)2
Solution: The ratio of the nth term of this series to the nth term of the Harmonic
P

1
series n
is
n=1
85 12
n2 − 85n + 12 1 − n + n2
=   → 1 , 0.
(n + 6)2 6 2
1+ n
By the Limit Comparison Test the given series is divergent, since the Harmonic
series is divergent.

2. SHOW ALL YOUR WORK!


Z 2
(a) [4 MARKS] Evaluate |x| dx .
−1
Solution:
Z 2 Z 0 Z 2
|x| dx = |x| dx + |x| dx
−1 −1 0
Z 0 Z 2
= (−x) dx + x dx
−1 0
" 2 #0 " 2 #2
x x
= − +
2 −1 2 0
1 5
= +2=
2 2

Ze3
dt
(b) [3 MARKS] Evaluate √ .
t 1 + ln t
1
dt
Solution: Under the substitution u = ln t, du = t
and

Ze3 Z 3 i √ √
dt du 1 3
√ = √ = 2(1 + u) 2 = 2( 4 − 1) = 2(2 − 1) = 2
0
t 1 + ln t 0 1+u
1

Z x2
d 2
(c) [3 MARKS] Evaluate et dt .
dx 0
Information for Students in Lecture Section 002 of MATH 141 2009 01 2212

Solution: With u = x2 ,
Z x2 Z u
d d 2 2
t2
et dt = eu 2x = e( x ) 2x = 2xe x ,
2 2 4
e dt =
dx 0 dx 0
by the Fundamental Theorem of Calculus.
(d) [4 MARKS] Evaluate
 !7 !7 !7 !7 
1  0 1 2 n − 1 
lim  + + + ... +  .
n→∞ n n n n n
R1
Solution: You are asked to evaluate the limit of a Riemann sum for 0 x7 dx =
h 8 i1
x
8 0
= 18 , (where the values of the function are taken at the left end-point of each
subinterval). Accordingly the value is 81 .

SHOW ALL YOUR WORK!

3. For each of the following series you are expected to apply one or more tests for conver-
gence or divergence and determine whether the series is convergent. In each case you
must answer 3 questions:
• Name the test(s) that you are using.
• Explain why the test(s) you have chosen is/are applicable to the given series.
• Use the test(s) to conclude whether or not the series is convergent.
X

1
(a) [4 MARKS]
n=1
(tanh n)2 + 1
n −n −2n
Solution: As n → ∞, tanh n = een −e +e−n
1−e
= 1+e 1
−2n → 1 = 1 , so the terms summed in

this series have limit 121+1 = 12 , 0. By the “Test for Divergence” this series cannot
converge.
X∞
2
(b) [4 MARKS] n2n e−n
n=1
Solution:
pn n2
nth root of nth term = n2n e−n2 = n2 e−n = n .
e
By l’Hospital’s Rule ( ∞

case) the limit of this nth root as n → ∞ is
n2 2n 2
lim = lim n = lim n = 0 < 1 .
n→∞ en n→∞ e n→∞ e

By the Root Test, the series is convergent.


Information for Students in Lecture Section 002 of MATH 141 2009 01 2213

X

n2 − 85n + 12
(c) [4 MARKS]
n=1
n(n + 6)2
Solution: The ratio of the nth term of this series to the nth term of the Harmonic
P

1
series n
is
n=1
85 12
n2 − 85n + 12 1 − n + n2
=  2 → 1 , 0 .
(n + 6)2 1 + 6n
By the Limit Comparison Test the given series is divergent, since the Harmonic
series is divergent.

4. BRIEF SOLUTIONS Express the value of each of the following as a definite integral
or a sum, product, or quotient of several definite integrals, but do not evaluate the inte-
gral(s). It is not enough to quote a general formula: your integrals must have integrand
and limits specific to the given problems, and should be simplified as much as possible,
except that you are not expected to evaluate the integrals.
(a) [3 MARKS] Expressed as integral(s) along the x-axis only, the area of the region
bounded by the parabola y2 = 2x + 6 and the line y = x − 1. An answer involving
integration along the y-axis will not be accepted.
DEFINITE INTEGRAL(S) ONLY (DO NOT EVALUATE)

Solution:
Z −1 p Z 5p 
2 2(x + 3) dx + 2(x + 3) − (x − 1) dx
−3 −1

(b) [3 MARKS] The volume of the solid obtained by rotating about the line y = 1 the
region bounded by the curves y = x3 and y = x2 . For this question you are to use
only the method of “washers”.
DEFINITE INTEGRAL(S) ONLY (DO NOT EVALUATE)
Information for Students in Lecture Section 002 of MATH 141 2009 01 2214

Solution: Only the first integral was required.


Z 1  2  2 
π 1 − x3 − 1 − x2 dx
0
Z 1 
= π −2x3 + x6 + 2x2 − x4 dx
0
" #1
2x4 x7 2x3 x5
= π − + + −
4 7 3 5 0
!
2 1 2 1 23π
= π − + + − = .
4 7 3 5 210
(c) [3 MARKS] The volume of the solid obtained by rotating about the line y = 1 the
region bounded by the curves y = x3 and y = x2 . For this question you are to use
only the method of “cylindrical shells”.
DEFINITE INTEGRAL(S) ONLY (DO NOT EVALUATE)

Solution: Only the first integral was required.


Z 1  1 
1
2π (1 − y) y 3 − y 2 dy
0
Z 1 
1 4 1 3
= 2π y 3 − y 3 − y 2 + y 2 dy
0
!
3 3 2 2 23π
= 2π − − + =
4 7 3 5 210
(d) [3 MARKS] The length of the curve whose equation is
x2 y2
+ = 1.
4 9
DEFINITE INTEGRAL(S) ONLY (DO NOT EVALUATE)
Information for Students in Lecture Section 002 of MATH 141 2009 01 2215

Solution: This is a special case of [7, Exercise 20, p. 553].

x 2 y2
+ = 1 ⇔ 9x2 + 4y2 = 36
4 9
dy 9x
⇒ =−
dx 4y
s p
!2
dy 81x2 + 16y2
⇒ 1+ =
dx 4|y|
Z 2 r
1 16 + 5x2
⇒ arc length = 2 dx
−2 2 4 − x2
where the factor 2 is needed because the integral is the length of either the top or
the bottom arc of the ellipse; this integral is improper. A cleaner solution is found
by parameterizing the curve as x = 2 cos θ, y = 3 sin θ (0 ≤ θ ≤ 2π). Then the arc
length is Z 2π p
4 sin2 θ + 9 cos2 θ dθ .
0

5. SHOW ALL YOUR WORK!


[12 MARKS] Evaluate the indefinite integral
Z 5
x +x
dx .
x4 − 16

Solution: The first step is to divide denominator into numerator:

x5 + x 17x
x5 + x = x(x4 − 16) + 17x ⇒ 4
= x+ 4 .
x − 16 x − 16
Next we could separate the resulting remainder fraction, whose numerator has degree
less than that of its denominator, into partial fractions:
17x Ax + B Cx + D
= + 2
x4− 16 x2 − 4 x +4
E F Cx + D
= + + 2
x−2 x+2 x +4
⇒ 17x = E(x + 2)(x + 4) + F(x − 2)(x2 + 4)
2

+Cx(x − 2)(x + 2) + D(x − 2)(x + 2)


Information for Students in Lecture Section 002 of MATH 141 2009 01 2216

We can determine the constants by a combination of the standard methods: giving x


convenient values, and equating coefficients of like powers of x:
17
x = 2 ⇒ 34 = 32E ⇒ E =
16
17
x = −2 ⇒ −34 = −32F ⇒ F =
16
x = 0 ⇒ 0 = 6E − 6F − 8D ⇒ D = 0
17
coeff x3 : ⇒ 0 = E + F + C ⇒ C = −
8
Hence !
x5 + x 17 1 1 2x
4
= x+ + − 2
x − 16 16 x − 2 x + 2 x + 4
Hence
Z
x5 + x x2 17 x2 − 4
dx = + ln +C
x4 − 16 2 16 x2 + 4
A better tactic would have been to make a substitution u = x2 in the integral prior to
separation into partial fractions:
Z Z Z !
17x 17 du 17 1 1 17 u − 4
dx = = − du = ln +C
x4 − 16 2 (u − 4)(u + 4) 16 u−4 u+4 16 u + 4

6. SHOW ALL YOUR WORK!


Showing all your work, evaluate each of the following:
Z
(a) [4 MARKS] cos x · cosh x dx
Solution: Using integration by parts,
u = cos x ⇒ du = − sin x dx
Z dv = cosh x dx ⇒ v = sinh x Z
⇒ cos x · cosh x dx = (cos x)(sinh x) + sinh x · sin x dx .

A second integration by parts yields


U = sin x ⇒ dU = cos x dx
Z dV = sinh x dx ⇒ V = cosh x Z
⇒ sin x · sinh x dx = (sin x)(cosh x) − cosh x · cos x dx
Z Z
⇒ cos x · cosh x dx = (cos x)(sinh x) + (sin x)(cosh x) − cos x · cosh x dx.
Information for Students in Lecture Section 002 of MATH 141 2009 01 2217

Now we can move the integral from the right side of the equation to join the same
integral with opposite sign on the left side, yielding
Z
2 cos x · cosh x dx = (cos x)(sinh x) + (sin x)(cosh x) + C

or Z
(cos x)(sinh x) + (sin x)(cosh x)
cos x · cosh x dx = + C0 .
2
Z1 √
(b) [5 MARKS] x2 + 2x + 5 dx
−3
Solution: We begin by completing the square:

Z1 √ Z1 p
x2 + 2x + 5 dx = (x + 1)2 + 4 dx .
−3 −3

Then the integrand can be changed to the form of X 2 + 1:
s
Z1 p Z1 !2
2
x+1
(x + 1) + 4 dx = 2 + 1 dx .
2
−3 −3

Here an appropriate substitution would be to set x+1


2
equal to either tan θ or sinh θ;
I have chosen to use the trigonometric substitution:
x+1 x+1
= tan θ ⇒ θ = arctan
2 2
2
⇒ dx = 2 sec θ dθ
Z1 √ Z π
4
⇒ 2
x + 2x + 5 dx = 2 | sec θ| · 2 sec2 θ dθ
− π4
−3
Z1 √ Z π
4
⇒ x2 + 2x + 5 dx = 8 sec3 θ dθ
0
−3

Finally we have to find an antiderivative of sec3 θ, which can be integrated by


“standard” methods, e.g., by integration by parts with u = sec θ, dv = sec2 θ dθ:

du = sec θ tan θ dθ
Information for Students in Lecture Section 002 of MATH 141 2009 01 2218

Z v = tan θ Z
3
sec θ dθ = sec θ tan θ − sec θ tan2 θ dθ
Z
= sec θ tan θ − sec θ(sec2 θ − 1) dθ
Z Z
3
= sec θ tan θ − sec θ dθ + sec θ dθ
Z
= sec θ tan θ − sec3 θ dθ + ln | sec θ + tan θ| + C
Z
⇒2 sec3 θ dθ = sec θ tan θ + ln | sec θ + tan θ| + C
Z
sec θ tan θ + ln | sec θ + tan θ|
⇒ sec3 θ dθ = + C0 .
2
Here
Z1 √ Z π
4 √ √
x2 + 2x + 5 dx = 8 sec3 θ dθ = 4 2 + 4 ln( 2 + 1) .
0
−3
Z
(c) [4 MARKS] sin2 x · cos2 x dx
Solution: One way to simplify this integral is to use the identity
sin 2x
sin x cos x = :
2
Z Z
2 1
sin x · cos x dx =2
sin2 2x dx
4
Z !
1 1 sin 4x
= (1 − cos 4x) dx = x− +C
8 8 4
x sin 4x
= − +C.
8 32
This could also have been simplified using the algorithm proposed in the textbook:
Z Z
2 2 1
sin x · cos x dx = (1 − cos 2x)(1 + cos 2x) dx
4Z
1
= (1 − cos2 2x) dx
4
Z !
1 1 + cos 4x
= 1− dx
4 2
Z
1
= (1 − cos 4x) dx etc.
8
Information for Students in Lecture Section 002 of MATH 141 2009 01 2219

7. SHOW ALL YOUR WORK!


Consider the curve C defined by

x = x(t) = 10 − 3t2
y = y(t) = t3 − 3t ,

where −∞ < t < +∞.


d2 y
(a) [8 MARKS] Determine the value of 2 at the points where the tangent is horizon-
dx
tal.
Solution:

dx
= −6t
dt
dy
= 3t2 − 3
dt
dy !
dy dt t2 − 1 1 1
= = =− t−
dx dx −2t 2 t
dt !
d2 y d dy
=
dx2 dx dx
!
d dy dt
= ·
dt dx dx
!
d dy
dt dx
=
dx
dt !!
d 1 1
− t−
dt 2 t
=
dx
dt !
1d 1
− t−
2 dt t
=
−6t
1
1 1 + 2
= − t2 = t + 1 .
2 −6t 12t3
Information for Students in Lecture Section 002 of MATH 141 2009 01 2220

dy
The tangent is horizontal when = 0, i.e., when t2 − 1 = 0 or t = ±1. At such
dx
parameter values
d2 y 2 1
2
= =± .
dx ±12 6
(b) [4 MARKS] Determine the area of the surface of revolution about the x-axis of the
arc n √ o
(x(t), y(t)) : − 3 ≤ t ≤ 0 .
Solution: The area of the surface of revolution about the x-axis is
s
Z 0 !2 !2 Z 0 p
dx dy
2π √ y(t) + dt = 2π √ (t3 − 3t) 36t2 + 9(t2 − 1)2 dt
− 3 dt dt − 3
Z 0
= 2π √ (t3 − 3t)(3t2 + 3) dt
− 3
Z 0
= 2π √ (t5 − 2t3 − 3t) dt
− 3
" #0
t6 2t4 3t2
= 6π − − = 27π .
6 4 2 √
− 3

8. SHOW ALL YOUR WORK!

(a) [5 MARKS] Showing detailed work, determine whether the following integral is
convergent; if it is convergent, determine its value:
Z 0
dx
2
.
−1 x 3

Solution: This integral is improper because the integrand is undefined, hence dis-
continuous at the right endpoint.
Z 0 Z b
dx dx
2
= lim− 2
by definition
−1 x 3 b→0 −1 x 3
h 1 ib 1
= lim− 3x 3 = 3 lim− b 3 − (−3) = 0 + 3 = 3.
b→0 −1 b→0

Since the limit exists, the improper integral is convergent.


(b) [5 MARKS] Determine whether the following series is conditionally convergent,
absolutely convergent, or divergent.
X∞
(−1)n
n=1
n − ln n
Information for Students in Lecture Section 002 of MATH 141 2009 01 2221

1
Solution: Let f (x) = . then
x − ln x

0
1 − 1x
f (x) = − .
(x − ln x)2
For x > 1, 1− 1x > 0, so f 0 (x) < 0. Thus the terms of the given series are monotonely
decreasing in magnitude, and alternating in sign.
We observe that
1
ln x
lim = lim x = 0;
x→∞ x x→∞ 1
it follows that the limit of the ratio of corresponding terms of the given series and
P
the series 1n is
1
n−ln n 1 1
lim 1 = lim ln n
= ln n
= 1.
n→∞
n
n→∞ 1− n 1 − lim
n→∞ n

Since the Harmonic Series diverges, this implies (by the Limit Ratio Test) that the
series of absolute values of the given series is divergent; and the given series is
either conditionally convergent or divergent. The limit of the terms of the series is
1 1 1
lim = lim · lim = 0 · 1 = 0.
n→∞ n − ln n n→∞ n n→∞ 1 − ln n
n

This, combined with the property that the terms of the given series are alternating
in sign and monotonely decreasing in magnitude, permits us to conclude by the
Leibniz Test that the series converges. Thus this alternating series is convergent,
but not absolutely convergent: it is conditionally convergent.
(c) [3 MARKS] Give an example of a sequence {an } with the property that lim an = 0
n→∞
X

but an = +∞. You are expected to give a formula for the general term an of
n=1
your sequence.
1
Solution: The Harmonic Series an = n
is one example of a series which has the
desired property.

9. SHOW ALL YOUR WORK!


[12 MARKS] The arc
r = 1 − cos θ (0 ≤ θ ≤ π)
divides the area bounded by the curve

r = 1 + sin θ (0 ≤ θ ≤ 2π)
Information for Students in Lecture Section 002 of MATH 141 2009 01 2222

1.5

0.5

0
-2 -1.5 -1 -0.5 0 0.5 1

Figure
  22: The curves with equations r = 1 − cos θ, (θ ≤ 0), and r = 1 + sin θ, and the point
1 π
,
2 2

into two parts. Showing


  all your work, carefully find the area of the part that contains
1 π
the point (r, θ) = 2 , 2 .
Solution: Since the functions defining the curves are both periodic with period 2π, we
can confine our attention to any parameter interval of length 2π. We need to determine
where the curves intersect. Solving the two given equations algebraically gives tan θ =
−1, implying that θ = 3π , 7π in the interval 0 ≤ θ ≤ 2π. The only point whose parameter
4 4
value is in the intervals given for the two arcs is θ = 3π4
. However, we always need to be
alert to the possibility that there are intersections of the curves in which the parameter
values could be different on the two curves. One place where precisely that occurs for
these two curves is the pole: on r = 1−cos θ the pole appears as (r, θ) = (0, 0), which
 is in

the restricted interval 0 ≤ θ ≤ π; on the cardioid r = 1 + sin θ the pole appears as 0, 3π
2
.
Information for Students in Lecture Section 002 of MATH 141 2009 01 2223

Could there be any other points of intersection? Any others could be found by solving
the variants of the equations obtainable by applying the transformation (r, θ) = (−r, θ+π)
repeatedly. We can show that there are no such points that appear in this way.
It follows that the region whose area we seek is bounded by two arcs:

• The arc of r = 1 − cos θ bounded by 0 ≤ θ ≤ 4
.
• One arc of r = 1 + sin θ: either − π2 ≤ θ ≤ 3π4
or 3π
4
≤ θ ≤ 3π2
. (In the first of these
π
I have, for convenience, used the negative value θ = − 2 . You might wish to object
that the prescribed interval for θ was 0 ≤ θ ≤ 2π. I could, instead have described
the first of these arcs as being made up of two pieces: one being 3π 2
≤ θ ≤ 2π, and
the second being 0 ≤ θ ≤ 3π 4
.)
 
But the point 21 , π2 is only in the region whose boundary contains the arc 3π4
≤ θ ≤ 3π 2
of
the curve r = 1 − cos θ.
 
To find the area, think of a line segment being drawn from the pole to 1 + √12 , 3π 4
. The
area is then the sum of the area below the line segment and the area above it, i.e.,
Z 3π Z 3π
1 4
2 1 2
(1 − cos θ) dθ + (1 + sin θ)2 dθ
2 0 2 3π
4
Z 3π ! Z 3π2 !
1 4 1 + cos 2θ 1 1 − cos 2θ
= 1 − 2 cos θ + dθ + 1 + 2 sin θ + dθ
2 0 2 2 3π4 2
" # 3π " # 3π
1 3θ sin 2θ 4 1 3θ sin 2θ 2
= − 2 sin θ + + − 2 cos θ −
2 2 4 0 2 2 4 3π
4

9π √ 1
= − 2− .
8 4
Information for Students in Lecture Section 002 of MATH 141 2009 01 2224

B.22 Supplementary Notes for the Lecture of March 23rd, 2009


Distribution Date: Monday, March 23rd, 2009, subject to further revision

Review of the last lecture


• After reviewing the previous lecture, I completed my treatment of the Integral Test.
• It time permits, I may return to further discussion of the use of integrals to bound series.
X

1
• Discussion included [1, Exercise 28, p. 704], , which we found to
m=1
n(ln n) (ln ln n) p
behave “similarly” to p-series.
• I introduced the Comparison Test(s), and mentioned only briefly the Limit Comparison
Test(s) (in the restricted form given in the body of the text, which requires that lim = L,
n→∞
where L is neither 0 nor +∞. These tests can be applied only to non-negative series, and
require having an appropriate series with which to compare. A series which is ultimately
larger than a positive, divergent series will also diverge; a non-negative series which is
ultimately smaller than a convergent, non-negative series will also converge. And these
conditions can be rephrased in terms of a limit, which must exist as a finite, positive
number.
• There are often many, significantly different choices of series with which to compare.
You need to work problems and check your solutions afterward in the manual to develop
the necessary experience.

B.22.1 §11.4 The Comparison Tests (conclusion)


The following related test is sometimes easier to use:
X X
Theorem B.90 (Limit Comparison Test) Let an be a series with positive terms. If bn
is a series of positive terms such that
an
lim =c>0
n→∞ bn

then X X
an diverges ⇔ bn diverges .
X X
an converges ⇔ bn converges .
(There is a sharper version of this theorem that permits the limit to be either 0 or ∞; but it is
delicate, and difficult for students to remember, so we usually do not discuss it in this course,
cf. [1, Exercises 40-42, p. 709].)
Information for Students in Lecture Section 002 of MATH 141 2009 01 2225

Estimating Sums This question will be illustrated by the solution below of part of [1, Exer-
cise 32, p. 709].
X

1 + 2n
Example B.91 ([7, Exercise 20, p. 735]) Determine whether the series converges
n=1
1 + 3n
or diverges.
2n
Solution: We could not compare the terms of this series naively with those of n , which is a
3
convergent geometric series because the terms here are slightly larger. However a number of
other possibilities suggest themselves. For example, we have
!n !n !n
1 + 2n 1 + 2n 1 2 2
≤ = + <2
1 + 3n 3n 3 3 3

which is the general term of a geometric series with common ratio 23 < 1, which is conver-
gent. Other comparisons are possible, some of them complicated to prove. For the purpose of
proving only convergence (and not bounding the series) one tries to find the simplest possible
comparison.
This problem can also be solved by applying the Limit Comparison Test, again in compar-
P  2 n
ison with 3
:

1 + 2n !
n 1 + 2n 3n
lim 1 + !3n = lim ·
n→∞ 2 n→∞ 2n 1 + 3n
3
 
= lim 2−n + 1 · 3−n + 1
n→∞
= 1 × 1 = 1.
P  2 n
Since the limit is a positive real number, and since the geometric series 3
converges, the
given series must also converge.

Example B.92 ([7, Exercise 34, p. 735]) Estimate the error when the sum of the first 10 terms
X∞
1 + cos n
of the following sum is used to approximate the sum of the series: .
n=1
n5
Solution: The “tail” of the series will be estimated from above. Observe that
1 + cos n 2
5
≤ 5.
n n
We know that
1 1
≤ for n − 1 ≤ x ≤ n.
n5 x 5
Information for Students in Lecture Section 002 of MATH 141 2009 01 2226

Comparing the sum of the terms on the left — interpreted as rectangles of width 1 lying under
the graph of f (x) = x15 — we have

X∞ Z ∞
1 dx 1
5
≤ 5
= 4.
n=11
n 10 x 10

It follows that the “tail” of the series — the sum beginning with term n = 11 — is no greater
than 2 × 10−4 . This gives an upper bound for the error if the series is truncated at term n = 10.
This method will not give a useful lower bound for the error, however, since we know only that
1 + cos n 0
5
≥ 5.
n n
This is certainly not the best possible lower bound, but we can’t do better in this course.

11.4 Exercises In some of the discussions below I give only hints as to the solution.
X

1 + sin n
[1, Exercise 12, p. 709] Determine whether the series converges or diverges: .
n=0
10n
Solution: The Limit Comparison Test cannot be used, since the terms behave erratically
because of the sine function, and there is not likely to exist a limit, no matter what series
you choose to compare with. But
1 + sin n 2
n
< n,
10 10
so the terms of the series are smaller than the terms of a geometric series with common
1
ratio 10 , which is less than 1, and must converge.
X∞ √
n
[1, Exercise 14, p. 709] Determine whether the series converges or diverges: .
n=0
n − 1
Solution: The terms can be made smaller by making the denominator larger, into n. That
yields a divergent p-series.
X

1
[1, Exercise 18, p. 709] Determine whether the series converges or diverges: .
n=0
2n + 3
Solution: The terms can be made smaller by making the denominator into 2n + 4. The
1
new series is a multiple 2 of a harmonic series. The harmonic series diverges, and that
property doesn’t change if we discard 2 terms at the beginning, nor if we then multiply
by 12 .
Information for Students in Lecture Section 002 of MATH 141 2009 01 2227

X

n!
[1, Exercise 30, p. 709] Determine whether the series converges or diverges.
n=1
nn
Solution: The general term is
n! 1 2 3 n−1 n
n
= · · · ... · ·
n n n n n n
1 2 2
≤ · ·1= 2
n n n
for n ≥ 3, which is the general term in a convergent p-series. Hence the given series
is convergent. It is impractical to use the limit comparison test here, because the limit
would be difficult to compute, and because it would be zero or infinite, depending on
which series is in the numerator; our version of this test does not permit the limit to be
either 0 or ∞.
X∞
1
[1, Exercise 31, p. 709] Determine whether the series sin converges or diverges.
n=1
n
sin 1n
Solution: We know the value of lim 1
to be 1; so the Limit Comparison Test shows
n→∞
n
this series diverges because the harmonic series diverges. Could we have used the Com-
parison Test? Unfortunately, the geometric argument that we used to prove the result
quoted earlier showed [1, p. 190-191] that
1 1
0 < sin < .
n n
In its present form this inequality is not useful for proving the divergence of the given
series, since the series known to be divergent occupies the outer position rather than
lying between the terms of the given series and 0.
X

1
[1, Exercise 32, p. 709] Determine whether the series converges or diverges.
1+ n1
n=1 n
Solution: It appears that the series “resembles” the Harmonic Series, and we will try to
compare it in the limit. The guess is a fortunate one, since the limit does exist.
1
1+ n 1 1
lim n 1 = lim 1
n→∞
n
n→∞ nn
1
= lim ln n
n→∞ e n

1
= ln n
by continuity of exponential
lim
en→∞ n
Information for Students in Lecture Section 002 of MATH 141 2009 01 2228

1
= = 1 by L’Hospital’s Rule
e0
We apply the Limit Comparison Test: since the Harmonic Series diverges and the ratio
of the terms of the given series to those of this divergent series approaches a non-zero
positive real number, the given series also diverges.
The same argument could have been used to prove that the series
X

1
1+ ln1n
n=1 n
is also divergent; the terms in this last series are smaller than those of the series we were
given.

B.22.2 §11.5 Alternating Series


I continue to follow the syllabus in the order of topics in the textbook. This order is the reverse
of that which I would prefer, in that the most basic results for this topic will appear in the next
section, where the textbook considers theorems about positive series. As with the preceding
section, we are investigating a specialized topic concerning series prior to meeting a more
general theorem in [1, §11.6] at the next lecture. The present section will broader the class of
series that we can work with, albeit to a very special class of series.
An alternating series is one in which the signs alternate. Our main interest, as in the
preceding sections will be in three questions, listed in the order of our priorities:
1. Does the series converge?
2. If the series converges, what is its sum?
3. If we truncate the series at a specific partial sum, can we obtain a “good” upper bound
for the error — i.e., for the difference between the sum of the series and the given partial
sum.
Our interest in alternating series derives partly from some specific series that will arise later in
[1, Chapter 11], in the portion of the chapter assigned to Calculus III. There we would meet
several important power series which represent some familiar functions; and many of those
series have alternating signs.
By the “Test for Divergence”, an alternating series cannot converge unless the limit of the
sequence of its terms is 0. For these series, and these series alone, we have a partial converse
of that test:
Theorem B.93 (Leibniz’s Alternating Series Test) An alternating series
X∞
(−1)n−1 bn for which:
n=1
Information for Students in Lecture Section 002 of MATH 141 2009 01 2229

(i) 0 ≤ bn+1 ≤ bn for all n; and

(ii) lim bn = 0
n→∞

converges.

Estimating Sums While the estimation of errors can be complicated for other series, the
following very simple bounds hold for alternating series:

Corollary B.94 (to the Leibniz Alternating Series Test) Continuing with the same notation
X

as the theorem, the remainder upon truncation of an alternating series (−1)n−1 bn (bn ≥ 0; n =
n=1
1, 2, . . .) at the N th term cannot exceed the very next term; more precisely,

X ∞
(−1)n−1 bn < |bN+1 | .
n=N+1

X

1
Example B.95 Determine whether the series (−1)n converges.
n=0
2n + 1
Solution: This is an alternating series. Since the denominators are increasing, while the nu-
merators stay fixed, the terms are decreasing; the limit of the denominators is ∞, so the terms
are approaching 0. By the Leibniz Theorem, the series converges. (It is beyond this course,
but it can be shown that this series converges to arctan 1, i.e., to π4 , and thereby provides a way
of approximating π to any desired accuracy. It is, however, not a good series for that purpose,
since it converges rather slowly.)
X

1
Example B.96 Determine whether the series (−1)n converges.
n=0
n+1
Solution: This is an alternating series. Since the denominators are increasing, while the nu-
merators stay fixed, the terms are decreasing; the limit of the denominators is ∞, so the terms
are approaching 0. By the Leibniz Theorem, the series converges. (It is beyond this course,
but this series can be proved to converge to ln(1 + 1), i.e., to ln 2.)
X

(−1)n n
Example B.97 ([7, Exercise 26, p. 740]) How many terms of the series do we need
n=1
4n
to add in order to find the sum to within an error of 0.002?
Solution: As n → ∞ lim 4nn = lim 4n 1ln n = 0. The terms are decreasing, since

n n+1 1
n
> n+1 ⇔ 4n > n + 1 ⇔ n > .
4 4 3
Information for Students in Lecture Section 002 of MATH 141 2009 01 2230

Thus the Leibniz Theorem is applicable, and we apply the error estimate associated with that
Theorem. If we truncate the series at the Nth term, then we know that the error will be less
N+1
than N+1 . If we can find N satisfying the condition that
4
N+1
< 0.002 , (89)
4N+1
we will have the desired accuracy. Since
d x 1 · 4 x − x · 4 x · ln 4 1 − x ln 4
= = ,
dx 4 x (4 x )2 4x

which is negative provided x > ln14 , which is certainly true if x ≥ 1. Thus we need only find the
first value of N for which inequality (89) holds. By repeated calculations we find the first value
is N = 5. That is, the series may be truncated by summing so that the last term we include is
(−1)5 5
and the error will be within the prescribed tolerance.55
45
The student should understand that the Leibniz estimate for error is not necessarily as sharp
as it might be. It is often possible to obtain much better approximations than the Leibniz esti-
mate might guarantee; such investigations are beyond this course. The Leibniz theorem states
that an alternating series whose terms are non-decreasing in magnitude and are approaching
0 is convergent. However, an alternating series whose terms are not decreasing, but whose
terms do approach 0, may still converge! For example, suppose that we modify the “alternat-
P

ing harmonic series” (−1)n+1 1n in the following way: term number 2n is the negative fraction
n=1
n
(−1)2 +1 21n . Let’s divide just these terms by 2. The resulting alternating series can be shown
to still be convergent; but it will converge to a sum which is less than the sum of the original
series; in fact, the reduction converges to a sum of
1
1 1
· 2 1
= .
2 1− 2
2

But the condition of the Leibniz Theorem that requires that the terms be monotonely decreasing
in magnitude is not satisfied, so the convergence cannot be inferred from the Leibniz Theorem.
Nonetheless, the series does converge (since the partial sums are equal to the difference of the
partial sums of two series known to converge).
55
We can actually sum this infinite series, using methods analogous to the method used to sum a geometric
4
progression. The sum can be shown to be − 25 . The sum of the first 5 terms is approximately -0.161133, so the
error is approximately 0.001133, which is within the desired tolerance.
Information for Students in Lecture Section 002 of MATH 141 2009 01 2231

11.5 Exercises
X

ln n
[1, Exercise 14, p. 713] Test for convergence or divergence the series (−1)n−1 .
n=1
n
Solution: The series is alternating. The limit of the terms can be seen by L’Hospital’s
Rule to be lim 1n = 0. To determine whether the terms are decreasing, we can consider
n→∞
!
d ln x 1 − ln x
= <0
dx x x2
for x > e. Hence the Leibniz Theorem applies, and the series must converge. It’s not
important where the terms start to decrease — only that they ultimately decrease.56

B.22.3 Solutions to Problems on the Final Examination in MATH 141 2007 01


Among the instructions for this examination was the following:
There are two kinds of problems on this examination, each clearly marked as
to its type.
• Most of the questions on this paper require that you
SHOW ALL YOUR WORK!
Their solutions are to be written in the space provided on the page where
the question is printed. When that space is exhausted, you may write on the
facing page. Any solution may be continued on the last pages, or the back
cover of the booklet, but you must indicate any continuation clearly on the
page where the question is printed!
• Some of the questions on this paper require only
BRIEF SOLUTIONS
for these you are expected to write the correct answer in the box provided;
you are not asked to show your work, and you should not expect partial marks
for solutions that are not correct.
You are expected to simplify your answers wherever possible.
You are advised to spend the first few minutes scanning the problems. (Please
inform the invigilator if you find that your booklet is defective.)

1. SHOW ALL YOUR WORK!


Your answers must be simplified as much as possible.
56
However, the Leibniz estimate for the error would not be valid until the terms have begun to decrease.
Information for Students in Lecture Section 002 of MATH 141 2009 01 2232

Z 2
(a) [2 MARKS] Evaluate |x|2 dx.
−1
Solution:
Z 2 Z 2 " #2
2 x3 2 8 − (−1)
|x| dx = x dx = = = 3.
−1 −1 3 −1 3
Z0
t4 dt
(b) [2 MARKS] Evaluate √ .
t5 + 1
1
5
Solution: Either
√ by observation, or by applying a substitution like u = t , or u =
t5 + 1, or u = t5 + 1, we can show that
Z0 i0 2
t4 dt 2 √5 √ 2 √
√ = t + 1 = (1 − 2) = − ( 2 − 1) .
t5 + 1 5 1 5 5
1

(c) [3 MARKS] Determine the value of


 !3 !3 !3 !3 
1  0 1 2 n − 1 
 + + + ... +  .
n n n n n

Solution: Note that the question did not ask for the value of the limit as n → ∞ —
it asked simply for the value of the sum (as a function of n).
 !3 !3 !3 !3 
1  0 1 2 n − 1 
 + + + ... + 
n n n n n
X
n−1
i3
i=0
=
n4
!2
(n − 1)n
2 (n − 1)2
= = .
n4 4n2
(Had the instructions been to evaluate the limit as n → ∞, then the given product
could have been interpreted as a left Riemann sum with n intervals of equal length
Z1 " 4 #1
1 3 x 1
∆n = n . The value would have been x dx = = .)
4 0 4
0
Information for Students in Lecture Section 002 of MATH 141 2009 01 2233

(d) [3 MARKS] Suppose it is known that f 0 (x) = 4 cosh x for all x. Showing all your
work, determine the value of f (1)− f (−1), expressed in terms of the values of either
exponentials or hyperbolic functions.
Solution: Integrating the given equation yields f (x) = 4 sinh x + C, where C is
some (fixed) real constant C. Since f 0 exists, f is continuous everywhere, so it will
be the same constant C that is involved throughout the interval [−1, +1]; hence

f (1) − f (−1) = 4 sinh 1 + C − (sinh(−1) + C)


= 4(sinh 1 + sinh 1)
= 8 sinh 1 .

Alternatively one could work with exponentials,


! beginning with f 0 (x) = 2 (e x + e−x ),
1
to show that f (1) − f (−1) = 4 e − .
e
Z x2
d t
(e) [4 MARKS] Evaluate et dt when x = 1 .
dx 12
Solution:
Z x2
Z 
d tt d  u tt  du
e dt =  e dt · where u = x2
dx 1
2
du 2 1 dx
du 2 (x )
2
= e( x )
u
= eu · · 2x
dx
(2x2 )
which simplifies to 2x · e x ; when x = 1, the value is 2e.

2. SHOW ALL YOUR WORK!


For each of the following series you are expected to apply one or more tests for conver-
gence or divergence to determine whether the series is absolutely convergent, condition-
ally convergent, or divergent. All tests used must be named, and all statements must be
carefully justified.
X

(−n − 2)n (n − 2)n
(a) [4 MARKS]
n=1
(2n2 + 1)n
Solution:
X

(−n − 2)n (n − 2)n X

(n2 − 4)n
= (−1)n .
n=1
(2n2 + 1)n n=1
(2n2 + 1)n
Information for Students in Lecture Section 002 of MATH 141 2009 01 2234

This is an alternating series. Applying the Root Test to the series of absolute values
of the terms of the given series, we have
s 2
n − 4 1 − n2
2 n 4
n (n − 4)
= =
(2n2 + 1)n 2n2 + 1 2 + n12

for n ≥ 2. As n → ∞, this ratio → 12 < 1 . Since the limit of the ratio is less
than 1, the original alternating series is absolutely convergent.
X∞
n!
(b) [4 MARKS] (−1)n+1 2 n
n=1
n2
n!
Solution: If we define an = (−1)n+1 2 n , we find, using the Ratio Test, that
n2

an+1 = (n + 1)n
2

an (n + 1)2 1
n2
= → ∞ > 1.
2(n + 1)
Hence the original series diverges.
X∞
1
(c) [4 MARKS] (−1)n sin
n=1
n
Solution: The series of absolute values may be compared, using the Limit Compar-
ison Test, with the Harmonic series, a positive series known to be divergent. Since
sin 1n
lim 1 = 1 , 0, the series of absolute values is also divergent, and the original
n→∞
n
series is either (1) conditionally convergent, or (2) divergent. But
! ! !
d 1 1 1
sin = cos · − 2 < 0
dx x x x
as x > 1. Hence the given series of alternating terms are monotonely decreasing
in magnitude. As n → ∞, sin 1n → sin 0 = 0, by the continuity of the sine func-
tion. Thus the conditions of the Alternating Series Test are satisfied, and the series
converges. We conclude that the given series is conditionally convergent.
3. BRIEF SOLUTIONS Express each of the following as a definite integral or a sum,
product, or quotient of several definite integrals, simplified as much as possible; you are
not expected to evaluate the integrals.
R is defined to be the region enclosed by the curves x + y = 6 and y = x2 ; C is the arc
y = 3 x (−1 ≤ x ≤ 2).
Information for Students in Lecture Section 002 of MATH 141 2009 01 2235

(a) [3 MARKS] The region R is rotated about the x-axis. Give an integral or sum of
integrals whose value is the volume of the resulting solid.
DEFINITE INTEGRAL(S) ONLY (DO NOT EVALUATE)

Solution: By solving the equations of the line x + y = 6 and the parabola y = x2 ,


we find the points of intersection to be (−3, 9) and (2, 4). Either of the following
integrals or sums of integrals was acceptable. (Students were not expected to eval-
uate the integrals; I have done so here in order to verify my own work, and to show
you that the integrals weren’t really hard to evaluate.)
i. Using the method of “Washers”, we find the volume to be
Z 2  Z 2 
2 2 2
π (6 − x) − (x ) dx = π −x4 + x2 − 12x + 36 dx
−3 −3

500π
which can be shown to be equal to .
3
ii. Using the method of cylindrical shells, we find the volume to be
Z 4 Z 9
√ √
(2πy) · (2 y) dy + (2πy) · (6 − y + y) dy
0 4

500π
whose value can again be shown to equal .
3
(b) [3 MARKS] The region R is rotated about the line x = 5. Give an integral or sum
of integrals whose value is the volume of the resulting solid.
DEFINITE INTEGRAL(S) ONLY (DO NOT EVALUATE)

Solution: Here again, students were expected only to state the integrals, but not to
evaluate them.
i. Using the method of cylindrical shells: We will be rotating about x = 5
vertical elements of area; the width can be taken to be ∆x, as integration will
Information for Students in Lecture Section 002 of MATH 141 2009 01 2236

be with respect to x. The element extends from a point (x, x2 ) to (x, 6 − x), so
its height is 6 − x − x2 . The volume is
Z 2 " 4 #2
2 x 4x3 11x2 1375π
2π(5 − x) · (6 − x − x ) dx = 2π − − + 30x = .
−3 4 3 2 −3 6
ii. Using the method of washers:
Z 4 Z 9
√ 2 √ 2 √
π (5 + y) − (−5 + r) dy + π ((5 + y)2 − (5 − (6 − y))2 ) dy
0 4
Z 4 Z 9
√ √ 1375π
= 20π y dy + π (24 + 10 y + 3y − y2 ) dy = .
0 4 6
(c) [3 MARKS] Express in terms of integrals — which you need not evaluate — the
average length that R cuts off from the vertical lines which it meets.
DEFINITE INTEGRAL(S) ONLY (DO NOT EVALUATE)

Solution: The length of the segment cut off from the line x = a has been shown
above to be 6 − a − a2 . The average of this function over the interval −3 ≤ a ≤ 2 is
the ratio R2
−3
(6 − a − a2 ) da 25
= .
2 − (−3) 6
Students weren’t asked to evaluate the integral. I have done so here to verify my
work; in this case I can’t verify by comparing answers obtained in two different
ways, but I can, at least, examine the magnitude of my answer, and decide whether
it is reasonable for the given data. For example, if my answer had been 25, I
would know something was wrong, by considering the sizes of the numbers being
averaged, and the maximum value obtained by the function on the given interval.
(d) [2 MARKS] Give an integral whose value is the length of C; you need not evaluate
the integral.
DEFINITE INTEGRAL(S) ONLY (DO NOT EVALUATE)
Information for Students in Lecture Section 002 of MATH 141 2009 01 2237

Solution: When y = 3 x , y0 = 3 x ln 3. The arc length is


Z 2 p
1 + (3 x ln 3)2 dx .
−1

(e) [3 MARKS] Given an integral whose value is the area of the surface generated by
rotating C about the line y = −1; you need not evaluate the integral.
DEFINITE INTEGRAL(S) ONLY (DO NOT EVALUATE)

Solution:
Z 2 p
2π (3 x + 1) 1 + 32x (ln 3)2 dx
−1

4. SHOW ALL YOUR WORK!


[12 MARKS] Evaluate the indefinite integral
Z x  x2 − 4 (x − 2) + 4
 dx .
x2 − 4 (x − 2)

Solution: The numerator of the integrand has degree not less than that of the denomina-
tor. Accordingly, the first step is to divide denominator into numerator and to integrate
the quotient separately, and to factorize the denominator and group like factors together:
Z x  x2 − 4 (x − 2) + 4 Z !
4
 dx = x+ dx
x2 − 4 (x − 2) (x + 2)(x − 2)2
Z
x2 4
= + dx .
2 (x + 2)(x − 2)2
Next we apply to the new integrand the method of partial fractions:
4 A B C
2
= + +
(x + 2)(x − 2) x + 2 x − 2 (x − 2)2
⇒ 4 = A(x − 2)2 + B(x + 2)(x − 2) + C(x + 2)
⇒ 4 = 4C when x = 2; and
4 = 16A when x = 2
Information for Students in Lecture Section 002 of MATH 141 2009 01 2238

implying that A = 14 and C = 1. Comparing coefficients of x2 on both sides of the


identity yields 0 = A + B, implying that B = − 41 . Now we can continue integration of
the original function:
Z x  x2 − 4 (x − 2) + 4 Z
x2 4
 dx = + dx
x2 − 4 (x − 2) 2 (x + 2)(x − 2)2
Z ! Z
x2 1 1 1 dx
= + − dx +
2 4 x+2 x−2 (x − 2)2

x2 1 x + 2 1
= + ln − +C.
2 4 x−2 x−2

5. SHOW ALL YOUR WORK!


Showing all your work, evaluate each of the following:
Z
(a) [4 MARKS] e−x · cos x dx
Solution: I will apply Integration by Parts twice. First, with u = e−x , dv = cos x dx
(so du = −e−x dx, v = sin x) yields
Z Z
e cos x dx = e sin x + e−x sin x dx ,
−x −x

and the second (applied to the integral resulting from the first application), with
U = e−x , dV = sin x dx (so dU = −e−x dx, V = − cos x) yields
Z Z
e cos x dx = e (sin x − cos x) − e−x cos x dx .
−x −x

By collecting both integral terms on one side, and dividing by 2, we obtain


Z
1
e−x cos x dx = e−x (sin x − cos x) + C .
2
Another way to solve this problem is to assume — from experience — that the
indefinite integral is equal to a sum of the form
Z
e−x · cos x dx = e−x (A sin x + B cos x) + C ,

where A and B are “undetermined” constants, to be determined. One could then


differentiate this equation, top obtain

e−x cos x = −e−x (A sin x + B cos x)e−x (A cos x − B sin x) .


Information for Students in Lecture Section 002 of MATH 141 2009 01 2239

This is an identity – true for all values of x. If we assign “convenient” values, for
π
example x = 0 and x = , we obtain equations
2
A − B = 1,
A + B = 0,
which we can solved, to obtain A = 12 , B = − 12 , yielding the same solution as found
earlier.
5
Z2
x
(b) [5 MARKS] √ dx
8 + 2x − x 2
1
−2
Solution:
5 5
Z2 Z2
x x
√ dx = p dx
8 + 2x − x2 9 − (x − 1)2
− 12 − 12
5
Z2
1 x
= q dx .
3
− 12
1 − ( x−1
3
)2

x−1 dx
I will use the substitution u = , so du = .
3 3
5
Z2 Z 1
1 x 1 2 3u + 1
q dx = √ .3 du
3 3 − 12 1 − u2
− 12
1 − ( x−1
3
)2
√ h i 21
−3 1 − u2 + arcsin u 1
=
 r  2 r 
 3 1   3 1 
= −3 + arcsin  − −3 − arcsin 
4 2 4 2
1 π π
= 2 arcsin = 2 · = .
2 6 3
Z !
1
(c) [4 MARKS] cos2 x + · tan2 x dx
cos2 x
Solution:
Z ! Z  
2 1 2 2 2 2
cos x + · tan x dx = sin x + tan x · sec x dx
cos2 x
Information for Students in Lecture Section 002 of MATH 141 2009 01 2240

Z !
1 − cos 2x 2 2
= + tan x · sec x dx
2
x sin 2x tan3 x
= − + +C.
2 4 3

6. SHOW ALL YOUR WORK!


Consider the arc C defined by

x = x(t) = cos t + t sin t


y = y(t) = sin t − t cos t ,

where 0 ≤ t ≤ π2 .

d2 y
(a) [6 MARKS] Determine as a function of t the value of .
dx2
Solution:

dx
= − sin t + sin t + t cos t = t cos t
dt
dy
= cos t − cos t + t sin t = t sin t
dt
dy
dy dt t sin t
= dx = = tan t when t , 0, π2 ,
dx dt
t cos t
2
! !
d y d dy d dy dt
= = ·
dx2 dx dx dt dx dx
 
d dy d
dt dx dt
tan t sec2 t 1
= dx
= = = ,
dt
t cos t t cos t t cos3 t

when t , 0, π2 .
(b) [6 MARKS] Determine the area of the surface generated by revolving C about the
y-axis.
Solution: The area of the surface generated is
Z p π
2
2π x(t) (t cos t)2 + (t sin t)2 dt
0
Z π2 Z π2
= 2π (cos t + t sin t)|t| dt = 2π (t cos t + t2 sin t) dt .
0 0
Information for Students in Lecture Section 002 of MATH 141 2009 01 2241

At this point I interrupt the calculations to integrate by parts. First, taking u = t,


dv = cos t dt (so du = dt, v = sin t),
Z Z
t cos t dt = t sin t − sin t dt = t sin t + cos t + C1 .

Then, taking u = t2 , dv = sin t dt (so du = 2t dt and v = − cos t),


Z Z
t sin t dt = −t cos t + 2 t cos t dt = −t2 cos t + 2(t sin t + cos t) + C2 .
2 2

Incorporating these results into the earlier integral yields


Z π2
2π (t cos t + t2 sin t) dt
0
h i π2
= 2π (t sin t + cos t) + (−t2 cos t + 2t sin t + 2 cos t)
0
π 
= 6π − 1 .
2

7. SHOW ALL YOUR WORK!

(a) [5 MARKS] Showing detailed work, determine whether the following integral is
convergent; if it is convergent, determine its value:
Z π
sec x dx .
π
2

π
Solution: The integrand has an infinite discontinuity at x = .
2
Z π Z π
sec x dx = lim + sec x dx
π
2
a→( π2 ) a

= lim + [ln | sec x + tan x|]πa


a→( π2 )

= lim + (ln | − 1 + 0| − ln | sec a + tan a|)


a→( π2 )

= 0 − lim + ln | sec a + tan a| .


a→( π2 )

Here both sec a and tan a are negative, as the argument is in the second quadrant,
so they both approach −∞. This implies that their sum also approaches −∞, and
that ln | sec a + tan a| approaches +∞; hence the improper integral approaches −∞,
and is not convergent.
Information for Students in Lecture Section 002 of MATH 141 2009 01 2242

X

4
(b) [5 MARKS] Showing all your work, carefully determine whether the series
n=3
n ln n
is convergent.
Solution: Since both factors in the denominator are increasing, the terms of this
positive series are decreasing monotonely. We can apply the Integral Test with the
function f (x) = x ln4 x . But
Z ∞
dx
= lim [ln ln x]b3 = +∞
3 x ln x b→∞
is a divergent, improper integral, so the series is also divergent.
(c) [3 MARKS] Showing all your work, determine whether the following sequence
converges; if it converges, find its limit:
a1 = 1.
a2 = 1.23
a3 = 1.2345
a4 = 1.234545
a5 = 1.23454545
a6 = 1.2345454545
etc., where each term after a2 is obtained from its predecessor by the addition on
the right of the decimal digits 45.
Solution:
!
45 1 1
1.23 + 45(0.0001 + 0.000001 + . . .) = 1.23 + 1+ + + ...
10000 100 1002
45 1
= 1.23 + 4 · 1
10 1 − 100
45 5
= 1.23 + = 1.23 +
9900 1100
1358 679
= = .
1100 550
8. SHOW ALL YOUR WORK!
[10 MARKS] The polar curves
r = 2 + 2 sin θ (0 ≤ θ ≤ 2π)
and
r = 6 − 6 sin θ (0 ≤ θ ≤ 2π)
Information for Students in Lecture Section 002 of MATH 141 2009 01 2243

divide the plane into several regions. Showing all your work, carefully find the area of
the region bounded by these curves which contains the point (r, θ) = (1, 0).
Solution: (see Figure 23 on page 2243)) Both of these curves are cardioids, but their sizes

-8 -4 0 4 8
0

-4

-8

-12

Figure 23: The cardioids with equations r = 2 + 2 sin θ, r = 6 − 6 sin θ

and orientations are different. The first step is to determine the points of intersection of
the curves. Solving the equations, by eliminating r “between” them, we obtain sin θ = 12 ,
so θ = π6 + 2nπ, 5π6
+2nπ, etc.
 For  θ in the given interval, then, we have obtained the
π 5π
points of intersection 3, 6 , 3, 6 .
(Are these the only points of intersection? We didn’tfind the pole to be a point of intersection

— but it is! It lies on the first cardioid as the point 0, 2 , and on the second cardioid as the
 
point 0, π2 — the same point, but appearing on the two curves with different coordinates! Could
Information for Students in Lecture Section 002 of MATH 141 2009 01 2244

there be yet other points of intersection? These could be only at points which, like the pole,
appeared with different sets of coordinates. The only other way in which different coordinates
can occur is because of the convention that (r, θ) and (−r, θ + π) are the same point. If we apply
the transformation (r, θ) → (−r, θ + π) to the cardioid r = 2 + 2 sin θ (0 ≤ θ ≤ 2π), we
obtain the equation −r = 2 − 2 sin θ (0 ≤ θ ≤ 2π); and, when we solve this equation with
the original equation of the second cardioid, i.e., r = 6 − 6 sin θ (0 ≤ θ ≤ 2π), we obtain
θ = π2 and r = 0, i.e., the pole. Similarly, if we apply the transformation (r, θ) → (−r, θ + π)
to the cardioid r = 6 − 6 sin θ (0 ≤ θ ≤ 2π), we obtain the equation −r = 6 + 6 sin θ (0 ≤
θ ≤ 2π); and, when we solve this equation with the original equation of the first cardioid, i.e.,
r = 2+2 sin θ (0 ≤ θ ≤ 2π), we obtain θ = 3π 2 and r = 0, i.e., again the pole. Finally, if we apply
the transformation to both of the original equations, we obtain sin θ = − 12 , r = −3, which yields
the same 2 points we found in the original solving of equations. A second application of the
transformation (r, θ) → (−r, θ + π) takes either equation back to its original form. To summarize,
  
we now know all the possible points of intersection: the pole, and the two points 3, π6 , 3, 5π 6 .)
The region that interests us is bounded by the two arcs:
!
3π 13π
r = 2 + 2 sin θ ≤θ≤ (90)
2 6
and (91)
π π
r = 6 − 6 sin θ ≤θ≤ (92)
6 2
Technically the first of these curves is described incorrectly, since the values shown for
the angle are not in the given interval; so we should write, instead,
!

r = 2 + 2 sin θ ≤ θ ≤ 2π , (93)
2
 π
r = 2 + 2 sin θ 0≤θ≤ , (94)
6
and (95)
π π
r = 6 − 6 sin θ ≤θ≤ . (96)
6 2
There are various ways in which the area can be calculated (see  Figure
 24 on page
2245)). One method is to first draw a line joining (0, 0) and 3, π6 . The area of the
π
Z2
1
subregion above the line is (6 − 6 sin θ)2 dt; the area of the region below the line is
2
π
6
Information for Students in Lecture Section 002 of MATH 141 2009 01 2245

1.5

0.5

0
0 0.5 1 1.5 2 2.5

-0.5

Figure 24: The region bounded by cardioids r = 2 + 2 sin θ, r = 6 − 6 sin θ and containing the
point (r, θ) = (1, 0)

π
Z6
1
(2 + 2 sin θ)2 dθ. To evaluate these definite integrals, observe that
2
− π2

Z Z
2
(1 − sin θ) dθ = (1 − 2 sin θ + sin2 θ) dθ
Z !
1 − cos 2θ
= 1 − 2 sin θ + dθ
2
3θ sin 2θ
= + 2 cos θ − +C.
2 4
In a similar fashion we can prove that
Z
3θ sin 2θ
(1 + sin θ)2 dθ = − 2 cos θ − +C.
2 4
Information for Students in Lecture Section 002 of MATH 141 2009 01 2246

Hence
π
Z2 " # π2
1 2 sin 2θ
(6 − 6 sin θ) dt = 27θ + 36 cos θ − 9
2 2 π
π 6
6
 √ √ 
27π  3 9 3 
= − 9π + 36 · − · 
2 2 2 2

63 3
= 9π −
4
π
Z6 " #π
1 2 sin 2θ 6
(2 + 2 sin θ) dt = 3θ + 4 cos θ −
2 2 −π
2
− π2
 √ √  !
 π 3 3  −3π
=  + 4 · −  − −
2 2 2 4

7 3
= 2π + ,
4

so the total area is 11π − 18 3. This area could have been computed in other ways. For
example, half the area of the smaller cardiod can be seen to be 3π, and from this one
could subtract the integral
π
Z2  2
1
(2 + 2 sin θ)2 − (6 − 6 sin θ)2 dθ ,
2
π
6


which can be shown to equal 18 3 − 8π. The integration of this difference of squares
depends on the fact that the two curves are “traced out” in the same direction; this should
be verified before using this “combined” method.
Information for Students in Lecture Section 002 of MATH 141 2009 01 2247

B.23 Supplementary Notes for the Lecture of March 25th, 2009


Distribution Date: Wednesday, March 25th, 2009, subject to further revision

B.23.1 §11.5 Alternating Series (conclusion)


At the last lecture I discussed the following theorem and corollary:

Theorem B.93 (Leibniz’s Alternating Series Test). An alternating series


X∞
(−1)n−1 bn for which:
n=1
(i) 0 ≤ bn+1 ≤ bn for all n; and
(ii) lim bn = 0
n→∞
converges.
Corollary B.94 to the Leibniz Alternating Series Test. Continuing with the
same notation as the theorem, the remainder upon truncation of an alternating
X

series (−1)n−1 bn (bn ≥ 0; n = 1, 2, . . .) at the N th term cannot exceed the very
n=1
next term; more precisely,

X ∞
(−1)n−1 bn < |bN+1 | .
n=N+1

11.5 Exercises (continued)


X
∞ √
n
n
[1, Exercise 10, p. 713] Test for convergence or divergence the series (−1) √ .
n=1
1 + 2 n
Solution: The general term of the given series is,

n n 1
(−1) √ = (−1)n
1+2 n √1 +2
n

1
whose magnitude → as n → ∞. Thus the sequence of terms of this series does not
2
have a limit as n → ∞, and the series cannot converge, by the “Test for Divergence”. In
fact, the other condition of the Leibniz Theorem, that the terms decrease in magnitude,
also fails to hold! But here we can say more: not only can we state that the Leibniz Test
yields no information, but we can state that the Test for Divergence shows that the series
actually diverges.
Information for Students in Lecture Section 002 of MATH 141 2009 01 2248

X

ln n
[1, Exercise 14, p. 713] Test for convergence or divergence the series (−1)n−1 .
n=1
n
Solution: The series is alternating. The limit of the terms can be seen by L’Hospital’s
Rule to be lim 1n = 0. We have determined this limit for 2 reasons:
n→∞

• as an application of “The” Test for Divergence — our result is inconclusive, and


gives no information about divergence;
• as a condition of the Leibniz Alternating Series Test — here our result is positive,
and, when combined with the yet unproved fact that the terms are decreasing in
magnitude, will show that the series does, indeed, converge.
To determine whether the terms are decreasing, we can consider
!
d ln x 1 − ln x
= <0
dx x x2
for x > e. Hence the Leibniz Theorem applies, and the series must converge. It’s not
important where the terms start to decrease — only that they ultimately decrease.57
X

π
[1, Exercise 17, p. 713] Test for convergence or divergence the series (−1)n sin .
n=1
n
Solution: This problem should be compared with [1, Exercise 31, p. 709] (cf. these
notes, p. 2227). In the earlier problem we proved the divergence of a series whose terms
resemble those of the present series, except that it was not alternating. But the present
series satisfies the conditions of the alternating series test, as the terms alternate in sign,
are monotonely decreasing (because the sine function is an increasing function to the
right of 0), and the limit of the terms is 0. Thus the series converges. When we have
covered [1, §11.6] we will be able to describe the present series as being Conditionally
Convergent — it is convergent, but loses that property if all terms are replaced by their
absolute value.
X∞
π
[1, Exercise 18, p. 713] Test for convergence or divergence the series (−1)n cos .
n=1
n
Solution: Since the magnitudes of the terms of this series approach 1 as n → ∞, the
terms of the series have no limit! By “The” Test for Divergence, this series diverges!
X

nn
[1, Exercise 19, p. 713] Test for convergence or divergence the series (−1)n (cf. [1, Ex-
n=1
n!
ercise 30, p. 709][1, Exercise 18, p. 720])
57
However, the Leibniz estimate for the error would not be valid until the terms have begun to decrease.
Information for Students in Lecture Section 002 of MATH 141 2009 01 2249

Solution: The terms may be factorized as


nn n n n n
(−1)n = (−1)n · · . . . · .
n! 1 2 n−1 n
While this is an alternating series, the limit of the terms cannot be 0, since the fractions
into which I have factored the terms are, in magnitude, all greater than or equal to 1;
thus the series must diverge, by “The” Test for Divergence; this also shows that not all
conditions of the Leibniz Alternating Series Test are satisfied, but from that we can only
conclude that that test does not apply, not that the series diverges.
X
∞  n n
[1, Exercise 20, p. 713] Test for convergence or divergence the series (−1)n .
n=1
5
Solution: This series also diverges, because the terms to not have limit 0.

B.23.2 §11.6 Absolute Convergence and the Ratio and Root Tests
Finally, after meeting, in [1, §§11.2–11.4] several tests for the convergence of positive series,
we see a reason for our interest in this type of series. To state the theorem we require two
definitions:
P P
Definition B.7 1. A series an is absolutely convergent if the series |an | is convergent.
P P
2. A series an is conditionally convergent if an is convergent but not absolutely con-
P P
vergent, i.e., if an converges but |an | diverges.
Note that, thus far, you must not interpret the word absolutely as an adverb modifying the
participle convergent; until the following theorem is available, you should treat absolutely
convergent as a two-word name for a property, nothing more. Now we state the theorem that
will permit a broader interpretation of the name.
Theorem B.98 A series which is absolutely convergent is convergent.
(I shall not prove the theorem in the lectures, but a short proof can be found in your textbook.)
Now that we have this theorem, we can interpret the word absolutely as a modifier —
if we drop the word from a statement, the resulting statement is still true; we didn’t need
such a reservation with the term conditionally convergent, since the definition stated that a
conditionally convergent series is convergent and . . . .
While investigation of the convergence of general series, i.e., series whose terms have both
plus and minus signs, can be difficult, we can begin by considering the series of absolute
values: if the resulting series converges, we can infer that the original series is convergent.
But, if the series of absolute values diverges, or if we are unable to say anything about it, then
we can make no inference at all.
Information for Students in Lecture Section 002 of MATH 141 2009 01 2250

The Ratio Test. The Ratio Test is a test for positive series which considers the limit of the
ratio of a term to its predecessor. The textbook states the test in terms of the absolute values of
terms of a general series, so we will present it in that variant:
|an+1 | P
Theorem B.99 (Ratio Test) 1. If lim = L < 1, then the series an is absolutely
n→∞ |an |
convergent.
|an+1 | P
2. If lim = L > 1, then the series an is divergent.
n→∞ |an |
|an+1 |
3. If lim = 1, or if the limit does not exist, then this test provides no information
n→∞ |an |
P
concerning the possible convergence of the series an .

The Root Test. The Root Test is another test for positive series; here also the textbook
presents it in a form that applies to all series.
pn P
Theorem B.100 (Root Test) 1. If lim |an | = L < 1, then the series an is absolutely
n→∞
convergent.
pn P
2. If lim |an | = L > 1, then the series an is divergent.
n→∞
pn
3. If lim |an | = 1, or if the limit does not exist, then this test provides no information
n→∞ P
concerning the possible convergence of the series an .

You will need to practice on many problems in order to become comfortable with all the tests
you have met, and to know the limitations of each of them. You should not be surprised if
some problems are amenable to the use of more than one test.

Rearrangements The textbook reports on the result of Bernhardt Riemann that the terms
of a conditionally convergent series may be written in another order to create series that will
converge to any given real number, and even to diverge.

Example B.101 ([7, Exercise 20, p. 746]) Determine whether the series
X∞
(−1)n
n=2
(ln n)n

is absolutely convergent, conditionally convergent, or divergent.


Information for Students in Lecture Section 002 of MATH 141 2009 01 2251

Solution: Consider the series of absolute values. The ratio of the (n+1)st term to its predecessor
is complicated, so we will look at the nth root of the nth term, obtaining
1
→0
ln n
as n → ∞. The given series is absolutely convergent. If we were interested only in checking
the convergence of the given series, then the Leibniz Alternating Series Test would have been
sufficient, and easy to use; but it would have given no information about absolute convergence.

Example B.102 ([7, Exercise 24, p. 746]) Determine whether the series
X

(−1)n
n=1
(arctan n)n

is absolutely convergent, conditionally convergent, or divergent.


Solution: I will apply the Root Test. The nth root of the absolute value of the nth term is
1 2
, which approaches < 1. Hence the series is absolutely convergent.
arctan n π

11.6 Exercises

[1, Exercise 3, p. 719] I modify the problem: Determine whether the series
X

1, 000, 000n
(−1)n
n=0
n!

is absolutely convergent, conditionally convergent, or divergent.


Solution: Apply the Ratio Test to the series of absolute values. The ratio of the (n + 1)th
100000 100000
term to the nth is , and → 0 as n → ∞. Thus, by the Ratio Test, the
n+1 n+1
series is absolutely convergent. This conclusion could also be proved in other ways.

[1, Exercise 12, p. 719] Determine whether the series


X

sin 4n
n=0
4n

is absolutely convergent, conditionally convergent, or divergent.


Solution: Neither the Root Test nor the Ratio Test is useful here, because the limits do
P
not exist. Compare the series of absolute values with the geometric series 41n . Since
the latter converges, so does the former. Thus the given series is absolutely convergent.
Information for Students in Lecture Section 002 of MATH 141 2009 01 2252

[1, Exercise 16, p. 720] Determine whether the series


X

3 − cos n
2
n=1 n3 − 2

is absolutely convergent, conditionally convergent, or divergent.


Solution: For n > 1000 — (we don’t need the best bound here) — this is a positive
series; the numerator behaves erratically, and we will not be able to calculate a limit
using the Ratio or Root Tests. But since, for large n, the denominator is larger than
2
n 3 , we would expect this series to diverge by comparison with the appropriate p-series.
More precisely, the divergence of the p-series with p = 32 implies the divergence of the
given series, since
3 − cos n 3−1 3−1 2
2
> 2 > 2
= 2 .
n3 − 2 n3 − 2 n3 n3
Information for Students in Lecture Section 002 of MATH 141 2009 01 2253

B.24 Supplementary Notes for the Lecture of March 30th, 2009


Distribution Date: Monday, March 30th, 2009, subject to further revision

B.24.1 §11.6 Absolute Convergence and the Ratio and Root Tests (conclusion)
Review of the last lecture After reviewing the Leibniz Alternating Series Test, I worked
examples on these series, and mentioned the concept of conditional convergence, to be defined
in the next section. In §11.6 we finally saw some justification for our development of tests for
the convergence of positive series.

• definitions of absolute and conditional convergence

• Theorem: Absolute convergence implies convergence.

• The Ratio Test and Root Test are essentially tests for the convergence of positive series,
but they are formulated in your textbook for any series, since the limits involved are each
applied to the absolute values of roots and/or ratios.

• In each of these tests a limit is investigated, and that limit must be different from 1 for
the test to yield any useful information. If the limit does not exist, or if the limit exists
and is equal to 1, the test fails to give any information.

• When the limit exceeds 1, that implies that the terms do not approach 0, and so diver-
gence follows from “The” Test for Divergence.

11.6 Exercises (continued)


X

2n
[1, Exercise 4, p. 719] Determine whether the series (−1)n−1 is absolutely convergent,
n=1
n4
conditionally convergent, or divergent.
Solution: This is an alternating series: the only test we have to apply directly to such
series is the Leibniz Alternating Series Test (and the Test for Divergence, which forms a
2n
part of the Leibniz Test). To determine lim 4 , where both numerator and denominator
n→∞ n
become infinite, we can use L’Hospital’s Rule. This will need to be applied 4 times, after
2n (ln 2)4
which we have lim = +∞ . The terms of the original series do not approach
n→∞ 24
+∞, since they are alternating in sign; but the non-existence of a limit shows that, in
particular, the limit of terms is not 0, so the Test for Divergence tells us that this series
must diverge. We don’t even need to check the second condition of the Leibniz Test
(which could be shown to fail). Thus, even though we thought we were applying the
Information for Students in Lecture Section 002 of MATH 141 2009 01 2254

Leibniz Test, the failure of convergence derives from the Test for Divergence, which is
one part of the Leibniz Test.
The preceding was correct, but naive. It would have been better to apply the Ratio Test,
since
2n+1
 n 4  n 4
(n + 1)4
lim = 2 lim = 2 lim = 2,
n→∞ 2n n→∞ n + 1 n→∞ n + 1

n4
implying that the given series diverges.
X

n2 2n
[1, Exercise 14, p. 719] Determine whether the series (−1)n−1 is absolutely conver-
n=1
n!
gent, conditionally convergent, or divergent.
Solution:

1. This is an Alternating Series, and we could have begun by applying the Leibniz
Test. We would find that the terms are decreasing only if
(n + 1)2 2n+1 n!
· 2 n <1
(n + 1)! n2

which is equivalent, after reduction, to n − 1 > 3; thus the condition of be-
ing monotonely decreasing is ultimately satisfied (here, for n ≥ 4). But how do
n2
we determine lim (−1)n+1 n ? Simply observe that the (n + 1)st term is obtained
n→∞ 2
2n + 2
from the nth by multiplying by 2
. This factor will be less than, say, 12 , when
n
(n − 2)2 > 8, i.e., when n ≥ 5. So, for such n, each term is less than 21 of its prede-
cessor; this shows that the nth term is bounded by a constant multiple of 21n , which
approaches 0 as n → ∞. Thus the terms approach 0, so the Test for Divergence
reveals nothing in itself! However, as we have now proved that both conditions
of the Leibniz Test apply to this Alternating Series, we can infer that the series is
convergent.
2. But, to answer the given question, we need to know whether the series is absolutely
convergent or conditionally convergent, so more information is required: we have
proved only that it is at least conditionally convergent.
3. To check for absolute convergence we need to apply one of our tests to the positive
series obtained by changing all the signs to +. Of the tests we have, the Integral Test
is not useful, since we don’t have any “nice” function available to take on the values
of n! at the integer points n; there does exist such a function, but its development
Information for Students in Lecture Section 002 of MATH 141 2009 01 2255

is far beyond this course. The obvious test to try is the Ratio Test, since the terms
are products, and the ratio of successive terms has many factors which cancel. ! We
2(n + 1) 1 1
find that the ratio of the (n + 1)st term to the nth is = 2 + 2 which
n2 n n
approaches 0 as n → ∞. Since this limit exists, and is less than 1, the Ratio Test
tells us that the series of absolute values is convergent — i.e., that the original series
is absolutely convergent.

[1, Exercise 17, p. 720] Determine whether the series


X

(−1)n
n=0
n ln n

is absolutely convergent, conditionally convergent, or divergent.


Solution: Here the ratio of the absolute value of the (n + 1)th term to that of the nth is
!
1 ln(n + 1)
1+ ·
n ln n
in which the first factor approaches 1, and the second (by L’Hospital’s Rule) also ap-
proaches 1. Thus the Ratio Test fails to give useful information about this series. We are
not helpless, however. Let’s first consider the series of absolute values. The terms are
decreasing and approaching 0, so we may use the Integral Test to show that it is diver-
gent. On the other hand, the original series is alternating, and the terms are decreasing
and approaching 0; hence, by the Leibniz Test, that series converges; consequently it is
conditionally convergent.

[1, Exercise 26, p. 720] Determine whether the series


2 2 · 6 2 · 6 · 10 2 · 6 · 10 · 14
+ + + + ...
5 5 · 8 5 · 8 · 11 5 · 8 · 11 · 14
is absolutely convergent, conditionally convergent, or divergent.
4n + 2
Solution: The ratio of the (n + 1)st term to the nth is , which approaches 43 > 1.
3n + 5
By the Ratio test, this series diverges.
P
[1, Exercise 30, p. 720] The terms of a series an are defined “recursively” by a1 = 1, an+1 =
2 + cos n P
√ · an (n ≥ 1). Determine whether an converges or diverges.
n

Solution: The ratio of the (n + 1)st to the nth term is 2 + cos n n (n ≥ 1). As n → ∞ the
numerator is bounded between 1 and 3, while the denominator becomes infinitely large;
Information for Students in Lecture Section 002 of MATH 141 2009 01 2256

by the Squeeze Theorem, applied to


an+1 3
0≤ ≤ √ ,
an n
an+1
we see that lim = 0. By the Ratio Test, this shows that the series converges abso-
n→∞ an
lutely.
X∞
(n!)2
[1, Exercise 32, p. 720] For which positive integers k is the series convergent?
n=1
(kn)!
Solution: The ratio of the (n + 1)st term to the nth is

((n + 1)!)2
(k(n + 1))! (n + 1)!(n + 1)!(kn)! (n + 1)2
= =
(n!)2 n!n!(kn + k)! (kn + k)(kn + k − 1) · . . . · (kn + 1)
(kn)!
in which the factors in the denominator are k in number, and all of them lie between
n + 1 and kn + k. As long as k > 2, this fraction has more 1st degree factors in the
denominator than in the numerator, so it will approach 0 as n → ∞, and we note that
(n + 1)2
0 < 1; when k = 2 the fraction is , which approaches 14 < 1 as n → ∞;
(2n + 2)(2n + 1)
when k = 1 the fraction is equal to n + 1, and approaches +∞. By the Ratio Test the
series will converge for k ≥ 2, and will diverge when k = 1.

B.24.2 §11.7 Strategy for Testing Series


This section is concerned with improving your methods for attacking series problems. Plan-
ning strategies really needs some experience: after you have worked a substantial number of
questions from earlier sections in the chapter, you might wish to read the 8 points that the
textbook gives on [1, p. 721]. These points will be of much more use to you after you have
accumulated some experience in applying the individual tests.

Exercise B.2 ([7, Problems 2 and 4, p. 748]) You are asked to test for convergence or diver-
X∞
n−1 X∞
n−1
gence the series 2
and (−1)n−1 2 .
n=1
n + n n=1
n + n
Solution:

1. Consider the given positive series.


Information for Students in Lecture Section 002 of MATH 141 2009 01 2257

(a) It is useful, where possible, to formulate a good conjecture (guess) about the out-
come of the investigation, in order to know whether to concentrate efforts on at-
tempting to prove convergence or divergence. I suggest that a valuable first strate-
gic stepX is to replace the general term by a “simpler” term: in this case I would
n
look at . Since this gives the harmonic series, a first guess in this case would
n
n2
be that the series diverges. I would thus try to use some test that relates the given
series to the harmonic series.
(b) The simplest method would be to use the Limit Comparison Test:
n−1
2 n−1
lim n + n = lim = 1 , 0.
n→∞ 1 n→∞ n + 1

n
Hence the given series and the Harmonic Series converge or diverge together. We
know that the Harmonic Series diverges; hence the given series also diverges.
(c) We could also have attacked this problem naively by using the Integral Test, with
x−1
f (x) = 2 . Since
x +x
−x2 + 3x x(x − 3)
f 0 (x) =  2
=−  ,
x2 + x x2 + x 2
the function is decreasing for n > 3; and

n−1 1 − 1n
lim = lim = 0.
n→∞ n2 + n n→∞ 1 + n

Then the series will converge or diverge according as the following improper inte-
gral converges or diverges:
Z ∞ Z ∞ !
x−1 1 1
dx = − + dx
3 x2 + x 3 x x+1
expanding by partial fractions
Z b !
1 2
= lim − + dx
b→∞ 3 x x+1
= lim [− ln x + 2 ln(x + 1)]b3
b→∞
" #b
(x + 1)2
= lim ln
b→∞ x 3
Information for Students in Lecture Section 002 of MATH 141 2009 01 2258

!
(b + 1)2 16
= lim ln − ln
b→∞ b 3
! !
1 16
= lim ln b + 2 + − ln
b→∞ b 3
= ∞.

Thus this integral diverges, and the given series must also diverge.
X

n−1
2. Now consider the series (−1)n−1 . We know from the preceding discussion
n=1
n2 + n
that the given alternating series does not converge absolutely. However, the Alternating
Series Test may be applied to show that the alternating series does converge. Thus the
series converges conditionally. So the answer to the question as stated is that the given
series does converge.

11.7 Exercises
P
∞ 3
[1, Exercise 10, p. 722] Test for convergence or divergence the series n2 e−n .
n=1
Solution: The terms suggest
R a function with an obvious antiderivative. More precisely,
2 −x3 3
if f (x) = x e , then f (x) dx = − 13 e−x + C. Thus it would appear that we could use
the Integral Test here. But let’s check that the conditions for that test are satisfied:

• Is the given series positive: YES


• Does the function f match the series values at integer terms: YES
• Is f (continuous and) positive (not just at the integer points): YES
• Is f decreasing? NOT CHECKED YET!
  3
f 0 (x) = x 2 − 3x3 e−x , a product of 3 factors. The first factor is positive for x > 0; the
last factor is an exponential, so it is always positive; and the middle factor is negative
 1
for x > 32 3 , so it is certainly negative for x ≥ 1. Thus f 0 < 0, and f is decreasing. The
Integral RTest tells us that the series will converge or diverge according as the improper

integral 1 f (x) dx converges or diverges. But
Z a
3 1  −a  1
x2 e−x dx = − e − e−1 → as a → ∞.
0 3 3e
From the convergence of the integral we infer the convergence of the given series.
Information for Students in Lecture Section 002 of MATH 141 2009 01 2259

P

[1, Exercise 12, p. 722] Test for convergence or divergence the series sin n.
n=1
Solution: Here is a case where the Test for Divergence is needed. None of the other
tests you know will be helpful. As n → ∞, sin n does not approach a limit. (However,
students in this course could not be expected to prove that fact rigorously without some
strong hint.) Thus the series cannot converge.
X

k+5
[1, Exercise 20, p. 722] Test for convergence or divergence the series .
k=1
5k
Solution: The series looks at first like a geometric series; but the numerators are increas-
ing, so we can’t compare with a geometric series; the Limit Comparison Test will also
X 1
not be helpful, at least not in comparing with the Geometric Series . But several
5k
other possibilities suggest themselves.
P  2 k
Compare with a geometric series which converges more slowly: e.g., with 5
. Here

k+5
k k+5 1
lim 5 !k = lim k = lim k = 0.
k→∞ 2 k→∞ 2 k→∞ 2 ln 2

Unfortunately, this limit being 0, we cannot use the form of the Limit Comparison
Test given in your book. We could use the version given in [1, Exercise 40(a), p.
709], but that was not studied in our course.
Ratio Test:
ak+1 1 k+6 1
lim = lim = <1
k→∞ak 5 k→∞ k+5 5
hence the given positive series converges.
Root Test:
ln(k + 5) ln(k + 5)
√k 1 √k 1 lim
1 k→∞
lim ak = lim k + 5 = lim e k = e k .
k→∞ 5 k→∞ 5 k→∞ 5
It can be shown by l’Hospital’s Rule that the exponent approaches 0, so the kth root
approaches 15 , which is less than 1.

In fact one can determine the actual sum here, but it involves techniques slightly beyond
the course.
Information for Students in Lecture Section 002 of MATH 141 2009 01 2260

P
∞  
1
[1, Exercise 23, p. 722] Test for convergence or divergence the series tan n
.
n=1

Solution: As n → ∞, 1n → 0, and tan n1 → 0 (since it is the ratio of sin 1n to cos n1 , and


cos n1 → cos 0 = 1, while sin 1n → 0). Thus the Test for Divergence does not eliminate
the possibility that the series may converge, just as the same test did not eliminate the
possibility that the Harmonic Series might converge. If you remember that

tan 1n sin 1n
lim 1
= lim
n→∞ n→∞ 1 · cos 1n
n n
sin 1n 1
= lim 1
· lim sec
n→∞
n
n→∞ n
= 1·1=1

you can apply the Limit Comparison Test, comparing the given series with the Harmonic
Series. Since the limit is a non-zero real number, and since the Harmonic Series is known
to diverge, the given series must also diverge.
X

1
[1, Exercise 36, p. 722] Test for convergence or divergence the series .
n=2
(ln n)ln n
Solution: Observe that

1 1 1 1 1
ln n
=  ln n
= (ln ln n)(ln n) = ln ln n
= ln ln n .
(ln n) eln ln n e eln n n

We can arrange for the exponent, i.e., ln ln n, to be greater than, for example, 2, by taking
2 P
n > ee = 1618.17799.... Thus we may compare the given series with the p-series n12 ,
which is known to converge, and infer that the given series also is convergent.
X
∞ 
√n 
[1, Exercise 38, p. 722] Test for convergence or divergence the series 2−1 .
n=1
Solution: I start with the observation that the following factorization holds:
 
an − bn = (a − b) an−1 + an−2 b + . . . + abn−2 + an−1 ,
√n
for any real numbers a and b, Taking a = 2 and b = 1, we have
 √n n
√n 2 −1
2 − 1 =  √ n−1  √ n−2 √n .
n n
2 + 2 + ... + 2 + 1
Information for Students in Lecture Section 002 of MATH 141 2009 01 2261

The
 √n numerator is equal to 1. All of the n terms in the denominator are smaller than
n
2 , so the denominator is smaller than 2n. Thus the terms are greater than terms in a
multiple of the harmonic series, so the series must diverge, by the Comparison Test.
We could have used the Limit Comparison Test also, since we know that
√n ln 2 ln 2
lim 2 = lim e n = elimn→∞ n e0 = 1 .
n→∞ n→∞

Now apply l’Hospital’s Rule:

√n ln 2 √n
− 2 2
2−1
lim = lim n
n→∞ 1 n→∞ 1
− 2
n n√
n
= lim (ln 2) 2 = ln 2 ,
n→∞

since  1
√n ln 2 lim ln 2
lim 2 = lim eln 2 n = lim e n = en→∞ n = e0 = 1 .
n→∞ n→∞ n→∞

By the Limit Comparison Test, the divergence of the harmonic series implies the diver-
gence of the given series. (Note that, if we alternate the signs in this problem, we obtain
a series whose terms are decreasing and approaching 0, so the Alternating Series Test
tells us that that series converges, i.e., that it is conditionally convergent.)
B.25 Supplementary Notes for the Lecture of Wednesday, April 01st,
2009
Distribution Date: Wednesday, April 01st, 2009, corrected on 12 April, 2009
subject to further correction

B.25.1 Final Examination in MATH 141 2008 01 (one version)


This examination was written during a labour disruption, when the services of Teaching As-
sistants were not available for grading purposes. The following additional instructions were
distributed with the examination.

VERSION n
McGILL UNIVERSITY
FACULTY OF SCIENCE
FINAL EXAMINATION
IMPORTANT ADDITIONAL INSTRUCTIONS
MATHEMATICS 141 2008 01 CALCULUS 2
EXAMINER: Professor W. G. Brown DATE: Monday, April 14th, 2008
ASSOCIATE EXAMINER: Mr. S. Shahabi TIME: 09:00 – 12:00 hours

A. Part marks will not be awarded for any part of any question worth [4 MARKS] or less.

B. To be awarded part marks on a part of a question whose maximum value is 5 marks or


more, a student’s answer must be deemed to be more than 75% correct.

C. While there are 100 marks available on this examination 80 MARKS CONSTITUTE A
PERFECT PAPER. You may attempt as many problems as you wish.

All other instructions remain valid. Where a problem requires that all work be shown, that
remains the requirement; where a problem requires only that an answer be written in a box
without work being graded, that also remains the requirement.
Students are advised to spend time checking their work; for that purpose you could verify
your answers by solving problems in more than one way. Remember that indefinite integrals
can be checked by differentiation.
Information for Students in Lecture Section 002 of MATH 141 2009 01 2263

W. G. Brown, Examiner.
Instructions
1. Fill in the above clearly.

2. Do not tear pages from this book; all your writing — even rough work — must be handed in.
You may do rough work for this paper anywhere in the booklet.

3. Calculators are not permitted. This is a closed book examination. Regular and translation
dictionaries are permitted.

4. This examination booklet consists of this cover, Pages 1 through 9 containing questions; and
Pages 10, 11, and 12, which are blank. Your neighbour’s version of this test may be different
from yours.

5. There are two kinds of problems on this examination, each clearly marked as to its type.

• Most of the questions on this paper require that you SHOW ALL YOUR WORK!
Their solutions are to be written in the space provided on the page where the question
is printed; in some of these problems you are instructed to write the answer in a box,
but a correct answer alone will not be sufficient unless it is substantiated by your work,
clearly displayed outside the box. When space provided for that work is exhausted, you
may write on the facing page . Any solution may be continued on the last pages, or the
back cover of the booklet, but you must indicate any continuation clearly on the page
where the question is printed!
• Some of the questions on this paper require only BRIEF SOLUTIONS ; for these you
are expected to write the correct answer in the box provided; you are not asked to show
your work, and you should not expect partial marks for solutions that are not correct.

You are expected to simplify your answers wherever possible.


You are advised to spend the first few minutes scanning the problems. (Please inform the
invigilator if you find that your booklet is defective.)

6. A TOTAL OF 100 MARKS ARE AVAILABLE ON THIS EXAMINATION.

1. SHOW ALL YOUR WORK!


Your answers must be simplified as much as possible.
Z
4 − 6x
(a) [2 MARKS] Evaluate dx .
1 + x2
Solution:
Z Z Z
4 − 6x dx 2x
dx = 4 − 3 dx
1 + x2 1 + x2 1 + x2
= 4 arctan x − 3 ln(1 + x2 ) + C
Information for Students in Lecture Section 002 of MATH 141 2009 01 2264

Z2 p
(b) [3 MARKS] Evaluate y2 y3 + 1 dy .
0
p
Solution: I apply the change of variable u = y3 (or v = y3 + 1), under which
du = 3y2 dy:
Z2 p Z 8 √
2 1
y y3 + 1 dy = u+1· du
0 3
0
1 2h i
3 8
= · (u + 1) 2
3 3 0
2  23 3
 52
= 9 − 12 = .
9 9
Z
(c) [3 MARKS] Evaluate sin(18 θ) · cos(30 θ) dθ .
Solution:
i. The easiest way to solve this problem is by using a trigonometric identity
which relates products to sums:
Z Z
1
sin(18 θ) cos(30 θ) dθ = (sin 48θ + sin(−12θ)) dθ
2
1 1
= − cos 48θ + cos 12θ + C .
96 24
ii. The problem could also be solved laboriously by two applications of inte-
gration by parts: Take u = sin 18θ, dv = cos 30θ dθ, implying that du =
1
18 cos 18θ, v = 30 sin 30θ:
Z Z
1 18
sin(18 θ) · cos(30 θ) dθ = sin 18θ · sin 30θ − cos 18θ · sin 30θ dθ .
30 30
A second application of integration by parts, with U = cos 18θ, dV = sin 30θ dθ,
1
implying that dU = −18 sin 18θ dθ, V = − 30 cos 30θ, yields
Z
sin(18 θ) · cos(30 θ) dθ
Z !
1 18 1 18
= sin 18θ · sin 30θ − − cos 18θ · cos 30θ − sin 18θ · cos 30θ dθ .
30 30 30 30
This equation may be solved for the desired indefinite integral, yielding
Z
sin(18 θ) cos(30 θ) dθ
Information for Students in Lecture Section 002 of MATH 141 2009 01 2265

!
1 1 18
=  2 sin 18θ · sin 30θ + 2 cos 18θ · cos 30θ + C
1 − 18 30 30
30
1
= (5 sin 18θ · sin 30θ + 3 cos 18θ · cos 30θ) + C .
96
The validity of this solution can be demonstrated by differentiation.
2. SHOW ALL YOUR WORK!
Z √
3
d
(a) [3 MARKS] Simplifying your answer as much as possible, evaluate earcsin z dz .
dx −x
Solution:
Z √ Z
3 −x
d arcsin z d
e dz = − √ earcsin z dz
dx −x dx 3
d
= −earcsin(−x) · (−x)
dx
= earcsin(−x) = e− arcsin x
(The last simplification was not required.)
(b) [4 MARKS] For the interval 2 ≤ x ≤ 5 write down the Riemann sum for the
function f (x) = 3 − x, where the sample points are the left end-point of each of n
subintervals of equal length.
5−2 3 3i
Solution: The intervals have length = ; f (2) = 1, xi = 2 + , f (xi ) =
! n n n
3i 3i
3− 2+ = 1 − . The Riemann sum is
n n
! !
3X 3X
n−1 n
3i 3(i − 1)
1− or 1−
n i=0 n n i=1 n

(c) [4 MARKS] Determine the value of the preceding Riemann sum as a function of
n, simplifying your work as much as possible. (NOTE: You are being asked to
determine the value of the sum as a function of n, not the limit as n → ∞.)
Solution:
 n−1  !
3 X 3 X 
n−1
3 3 n(n − 1)
 1− i = n− ·
n  i=0 n i=0  n n 2
3 9
= − + .
2 2n
Information for Students in Lecture Section 002 of MATH 141 2009 01 2266

3. SHOW ALL YOUR WORK!


For each of the following series determine whether the series diverges, converges condi-
tionally, or converges absolutely. All of your work must be justified; prior to using any
test you are expected to demonstrate that the test is applicable to the problem.
X ∞ !
1
(a) [4 MARKS] √
n=3 n ln n

Solution:
√ Since x, and ln x are both increasing functions of x, so is their compo-

sition ln x; since x is also an increasing function of x, so is the product x ln x.
Thus the denominators of these fractions are increasing, and the fractions must be
decreasing; (this is because √the function 1x is a decreasing function of x). We ob-
serve also that the limit of n n as n → ∞ is infinite; hence the limit of the terms of
the series is 0. As the function x ln1 x is continuous, we have proved that the condi-
tions of the integral test have been satisfied: the series will converge iff the integral
Z∞
1
dx converges. (The fact that the function is decreasing could also have
x ln x
3
been shown by differentiation.)
Z a √ ia √ √
dx
√ = 2 ln x = 2 ln a − 2 ln 3 → ∞ as a → ∞ .
3
3 x ln x

We conclude, by the Integral Test that the given series diverges. (The proof that the
terms were the values of a function which satisfied the conditions of the theorem
was an essential part of the solution.)
X∞ √
n+1 4n + 5
(b) [4 MARKS] (−1)
n=1
3n + 10
Solution:√We may begin by considering the corresponding series of absolute val-
X∞
4n + 5
ues, . If we attempt to apply the Ratio Test, we find that the limit of
n=1
3n + 10
the ratios of successive terms is 1, so that test is useless here. But
Xwe can apply the
− 12
Limit Comparison Test, comparing with the divergent p-series n :
√ q
4n + 5 4 + 5n 2
lim 3n + 10 = lim = > 0.
n→∞ 1 n→∞ 3 + 10 3
n
1
n2
Since this limit is positive, and since the p-series to which we have compared is
divergent, we may conclude that the series of absolute values is also divergent; and
Information for Students in Lecture Section 002 of MATH 141 2009 01 2267

hence that the original series we were given is not absolutely convergent. But that
still leaves open the question of whether that series is conditionally convergent or
divergent. I will check whether the conditions of the Leibniz Alternating Series
Test are satisfied:
 q 
√  5
4 + n 
4n + 5  1 
lim = lim  √ · =0
n→∞ 3n + 10 n→∞   n 3 

1 1 √
√  ·2· √ · (3x + 10)) − ( 4x + 5 · 3)
d  4x + 5  2 4x + 5
  =
dx 3x + 10 (3x + 10)2
−9x − 5
= √ < 0,
(3x + 10)2 4x + 5
hence we may apply the Leibniz Test, and conclude that the given series is conver-
gent. Since it has been shown to not be absolutely convergent, it is conditionally
convergent.
X∞ ! !!
−1 1 −1 1
(c) [4 MARKS] cot − cot
n=1
n+1 n
Solution: This is a telescoping series:

X
N ! !!
−1 1 −1 1 1
cot − cot = − cot−1 1 + cot−1 .
n=0
n+1 n N+2

As N → ∞ this partial sum approaches − cot−1 1 + cot−1 0 = − π4 + π


2
= π4 , so this is
1
the value to which the series converges. Since the derivative of cot−1 x is − ,
1 + x2
the function is monotonely decreasing. Hence the terms of the given series are all
positive: a positive series which is convergent is absolutely convergent.
(The problem did not require the student to find the precise value of the sum. If
one did not notice that the given series was telescopic, one could still have applied
the integral test to prove that it is absolutely convergent. It can be seen that
Z ∞ !
−1 1 −1 1
cot − cot dx
0 x+1 x
Z ∞
= (arctan(x + 1) − arctan(x)) dx
0
" ! !#a
ln(1 + (x + 1)2 ) ln(1 + x2 )
= lim arctan(x + 1) − − arctan x −
a→∞ 2 2 1
Information for Students in Lecture Section 002 of MATH 141 2009 01 2268

" #a !
1 1 + x2 1 2
= lim arctan(x + 1) − arctan x + ln = 0 − ln .
a→∞ 2 1 + (x + 1)2 1 2 5

4. BRIEF SOLUTIONS R is defined to be the region in the first quadrant enclosed by the
curves 2y = x, y = 2x, and x2 + y2 = 5.

(a) [4 MARKS] The region R is rotated about the line x = −1. Give an integral or sum
of integrals whose value is the volume of the resulting solid.
Solution:
Using “Washers”:
Z 1 ! Z 2 p 2  y 2 !
2 4 2
π (2y + 1) − ( + 1) dy + π 5 − y2 + 1 − + 1 dy
0 2 1 2
Using Cylindrical Shells:
Z 1  Z 2
x √
2
x
2π (x + 1) 2x − dx + 2π (x + 1) 5 − x − dx
0 2 1 2
(b) [4 MARKS] Let L(a) denote the length of the portion of line y = a which lies inside
R. Express in terms of integrals — which you need not evaluate — the average of
the positive lengths L(a).
Solution:
Z 1 ! Z 2 p ! !
1 4 2
4
2y − dy + 5−y − dy
2 0 2 1 2
(c) [4 MARKS] Let C1 be the curve x(t) = t, y(t) = cosh t (0 ≤ t ≤ ln 2). Simplifying
your answer as much as possible, find the length of C1 .
Solution:
dx
= 1
dt
dy
= sinh t
s dt
!2 !2 p
dx dy
+ = 1 + sinh2 t = | cosh t| = cosh t
dt dt
Z ln 2
Length = cosh t dt
0
= [sinh t]ln
0
2

= sinh ln 2 − sinh 0
Information for Students in Lecture Section 002 of MATH 141 2009 01 2269

3ln 2 − e− ln 2
= −0
2
1
2−
= 2 = 3.
2 4
Information for Students in Lecture Section 002 of MATH 141 2009 01 2270

B.26 Supplementary Notes for the Lecture of Monday, April 06th, 2009
Distribution Date: Monday, April 06th, 2009, subject to correction

B.26.1 Final Examination in MATH 141 2008 01 (Version 1, continued)


5. SHOW ALL YOUR WORK!
(a) [8 MARKS] Evaluate the indefinite integral
Z
36
dx .
(x + 4)(x − 2)2
Solution: We observe that the degree of the numerator of the integrand is less than
the degree of the denominator; thus we do not need to divide denominator into
numerator and obtain a quotient and remainder. Next we need to find a Partial
Fraction decomposition of the integrand. Assuming a decomposition of the form
36 A B C
2
= + +
(x + 4)(x − 2) x + 4 x − 2 (x − 2)2
we obtain, by taking the right side to a common denominator,
36 = A(x − 2)2 + B(x + 4)(x − 2) + C(x + 4) .
Students should know two methods to find the values of these constants: either,
by assigning “convenient” values to x and thereby obtaining enough equations that
can be solved for the coefficients, or by comparing coefficients of powers of x on
the two sides of the equation. These methods, or a combination of them, yield
A = 1, B = −1, C = 6. Hence
Z Z !
36 1 1 6
dx = − + dx
(x + 4)(x − 2)2 x + 4 x − 2 (x − 2)2

x + 4 6
= ln − +C.
x−2 x−2
Z∞
36
(b) [4 MARKS] Determine whether dx converges. If it converges,
(x + 4)(x − 2)2
3
find its value.
 
Z  1 + 4 
a
36  a − 6 
dx = ln  − (ln 7 − 6)
3 (x + 4)(x − 2)2  2 a − 2 
1 −
a
→ 0 − 0 − (ln 7 − 6) = 6 − ln 7 as a → ∞.
Since the limit exists, the integral converges to the limiting value, 6 − ln 7.
Information for Students in Lecture Section 002 of MATH 141 2009 01 2271

6. SHOW ALL YOUR WORK!


Showing all your work, evaluate each of the following:
Z p

(a) [4 MARKS] e x dx
Solution: This integral
√ can be solved by a series of substitutions; for example, the
x
substitution u = , which implies that x = 4u2 , dx = 8u du yields
2
Z p Z √ Z
√ x
t
e dt = e dx = 8 u · eu du ,
2

which can then be integrated by parts, as follows: with u = u, dv = eu du, du = du,


v = eu , so
Z p Z

e dt = 8 u · eu du
t

Z !
u u
= 8 u · e − e du
= 8 ((u − 1)eu ) + C
 √  √x
= 4 x−8 e 2 +C.

Z0
x
(b) [5 MARKS] √ dx
3 − 4x − 4x2
− 21
Solution:
√ p
3 − 4x − 4x2 = 4 − (1 + 2x)2
s
!2
1 + 2x
= 2 1− .
2
Z0 Z
x 1 0 x
√ dx = q   dx
3 − 4x − 4x2 2 − 12 1+2x 2
− 12 1− 2
Z 1 1
1 2 u− 2
= √ du
2 0 1 − u2
" # 12
1 √ 1
= − 1 − u2 − · arcsin u
2 2 0
Information for Students in Lecture Section 002 of MATH 141 2009 01 2272

 √  
1  3 1 1  
= − − arcsin  − (−1 − 0)
2 2 2 2
 √  √
1  3 1 π  1 π 3
= − − · + 1 = − − .
2 2 2 6 2 24 4
Z π
(c) [4 MARKS] sin2 t cos4 t dt .
0
Solution: To evaluate integrals of this type the method of the textbook is to replace
the square powers of sines and cosines by functions of the cosine of twice the angle.
The following is a variant of that method, using both the sine and the cosine of the
double angle.
Z π Z π
2 4 (2 sin t · cos t)2
sin t cos t dt = · cos2 t dt
0 0 4
Z π ! !
sin2 2t 1 + cos 2t
= · dt
0 4 2
Z π !
sin2 2t sin2 2t · cos 2t
= + dt
0 8 8
Z 2 !
π sin 2t (2 sin 2t · cos 2t)2
= −0 + 2 dt
8 3
Z π !
1 − cos 4t sin2 2t
= + dt
0 16 32
" #π
t sin 4t sin3 2t
= − +
16 64 48 0
π
= .
16

7. SHOW ALL YOUR WORK!


Consider the curve C2 defined by x = x(t) = 1 + e−t , y = y(t) = t + t2 .

(a) [2 MARKS] Determine the coordinates of all points where C2 intersects the x-axis.
Solution:

y = 0 ⇒ t + t2 = 0 ⇒ t = 0, −1
t = 0 ⇒ (x, y) = (2, 0)
t = −1 ⇒ (x, y) + (1 + e, 0) ,

so the points of intersection with the x axis are (2, 0) and (1 + e, 0).
Information for Students in Lecture Section 002 of MATH 141 2009 01 2273

(b) [2 MARKS] Determine the coordinates of all points of C2 where the tangent is
horizontal.
Solution:

dx
= −e−t
dt
dy
= 1 + 2t
dt
dy
dy dt dy 1
=0 ⇒ dx
=0⇒
=0⇒t=−
dx dt
dt 2
!
1 √ 1
t=− ⇒ (x, y) = 1 + e, − .
2 4

(c) [6 MARKS] Determine the area of the finite region bounded by C2 and the x-axis.
Z2
Solution: The area is y dx =
1+ 1e

Z 0 Z 0 
dx 
y(t) · (t) dt = t + t2 −e−t dt
−1 dt −1
h  i0
= e−t (t + 1) + e−t t2 + 2t + 2
−1
h  i0
−t 2
= e · t + 3t + 3
−1
= 3 − e(1 − 3 + 3) = 3 − e .

The preceding evaluation required two indefinite integrals that could be obtained
by integration by parts:
• Taking u = t, v0 = e−t , which imply that u0 = 1, v = −e−t ,
Z Z
t · e dt = −te + e−t dt = −e−t (t + 1) + C
−t −t

• Taking U = t2 , V 0 = e−t , which imply that U 0 = 2t, V = −e−t ,


Z Z
t · e dt = −t e + 2t · e−t dt = −(t2 + 2t + 2)e−t + C .
2 −t 2 −t
Information for Students in Lecture Section 002 of MATH 141 2009 01 2274

B.27 Supplementary Notes for the Lecture of Wednesday, April 08th,


2009
Distribution Date: Wednesday, April 08th, 2009, subject to correction

B.27.1 Final Examination in MATH 141 2008 01 (Version 1, conclusion)


8. SHOW ALL YOUR WORK!

(a) [5 MARKS] Showing all your work, determine whether the series
X

√ √ √ 
n n+2− n−2
n=2

is convergent or divergent.
Solution: This is a telescoping series.

X
N
√ √ √ 
n n+2− n−2
n=2
X
N
√ √ X
N−2
√ √
= n· n+2− m· m+2
n=2 m=0
√ p p
= − 3 + N(N + 2) + (N − 1)(N + 1)
→ ∞ as N → ∞ .

As the partial sums are approaching infinity, they do not have a finite limit, and the
series is, by definition, divergent.
Alternatively, one could argue that

√ √ √  n (n + 2 − n + 2)
n n+2− n−2 = √ √
n+2+ n−2

4 n
= √ √
n+2+ n−2
4
= q q → 2 , 0 as n → ∞.
2 2
1+ n + 1− n

By “The” Test for Divergence, the series diverges.


Information for Students in Lecture Section 002 of MATH 141 2009 01 2275

(b) [5 MARKS] Showing all your work, determine whether the following sequence
converges; if it converges, find its limit:
a1 = 3.
a2 = 3.14
a3 = 3.1414
a4 = 3.141414
a5 = 3.14141414
a6 = 3.1414141414
etc., where each term after a2 is obtained from its predecessor by the addition on
the right of the decimal digits 14.
!
1 1 1
an = 3 + 0.14 1 + + + ... +
100 1002 100n−1
 
1
0.14 1 − 100 n 0.14 311
= 3+ 1
→3+ = as n → ∞ .
1 − 100 0.99 99

9. SHOW ALL YOUR WORK!


Curves C3 and C4 , respectively represented by polar equations
r = 4 + 2 cos θ (0 ≤ θ ≤ 2π) (97)
and
r = 4 cos θ + 5 (0 ≤ θ ≤ 2π) , (98)
divide the plane into several regions.
(a) [8 MARKS] Showing all your work, carefully find the area of the one region which
is bounded by C3 and C4 and contains the pole.
Solution: (cf. Figure 25 on page 2276 of these notes) Solving the given!equations !
2π 4π 2π 4π
yields θ = , and r = 3. The points of intersection are 3, , 3, .
3 3 3 3
(Strictly speaking, the student should also solve using equations −r = 4 + cos(−θ)
and −r = 4 cos(−θ)+5, taking all 2×2 combinations of the equations, in case a point
of intersection appeared on the two given curves with different sets of coordinates;
that would not have yielded any new points of intersection in this problem. It could
also have been necessary to check for the curves’ possibly passing through the pole
— but that does not happen in this problem, since, when we set r = 0, we obtain
an equation for θ which cannot be solved.)
There are several different ways of finding the area in question.
Information for Students in Lecture Section 002 of MATH 141 2009 01 2276

-2 0 2 4 6 8
0

-2

-4

-6

Figure 25: The curves with equations r = 4 + 2 cos θ, r = 4 cos θ + 5

i. Find the area of the small oval (a limaçon) and subtract that of the region
on the left. The area of the limaçon is
Z Z
1 2π 2 1 2π
(2 cos θ + 4) dθ = (2(1 + cos 2θ) + 16 cos θ + 16) dθ
2 0 2 0
1
= [18θ + sin 2θ + 16 sin θ]2π
0 = 18π .
2
The area of the region to be subtracted is
Z
1 π 
2· (4 + 2 cos θ)2 − (4 cos θ + 5)2 dθ
2 2π3
Information for Students in Lecture Section 002 of MATH 141 2009 01 2277

Z π
= ((16 + 16 cos θ + 2(1 + cos 2θ)) − (8(1 + cos 2θ) + 40 cos θ + 25)) dθ

3

= [−15θ − 3 sin 2θ − 24 sin θ]π2π


3
 √ √ 
 3 3 
= (−15π − 0 − 0) − −10π + 3 · − 24 · 
2 2

21 3
= − 5π ,
2
√ √
21 3 21 3
so the area of the region in question is 18π − + 5π = 23π − .
2 2
ii. Find the area of the larger outer oval and subtract the area of the region
to the right of the desired region. The area bounded by the larger oval is
Z Z
1 2π 2 1 2π
(4 cos θ + 5) dθ = (8(1 + cos 2θ) + 40 cos θ + 25) dθ
2 0 2 0
1
= [33θ + 4 sin 2θ + 40 sin θ]2π 0 = 33π .
2
The area to be subtracted is
Z 2π3  
1 2π
2· (4 cos θ + 5)2 − (4 + 2 cos θ)2 dθ = [15θ + 3 sin 2θ + 24 sin θ]03
2 0
 √ √ 
 3 3 24 3 
= 10π − +  − 0
2 2

21 3
= 10π + .
2
Thus the net area is equal to
 √  √
 21 3  21 3
33π − 10π +  = 23π − .
2 2
iii. Join the pole by line segments to the points of intersection of the two
curves. Then compute the areas of the two subregions of the desired region
bounded by these line segments. The area to the right of the line segments is
Z 2π3 Z 2π3
1 2
2 (2 cos θ + 4) dθ = (2(1 + cos 2θ) + 16 cos θ + 16) dθ
2 0 0

= [18θ + sin 2θ + 16 sin θ]03
 √ √  √
 3 16 3  15 3
= 12π − +  − 0 = 12π + .
2 2 2
Information for Students in Lecture Section 002 of MATH 141 2009 01 2278

The area to the left of the line segments is


Z
1 π
2 (4 cos θ + 5)2 dθ
2 2π3
= [33θ + 4 sin 2θ + 40 sin θ]π2π
3

= 11π − 18 3 .

21 3
Summing these two areas yields 23π − .
2
(b) [4 MARKS] Find another equation — call it (98*) — that also represents C4 , and
has the property that there do not exist coordinates (r, θ) which satisfy equations
(97) and (98*) simultaneously. You are expected to show that equations (97) and
(98*) have no simultaneous solutions.
Solution: In my discussion of the preceding part of the problem I have shown (by
replacing (r, θ) by (−r, θ + π)) that an alternative equation for the second curve is
−r = 4 cos(−θ)+5, equivalently r = −4 cos θ−5. When this equation is solved with
3
r = 4 + 2 cos θ, we obtain, as a consequence, that cos θ = − , which is impossible.
2
Information for Students in MATH 141 2009 01 3001

C Problem Assignments from Previous Years


C.1 1998/1999
The problem numbers listed below refer to the textbook in use at that time, [31], [33]. For
many of the problems there are answers in the textbook or in the Student Solution Manual
[34].

C.1.1 Assignment 1

§5.2: 5, 11, 15, 21, 29

§5.3: 3, 9, 15, 35, 47

§5.4: none

§5.5: 17, 27, 33, 41

§5.6: 47, 55, 59, 65

§5.7: 21, 27, 33, 39, 45, 51, 57

§5.8: 33, 39, 45, 51, 57

C.1.2 Assignment 2

§6.1: none

§6.2: 3, 9, 15, 21, 27, 31, 35, 41

§6.3: 3, 9, 15, 21, 27, 31, 39, 43

§6.4: 3, 9, 15, 21, 27, 31, 35, 41

§3.8: none

Chapter 7: none
Information for Students in MATH 141 2009 01 3002

C.1.3 Assignment 3

§8.2: 5, 13, 21, 29, 39, 45, 53

§9.2: 5, 13, 21, 29, 39

§9.3: 5, 13, 21, 29, 39, 41

§9.4: 5, 13, 21, 29, 39

§9.5: 5, 9, 17, 21, 29, 33

§9.6: 5, 9, 17, 21, 29, 33

C.1.4 Assignment 4

§9.7: 13, 17, 21, 25, 29, 33

§9.8: 21, 23, 29, 33, 39

§10.2: 39, 41, 43, 45, 47, 49, 51, 53, 57

§10.3: 9, 13, 17, 21, 23, 29, 33, 35

§10.4: 3, 5, 9, 13

C.1.5 Assignment 5

§11.2: 9, 17, 23, 33, 39

§11.3: 3, 9, 15, 21, 29, 35, 47

§11.4: 3, 9, 15, 21, 29, 35, 45, 47

§11.5: 3, 9, 15, 21, 23

§11.6: 3, 9, 15, 21, 29

§11.7: 3, 9, 15, 21, 27, 33


Information for Students in MATH 141 2009 01 3003

C.2 1999/2000
(Students had access to brief solutions that were mounted on the web.)

C.2.1 Assignment 1
Before attempting problems on this assignment you are advised to try some “easy” problems
in the textbook. In most of the following problems there is a reference to a “similar” problem
in the textbook. You should always endeavour to show as much of your work as possible, and
to reduce your solution to “simplest terms”. Remember that the main reason for submitting
this assignment is to have an opportunity for your tutor to grade your work; the actual grade
obtained should be of lesser significance.
In Exercises 1-5 below, evaluate the indefinite integral, and verify by differentiation:
Z !
3 1
2 −3
1. (cf. [31, Exercise 5.2.5, p. 294]) − 5x 2 − x + 4x dx
x4
Z !
3 2
2. − dx
x 1 + x2
Z  
2
3. (cf. [31, Exercise 5.2.13, p. 294]) xe x − e4x dx
Z

4. (cf. [31, Exercise 5.2.19, p. 294]) (1 − x)(2x + 3)2 dx
Z
5. (cf. [31, Exercise 5.2.27, p. 294]) (4 cos 8x − 2 sin πx + cos 2πx − (sin 2π)x) dx

6. (cf. [31, Example 5.2.8, p. 289]) Determine the differentiable function y(x) such that
dy 1  1 π
= √ and y 2− 2 = .
dx 1 − x2 2
7. (This is [31, Exercise 5.2.51, p. 295]
! written in purely mathematical terminology.) Solve
d dy dy
the initial value problem: = sin x, where y = 0 and = 0 when x = 0. [Hint:
dx dx dx
dy
First use one of the initial values to determine the general value of from the given
dx
“differential equation”; then use the second initial value to determine y(x) completely.]

8. ([31, Exercise 5.3.4, p. 306]) Write the following in “expanded notation”, i.e. without
P X
6
using the symbol : (2 j − 1).
j=1
Information for Students in MATH 141 2009 01 3004

9. (cf. [31, Exercise 5.3.18, p. 306]) Write the following sum in “summation notation”:
x3 x5 x7 x999
x− + − + ... ±
3 5 7 999
where the signs are alternating +, −, +, −, ... The sign of the last term has not been given
— you should determine it.
10. (cf. [31, Example 5.3.6, p. 302]) Given that
X
n
n(n + 1) X
n
n(n + 1)(2n + 1) X
n
n2 (n + 1)2
i= , i2 = , i3 = ,
i=1
2 i=1
6 i=1
4

(n + 1)3 + (n + 2)3 + ... + (2n)3


determine lim .
n→∞ n4

C.2.2 Assignment 2
1. Evaluate the following integrals:
Z 3
(a) (x − 1)4 dx
1
Z 1
(b) (2e x − 1)2 dx
Z0 π
(c) sin 4x dx.
0

2. Interpreting the following integral as the area of a region, evaluate it using known area
formulas: Z 6√
36 − x2 dx.
0

3. Use properties of integrals to establish the following inequality without evaluating the
integral: Z 1 Z 1
1 1
√ dx ≤ 3
dx.
0 1+ x 0 1+ x

4. Deduce the Second Comparison Property of integrals from the First Comparison Prop-
erty [31, p. 325, §5.5].
5. Apply the Fundamental Theorem of Calculus [31, p. 331, §5.6] to find the derivative of
the given function: Z x
(t2 + 2)15 dt.
−1
Information for Students in MATH 141 2009 01 3005

6. Differentiate the functions


Z x3
(a) cos t dt
0
Z 3x
(b) sin t2 dt.
1

dy √
7. Solve the initial value problem = 1 + x2 , y(1) = 5 . Express your answer in
dx
terms of a definite integral (which you need not attempt to evaluate). This problem can
be solved using the methods of [31, Chapter 5].

8. Evaluate the indefinite integrals:


Z √
(a) 2x 3 − 2x2 dx
Z
(b) x2 sin(3x3 ) dx
Z
x+3
(c) 2
dx
x + 6x + 3
9. Evaluate the definite integrals:
Z 8 √
(a) t t + 2 dt
0
Z π/2
(b) (1 + 3 sin η)3/2 cos η dη
0
Z π
(c) sin2 2t dt.
0

10. Sketch the region bounded by the given curves, then find its area:

(a) x = 4y2 , x + 12y + 5 = 0


π
(b) y = cos x, y = sin x, 0≤x≤ .
4
x 2 y2
11. Prove that the area of the ellipse + = 1 is A = πab. This problem can be
a2 b2
solved using the methods of [31, Chapter 5]. It is not necessary to use methods of [31,
Chapter 9].

UPDATED TO April 12, 2009


Information for Students in MATH 141 2009 01 3006

C.2.3 Assignment 3
In all of these problems you are expected to show all your work neatly. (This assignment is
only a sampling. Your are advised to try other problems from your textbook; solutions to some
can be found in the Student Solution Manual [32].)

1. [31, Exercise 6.1.6, p. 382] As n → ∞, the interval [2, 4] is to be subdivided into n


subintervals of equal length ∆x by n − 1 equally spaced points x1 , x2 , ..., xn−1 (where
Xn
1
x0 = 2, xn = 4). Evaluate lim ∆x by computing the value of the appropriate
n→∞
i=1
x i
related integral.

2. (a) [31, Exercise 6.2.6, p. 391] Use the method of cross-sections to find the volume of
the solid that is generated by rotating the plane region bounded by y = 9 − x2 and
y = 0 about the x-axis.
(b) (cf. Problem 2a) Use the method of cylindrical shells to find the volume of the solid
that is generated by rotating the plane region bounded by y = 9− x2 and y = 0 about
the x-axis.
(c) Use the method of cross-sections to find the volume of the solid that is generated
by rotating the plane region bounded by y = 9 − x2 and y = 0 about the y-axis.
(d) (cf. Problem 2c) Use the method of cylindrical shells to find the volume of the solid
that is generated by rotating the plane region bounded by y = 9− x2 and y = 0 about
the y-axis.

3. (a) [31, Exercise 6.2.24, p. 392] Find the volume of the solid that is generated by
rotating around the line y = −1 the region bounded by y = 2e−x , y = 2, and x = 1.
(b) (cf. Problem 3a) Set up an integral that would be obtained if the method of cylin-
drical shells were used to represent the volume of the solid that is generated by
rotating around the line y = −1 the region bounded by y = 2e−x , y = 2, and x = 1.
YOU ARE NOT EXPECTED TO EVALUATE THE INTEGRAL.

4. (cf. [31, Exercise 6.2.40, p. 392]) The base of a certain solid is a circular disk with di-
ameter AB of length 2a. Find the volume of the solid if each cross section perpendicular
to AB is an equilateral triangle.

5. (a) [31, Exercise 6.3.26, p. 401] Use the method of cylindrical shells to find the volume
of the solid generated by rotating around the y-axis the region bounded by the
1
curves y = , y = 0, x = 0, x = 2.
1 + x2
Information for Students in MATH 141 2009 01 3007

(b) (cf. Problem 5a) Use the method of cross sections to find the volume of the solid
generated by rotating around the y-axis the region bounded by the curves y =
1
, y = 0, x = 0, x = 2.
1 + x2
e x + e−x
6. (cf. [31, Exercise 7.3.69, p. 450]) Find the length of the arc of the curve y =
2
between the points (0, 1) and (ln 2, 2).

7. (a) [31, Exercise 6.4.30, p. 411] Find the area of the surface of revolution generated
by revolving the arc of the curve y = x3 from x = 1 to x = 2 around the x-axis.
(b) (cf. 7a) Set up an integral for, BUT DO NOT EVALUATE, the area of the surface
of revolution generated by revolving the arc of the curve y = x3 from x = 1 to x = 2
around the y-axis.

8. [31, Exercise 7.2.44, p. 442] Evaluate the indefinite integral


Z
x+1
2
dx
x + 2x + 3
Z x
t2
9. (cf. [31, Exercise 7.2.36, p. 442]) Determine the value of the function f (x) = dt
−1 8 − t3
for any point x < 2.

10. (cf. [31, Exercise 7.3.70, p. 450]) Find the area of the surface generated by revolving
around the x-axis the curve of Problem 6.

C.2.4 Assignment 4
1. Differentiate the functions:

(a) sin−1 (x50 )


(b) arcsin(tan x)
(c) cot−1 e x + tan−1 e−x

2. Showing all your work, evaluate the integrals:


Z
dx
(a) √
1 − 4x2
Z
dx
(b) √
2 x(1 + x)
Information for Students in MATH 141 2009 01 3008

Z
ex
(c) dx
1 + e2x
Z √ √
cot y csc y
(d) √ dy
y
Z
(ln t)8
(e) dt
t
Z
(f) tan4 2x sec2 2x dx

(g) THIS PROBLEMZ SHOULD BE OMITTED. IT MAY BE INCLUDED IN AS-


x2
SIGNMENT 5. √ dx
16x2 + 9
3. Use integration by parts to compute the following integrals. Show all your work.
Z
(a) t cos t dt
Z

(b) y ln y dy

(c) THIS PROBLEM Z SHOULD BE OMITTED. IT MAY BE INCLUDED IN AS-


SIGNMENT 5. x2 arctan x dx
Z
(d) csc3 x dx
Z
(e) ln(1 + x2 ) dx

4. Showing all your work, evaluate the following integrals:


Z
(a) cos2 7x dx
Z
(b) cos2 x sin3 x dx
Z
sin3 2x
(c) dx
cos2 2x
Z
(d) sec6 2t dt
Information for Students in MATH 141 2009 01 3009

C.2.5 Assignment 5
Z
x3
1. [31, Exercise 9.5.6, p. 540] Find dx. (Your solution should be valid for x
x2 + x − 6
in any one of the intervals x < 3, −3 < x < 2, x > 2.)
Z
1
2. [31, Exercise 9.5.8, p. 540] Find dx.
(x + 1)(x2 + 1)
Z
x2
3. (a) [31, Exercise 9.5.23] Find dx.
(x + 2)3
(b) Find the volume of the solid of revolution generated by the region bounded by
x
y= 3
, y = 0, x = 1, and x = 2 about the x-axis.
(x + 2) 2
(c) Find the volume of the solid of revolution generated by the region bounded by
x
y= 3
, y = 0, x = 1, and x = 2 about the y-axis.
(x + 2) 2
4. [31, Exercise 9.5.38, p. 540] Make a preliminary substitution before using the method
of partial fractions: Z
cos θ
2

sin θ(sin θ − 6)
5. [31,
Z Exercise 9.6.6, p. 547] Use trigonometric substitutions to evaluate the integral
x2
√ dx.
9 − 4x2
6. [31,
Z Exercise 9.6.26, p. 547] Use trigonometric substitutions to evaluate the integral
1
dx.
9 + 4x2
7. [31,√Exercise 9.6.35, p. 547] Use trigonometric substitutions to evaluate the integral
Z
x2 − 5
dx.
x2
Z √
8. [31, Exercise 9.7.14, p. 553] Evaluate the integral x 8 + 2x − x2 dx.

9. [31, Exercise 9.8.17, p. 561] Determine Z whether the following improper integral con-

x
verges; if it does converge, evaluate it: 2
dx.
−∞ x + 4

10. [31, Exercise 9.8.27, p. 561] Determine Z whether the following improper integral con-

verges; if it does converge, evaluate it: cos x dx.
0
Information for Students in MATH 141 2009 01 3010

11. (cf. [31, Exercise 9.8.14, p. 561]) DetermineZ +8whether the following improper integral
1
converges; if it does converge, evaluate it: 2
dx.
−8 (x + 4) 3

12. [31, Exercise 10.2.2, p. 580] Find two polar coordinate representations, one with r ≥ 0,
and the other with r ≤ 0 for the points with the following rectangular coordinates:

(a) (−1, −1),



(b) ( 3, −1),
(c) (2, 2),

(d) (−1, 3),
√ √
(e) ( 2, − 2),

(f) (−3, 3).

13. For each of the following curves, determine — showing all your work — equations in
both rectangular and polar coordinates:

(a) [31, Exercise 10.2.20, p. 580] The horizontal line through (1, 3).
(b) [31, Exercise 10.2.26, p. 580] The circle with centre (3, 4) and radius 5.

14. (a) [31, Exercise 10.2.56, p. 581] Showing all your work, find all points of intersection
of the curves with polar equations r = 1 + cos θ and r = 10 sin θ.
(b) Showing all your work, find all points of intersection of the curves with polar equa-
tions r2 = 4 sin θ and r2 = −4 sin θ.
[Note: The procedure sketched in the solution of [31, Example 10.2.8, p. 579] for finding
points of intersection is incomplete. Your instructor will discuss a systematic procedure
in the lectures.]

C.2.6 Assignment 6
1. Find the area bounded by each of the following curves.

(a) r = 2 cos θ,
(b) r = 1 + cos θ.

2. Find the area bounded by one loop of the given curve.

(a) r = 2 cos 2θ,


(b) r2 = 4 sin θ.
Information for Students in MATH 141 2009 01 3011

3. Find the area of the region described.



(a) Inside both r = cos θ and r= 3 sin θ.
(b) Inside both r = 2 cos θ and r = 2 sin θ .
4. Eliminate the parameter and then sketch the curve.
(a) x = t + 1, y = 2t2 − t − 1.
(b) x = et , y = 4e2t .
(c) x = sin 2πt, y = cos 2πt; 0 ≤ t ≤ 1. Describe the motion of the point (x(t), y(t)) as
t varies in the given interval.
5. Find the area of the region that lies between the parametric curve x = cos t, y =
sin2 t, 0 ≤ t ≤ π, and the x-axis.
6. Find the arc length of the curve x = sin t − cos t, y = sin t + cos t; π/4 ≤ t ≤ π/2.
7. Determine whether the sequence an converges, and find its limit if it does converge.
n2 − n + 7
(a) an = ,
2n3 + n2

1 + (−1)n n
(b) an = ,
(3/2)n
(c) an = n sin πn,
!n
n−1
(d) an = .
n+1
8. Determine, for each of the following infinite series, whether it converges or diverges. If
it converges, find its sum.
(a) 1 + 3 + 5 + 7 + . . . + (2n − 1) + . . . ,
(b) 4 + 43 + . . . + 34n + . . . ,
X∞
(c) (5−n − 7−n ),
n=1
∞ 
X e n
(d) .
n=1
π
∞  n
X x
9. Find the set of all those values of x for which the series is a convergent geomet-
3
n=1
ric series, then express the sum of the series as a function of x.
Information for Students in MATH 141 2009 01 3012

10. Find the Taylor polynomial in powers of x − a with remainder by using the given values
of a and n.

(a) f (x) = sin x; a = π/6, n = 3.


1
(b) f (x) = ; a = 5, n = 5 .
(x − 4)2
11. Find the Maclaurin series of the function e−3x by substitution in the series for e x .

12. Find the Taylor series for f (x) = ln x at the point a = 1.

13. Use comparison tests to determine whether each of the following infinite series converge
or diverge.
X

1
(a) ,
n=1
1 + 3n
X∞ √
n
(b) 2
,
n=1
n +n
X

sin2 (1/n)
(c) .
n=1
n2

C.3 2000/2001
(In the winter of the year 2001 Assignments based on WeBWorK were used, although the
experiment had to be terminated in mid-term because of technical problems.)

C.4 2001/2002
This was the first time WeBWorK assignments were used exclusively in this course.

C.5 MATH 141 2003 01


WeBWorK assignments were used exclusively for assignments. The questions are not avail-
able for publication.

C.6 MATH 141 2004 01


WeBWorK assignments were used exclusively for assignments. The questions are not avail-
able for publication.
Information for Students in MATH 141 2009 01 3013

C.7 MATH 141 2005 01


WeBWorK assignments were used for online assignments; the questions are not available for
publication. In addition, these written assignments were intended to provide students with in-
dividualized opportunities to work problems for which the textbook often provided examples;
at the time, appropriate materials available from WeBWorK for this purpose were limited. The
individualization was often based on the student number.

C.7.1 Written Assignment W1


Your written assignments will usually be mounted on the WeBWorK site, and will usually be
individualized, that is, your problems will not be exactly the same as those of other students.
This first written assignment is based on your WeBWorK assignment R1 . Subsequent written
assignments may be designed in other ways. Because of its general form, it was possible to
release this assignment in the document Information for Students in MATH 141 2005 01; some
of the other assignments may appear only on your WeBWorK site.
Your completed assignment must be submitted together with your solutions to quiz Q1 ,
inside your answer sheet for that quiz. No other method of submission is acceptable.

Purpose of the written assignments These assignments are designed to help you learn how
to write full solutions to problems. While they carry a very small weight in the computation of
your final grade, conscientious completion of the assignments should help you substantially in
learning the calculus, and help prepare you for your final examination.

Certificate Your assignment will not be graded unless you attach or include the following
completed certificate of originality:

I have read the information on the web page

http://www.mcgill.ca/integrity/studentguide/,

and assert that my work submitted for W1 and R1 does not violate McGill’s
regulations concerning plagiarism.

Signature(required) Date(required)

The assignment questions Your assignment consists of the following problems on your ver-
sion of R1 :
Information for Students in MATH 141 2009 01 3014

##3, 4, 5, 6, 7, 8

(Teaching Assistants are not primarily checking for plagiarism; but, if they detect it, they may
be obliged to report any apparent violations to the Associate Deans.)

Complete solutions are required It is not enough to give the correct answer; in fact, the
numerical answer alone may be worth 0 marks. You should submit a full solution, similar to
solutions in Stewart’s textbook or the Student Solutions Manual, which are where you should
look if you have doubts about the amount of detail required in a solution.

Use of calculators You are expected to complete the entire assignment without the use of
a calculator. In particular, you are expected to be familiar with the values of trigonometric
functions at “simple” multiples and submultiples of π.

Not all problems may be graded On all of the written assignments it is possible that the
Teaching Assistant will grade only a small number of the solutions you submit. The numbers
of the questions that will be graded will not be announced in advance, even to the tutor. For
that reason you are advised to devote equal attention to all of the problems.

C.7.2 Written Assignment W2


Written assignment W2 is based on your WeBWorK assignment R3 , but some problems are
being modified. Your completed assignment must be submitted together with your solutions to
quiz Q2 , inside your answer sheet for that quiz. No other method of submission is acceptable.

Purpose of the written assignments These assignments are designed to help you learn how
to write full solutions to problems. While they carry a very small weight in the computation of
your final grade, conscientious completion of the assignments should help you substantially in
learning the calculus, and help prepare you for your final examination.

Certificate Your assignment will not be graded unless you attach or include the following
completed certificate of originality, signed in ink:
Information for Students in MATH 141 2009 01 3015

I have read the information on the web page

http://www.mcgill.ca/integrity/studentguide/,

and assert that my work submitted for W2 and R3 does not violate McGill’s
regulations concerning plagiarism.

Signature(required) Date(required)

The assignment questions Your assignment consists of the following problems on your ver-
sion of R1 :

1. Problem 1 of R3 , solved by integration with respect to x. Include in your solution a rough


sketch of the region, showing a typical element of area.

2. Problem 1 of R3 solved by integration with respect to y. This will require rewriting the
equations of the curves appropriately. You may assume without proof that
Z
ln x dx = x(ln x − 1) + C ,

a fact which you will see derived later in the course. Include in your solution a rough
sketch of the region, showing a typical element of area.

3. Problem 6 of R3 , evaluated using the Method of Washers. Include in your solution a


rough sketch of the plane region which generates the solid, showing a typical element of
area which will generate a typical washer.

4. Problem 6 of R3 , evaluated using the Method of Cylindrical Shells. Include in your


solution a rough sketch of the plane region which generates the solid, showing a typical
element of area which will generate a typical cylindrical shell.

Complete solutions are required It is not enough to give the correct answer; in fact, the
numerical answer alone may be worth 0 marks. You should submit a full solution, similar to
solutions in Stewart’s textbook or the Student Solutions Manual, which are where you should
look if you have doubts about the amount of detail required in a solution.

Use of calculators You are expected to complete the entire assignment without the use of a
calculator.
Information for Students in MATH 141 2009 01 3016

Not all problems may be graded On all of the written assignments it is possible that the
Teaching Assistant will grade only a small number of the solutions you submit. The numbers
of the questions that will be graded will not be announced in advance, even to the tutor. For
that reason you are advised to devote equal attention to all of the problems.

C.7.3 Written Assignment W3


Unlike the preceding written assignments Written Assignment W3 is not directly based on your
WeBWorK assignments, although some problems will be similar to WeBWorK assignment
problems. Your completed assignment must be submitted together with your solutions to quiz
Q3 , inside your answer sheet for that quiz. No other method of submission is acceptable.

Certificate Your assignment will not be graded unless you attach or include the following
completed statement of originality, signed in ink:

I have read the information on the web page

http://www.mcgill.ca/integrity/studentguide/,

and assert that my work submitted for W3 does not violate McGill’s regula-
tions concerning plagiarism.

Signature(required) Date(required)

The assignment questions The parameters in these problems are based on the digits of your
9-digit McGill student number, according to the following table:

Parameter name: A B C D E F G H J
Your student number:
Before starting to solve the problems below, determine the values of each of these integer
constants; then substitute them into the descriptions of the problems before you begin your
solution.
Z  
1. Showing all your work, systematically determine Ax2 + Bx + C e−x dx by repeated
integration by parts: no other method of solution will be accepted. Verify by differenti-
ation that your answer is correct.
Information for Students in MATH 141 2009 01 3017

2. Showing all your work, use trigonometric or hyperbolic substitutions to evaluate


Z Z Z
du du du
each of p , p , p .
u2 − (F + 1)2 u2 + (F + 1)2 −u2 + (F + 1)2

Verify that your answers are correct by differentiation.


j k h i
3. Let K = J+4 2
= J+42
.58 Showing all your work, develop for this integer K a reduction
formulaZ of the following type that can be used to evaluate
In (x) = xn (sin(K x)) dx in terms of In−2 (x):
Z Z
n n n−1
x (sin K x) dx = L · x cos K x + M · x sin K x + N xn−2 (sin K x) dx

where n ≥ 2 and L, M, N are constants that you are expected to determine only by
integration by parts. Again showing
R all your work, use the reduction formula you have
just determined to evaluate x2 sin K x dx, and test by differentiation the answer that it
gives — you should recover x2 sin K x.

4. Showing all your work, evaluate both of the integrals


Z Z
F+1 2
sin x · cos x dx and sinF+2 x · cos2 x dx

Complete solutions are required It is not enough to give the correct answer; in fact, the
final answer alone may be worth 0 marks. You should submit a full solution, similar to solu-
tions in Stewart’s textbook or the Student Solutions Manual, which are where you should look
if you have doubts about the amount of detail required in a solution.

Not all problems may be graded On all of the written assignments it is possible that the
Teaching Assistant will grade only a small number of the solutions you submit. The numbers
of the questions that will be graded will not be announced in advance, even to the tutor. If for
no other reason, you are advised to devote equal attention to all of the problems.

C.7.4 Written Assignment W4


Your completed assignment must be submitted together with your solutions to quiz Q4 , inside
your answer sheet for that quiz. No other method of submission is acceptable.
58
Determine Khfrom i J using the greatest integer function, defined
j k in your textbook, page 110. Your textbook
uses the notation 2 , but some authors write the function as J+4
J+4
2 .
Information for Students in MATH 141 2009 01 3018

Certificate Your assignment will not be graded unless you attach or include the following
completed statement of originality, signed in ink:

I have read the information on the web page

http://www.mcgill.ca/integrity/studentguide/,

and assert that my work submitted for W4 does not violate McGill’s regula-
tions concerning plagiarism.

Signature(required) Date(required)

The assignment questions Some of the parameters in these problems are based on the digits
of your 9-digit McGill student number, according to the following table:

Parameter name: A B M D E F G H J
Your student number:
Before starting to solve the problems below, determine the values of each of these integer
constants; then substitute them into the descriptions of the problems before you begin your
solution. It is not enough to give the correct answer; in fact, the final answer alone could be
worth 0 marks. You should submit a full solution, similar to solutions in Stewart’s textbook or
the Student Solutions Manual, which are where you should look if you have doubts about the
amount of detail required in a solution. Not all problems may be graded.

1. Showing
Z all your work, systematically determine the value of the following integral:
x
dx . Verify your work by differentiation of your answer: you
(x + J) (x − H − 1)1
2
should recover the integrand. (Systematically means that you to use the methods you
learned in this course for the treatment of problems of this type, even if you happen to
see some other method that could be used in this particular example.)

2. Showing all your work, use a substitution to transform the integrand into a rational
function, then integrate the particular integral that is assigned for your particular value
of G:
Z
1
If G = 1,4,or 7: √ dx
x− x+2
Z
cos 2x
If G = 0, 2, 5, or 8: dx
sin2 2x + sin 2x
Information for Students in MATH 141 2009 01 3019

Z
1
If G = 3, 6, or 9: √ dx
x−2−2 x+1
3. Problem 55, pages 504-505 of your textbook, describes a substitution discovered by Karl
Weierstrass (1815-1897) for the evaluation of rational functions of sin x and cos x into
an ordinary rational function. It states that if, for x such that −π < x < π, we define
1 − t2 2t
t = tan 2x , then cos x = and sin x = and that, at a consequence, dx =
1 + t2 1 + t2
2
dt . You are not asked to verify these facts. You are asked to use the substitution
1 + t2
to transform the following integral and then to evaluate it:
Z π
2 1
dx
π
3
1 + sin x − cos x

4. Use the trigonometric


R identities given in your textbook on page 487 to evaluate the fol-
lowing integral: cos Mx · cos Dx · sin x dx where M and D are the digits of your student
number, defined above.

5. Showing all your work, determine whether each of the following integrals is convergent
or divergent. If it is convergent, determine its value (again showing all your work):
Z 4
1
(a) dx
0 2+E ·F
Z ∞ 2
x −E−F
(b) dx
0 x2 + E + F
Z B+4
π
1
(c) dx
0 x sin((G + 2)x)

C.7.5 Written Assignment W5


Your completed assignment must be submitted together with your solutions to quiz Q5 , inside
your answer sheet for that quiz. No other method of submission is acceptable.

Certificate Your assignment will not be graded unless you attach or include the following
completed statement of originality, signed in ink:
Information for Students in MATH 141 2009 01 3020

I have read the information on the web page

http://www.mcgill.ca/integrity/studentguide/,

and assert that my work submitted for W5 does not violate McGill’s regula-
tions concerning plagiarism.

Signature(required) Date(required)

The assignment questions Some of the parameters in these problems are based on the digits
of your 9-digit McGill student number, according to the following table:

Parameter name: A B D E F G H J K
Your student number:
Before starting to solve the problems below, determine the values of each of these integer
constants; then substitute them into the descriptions of the problems before you begin your
solution. It is not enough to give the correct answer; in fact, the final answer alone could be
worth 0 marks. You should submit a full solution, similar to solutions in Stewart’s textbook or
the Student Solutions Manual, which are where you should look if you have doubts about the
amount of detail required in a solution. Not all problems may be graded.
 π 
1. For the point with polar coordinates H, − 2 ,
K +5
(a) find four other pairs of polar coordinates, two with r ≤ 0 and two with r > 0:
(b) Find the cartesian coordinates, assuming that the positive x-axis is along the polar
axis, the origin is at the pole, and the positive y-axis is obtained by turning the polar
axis through a positive angle of π2 .
2. For the point whose cartesian coordinates are (F 2 +1, F 2 −2), determine polar coordinates
(r, θ) with the following properties:
(a) r > 0, 0 ≤ θ < 2π
(b) r < 0, 0 ≤ θ < 2π
5π 9π
(c) r > 0, 2
≤θ< 2

3. For the following curve given in polar coordinates, determine, showing all your compu-
π
tations, the slope of the tangent at the point with θ = :
4
r = A + cos(B + 2)θ .
Information for Students in MATH 141 2009 01 3021

4. Showing all your work, find the area contained between the outer loop and the inner loop
of the curve
r = 1 + 2 sin θ .
Explain carefully how you have established the limits for your definite integral.

5. The curve C is given by the parametric equations

x = 1 + (C + 2)t2
y = t − (E 2 − 2)t3

d2 y
Showing all your work, determine the value of (t).
dx2
6. Giving an explanation, determine whether or not the following sequences converge.
Where a sequence converges, find its limit.

(a) an = ln n − ln(n + A + 1)
ln n
(b) an =
ln((A + 2)n)
√ √
(c) an = n2 + A2 − n2 + A2 + 2

C.8 MATH 141 2006 01


C.8.1 Solution to Written Assignment W1
Release Date: Friday, 27 January, 2006
The Assignment was posted to the class via WeBCT on January 2nd, 2006

Instructions to Students

1. Your solution to this assignment should be brought to your regular tutorial, during the
week 16-20 January, and folded inside your solution sheet to the quiz which will be
written at that time.

2. Your TA may make special arrangements for submission at other times, but, no solutions
are ever accepted after the end of the week.

3. Your solution must use the data on your own WeBWorK assignment, as described be-
low.
Information for Students in MATH 141 2009 01 3022

The Assignment Question Problem 7 of WeBWorK assignment R1 requires that you eval-
uate a definite integral of the form
Z C
(Ex2 − Ax + B) dx,
D

where A, B, C, D, E are various combinations of constants, individualized for each student.


Your written assignment is to evaluate a simplified version of this integral as the limit of a
sequence of Riemann sums, using left endpoints59 . You may simplify your problem only in
the following way: you may take the lower limit of the integral (here called D) to be 0; this
is purely to make the algebra a little easier. You should model your proof on that given in [7,
Example 2(b), pp. 383-385]. You are expected to provide a full solution for your version of the
problem — it is not enough to supply the correct answer, and you must solve the version of
the problem on your own WeBWorK assignment R1 . In your solution you will need to use the
well known formulæ for the sums of the 1st and 2nd powers of the natural numbers from 1 to n;
these formulæ are in your textbook [7, Formulæ ##4, 5, p. 383], and you will not be expected
to prove them here.60 (This is a time-consuming exercise; the purpose of the assignment is to
ensure that you will have correctly solved a problem of this type during the term.)

The Solution (This solution has been composed with variables, so that it will produce valid
numerical solutions for all students. The particular solution for your version of the problem
should look much simpler, because, in place of all variables except n, there will be specific
integers.)
The length of each of n subintervals is
C−D
∆x = .
n
C − D 
Thus xi = D + i∆x = D + i . Since we are using left endpoints we apply [7, top
n
formula, p. 381]
Z C  
Ex2 − Ax + B dx
D
X
n  
2
= lim Exi−1 − Axi−1 + B · ∆x
n→∞
i=1
X
n !
2 2D(C − D) (C − D)2
= lim E D + · (i − 1) + · (i − 1)2
n→∞
i=1
n n2
59
Note that the solution given in the textbook uses right endpoints.
60
Of course, these formulæ may be proved by induction, and students who took MATH 140 2005 09 should
know how to write up such proofs if they had to.
Information for Students in MATH 141 2009 01 3023

 C−D  
−A D + · (i − 1) + B · ∆x
 n 
 Xn
(2DE − A)(C − D) X
n
E(C − D)2 X
n 
2
 2
= lim (ED − AD + B) 1+ (i − 1) + 2
(i − 1)  · ∆x
n→∞
i=1
n i=1
n i=1
Now
X
n
1 = sum of n 1’s
i=1
= n
X
n X
n−1
(i − 1) = j
i=1 j=1
(n − 1)n
= by [7, Formula 4, p. 383]
2
X
n X
n−1
(i − 1)2 = j2
i=1 j=1
(n − 1)n(2n − 1)
= by [7, Formula 5, p. 383].
6
Substituting the values of these sums yields
Z C  
Ex2 − Ax + B dx
D
C−D (2DE − A)(C − D) (n − 1)n
= lim · (ED2 − AD + B)n + ·
n→∞ n n 2
2
!
E(C − D) (n − 1)n(2n − 1)
+ ·
n2 6
!
2 (2DE − A)(C − D)2 1
= lim (ED − AD + B)(C − D) + 1−
n→∞ 2 n
3
! !!
E(C − D) 1 1
+ · 1− 2−
6 n n
(2DE − A)(C − D)2 E(C − D)3
= (ED2 − AD + B)(C − D) + ·1+ ·1·2
2 6
(2DE − A)(C − D)2 E(C − D)3
= (ED2 − AD + B)(C − D) + +
    2 3
3
E C −D 3 A C −D2 2
= − + B(C − D)
3 2
It was suggested that you simplify your problem by taking D = 0; in that case the value would
have worked out to be
EC 3 AC 2
− + BC .
3 2
Information for Students in MATH 141 2009 01 3024

The Grading Scheme The assignment was to be graded OUT OF A TOTAL OF 10 MARKS.
(In the version of this solution circulated to TA’s, there followed some technical details about the grad-
ing.)

C.8.2 Solution to Written Assignment W2


Release Date: Mounted on the Web on February 8th, 2006; corrected on March 16th, 2006
Solutions were to be submitted inside answer sheets to Quiz Q2 , at tutorials during the period
January 30th – February 2nd, 2006.

The Problem. In [7, Example 6, p. 441] of your textbook, the author solves the following
problem: “Find the area enclosed by the line y = x − 1 and the parabola y2 = 2x + 6,” by
integrating with respect to y. The textbook then states the following: “We could have found
the area in Example 6 by integrating with respect to x instead of y, but the calculation is
much more involved. It would have meant splitting the region in two, and computing the
areas labelled A1 and A2 in Figure 14. The method we used in Example 6 is much easier.” Your
assignment is to solve the problem by integrating with respect to x, and you may find the figure
in the√textbook helpful. The computation √ is not really very hard, but it does involve working
with 2. You should not approximate 2; just write it that way and work with it carefully,
and the square roots will cancel by the time you finish your solution.
You know the numerical answer to this question; the purpose of the assignment is to nudge
you into solving problems in two ways, so that you can verify your work; and to show you that
there really is nothing to fear, even if you do happen to choose the “wrong” way to approach a
problem.
Your TA will be alert to the possibility that students might be copying their solutions from
others. Please write up your own solution, so that your TA will not have to waste everyone’s
time by sending exact copies to the disciplinary officer of the Faculty. You need to know how
to solve problems of this type, since you could be expected to demonstrate that ability at a
future quiz or on the examination.

The Solution. The integration must be carried out separately for two subregions. This is
because the area to the left of the line x = −1 is bounded above and below by the parabola;
while the area to the right of x = −1 is bounded above by the parabola, and below by the line
y = x − 1. The method you know for finding the area between two curves requires that the
equations of the curves be written as the graphs of functions of x. But the parabola y2 = 2x + 6
is not the graph of a function, since it crosses some vertical lines twice. However, we can
factorize the equation in the form
 p  p 
y − 2(x + 3) y + 2(x + 3) = 0 :
Information for Students in MATH 141 2009 01 3025

the parabola
√ is the graph of two functions — the upper arm √ of the parabola is the graph of
y = 2(x + 3), while the lower arm is the graph of y √ = − 2(x + 3). To the left of x = −1
the region
√ A 1 is bounded above by the graph of y = 2(x + 3), and below by the graph of
y = − 2(x + 3); thus √ the height of √ the vertical element
√ of area is the difference between these
two functions, i.e., 2(x + 3) − (− 2(x + 3)) = 2 2(x + 3), and this will be the integrand √ for
the integral; the area to the right of x = −1 is bounded above by the parabola √ y = 2(x + 3),
and below by the line y = x−1, so the element of area for the integral will be 2(x + 3)−(x−1).
The total area is thus
Z −1 p Z 5p 
2 2(x + 3) dx + 2(x + 3) − (x − 1) dx
−3 −1
" #−1 " !#5
√ 2 3 √ 2 3 x 2
= 2 2 · · (x + 3) 2 + 2 · · (x + 3) 2 − −x
3 −3 3 2 −1
√  √ !  √ !
4 2 h 32 i  2 2 3 25 
 
 2 2 1 
2 − 0 +  − 5  −  + 1 
3
= · 82 − · 22 −
3 3 2 3 2
" #
16 56 16 38
= + −6 = + = 18.
3 3 3 3

The Grading Scheme. The assignment was to be graded out of a maximum of 15 marks.
(In the version of this document prepared for TA’s there were additional details on the marking
scheme.)

C.8.3 Solutions to Written Assignment W3


Release Date: March 25th, 2006
Solutions were to be submitted inside answer sheets to Quiz Q3 , at tutorials during the period
13-16 February, 2006

The Problems
Z
1. Use two integrations by parts to evaluate the integral (sin x) · (sinh x) dx. You will
need, at the last step, to solve an equation. The solution should resemble
R the solution in
x
your textbook to [7, Example 4, p. 478], where the author evaluates e sin x dx.

2. Write your student number (9 digits) in the following chart.

A1 A2 A3 A4 A5 A6 A7 A8 A9
Student # :
UPDATED TO April 12, 2009
Information for Students in MATH 141 2009 01 3026

Some of the digits are to be used in solving the following problem.

(a) You are to evaluate the following integral by integration by parts:


Z  
A7 x2 + A8 x1 + A9 eA1 x dx

Your answer should be simplified as much as possible.


(b) Then you are to differentiate the function you have obtained, to verify that it is
indeed an antiderivative of the given integrand.
Remember the rules: McGill’s Code of Student Conduct and Disciplinary Procedures appears in
the Handbook on Student Rights and Responsibilities (PDF English version - French version). Article
15(a) of the Code, which is devoted to plagiarism, reads as follows:

No student shall, with intent to deceive, represent the work of another person as his or
her own in any academic writing, essay, thesis, research report, project or assignment
submitted in a course or program of study or represent as his or her own an entire essay or
work of another, whether the material so represented constitutes a part or the entirety of
the work submitted.

Please don’t cause yourself embarrassment and waste everyone’s time: it isn’t worth it, and you really
need to learn how to solve these kinds of problems yourself.

The Solutions

1. Let u = sin x, v0 = sin x. Then u0 = cos x, v = cosh x (or cosh x plus any real constant
— I have taken the constant to be 0, as all choices of constant here will lead to the same
solution.)
Z Z
(sin x) · (sinh x) dx = (cos x) · sinh x − (cosh x) · (cos x) dx .
Z
To evaluate (cosh x) · (cos x) dx I will set U = cos x, V 0 = cosh x, so U 0 = − sin x,
V = sinh x.
Z Z
(cosh x) · (cos x) dx = (cos x) · sinh x + (sinh x) · sin x dx .

Combining these results yields


Z Z
(sin x) · (sinh x) dx = (sin x) · (cosh x) − (cos x) · (sinh x) − (sin x) · (sinh x) dx
Information for Students in MATH 141 2009 01 3027

which we may solve by moving the two integral terms to the same side of the equation:
Z
2 (sin x) · (sinh x) dx = (sin x) · (cosh x) − (cos x) · (sinh x) + C

or Z
(sin x) · (cosh x) − (cos x) · (sinh x)
(sin x) · (sinh x) dx = + C0
2
2. (a) In the first integration by parts I set

u = A7 x2 + A8 x1 + A9
dv = eA1 x dx
⇒ du = (2A7 x + A8 )dx,
1 A1 x
v = e .
A1
Hence
Z  
A7 x2 + A8 x1 + A9 eA1 x dx
  Z
2 1 A1 x 1
= A7 x + A8 x + A9 · e − (2A7 x + A8 ) · eA1 x dx
A1 A1
! Z !
A7 2 A8 1 A9 A1 x 2A7 A8
= x + x + ·e − ·x+ · eA1 x dx
A1 A1 A1 A1 A1
To evaluate the subtracted integral we must apply Integration by Parts a second
time:
2A7 A8
U = ·x+
A1 A1
A1 x
dV = e dx
2A7
⇒ dU = dx
A1
1 A1 x
V = e , so
A1
Z ! ! Z
2A7 A8 2A7 A8 1 A1 x 2A7 A1 x
·x+ · eA1 x dx = ·x+ · e − e dx
A1 A1 A1 A1 A1 A21
!
2A7 A8 2A7
= 2
· x + 2 · e A1 x − 3 e A1 x
A1 A1 A1
!
2A7 A8 2A7
= 2
· x + 2 − 3 · eA1 x
A1 A1 A1
Information for Students in MATH 141 2009 01 3028

Combining our results yields


Z  
A7 x2 + A8 x1 + A9 eA1 x dx
! !!
A7 2 A8 2A7 1 A9 A8 2A7
= x + − 2 x + − + 3 · eA1 x + C
A1 A1 A1 A1 A21 A1

(b) Remember that you were to differentiate the preceding product of polynomial and
exponential by the Product Rule to show that, indeed, its derivative is the given
integrand.

C.8.4 Solutions to Written Assignment W4


Release Date: March 25th, 2006
Solutions were to be submitted at inside answer sheets to Quiz Q4 , at tutorials during the
week 06 – 09 March, 2006

The Problems This assignment is based on your WeBWorK assignment R5 . You are asked
to write out complete solutions to the following modifications of your versions of problems on
that assignment. Note that, in some cases, the question asks for more than was asked on the
assignment. You are not permitted to use a Table of Integrals.

1. Problem 10:

(a) Evaluate the following integral with the specific values of the constants given in
your own WeBWorK assignment:
Z
Ex2 + Ex + F
dx .
x3 + Gx2 + Hx + J
HINT: −C is a root of the denominator.
(b) No marks will be given unless you verify your integration by differentiating your
answer.

2. Problem 12:

(a) Evaluate the following integral with the specific values of the constants given in
your own WeBWorK assignment:
Z
Bx + C
dx .
(x2 + A2 )2
Information for Students in MATH 141 2009 01 3029

(b) No marks will be given unless you verify your integration by differentiating your
answer.

3. Problem 16: Determine whether the following integral (with the constants given in your
own WeBWorK assignment) is divergent or convergent. If it is convergent, evaluate it.
If not, prove that fact. Z A
1
2
dx .
−∞ x + 1

In each case your solution should begin by your writing out the full problem with your data,
so your TA does not have to look up your data on the WeBWorK system.
While these problems were generated by WeBWorK, they now constitute a conventional
mathematics assignment, and it does not suffice to make unsubstantiated statements. You are
expected to prove everything you state. Thus, for example, in Problem 10, you have to show
how you use the fact that a certain number is stated to be a root of the denominator.
Remember the rules: McGill’s Code of Student Conduct and Disciplinary Procedures appears in
the Handbook on Student Rights and Responsibilities (PDF English version - French version). Article
15(a) of the Code, which is devoted to plagiarism, reads as follows:

No student shall, with intent to deceive, represent the work of another person as his or
her own in any academic writing, essay, thesis, research report, project or assignment
submitted in a course or program of study or represent as his or her own an entire essay or
work of another, whether the material so represented constitutes a part or the entirety of
the work submitted.

Please don’t cause yourself embarrassment and waste everyone’s time: it isn’t worth it, and you really
need to learn how to solve these kinds of problems yourself.

The Solutions
x
1. Under a change of variable of the form u = , the integral reduces to one of the form
Z A
Ku + L Ku
2 du. The term 2 du simplifies under a substitution v = u2 to a multiple
Zu + 1
2
Zu + 1
dv L
of . The term  du simplifies under a substitution θ = arctan u,
(v + 1)2
u2 + 1 2
tan θ
eventually proving to be a multiple of θ + , etc.
2(tan2 θ + 1)
2. An antiderivative is arctan x. This is evaluated between an upper limit of some constant
A, and a lower limit we may call B. We need to observe that lim arctan B = − π2 .
B→−∞
Information for Students in MATH 141 2009 01 3030

C.8.5 Solutions to Written Assignment W5


Release Date: Solutions were to be submitted inside answer sheets to Quiz Q5 , at tutorials
during the week 20 – 23 March, 2006

The Problems This assignment will be graded out of a maximum of 20 MARKS.


Write your student number (9 digits) in the following chart.

A1 A2 A3 A4 A5 A6 A7 A8 A9
Student # :
Some of the digits are to be used in solving the following problems.
1. Consider the curve in the plane defined parametrically by
x(t) = t2 + 1
y(t) = A7 t2 + A8 t + A9
(a) [3 MARKS] Showing all your work, determine the slope of the tangent to the curve
at the point with parameter value t = 1.
d2 y
(b) [6 MARKS] Showing all your work, determine the value of 2 at the point with
dx
general parameter value t (t , 0). For this problem you must not substitute in any
formula from your class notes or any textbook; you are expected to determine the
2nd derivative by differentiation, for example in the manner similar to that done in
your textbook, Example 1, page 661.
(c) [1 MARK] Determine the range of values of t — if there are any such values —
where the curve is concave upward.
2. This problem is based on Problem 12 on your WeBWorK assignment R8 . For the given
arc of the given curve,
(a) [8 MARKS] determine the area of the surface of revolution of that arc about the
x-axis;
(b) [2 MARKS] set up an integral for the area of the surface of revolution of that arc
about the y-axis, but, in this case only, do not evaluate the integral.
In each case you are expected to show all your work.
Remember the rules: McGill’s Code of Student Conduct and Disciplinary Procedures appears in
the Handbook on Student Rights and Responsibilities (PDF English version - French version). Article
15(a) of the Code, which is devoted to plagiarism, reads as follows:
Information for Students in MATH 141 2009 01 3031

No student shall, with intent to deceive, represent the work of another person as his or
her own in any academic writing, essay, thesis, research report, project or assignment
submitted in a course or program of study or represent as his or her own an entire essay or
work of another, whether the material so represented constitutes a part or the entirety of
the work submitted.
Please don’t cause yourself embarrassment and waste everyone’s time: it isn’t worth it, and you really
need to learn how to solve these kinds of problems yourself.

The Solutions
1. (a)
x(t) = t2 + 1
y(t) = A7 t2 + A8 t + A9
dx
= 2t
dt
dy
= 2A7 t + A8
dt
dy
dy dt 2A7 t + A8
= dx
=
dx dt
2t

dy A8
= A7 +
dx t=1 2
(b)
!
d2 y d 2A7 t + A8
=
dx2 dx 2t
!
d 2A7 t + A8 dt
= ·
dt 2t dx
!
d 2A7 t + A8 1
= · dx
dt 2t
! dt
d 2A7 t + A8 1
= ·
dt 2t 2t
(2A7 )(2t) − (2A7 t + A8 )(2)
=
(2t)3
A8
= − 3
4t
(c) If the student’s A8 is 0, the curve is flat. Otherwise it is concave upwards precisely
when t < 0.
Information for Students in MATH 141 2009 01 3032

2. CORRECTED ON 20 MARCH 2006. The following solution is valid only


when K, the upper limit of integration, is sufficiently small. The correct so-
lution is much more complicated, as we need to ensure that the respective
factors cos θ + θ sin θ and sin θ − θ cos θ remain positive. I have corrected my
original version by inserting absolute signs. However, the evaluation of these
integrals is complicated when we attempt to break the integral up into parts
based on the signs of these factors. This is certainly more difficult than was
intended from students in this course. Please exercise good judgment in grad-
ing the two parts of this problem, and do not expect those students for whom
the upper limit K is large to obtain the correct area.

x(θ) = a(cos θ + θ sin θ)


y(θ) = a(sin θ − θ cos θ)
⇒ x0 = aθ cos θ
y0 = aθ sin θ
p
⇒ (x0 )2 + (y0 )2 = |aθ|
(a) about the x-axis, PROVIDED K IS SUFFICIENTLY SMALL,
Z K Z K Z K !
2
Area = 2πa| sin θ − θ cos θ| · |θ| dθ = 2πa θ sin θ dθ − θ cos θ dθ
0 0 0

for positive K sufficiently small. We must integrate by parts.


Z Z
θ sin θ dθ = −θ cos θ + cos θ dθ

Z = −θ cos θ + sin
Z θ+C
θ cos θ dθ = θ2 sin θ − 2
2
θ sin θ dθ

Z = θ2 sin θ + 2θ cos θ − 2 sin θ + C 0


(θ sin θ − θ2 cos θ) dθ = −3θ cos θ + 3 sin θ − θ2 sin θ + C 00

Hence the area of the surface of revolution is −3K cos K + 3 sin K − K 2 sin K.
(b) about the y-axis, PROVIDED K IS SUFFICIENTLY SMALL,
Z K Z K Z K !
2
Area = 2πa| cos θ + θ sin θ| · |θ| dθ = 2πa θ cos θ dθ + θ sin θ dθ .
0 0 0

C.9 MATH 141 2007 01


The use of written assignments was discontinued in 2007.
Information for Students in MATH 141 2009 01 3033

D Quizzes from Previous Years


D.1 MATH 141 2007 01
D.1.1 Draft Solutions to Quiz Q1
Distribution Date: Mounted on the Web on 4 February, 2007
corrected 12 February, 2009 — subject to further corrections

There were four different types of quizzes, for the days when the tutorials are scheduled. Each
type of quiz was generated in multiple varieties for each of the tutorial sections. The order of
the problems in the varieties was also randomly assigned. All of the quizzes had a heading that
included the instructions
• Time = 30 minutes

• No calculators!

• Show all your work: marks are not given for answers alone.

• Enclose this question sheet in your folded answer sheet.


In the following I will either provide a generic solution for all varieties, or a solution to one
typical variety.

Monday version
1. [5 MARKS] Use Part 1 of the Fundamental Theorem of Calculus to find the derivative
of the function Z a
g(x) = b tan(t) dt ,
x
(where a and b are constants). Then use Part 2 of the Fundamental Theorem to evaluate
g(x), by first verifying carefully that ln | sec x| is an antiderivative of tan x.
Solution:

(a) Part 1 of the Fundamental Theorem gives the derivative of a definite integral as a
function of its upper index of integration. Here the variable is the lower index of
integration.
Z a
d
b tan(t) dt
dx x
Z x !
d
= − b tan(t) dt
dx a
Information for Students in MATH 141 2009 01 3034

Z x
d
= − b tan(t) dt
dx a
= −b tan x .
Some students may quote a variant of Part 1 which gives the derivative of a definite
integral with respect to the lower index, and this should be accepted if work has
been shown.
(b) Students were expected to first find the derivative of ln | sec x|. Since this is a com-
position of 2 functions, the Chain Rule will be needed. Let u = sec x. Then
d d d
ln | sec x| = ln |u| · secx
dx du dx
1
= · sec x tan x
u
1
= · sec x tan x
sec x
= tan x .
Hence
sec a cos x
g(x) = b ln | sec t|ax = b ln = b ln .
sec x cos a
2. [5 MARKS] Find an antiderivative of the integrand of the integral
Zb  
k + `y + my2 dy,
a
(where a, b, k, `, m are constants), and then use the Fundamental Theorem of Calculus to
evaluate the integral. You are not expected to simplify your numerical answer.
Solution:
(a) One antiderivative of ky0 + `y1 + my2 is
y1 y2 y3
k· +`· +m· .
1 2 3
(b) Hence
Zb  
k + `y + my2 dy
a
" #b
y1 y2 y3
= k· +`· +m·
1 2 3 a
1 2
! !
b b b3 a1 a2 a3
= k· +`· +m· − k· +`· +m· .
1 2 3 1 2 3
Information for Students in MATH 141 2009 01 3035

3. [10 MARKS] Use Part 1 of the Fundamental Theorem of Calculus to find the derivative
of the function Z √x
cos t
g(x) = b dt ,
a t
where a, b are constants.
Solution:

(a) Denote the upper index of the integral by u(x) = x.
(b) Then
Z √x
d d cos t
g(x) = b dt
dx dx a t
Z u(x)
d cos t
= b dt
dx a t
Z u(x)
d cos t du(x)
= b dt ·
du a t dx
cos u du(x)
= b ·
u dx
cos u 1
= b · √
u 2 x

cos x 1
= b √ · √
x 2 x

cos x
= b
2x
4. [10 MARKS] Showing all your work, determine all values of x where the curve y =
Zx
1
dt is concave upward, where a, b are constants. (Each version of this
1 + at + bt2
0
quiz contained specific values for the constants a, b.)
Solution:
(a) By Part 1 of the Fundamental Theorem,
1
y0 (x) = .
1 + ax + bx2
(b) Differentiating a second time yields
!
00 d 1
y (x) =
dx 1 + ax + bx2
Information for Students in MATH 141 2009 01 3036

 
  d 
1 
= −   
 · 1 + ax + bx 2
1 + ax + bx2 2 dx
a + 2bx
= −  .
1 + ax + bx2 2

(c) The curve is concave upward where y00 > 0:


a + 2bx
− 2 > 0 ⇔ −(a + 2bx) > 0
1 + ax + bx2
since the denominator is a square, hence positive
⇔ 
2bx < −a



a
x < − 2b when b>0


 a
 x > − 2b when b<0
⇔ 



 never concave upward when b = 0, a > 0

 always concave upward when b = 0, a < 0

Tuesday version
Zc Zc Zb
1. [5 MARKS] If f (x) dx = k and f (x) dx = `, find f (x) dx. Show your work.
a b a
Solution:

(a)
Zc Zb Zc
f (x) dx = f (x) dx + f (x) dx .
a a b

(b) Hence
Zc Zc Zb
f (x) dx = f (x) dx − f (x) dx .
b a a

(c)
= `−k.
Z a √
2. [5 MARKS] Find an antiderivative of the integrand of the integral x dx, and then
0
use the Fundamental Theorem of Calculus to evaluate the integral. You are not expected
to simplify your numerical answer, but no marks will be given unless all your work is
clearly shown.
Information for Students in MATH 141 2009 01 3037

1
(a) One antiderivative of x 2 is
1 1 2 3
1
· x 2 +1 = x 2 .
2
+1 3

(b) Z " #a
a √ 2 32 2  23  2 3
x dx = x = a − 0 = a2 .
0 3 0 3 3
3. [10 MARKS] Use Part 1 of the Fundamental Theorem of Calculus to find the derivative
Zx2 √
of the function b 1 + tc dt.
a
Solution:

(a) Let the upper index of the integral be denoted by u = x2 . Then


(b)

Zx2 √ Zu √
d d
b 1 + tc dt = b 1 + tc dt
dx dx
a a
Zu √
d du
= b 1 + tc dt ·
du dx
a
√ du
= b 1 + uc ·
√ dx
c
= b 1 + u · 2x

= b 1 + x2c · 2x
Z x Z t2
a + ub
4. [10 MARKS] If F(x) = f (t) dt, where f (t) = du and a, b are constants,
1 1 u
00
find F (2).
Solution:

(a) Applying Part 1 of the Fundamental Theorem yields


Z x2
0 a + ub
F (x) = f (x) = du .
1 u
Information for Students in MATH 141 2009 01 3038

(b) A second application of Part 1 of the Fundamental Theorem yields


Z x2
00 0d a + ub
F (x) = f (x) = du .
dx 1 u

(c) Denote the upper index of the last integral by v = x2 .


(d)
Z x2 Z v
d a + ub d a + ub
du = du
dx 1 u dx 1 u
Z v
d a + ub dv
= du ·
dv 1 u dx
b
a+v dv
= ·
v dx
a + vb
= · 2x
v
a + x2b
= 2
· 2x
x 
2 a + x2b
= .
x

Wednesday version

1. [5 MARKS] Use Part 2 of the Fundamental Theorem of Calculus to evaluate the integral
Zbπ
cos θ dθ, where a, b are given integers. No marks will be given unless all your work

is clearly shown. Your answer should be simplified as much as possible.
Solution:

(a) One antiderivative of cos θ is sin θ.


(b)
Zbπ
cos θ dθ = [sin θ]bπ
aπ = sin(bπ) − sin(aπ) .

(c) Students were expected to observe that the value of the sine at the given multiples
of π is 0, so the value of the definite integral is 0.
Information for Students in MATH 141 2009 01 3039

X
n
2. [5 MARKS] Express lim axi sin xi ∆x as a definite integral on the interval [b, c],
n→∞
i=1
which has been subdivided into n equal subintervals.
Solution: Z c
ax sin x dx .
b

3. [10 MARKS] Use Part 1 of the Fundamental Theorem of Calculus to find the derivative
of the function Z bx 2
t +c
g(x) = 2
dt ,
ax t − c
where a, b, c are positive integers.
Solution:

(a) The Fundamental Theorem gives the derivative of a definite integral with respect to
the upper limit of integration, when the lower limit is constant. The given integral
must be expressed in terms of such specialized definite integrals.
Z bx 2 Z 0 2 Z bx 2
t +c t +c t +c
g(x) = 2
dt = 2
dt + dt
ax t − c ax t − c 0 t2 − c
Z ax 2 Z bx 2
t +c t +c
=− 2
dt + dt
0 t −c 0 t2 − c

Zbx
t2 + c
(b) For the summand dt, let u = bx. Then
t2 − c
0

Zbx Zu
d t2 + c d t2 + c
2
dt = dt
dx t −c dx t2 − c
0 0
Zu
d t2 + c du
= 2
dt ·
du t −c dx
0
u2 + c du
= 2 ·
u − c dx
u2 + c
= 2 ·b
u −c
(bx)2 + c
= ·b
(bx)2 − c
Information for Students in MATH 141 2009 01 3040

Z ax 2
t +c
(c) For the summand dt, let u = ax. Then, analogously to the preceding
0 t2 − c
step, Z ax 2
d t +c (ax)2 + c
dt = · a.
dx 0 t2 − c (ax)2 − c
(d)
(bx)2 + c (ax)2 + c
g0 (x) = · b − · a.
(bx)2 − c (ax)2 − c
4. [10 MARKS] Find a function f (x) such that
Z x
f (t) √
k+ 2
dt = ` x (99)
a t
for x > 0 and for some real number a; k and ` are constants given in the question,
different constants to different students. (HINT: Differentiate the given equation.)
Solution: A more proper wording of the problem would have been “Find a function f (x)
and a real number a such that...”.

(a) Assume that equation (99) holds. Then differentiation of both sides with respect to
x yields
f (x) 1 1
0+ 2 = ·`· √ .
x 2 x
3
(b) We solve the preceding equation to obtain that f (x) = 2` x 2 .
(c) Substitution into equation (99) yields
Z
` x − 21 √
k+ t dt = ` x .
2 a
We know how to integrate powers of t:
#x
` √ √
k+ 2· t = ` x.
cot a


(d) The preceding reduces to k = ` a, which may be solved to obtain
!2
k
a= .
`
Information for Students in MATH 141 2009 01 3041

Thursday version
π
Zb
1. [5 MARKS] Evaluate the integral sin t dt.
π
a

Solution:
(a) An antiderivative of sin t is − cos t.
(b)
π
Zb
π
sin t dt = [− cos t] πb
a
π
a

(c) [1 MARK] Your answer should be simplified as much as possible.


2. [5 MARKS] Evaluate the Riemann sum for f (z) = a−x2 , (0 ≤ x ≤ 2) with 4 subintervals,
taking the sample points to be the right endpoints. It is not necessary to simplify the final
numerical answer.
Solution:
(a) The interval 0 ≤ x ≤ 2 is divided by 3 points into 4 subintervals of length ∆x =
2
4
= 12 .

(b) The point
  xi selected in the ith interval will always be the right end-point, i.e.,
xi = i 12 (i = 1, 2, 3, 4).
(c) The Riemann sum is
X !
1X
4 4
i2
f (xi∗ )∆x = a− .
i=1
2 i=1 4

3. [10 MARKS] Use Part 1 of the Fundamental Theorem of Calculus to find the derivative
Zbx
of the function cos (tc ) dt, where a, b, c are real numbers.
cos x
Solution:
(a) The Fundamental Theorem gives the derivative of a definite integral with respect to
the upper limit of integration, when the lower limit is constant. The given integral
must be expressed in terms of such specialized definite integrals.
Zbx Z0 Zbx
cos (tc ) dt = cos (tc ) dt + cos (tc ) dt
cos x cos x 0
Information for Students in MATH 141 2009 01 3042

Z
cos x Zbx
= − cos (t ) dt + cos (tc ) dt
c

0 0

Zbx
(b) For the summand cos (tc ) dt, let u = bx. Then
0

Zbx Zu
d c d
cos (t ) dt = cos (tc ) dt
dx dx
0 0
Zu
d du
= cos (tc ) dt ·
du dx
0
= cos (uc ) · b
= cos ((bx)c ) · b
Z
cos x

(c) For the summand cos (tc ) dt, let v = cos x.


0

Z
cos x Zv
d d
cos (tc ) dt = cos (tc ) dt
dx dx
0 0
Zv
d dv
= cos (tc ) dt ·
dv dx
0
= cos (vc ) · (− sin x)
= cos (cosc x) · (− sin x)

(d)
Zbx
cos (tc ) dt = − cos (cosc x) · (− sin x) + cos ((bx)c ) · b .
cos x



 0 if x < 0

 Z x

 x if 0 ≤ x ≤ a
4. [10 MARKS] Let f (x) =   and g(x) = f (t) dt, where a is


 2a − x if a < x < 2a 0

 0 if x > 2a
a positive constant. Showing all your work, find a formula for the value of g(x) when
a < x < 2a.
Information for Students in MATH 141 2009 01 3043

Solution:

(a) The interval where we seek a formula is the third interval into which the domain
has been broken. For x in this interval the integral can be decomposed into
Z x Z a Z x
f (t) dt = f (t) dt + f (t) dt .
0 0 a

The portion of the definition of f for x < 0 is of no interest in this problem, since
we are not finding area under that portion of the curve; the same applies to the
portion of the definition for x > 2a.
(b)
Z a Z a
f (t) dt = t dt
0 0
" #
2 t=a
t a2
= = .
2 t=0 2

(c)
Z x Z x
f (t) dt = (2a − t) dt
a a
" #t=x
t2
= 2at −
2 t=a
! !
x2 2 a2
= 2ax − − 2a − .
2 2

(d) ! !
a2 x2 2 a2 x2
g(x) = + 2ax − − 2a − = 2ax − − a2 .
2 2 2 2

D.1.2 Draft Solutions to Quiz Q2


Distribution Date: Posted on the Web on 28 February, 2007
Caveat lector! There could be misprints or errors in these draft solutions.

There were four different types of quizzes, for the days when the tutorials are scheduled. Each
type of quiz was generated in multiple varieties for each of the tutorial sections. The order of
the problems in the varieties was also randomly assigned. All of the quizzes had a heading that
included the instructions
• Time = 30 minutes
Information for Students in MATH 141 2009 01 3044

• No calculators!
• Show all your work: marks are not given for answers alone.
• Enclose this question sheet in your folded answer sheet.
In the following I will either provide a generic solution for all varieties, or a solution to one
typical variety.

Monday version
Z !
b 1
1. [5 MARKS] Evaluate the integral x +a+ 2 dx, (where a and b are given
x +1
positive integers).
Solution:
Z
xb+1
(a) xb dx = + C1 ,
b+1
Z
(b) a dx = ax + C2
Z
1
(c) 2
dx = arctan x + C3
x +1
Z !
b 1 xb+1
(d) x +a+ 2 dx = + ax + arctan x + C.
x +1 b+1
Z  
2. [5 MARKS] Use a substitution to evaluate the indefinite integral t2 cos a − t3 dt,
(where a is a given real number).
Solution:
(a) Try the substitution u = t3 .
(b) du = 3t2 dt ⇒ t2 dt = 13 du.
(c)
Z   Z
2 3 1
t cos a − t dt = cos(a − u) du
3
1
= − sin(a − u) + C
3
1
= − sin(a − t3 ) + C.
3
(Some students may wish to employ a second substitution v = a − u. Alternatively,
a better substitution for the problem would have been to take u = a − t3 .)
Information for Students in MATH 141 2009 01 3045

3. [10 MARKS] Find the area of the region bounded by the parabola y = x2 , the tangent
line to this parabola at (a, a2 ), and the x-axis, (where a is a given real number).
Solution: This area can be computed by integrating either with respect to y or with
respect to x.
Integrating with respect to y: (a) Since y0 = 2x, the tangent line through (a, a2 ) has
equation
y − a2 = 2a(x − a) ⇔ y = 2ax − a2 .
(b) To integrate with respect to y we need to express the equations of the parabola
and the line in the form
x = function of y .

The branch of the parabola to the right of the y-axis is x = y. The line has
y a
equation x = + .
2a 2 !
y + a2 √
(c) The area of the horizontal element of area at height y is − y ∆y.
2a
(d) The area is the value of the integral
Z a2 !
y + a2 √
− y dy .
0 2a
(e) Integration yields
" 2 #a2 !
y ay 2 32 1 1 2 3 1 3
+ − y = + − a = a .
4a 2 3 0 4 2 3 12

Integrating with respect to x: (a) As above, the tangent line is y = 2ax − a2 . Its
a
intercept with the x-axis is at x = .
2
a  
(b) The area of the vertical element of area at horizontal position x ≤ is x2 − 0 ∆x.
2  
(c) The area of the vertical element of area at horizontal position x ≥ 2a is x2 − (2ax − a2 ) dx =
(x − a)2 ∆x.
(d) The area of the region is the sum
Z a2 Z a
2
x dx + (x − a)2 dx .
a
0 2

(e) Integration yields


" # a2 " #a
x3 (x − a)3 a3
+ − = .
3 0 3 a 12
2
Information for Students in MATH 141 2009 01 3046

4. [10 MARKS] Find the volume of the solid√obtained by rotating the region bounded by
the given curves about the line y = 1: y = n x, y = x, where n is a given positive integer.
Solution: A favoured method of solution was not prescribed.

Using the method of “washers”: (a) Find the intersections of the curves bounding the
region. Solving the 2 equations yields the points (x, y) = (0, 0), (1, 1).
(b) Find the inner and outer dimensions of the washer. Since the axis of revolution
is a horizontal line, the element of area being rotated is vertical. For√arbitrary
x the lower point on the element is (x, x); the upper point is (x, n x). √n The
distances of these points from the axis are, respectively 1 − x and 1 − x.
(c) The volume of the “washer” is, therefore,
 √ 
π −(1 − x)2 + (1 − n x)2 ∆x .

(d) Correctly evaluate the integral:


Z 1
√ 
π −(1 − x)2 + (1 − n x)2 dx
0
Z 1 
1 2
= π −2x + x2 + 2x n − x n dx
0
" #1
2 x2 2n n+1 n n+2
= π −x + + xn − xn
3 n+1 n+2 0
!
1 2n n (n − 1)(n + 4)π
= π −1 + + − =
3 n+1 n+2 3(n + 1)(n + 2)

Using the method of cylindrical shells: (a) Find the intersections of the curves bound-
ing the region. Solving the 2 equations yields the points (x, y) = (0, 0), (1, 1).
(b) Find the inner and outer dimensions of the washer. Since the axis of revolution
is a horizontal line, the element of area being rotated is also horizontal. For
arbitrary y the left endpoint on the element is (yn , y); the right endpoint is (y, y).
The length of the element is, therefore, y−yn ; the distances of the element from
the axis of symmetry is 1 − y.
(c) The volume of the cylindrical shell element of volume is, therefore,

2π(1 − y) · (y − yn ) · ∆y .

(d) Correctly evaluate the integral:


Z 1
2π (1 − y)(y − yn ) dy
0
Information for Students in MATH 141 2009 01 3047

Z 1 
= 2π −yn + yn+1 + y − y2 dy
0
" #1
1 n+1 1 n+2 1 2 1 3
= 2π − y + y + y − y
n+1 n+2 2 3 0
!
1 1 1 1
= 2π − + + − −0
n+1 n+2 2 3
!
1 1 (n − 1)(n + 4)π
= 2π − =
6 (n + 1)(n + 2) 3(n + 1)(n + 2)

Tuesday version
Z
1. [5 MARKS] Evaluate the integral (a − t)(b + t2 ) dt.

Solution:

(a) Expand the product in the integrand:


Z Z  
2
(a − t)(b + t ) dt = ab − bt + at2 − t3 dt .

(b) Integrate term by term:


Z   b a 1
ab − bt + at2 − t3 dt = ab · t − · t2 + · t3 − · t4 + C .
2 3 4
Z
2. [5 MARKS] Using a substitution, evaluate the indefinite integral cosn x sin x dx, where
n is a fixed, positive integer.
Solution:

(a) Use new variable u, where du = − sin x dx; one solution is u = cos x.
(b)
Z Z
n
cos x sin x dx = − un du
un+1
= − +C
n+1
1
= − cosn+1 x + C
n+1
Information for Students in MATH 141 2009 01 3048

3. [10 MARKS] Find the area of the region bounded by the parabola x = y2 , the tangent
line to this parabola at (a2 , a), and the y-axis, where a is a fixed, positive real number.
Solution: The solution is analogous (under the exchange x ↔ y) to that given for Prob-
lem 3 of the Monday quiz.
4. [10 MARKS] Find the volume of the solid obtained by rotating the region bounded by
y = xn and x = yn about the line x = −1, where n is a given positive integer.
Solution:
Case I: n is even
Using the method of “washers”: (a) Find the intersections of the curves bound-
ing the region. Solving the 2 equations yields the points (x, y) = (0, 0), (1, 1).
(b) Find the inner and outer dimensions of the washer. Since the axis of revo-
lution is a vertical line, the element of area being rotated is horizontal. For
1
arbitrary y the farther endpoint on the element is (y n , y); the nearer end-
point is (yn , y). The distances of these points from the axis are, respectively

1 + n y and 1 + yn .
(c) The volume of the “washer” is, therefore,
 √ 
π −(1 + yn )2 + (1 + n y)2 ∆y .
(d) Correctly evaluate the integral:
Z 1
√ 
π −(1 + y)2 + (1 + n y)2 dy
0
Z 1 
1 2
= π 2y n + y n − 2yn − y2n dy
0
" #1
2n n+1 n n+2 2 n+1 1 2n+1
= π yn + yn − y − y
n+1 n+2 n+1 2n + 1 0
2(n − 1)(3n2 + 7n + 3)π
=
(n + 1)(n + 2)(2n + 1)
Using the method of cylindrical shells: (a) Find the intersections of the curves
bounding the region. Solving the 2 equations yields the points (x, y) =
(0, 0), (1, 1).
(b) Since the axis of revolution is a vertical line, the element of area being
rotated is also vertical. For arbitrary x the top endpoint on the element is
1
(x, x n ); the lower endpoint is (x, xn ). The length of the element is, there-
1
fore, x n − xn ; the distance of the element from the axis of symmetry is
1 + x.
Information for Students in MATH 141 2009 01 3049

(c) The volume of the cylindrical shell element of volume is, therefore,
 1 
2π(1 + x) · x n − xn .
(d) Correctly evaluate the integral:
Z 1
1
2π (1 + x)(x n − xn ) dx
0
Z 1 
1 n+1
= 2π x n − xn + x n − xn+1 dx
0
!
n 1 n 1
= 2π − + −
n + 1 n + 1 2n + 1 n + 2
2(n − 1)(3n2 + 7n + 3)π
=
(n + 1)(n + 2)(2n + 1)
Case II: n is odd
Using the method of “washers”: (a) Find the intersections of the curves bound-
ing the region. Solving the 2 equations yields the points (x, y) = (0, 0), (±1, ±1).
Here there is an issue of interpretation. The textbook usually permits the
word region to apply to one that may have more than one component;
some authors would not wish to apply the term in such a situation. I will
follow the textbook, and permit a region here to have two components.
(b) Find the inner and outer dimensions of the washer. Since the axis of rev-
olution is a vertical line, the element of area being rotated is horizontal.
But there are two kinds of elements, depending on whether y is positive
or negative. For arbitrary, positive y the farther endpoint on the element is
1
(y n , y); the nearer endpoint is (yn , y). The distances of these points from

the axis are, respectively 1 + n y and 1 + yn . For arbitrary, negative y the
1
nearer endpoint on the element is (y n , y); the farther endpoint is (yn , y).

The distances of these points from the axis are, respectively 1 + n y and
1 + yn (both of which are less than 1).
(c) The volume of the “washer” is, therefore,

π −(1 + yn )2 + (1 + n y)2 ∆y .
(d) Correctly evaluate the integral:
Z 1
π −(1 + y)2 + (1 + √n y)2 dy
−1
Z 1 
1 2
= π 2y n + y n − 2yn − y2n dy
0
Information for Students in MATH 141 2009 01 3050

Z 0 
1 2
+π −2y n − y n + 2yn + y2n dy
−1
" #1
2n n+1 n n+2 2 n+1 1 2n+1
= π yn + yn − y − y
n+1 n+2 n+1 2n + 1 0
" #0
2n n+1 n n+2 2 n+1 1 2n+1
+π − yn − yn + y + y
n+1 n+2 n+1 2n + 1 −1
2(n − 1)(3n2 + 7n + 3)π 2(n − 1)(n2 + 3n + 1)π
= +
(n + 1)(n + 2)(2n + 1) (n + 1)(n + 2)(2n + 1)
4(n − 1)π
= .
n+1
Using the method of cylindrical shells: (a) Find the intersections of the curves
bounding the region. Solving the 2 equations yields the points (x, y) =
(0, 0), (±1, ±1).
(b) Since the axis of revolution is a vertical line, the element of area being
rotated is also vertical. For arbitrary, positive x the top endpoint on the el-
1
ement is (x, x n ); the lower endpoint is (x, xn ); for arbitrary, negative x the
1
bottom endpoint on the element is (x, x n ); the upper endpoint is (x, xn ).
1
The length of the element is, therefore, x n − xn ; the distance of the ele-
ment from the axis of rotation is 1 + x.
(c) The volume of the cylindrical shell element of volume is, therefore,
1
2π(1 + x) · x n − xn .

(d) Correctly evaluate the integral:


Z 1 1
n
2π (1 + x) x − x dx
n

−1
Z 1
1 n+1
x n − x + x n − x dx
n+1
n
= 2π
−1
!
n 1 n 1
= 2π − + −
n + 1 n + 1 2n + 1 n + 2
!
n 1 n 1
+2π − − +
n + 1 n + 1 2n + 1 n + 2
4(n − 1)π
=
n+1

Wednesday version
Information for Students in MATH 141 2009 01 3051

Z
sin 2t
1. [5 MARKS] Showing all your work, evaluate the indefinite integral dt.
cos t
Solution:

(a) Apply a “double angle” formula:


Z Z Z
sin 2t 2 sin t cos t
dt = dt = 2 sin t dt .
cos t cos t

(b) Complete the integration:


Z
2 sin t dt = −2 cos t + C .

Z
ex
2. Using a substitution, evaluate the indefinite integral dx, where a is a non-zero
ex + a
real number.
Solution:

(a) [2 MARKS] Try the substitution u = e x + a, so du = e x dx.


(b) Z Z
ex du
x
dx = = ln |u| + C = ln |e x + a| + C .
e +a u
(If the constant a is positive, then the absolute signs are not required.)

3. [10 MARKS] Find the number b such that the line y = b divides the region bounded
by the curves y = ax2 and y = k into two regions with equal area, where a, k are given
positive constants.
Solution:

(a) Determine the range of values for integration by finding the r intersections
 of the
 k 
bounding curves: solving the equations yields the points ∓ , k .
a 
(b) Determine the portion of the full area which is below the line y = b. We begin by
repeating
 r the calculation of the preceding part: the corner points have coordinates
 b 
∓ , b . The area is
a 

Z √ ba " 3
# √ ba r
2 ax 4 b
√ b (b − ax ) dx = 2 bx − 3 = b
3 a
.
− a 0
Information for Students in MATH 141 2009 01 3052

(c) As a special case of the foregoing, or r by a separate calculation, we can conclude


4 k
that the area of the entire region is k .
3 a
(d) The condition of the problem is that
r r
4 b 1 4 k
b = · k
3 a 2 3 a
3 3
which is equivalent to 4b = k , and implies that the line should be placed where
2
b = 2− 3 k.
4. [10 MARKS] Use the method of cylindrical shells to find the volume generated by ro-
tating the region bounded by the given curves about the specified axis.

y= x−1 , y=0 , x=a

about y = b, where a, b are fixed positive constants, and b ≥ a − 1 .
Solution:
(a) Solve
√ equations to determine the limits of integration.
 √  Solving x = a with y =
x − 1 yields the single point of intersection a, a − 1 .
(b) The horizontal element of area at height y which generates the cylindrical shell has
left endpoint (1 + y2 , y) and right endpoint (a, y), so its length is a − (1 + y2 ).
(c) The distance of the horizontal element of area which generates the shell from the
axis of rotation is b − y.
(d) Set up the integral for the volume by cylindrical shells:
Z √a−1  
2π (b − y) a − (1 + y2 ) dy .
0

(e) Evaluate the integral


Z √
a−1  
2π (b − y) a − (1 + y2 ) dy
0
Z √
a−1  
= 2π b(a − 1) − (a − 1)y − by2 + y3 dy
0
" # √a−1
a−1 2 b 3 1 4
= 2π b(a − 1)y − y − y + y
2 3 4 0
!
3 a−1 b 3 1 2
= 2π b(a − 1) −
2 (a − 1) − (a − 1) + (a − 1)
2
2 3 4
!
3 2 1 √
= 2π(a − 1) 2 ·b− a−1 .
3 4

UPDATED TO April 12, 2009


Information for Students in MATH 141 2009 01 3053

Thursday version
1. [5 MARKS] Showing all your work, evaluate the integral by making a substitution:
Z
b
dx ,
(1 + ax)3
where a, b are non-zero constants.
Solution:

(a) A substitution which suggests itself is u = 1 + ax, implying that du = a dx, so


dx = 1a du.
(b) Z Z
b b
du b b
3
dx = 3
= − u−2 + C = − +C.
(1 + ax) ua 2a 2a(1 + ax)2
Z
2. [5 MARKS] Evaluate the indefinite integral seca x tan x dx, where a is a constant,
positive integer.
Solution:

(a) Try the substitution given by du = sec x·tan x dx, of which one solution is u = sec x.
(b) Z Z
a ua seca x
sec x tan x dx = ua−1 du = +C = +C.
a a
Some students may have integrated by sight.

3. [10 MARKS] Find the number b such that the line divides the region bounded by the
curves x = ay2 and x = k into two regions with equal area.
Solution: The solution is analogous (under the exchange x ↔ y) to that given for Prob-
lem 3 of the Wednesday quiz.

4. [10 MARKS] The region bounded by the given curves is rotated about the axis x = −1.
Find the volume of the resulting solid by any method:

y = 5, y = x2 − ax + b

Solution: Because there are constraints on the constants, I will work just one variant,
with a = 3, b = 7.

Using the method of cylindrical shells: (a) To find the extremes of integration, we
solve the equations y = 5 and y = x2 − 3x + 7, obtaining (x, y) = (1, 5), (2, 5).
Information for Students in MATH 141 2009 01 3054

(b) The height of a vertical element of area which generates a cylindrical shell is,
at horizontal position x, 5 − (x2 − 3x + 7) = −x2 + 3x − 2.
(c) The distance of that vertical element of area from the axis of revolution is 1+ x.
(d) The volume is given by the integral
Z 2
2π (1 + x)(−x2 + 3x − 2) dx
1
(e) Evaluating the integral:
Z 2
2π (1 + x)(−x2 + 3x − 2) dx
1
Z 2 
= 2π −x3 + 2x2 + x − 2 dx
1
" #2
1 4 2 3 1 2
= 2π − x + x + x − 2x
4 3 2 1
!
16 1 2 1
= 2π −4 + +2−4+ − − +2
3 4 3 2

=
6
Using the method of “washers” (a) To find the lowest point on the parabola, we solve
x2 − 3x + 7 ≥ 0. This can be done by completing the square, ! or by using the
3 19
calculus to find the local minimum. We find it to be , .
2 4
(b) The horizontal element generating the “washer” at height y extends between
the solutions in x to the equation y = x2 − 3x + 7; these are
p
3 ± 4y − 19
x= .
2
(c) The volume of the “washer” at height y is, therefore,
 p 2  p 2 
 3 + 4y − 19 
   3 − 4y − 19  
π 1 +  − 1 +   ∆y

2 2
p
= 5π 4y − 19 ∆y
Z 5p
(d) The volume is given by the integral 5π 4y − 19 dy.
19
4
(e) Evaluation of the integral:
Z 5p " #5
2 1 3 5π
5π 4y − 19 dy = 5π · · (4y − 19) 2 = .
19
4
3 4 19 6
4
Information for Students in MATH 141 2009 01 3055

D.1.3 Draft Solutions to Quiz Q3


Distribution Date: Posted on the Web on 21 March, 2007
Caveat lector! There could be misprints or errors in these draft solutions.

There were four different types of quizzes, for the days when the tutorials are scheduled. Each
type of quiz was generated in multiple varieties for each of the tutorial sections. The order of
the problems in the varieties was also randomly assigned. All of the quizzes had a heading that
included the instructions

• Time = 30 minutes

• No calculators!

• Show all your work: marks are not given for answers alone.

• Enclose this question sheet in your folded answer sheet.

In the following I will either provide a generic solution for all varieties, or a solution to one
typical variety.

Monday version
Z
1. [6 MARKS] Evaluate the integral t3 eat dt, where a is a non-zero constant.

Solution: This problem requires several consecutive applications of integration by parts


to reduce the exponent of the power of t to 0:
1
u = t3 , dv = eat dt ⇒ du = 3t2 dt, v = eat
Z Z a
1 3
⇒ t3 eat dt = · t3 eat − t2 eat dt
a a
1
U = t2 , dV = eat dt ⇒ dU = 2t dt, V = eat
Z ! a Z
3 at 1 3 3 2 at 6
⇒ t e dt = · t − 2t e + 2 teat dt
a a a
1
ũ = t, dṽ = eat dt ⇒ dũ = dt, ṽ = eat
Z a ! Z
3 at 1 3 3 2 6 at 6
⇒ t e dt = · t − 2 · t + 3t e − 3 eat dt
a a a a
!
1 3 3 2 6 6 at
= · t − 2 · t + 3t − 4 e + C
a a a a
Information for Students in MATH 141 2009 01 3056

the correctness of which integration may be verified by differentiation of the product on


the right.

2. [8
Z MARKS] Showing all your work, find a reduction formula for the indefinite integral
cosn ax dx, where a is a non-zero constant, and n is an integer not less than 2.

Solution:

(a) Introduce a symbol for the general indefinite integral sought:


Z
In = cosn ax dx

(b) Integration by parts:

u = cosn−1 ax ⇒ du = −a(n − 1) cosn−2 ax · sin ax dx


1
dv = cos ax dx ⇒ v = · sin ax
a Z
1
In = · cos ax · sin ax + (n − 1) cosn−2 ax · sin2 ax dx
n−1
a Z
1  
= · cos ax · sin ax + (n − 1) cosn−2 ax · 1 − cos2 ax dx
n−1
a
1
= · cosn−1 ax · sin ax + (n − 1)In−2 − (n − 1)In .
a
(c) Solve the last equation for In
1 n−1
· sin ax · cosn−1 ax +
In = In−2 + C .
an n
Z
1
3. [4 MARKS] Showing all your work, evaluate the integral √ dx , where a is
x 2 x 2 − a2
a non-zero constant.
Solution: The following solution uses a trigonometric substitution; it is also possible to
solve this problem using a hyperbolic substitution.
As is customary, I will proceed mechanically, taking square roots where necessary with-
out much attention to the sign choices; and then verify at the end by differentiation that
this process has produced a valid indefinite integral. This procedure can be made rigor-
ous by defining the new variable in terms of an inverse trigonometric function of x. This
is a useful exercise, but becomes extremely complicated in this case, because we would
have to work with either the inverse cosine or the inverse secant, and the textbook we
Information for Students in MATH 141 2009 01 3057

are using chooses different domains for these two functions. So I will avoid the niceties
and proceed as described.
I propose to use a substitution which provides that x = a sec θ. Then

dx = a sec θ · tan θ · dθ ,

and
Z Z
1 1
√ dx = cos θ dθ
x2 x2 − a2 a2
1
= sin θ + C
a2
1
= tan θ · cos θ + C
a2
sec2 θ − 1
= 2
+C
√a sec θ
x2 − a2
= +C,
a2 x
the correctness of which integration may be verified by differentiation of the quotient on
the right.
Z `
x(x − a)(x − b) + c
4. [8 MARKS] Showing all your work, evaluate the integral dx,
k (x − a)(x − b)
where a, b are distinct constants, c is a non-zero constant, and the limits of integration
k, ` are also prescribed. (The integrand was not presented to students in factored form.)
Solution:

(a) Since the degree of the numerator is not less than the degree of the denominator,
begin by dividing the denominator into the numerator, obtaining a quotient and a
remainder:
Z Z
x(x − a)(x − b) + c c
dx = x+ dx .
(x − a)(x − b) (x − a)(x − b)

(b) Expand the fraction into partial fractions. Assume that


c A B
= + ,
(x − a)(x − b) x − a x − b
take to a common denominator, and equate the resulting polynomials:

c = A · (x − b) + B · (x − a) .
Information for Students in MATH 141 2009 01 3058

(c) Now either equate coefficients of like powers of x, or, equivalently, give x succes-
sive values x = a and x = b:
c = A(a − b) c
⇒A= = −B .
c = B(b − a) a−b

(d) The integration reduces to


Z Z !
x(x − a)(x − b) + c c 1 c 1
dx = x+ · − · dx .
(x − a)(x − b) a−b x−a a−b x−b

(e) Complete the integration:


Z
x(x − a)(x − b) + c x2 c
dx = + (ln |x − a| − ln |x − b|) + C
(x − a)(x − b) 2 a−b

x2 c x − a
= + ln +C,
2 a − b x − b
the correctness of which integration may be verified by differentiation of the func-
tion on the right.
(f) Provided it is convergent the given definite integral can now be evaluated:
Z " #`
`
x(x − a)(x − b) + c x2 c x − a
dx = + ln
k (x − a)(x − b) 2 a − b x − b k
!
`2 − k2 c ` − a k − a
= + ln − ln
2 a−b ` − b k − b
!
`2 − k2 c (` − a)(k − b)
= + ln
2 a−b (` − b)(k − a)

(g) All of the preceding is based on the integral being convergent. In some of the
versions the integral was divergent. This was because at least one of the roots of
the polynomial which is the denominator of the integrand was contained in the
interval of integration. In such a case the integral can be seen to diverge.
√1
Z3
earctan y
5. [4 MARKS] Evaluate the integral dy . I have stated the problem with just one
1 + y2
−1
pair of possible limits for the integral; the variations of the problem included several
possible limits in each case, for each of which students should have been familiar with
the arctangent.
Information for Students in MATH 141 2009 01 3059

Solution: For simplicity, I work a specific instance of this problem. Use the substitution
dy
u = arctan y, so du = . Then an antiderivative can be obtained as follows
1 + y2
Z arctan y Z
e
dy = eu du = eu + C = earctan y + C
1 + y2
so the definite integral is equal to
h i √1 π π
earctan y 3
= e 6 − e− 4 .
−1

Alternatively, the substitution may be executed in the definite integral, replacing the
lower limit of −1 by arctan(−1) = − π4 , and the upper limit of √13 by arctan √13 = π6 .

Tuesday version
Z
1. [4 MARKS] Showing all your work, evaluate the integral x2 cos ax dx, where a is a
non-zero constant.
Solution: Two applications of integration by parts will be used to reduce the exponent
of the power of x to 0.

(a)

u = x2 ⇒ du = 2x dx
1
dv = cos ax dx ⇒ v = sin ax
Z a Z
2
2 x 2
x cos ax dx = · sin ax − x · sin ax dx
a a

(b)

U = x ⇒ dU = dx
1
dV = sin ax dx ⇒ V = − · cos ax
Z a Z !
2
2 x 2 x 1
x cos ax dx = · sin ax − − · cos ax + cos ax dx
a a a a
Z
x2 2 2
= · sin ax + 2 x · cos ax − 2 cos ax dx
a a a
x2 2 2
= · sin ax + 2 x · cos ax − 3 sin ax + C
a a a
Information for Students in MATH 141 2009 01 3060

The integration can be checked by differentiation of the alleged antiderivative.


Z
2. [9 MARKS] Showing all your work, find a reduction formula for the integral xn eax dx,
where a is a non-zero constant.
Solution:

(a) Introduce a symbol for the general indefinite integral sought:


Z
In = xn eax dx .

(b) Integration by parts:

u = xn ⇒ du = nxn−1 dx
1
dv = eax dx ⇒ v = · eax
a Z
1 ax n
In = ·e − xn−1 eax dx
a a
1 ax n
= · e − In−1 .
a a
which is the desired reduction formula.
Z
3. [4 MARKS] Showing your work, evaluate the integral sin3 ax · cos2 ax dx, where a is
a non-zero constant.
Solution: This integral is easily evaluated by a substitution giving du = constant ×
sin ax dx. So a convenient substitution is u = cos ax, which yields du = −a sin ax dx.
Z Z
3 −du
2
sin ax · cos ax dx = sin2 ax · u2 ·
Z  a
 −du
= 1 − cos2 ax · u2 ·
Z  a
 −du
= 1 − u2 · u2 ·
Z  a
1 
= u4 − u2 du
a
!
1 u5 u3
= − +C
a 5 3
!
1 cos5 ax cos3 ax
= − +C,
a 5 3
Information for Students in MATH 141 2009 01 3061

which integration may be checked by differentiation. (Of course, there are other, equiv-
alent ways of expressing this indefinite integral.)
Z
x3
4. [4 MARKS] Showing all your work, evaluate the integral √ dx, where a is a
x 2 + a2
given non-zero constant.
Solution: To simplify the surd in the denominator one may use either a trigonometric
or a hyperbolic substitution. For students in this course a trigonometric substitution is
usually a better choice. To arrange that x = a tan u, we use a substitution
x
u = arctan , (100)
a
and dx = a sec2 u du. We may assume that a > 0. The interval of validity for substitution
(100) is − π2 < x < π2 , in which the secant is positive.
Z Z 3 3
x3 a tan u
√ dx = · a sec2 u du
2
x +a 2 |a sec u|
Z
3
= a tan2 u · sec u tan u du
Z   d
3
= a sec2 u − 1 · sec u du
du
3
!
3 sec u
= a − sec u + C
3
effectively by substitution U = sec u
1 p
= (a tan u)2 − 2a2 (a tan u)2 + a2 + C
3
1 2 √
= x − 2a2 x2 + a2 + C
3
which integration may be verified by differentiation.
5. [9 MARKS] (To simplify the exposition of theZ solution, I work a specific example here.)
x + 21
Showing all your work, evaluate the integral dx.
(x + 9)(x − 5)
Solution: Since the degree of the numerator is less than that of the denominator, we can
dispense with the first step of dividing denominator into numerator.
(a) We need to expand the integrand into a sum of partial fractions; fortunately the
factorization of the denominator has been given. Assuming there are constants
A, B such that
x + 21 A B
= +
(x + 9)(x − 5) x + 9 x − 5
Information for Students in MATH 141 2009 01 3062

and transforming all fractions to have a common denominator, we find that


x + 21 = A(x − 5) + B(x + 9) .
(b) The values of A, B may be obtained either by comparing coefficients of like powers
of x, or by assigning to x successively the “convenient” values 5, -9: we obtain that
13
26 = 14B ⇒ B =
7
6
12 = −14A ⇒ A = − .
7
(c) We may now complete the integration:
Z Z !
x + 21 1 6 13
dx = − + dx
(x + 9)(x − 5) 7 x+9 x−5
1
= − (−6 ln(x + 9) + 13 ln(x − 5)) + C
7
which can also be expressed in other, equivalent ways. This integration may be
verified by differentiation.

Wednesday version
Za
1. [4 MARKS] Showing your work, evaluate (x2 + 1)e−x dx , where a is a given constant.
0
Solution: I will integrate by parts twice, in order to reduce the degree of the polynomial
factor of the integrand.
(a) First integration by parts:
u = x2 + 1 ⇒ du = 2x dx
dv = e−x dx ⇒ v = −e−x
Za h i Z a
2 −x 2 −x  a
(x + 1)e dx = (x + 1) −e + 2x · e−x dx .
0
0
0

(b) Second integration by parts:


U = 2x ⇒ dU = 2 dx
dV = e−x dx ⇒ V = −e−x
Za h i Z a
2 −x 2 −x  a
(x + 1)e dx = (x + 1 + 2x) −e + 2 · e−x dx .
0
0
0
Information for Students in MATH 141 2009 01 3063

(c) Completion of the integration:


Za h ia
(x2 + 1)e−x dx = (x2 + 2 + 2x) −e−x
0
0
= −(a2 + 2a + 3)e−a + 3.
π
Z2
2. [9 MARKS] Showing all your work, find a reduction formula for the integral sinn ax dx,
0
where a is a given integer.
Solution: Assume n is an integer greater than 1.
(a) Introduce a symbol for the definite integral sought:
Z π2
In = sinn ax dx .
0

(b) Integration by parts:


cos ax
dv = sin ax dx ⇒ v = − dx
a
u = sinn−1 ax ⇒ du = a(n − 1) sinn−2 ax · cos ax dx
" # π2 Z π2
1 n−1
In = − · sin ax · cos ax + (n − 1) sinn−2 ax · cos2 ax dx
a 0 0

(c) Decomposition of integral:


Z π2 Z π2  
n−2 2
sin ax · cos ax dx = sinn−2 ax · 1 − sin2 ax dx
0 0
Z π2 Z π2
n−2
= sin ax dx − sinn ax dx
0 0
= In−2 − In

(d) Solution of equation to obtain reduction formula


" # π2
1 n−1
In = − · sin ax · cos ax + (n − 1) (In−2 − In )
a 0
" # π2
1 n−1
⇒ nIn = − · sin ax · cos ax + (n − 1)In−2
a 0
1 n−1
⇒ In = (0 − 0) + In−2
n n
Information for Students in MATH 141 2009 01 3064

Because of the choice of limits and the fact that a is an integer, the “net change” is
0. Thus we obtain a very simple relationship, which can be solved. Students were
not asked to complete this part of the solution. For example, it is possible to prove
by induction that, if n = 2m, an even, positive integer, then
2m − 1 2m − 3 3
I2m = · · . . . · I0
2m 2m − 2 2
(2m)! π
= m · .
4 m!m! 2
(This is Exercise 44, page 481 in the textbook.)
Z
x2
3. [4 MARKS] Showing all your work, evaluate the integral  23 dx, where a is a
a2 − x 2
non-zero constant.
Solution: Without limiting generality we take a > 0. A trigonometric substitution can
simplify this integral. One such substitution would have x = a sin u; more precisely,
u = arcsin ax , defined for − π2 ≤ x ≤ π2 , in which interval the cosine and secant are
positive. Then dx = √a du 2 .
1−u
Z Z
x2 a2 sin2 u
 32 = · a cos u du
a2 − x 2 a3 cos3 u
Z   Z  
2
= tan u du = sec2 u − 1 du
= tan u − u + C
sin u
= −u+C
cos u
sin u
= p −u+C
2
1 − sin u
x
x
= q a − arcsin + C
2 a
1 − ax2
x x
= √ − arcsin + C
a2 − x 2 a

which may be verified by differentiation.


Z `
x(x − a)(x − b) + c
4. [9 MARKS] Showing all your work, evaluate the integral dx,
k (x − a)(x − b)
where a, b, k, ` are distinct constants such that the integrand is defined throughout the
Information for Students in MATH 141 2009 01 3065

given interval, and c is a non-zero constant. (The integrand was not presented to students
in factored form.)
Solution:

(a) Since the degree of the numerator is not less than the degree of the denominator,
begin by dividing the denominator into the numerator, obtaining a quotient and a
remainder:
Z Z
x(x − a)(x − b) + c c
dx = x+ dx .
(x − a)(x − b) (x − a)(x − b)

(b) Expand the fraction into partial fractions. Assume that


c A B
= + ,
(x − a)(x − b) x − a x − b
take to a common denominator, and equate the resulting polynomials:

c = A · (x − b) + B · (x − a) .

(c) Now either equate coefficients of like powers of x, or, equivalently, give x succes-
sive values x = a and x = b:
c = A(a − b) c
⇒A= = −B .
c = B(b − a) a−b

(d) The integration reduces to


Z Z !
x(x − a)(x − b) + c c 1 c c
dx = x+ · − · dx .
(x − a)(x − b) a−b x−a a−b x−b

(e) Complete the integration:


Z
x(x − a)(x − b) + c x2 c
dx = + (ln(x − a) − ln(x − b)) + C
(x − a)(x − b) 2 a−b
x2 c x−a
= + ln +C,
2 a−b x−b
the correctness of which integration may be verified by differentiation of the func-
tion on the right.
Information for Students in MATH 141 2009 01 3066

(f) Evaluate the definite integral


Z ` " 2 #`
x(x − a)(x − b) + c x c
dx = + (ln |x − a| − ln |x − b|)
k (x − a)(x − b) 2 a−b k
" 2 #`
x c x−a
= + ln
2 a−b x−b k

` − k2
2
c (` − a)(k − b)
= + ln
2 a − b (` − b)(k − a)
Z
5. [4 MARKS] Showing all work, evaluate the integral sin3 ax dx, where a is a positive
integer.
Solution:
Z Z  
3
sin ax dx = 1 − cos2 ax · sin ax dx

Z under substitution u = cos ax, where du = −a sin ax dx


1
= (1 − u2 )(− ) du
a
3
!
1 u
= − u− +C
a 3
cos ax cos3 ax
= − + +C.
a 3a

Thursday version
Z b √
1. [4 MARKS] Showing all your work, evaluate the integral e t dt.
a
√ 2

Solution: Begin with a substitution t = u, so t = u , dt = 2u du. When t = a, u = a,
etc.: Z √ Z
t
e dt = 2ueu du .

Now apply integration by parts:


U = u ⇒ dU = du
dVZ = 2eu du ⇒ V = 2eu Z
2ueu du = u · 2eu − 2eu du
= u · 2eu − 2eu + C = 2(u − 1)eu + C
√ √
= 2( t − 1)e t + C
Information for Students in MATH 141 2009 01 3067

h √ √ ib
The definite integral given is then equal to 2( t − 1)e t .
a
Z
2. [9 MARKS] Showing all your work, find a reduction formula for the integral (ln(ax + 1))n dx,
where a is a given, positive constant.
Solution: This problem is a slight generalization of Exercise 45, p. 481 in the textbook,
an odd-numbered problem for which there is a solution in the Student Solutions Manual,
and also hints on one of the CD-Roms supplied with the textbook.

(a) Introduce a symbol for the definite integral sought:

In = (ln(ax + 1))n dx .

(b) Change the variable (a step which is helpful, but not necessary)

u = ax + 1 ⇒ duZ= a dx
1
In = (ln u)n du
a

(c) Integration by parts:


1
U = (ln u)n ⇒ dU = n(ln u)n−1 ·
u
dV = 1 du ⇒ V = u Z
n
In = u(ln u) − n (ln u)n−1 du
Z
= (ax + 1)(ln(ax + 1)) − na n
(ln(ax + 1))n−1 dx
= (ax + 1)(ln(ax + 1))n − na · In−1
Z
3. [4 MARKS] Showing all your work, evaluate the integral cos4 at dt, where a is a
given non-zero constant.
Solution: We have to apply the following double angle identity twice:

cos 2θ = 2 cos2 θ − 1 .

Z Z !2
4 1 + cos 2at
cos at dt = dt
2
Z Z Z !
1 2
= cos 2at dt + 2 cos 2at dt + 1 dt
4
Information for Students in MATH 141 2009 01 3068

Z Z Z !
1 1 + cos 4at
= dt + 2 cos 2at dt + 1 dt
4 2
1 1 1 1 3
= · sin 4at + · sin 2at + t + C
8 4a 2 2a 8
1 1 3
= sin 4at + sin 2at + t + C
32a 4a 8
which may be verified by differentiation. (Of course, the integral may be expressed in
other ways under transformation by trigonometric identities.)
Z
dx
4. [4 MARKS] Showing your work, evaluate the integral √ , where a is a given
x x2 + a
positive integer, not a perfect square.
Solution: The surd in the denominator may be simplified by either a trigonometric or a
hyperbolic substitution. For students in this course the trigonometric substitutions are
usually easier.
√ √
x = a · tan θ ⇒ dx = a sec2 θ dθ
Z Z
dx sec2 θ dθ
√ = √ .
x x2 + a tan θ · a · | sec θ|
The actual substitution is given by θ = arctan √xa , valid for − π2 < x < π2 . In that interval
the secant function is positive, so the absolute signs may be dropped. The integral is
equal to
Z
1 1
√ csc θ dθ = √ ln | csc θ − cot θ| + C
a a

1 sec θ − 1
= √ ln +C
a tan θ

1 tan2 θ − 1
= √ ln + C
a tan θ

1 x2 + a − √a
= √ ln + C
a x
which can be verified by differentiation.
Z`
x(x − a)(x − b) + c
5. [9 MARKS] Showing all your work, evaluate the integral dx,
(x − a)(x − b)
k
where a, b, k, ` are distinct constants such that a, b are not contained in the interval whose
end-points are k, `, and c is a non-zero constant.
Solution: I will first determine an indefinite integral.
Information for Students in MATH 141 2009 01 3069

(a) Since the degree of the numerator is not less than the degree of the denominator,
begin by dividing the denominator into the numerator, obtaining a quotient and a
remainder:
Z Z
x(x − a)(x − b) + c c
dx = x+ dx .
(x − a)(x − b) (x − a)(x − b)

(b) Expand the fraction into partial fractions. Assume that


c A B
= + ,
(x − a)(x − b) x − a x − b
take to a common denominator, and equate the resulting polynomials:

c = A · (x − b) + B · (x − a) .

(c) Now either equate coefficients of like powers of x, or, equivalently, give x succes-
sive values x = a and x = b:
c = A(a − b) c
⇒A= = −B .
c = B(b − a) a−b

(d) The integration reduces to


Z Z !
x(x − a)(x − b) + c c 1 c c
dx = x+ · − · dx .
(x − a)(x − b) a−b x−a a−b x−b

(e) Complete the integration:


Z
x(x − a)(x − b) + c x2 c
dx = + (ln |x − a| − ln |x − b|) + C
(x − a)(x − b) 2 a−b

x2 c x − a
= + ln +C,
2 a − b x − b
the correctness of which integration may be verified by differentiation of the func-
tion on the right.
(f)
Z " 2 #`
`
x(x − a)(x − b) + c x c x − a
dx = + ln
k (x − a)(x − b) 2 a − b x − b k
Information for Students in MATH 141 2009 01 3070

D.1.4 Draft Solutions to Quiz Q4


Distribution Date: Posted on the Web on 06 April, 2007; corrected on 09 April, 2007.
Caveat lector! There could be misprints or errors in these draft solutions.

There were four different types of quizzes, for the days when the tutorials are scheduled. Each
type of quiz was generated in multiple varieties for each of the tutorial sections. The order of
the problems in the varieties was also randomly assigned. Each version of the quiz was graded
out of a maximum of 30 marks, but 2 of the versions had 5 problems and 2 had 4 problems.
All of the quizzes had a heading that included the instructions
• Time = 30 minutes

• No calculators!

• Show all your work: marks are not given for answers alone.

• Enclose this question sheet in your folded answer sheet.


In the following I will either provide a generic solution for all varieties, or a solution to one
typical variety.

Monday version
1. [6 MARKS] Showing all of your work, find the length of the following curve for the
interval 0 < a ≤ u ≤ b : !
eu + 1
y = ln u .
e −1
Solution:

(a)
!
dy eu − 1 eu (eu − 1) − (eu + 1)eu
= u
dx e +1 (eu − 1)2
2eu
= − 2u
e −1
!2 !2
dy e2u + 1
1+ = 2u
= coth2 u
dx e −1

(b) s
Z b !2 Z b
dy
arc length = 1+ du = | coth u| du
a dx a
Information for Students in MATH 141 2009 01 3071

(c) Successful completion of the integration:


Z b Z b
| coth u| du = coth u du
a a
since a < b
[ln sinh u]ba
!
sinh b eb − e−b
= ln = ln a
sinh a e − e−a

3.0

2.5

2.0

1.5

1.0

0.5

0.0

−1.5 −1.0 −0.5 0.0 0.5 1.0 1.5

Figure 26: The limaçon r = 1 + 2 sin θ

2. [10 MARKS] (see Figure 26 on page 3071) The graph of the following curve is given.
Information for Students in MATH 141 2009 01 3072

Showing detailed work, find the area that is enclosed between the inner and the outer
loops: r = a(1 + 2 sin θ), where a is a positive constant.
Solution:

(a) Determination of the limits of integration: we need first to locate where the curve
crosses itself. Since its formula is in terms of sin θ, the curve is periodic with period
(at most) 2π. As θ ranges over the values from 0 to 2π, the values r(θ) range over
uniquely determined values. How then can the curve cross itself? This can happen
either
i. at points (r(θ1 ), θ1 ) and (r(θ1 + π), θ1 + π) where r(θ1 + π) = −r(θ1 ); or
ii. at the pole, where r = 0 for two distinct values of θ.
The first possibility would, for the present curve, require that

1 + 2 sin(θ1 + π) = − (1 + 2 sin θ1 )

which is equivalent to

2 + 2 sin θ1 + 2 sin(θ1 + π) = 0

which is equivalent to 2 = 0, a contradiction. Thus the present curve can cross


itself only at the pole. That occurs where 1 + 2 sin θ = 0, i.e., where sin θ = − 12 .
The values of θ satisfying this equation are 2nπ− π6 and (2n+1)π+ π6 , where n is any
integer. The outer loop of this lima¸con is traced out, for example, for − π6 ≤ θ ≤ 7π
6
.
7π 11π
The inner loop is traced for 6 ≤ θ ≤ 6 .
(b) The area of the region bounded by the larger, outer loop is
Z 7π
2
6 1
a · (1 + 2 sin θ)2 dθ
− π6 2
Z 7π
6 1  
= a 2
· 1 + 4 sin θ + 4 sin2 θ dθ
− π6 2
Z 7π
2
6 1
= a · (1 + 4 sin θ + 2 − 2 cos 2θ) dθ
− π6 2
Z 7π
a2 6
= (3 + 4 sin θ − 2 cos 2θ) dθ
2 − π6

a2 7π
= [3θ − 4 cos θ − sin 2θ]−6 π
2 6
!
2
a 7π 7π 7π a2  π −π −π 
= − 4 cos − sin − − − 4 cos − sin
2 2 6 3 2 2 6 3
Information for Students in MATH 141 2009 01 3073

 √ 

 3 3 
= a2 2π +  .
2

(c) The area of the inner loop is


Z 11π
6 1
2
a · (1 + 2 sin θ)2 dθ

6
2
a2 11π
= [3θ − 4 cos θ − sin 2θ] 7π6
2 6
! !
a2 11π 11π 11π a2 7π 7π 7π
= − 4 cos − sin − − 4 cos − sin
2 2 6 3 2 2 6 3
 √ 
 3 3 
= a2 π −  .
2

(d) The area of the region between the loops is the excess of the area inside the outer
loop over the area inside the inner loop, i.e.,
 √   √ 

 
 3 3 
 
 3 3  √
a2 2π +  − π − 2
 = a (π + 3 3) .
2 2
Note that a cleaner way of solving this problem would have been to first integrate
from 0 to 2π, which would give the area between the loops plus twice the area
inside the smaller loop; and then to subtract twice the area inside the smaller loop.
This method is better because the first integral is very easy to evaluate, since the
periodic terms contribute nothing.
This curve is discussed in Exercise 10.4.21, on page 683 of the textbook, and is solved
in the Student Solutions Manual and also on one of the CD-Roms which accompany the
textbook.
3. [4 MARKS] Showing full details of your work, find the exact length of the curve x =
et + e−t , y = a − 2t, 0 ≤ t ≤ b, where a, b are constants.
Solution:
!2 !2
dx dy 2
+ = et − e−t + 4
dt dt
2
et + e−t
=
Z b

arc length = et + e−t dt
0
 b
= e − e−t
t
0 = eb − e−b = 2 sinh b .
Information for Students in MATH 141 2009 01 3074

4. [4
( MARKS] Find !) the value of the limit for the sequence. If it diverges, prove that fact:
3n
arctan .
3n + 1
3n 1
Solution: As n → ∞, 3n+1 = 1 − 3n+1 → 1. Since the arctangent function is continuous
at the point 1, the limit of the sequence is the arctangent of 1, i.e., π4 .

5. [6 MARKS] The given curve  is rotated about the y-axis. Find the area of the resulting
surface: x = 2 √1 2 y2 − ln y , (1 ≤ y ≤ a), where a is a real constant greater than 1.
Solution:

(a)
!
dx 1 1
= √ 2y −
dy 2 2 y
!2 !
dx 1 2 1
1+ = 1 + 4y + 2 − 4
dy 8 y
!
1 2 1
= 4y + 2 + 4
8 y
!!2
1 1
= √ 2y +
2 2 y

(b)
s
Z a !2
dx
surface area = 2πx 1 + dy
1 dy
Z !
π a 2  1
= y − ln y 2y + dy
4 1 y
Z !
π a 3 ln y
= 2y + y − 2y ln y − dy
4 1 y
" #a Z
π y4 y2 1 2 π a
= + − (ln y) − y ln y dy
4 2 2 2 1 2 1
R y2
(c) One way to integrate y ln y dy is by parts, with u = ln y, v0 = y: u0 = 1y , v = 2
,
Z Z
y2 y
y ln y dy = · ln y − dy
2 2
y2
= (2 ln y − 1) + C .
4
Information for Students in MATH 141 2009 01 3075

Another way is to use the substitution w = y2 , so dw = 2y dy, ln w = 2 ln y:


Z Z Z
ln w dw 1 1
y ln y dy = · = ln w dw = w (ln w − 1) + C etc.
2 2 4 4
Thus the surface area is
" #a
π 2 3 1 2 y2
y + ln y − (ln y) − (2 ln y − 1)
4 3 2 4 1
4 2 2
!
π a a 1 2 a
= + − (ln a) − (2 ln a − 1) − 1
4 2 2 2 4

Tuesday version
3
1. [4 MARKS] Showing detailed work, find the arc length function for the curve y = ax 2
with starting point P0 (1, a), where a is a positive constant. That is, find a function f (x)
whose value is the distance along the curve from the starting point to the point with
abscissa x.
Solution:
(a)
dy 3 √
= a· · x
dx 2
!2
dy 9a2 x
1+ = 1+
dx 4
(b)
Z r
x
9a2 t
f (x) = 1+ dt
1 4
 ! 3 x
 2 4 9a 2 2
t 
=  · 2 1 + 
3 9a 4
1
 3  3
4 + 9a x − 4 + 9a2 2
2 2
= .
27a2
2. [6 MARKS] Showing detailed work, find the area of the surface obtained by rotating the
following curve about the x-axis:
x2 ln x
y= − (a ≤ x ≤ b)
4 2
where a, b are two positive real constants, a < b.
Solution:
Information for Students in MATH 141 2009 01 3076

(a)
!
dy 1 1
= x−
dx 2 x
!2 !
dy 1 2 1
1+ = 1+ x + 2 −2
dx 4 x
!
1 2 1
= x + 2 +2
4 x
!!2
1 1
= x+
2 x

(b)
s
Z b !2
dy
surface area = 2πy 1 + dx
a dx
Z b 2 ! !
x ln x 1
= π − x+ dx
a 4 2 x
Z b 3 !
x x x ln x ln x
= π + − − dx
a 4 4 2 2x
" #b Z b
1 4 x2 1 2
= π x + − (ln x) − π x ln x dx
16 8 4 a a

R x2
(c) One way to integrate x ln x dx is by parts, with u = ln x, v0 = x: u0 = 1x , v = 2
,
Z Z
x2 x
x ln x dx = · ln x − dx
2 2
x2
= (2 ln x − 1) + C .
4
Another way is to use the substitution w = x2 , so dw = 2x dx, ln w = 2 ln x:
Z Z Z
ln w dw 1 1
x ln x dx = · = ln w dw = w (ln w − 1) + C etc.
2 2 4 4
Thus the surface area is
" #b
x4 x3 1 x2
π + − (ln x)2 − (2 ln x − 1)
16 8 4 4 a
Information for Students in MATH 141 2009 01 3077

3. [4 MARKS] Find the exact length of the curve given by

x = et cos t y = et sin t (0 ≤ t ≤ a)

where a is a positive constant.


Solution:

(a)
dx
= et (cos t − sin t)
dt
dy
= et (sin t + cos t)
dt
!2 !2
dx dy
+ = e2t (2 cos2 t + 2 sin2 t) = 2e2t
dt dt

(b) The arc length is Z a √ t √


2e dt = 2(ea − 1) .
0

4. [10 MARKS] Working only with polar coordinates, find the area of the region that lies
inside the first curve and outside the second curve: r = b sin θ, r = a, where a and b are
positive constants.
Solution:

(a) Both of these curves are circles; we need to determine the coordinates of the points
of intersection. Solving the equations yields
a
r=a sin θ =
.
b
 
One point of intersection will be (r, θ) = a, arcsin ba . Another point of intersection
 
will be (r, θ) = a, π − arcsin ab — remember that the values of the arcsine function
h i
are in the interval − π2 , π2 . It appears from a drawing that we have all the points of
intersection. If we solve the equation r = −a with r = b sin θ we obtain precisely
the same points, albeit with different coordinates. If we attempt to replace the
equation r = b sin θ with that obtained under the identification (r, θ) → (−r, θ + π)
there is no change. This algebraic investigation discloses all possible points of
intersection except the pole, which must be checked separately. But the pole cannot
lie on r = a, since the pole has first coordinate 0 always. Thus we have, indeed,
found all the points of intersection.
Information for Students in MATH 141 2009 01 3078

(b) The area bounded by the arcs can be considered to consist of the disk r = b sin θ
diminished by a sector of the circle r = a for arcsin ba ≤ θ ≤ π − arcsin ab , together
with two small segments of the disk r = b sin θ bounded by the rays θ = arcsin ab
and θ = π − arcsin ab . What I have provided is one prescription for computing the
area. An easier way would be to take the integral
Z a
1 π−arcsin b  
(b sin θ)2 − a2 dθ
2 arcsin ab
Z π2  
= (b sin θ)2 − a2 dθ
a
arcsin b

by symmetry around the line θ = π2


Z π2 ! !
2 1 − cos 2θ 2
= b − a dθ
arcsin ba 2
Z π2 ! !
b2 2 b2
= − a − cos 2θ dθ
arcsin ba 2 2
" 2 ! # π2
b 2 b2
= − a θ − sin 2θ
2 4 arcsin ab
2  
2
b − 2a π a a√ 2
= − arcsin + b − a2
2 2 b 2
5. [6 MARKS] Determine whether the series is convergent or divergent. If it is convergent,
X∞ ( )
b
find its sum. Otherwise prove that it is divergent: , where a, b are positive
n=1
n(n + a)
integers.
Solution:
(a) Expand the general term into partial fractions: there exist constants A, B such that
b A B
= + .
n(n + a) n n + a
To determine the coefficients A, B we can proceed in several ways. If we take the
fractions to a common denominator n(n + a), we obtain the polynomial identity
b = A · (N + A) + b · a .
In this identity, if we set the variable n equal to −a, we obtain that b = B(−a), so
B = − ab ; and, setting n = 0, we obtain A = ba , hence
!
b b 1 1
= − .
n(n + a) a n n + a
Information for Students in MATH 141 2009 01 3079

(b) For sufficiently large N the Nth partial sum is equal to


X
N !
b
n=1
n(n + a)
!
b 1 1 1 1 1 1
= + + ... + − − − ... −
a 1 2 a−1 N+1 N+2 N+a
!
b 1 1 1
→ + + ... + − 0 − 0 − ... − 0
a 1 2 a−1
as N → ∞. Hence the series converges to the sum
!
b 1 1 1
+ + ... + .
a 1 2 a−1

Convergence could be proved in other ways, thereby earning the student part marks. For
example, using the Comparison or Limit Comparison Tests, or the Integral Test.

Wednesday version
1. [10 MARKS] Showing detailed work, find all points different from the origin on the
following curve where the tangent is horizontal; a is a positive constant:

x = a(cos θ − cos2 θ), y = a(sin θ − sin θ cos θ) .

Solution:

(a)
dx
= a (− sin θ − 2 cos θ(− sin θ))

= a(sin θ)(2 cos θ − 1)
dy  
= a − cos θ + 2 cos2 θ − 2 sin2 θ
dθ  
= a − cos θ + 2 cos2 θ − 1
= a(2 cos θ + 1)(cos θ − 1)
dx
Actually, you weren’t expected to find dt
.
dy
(b) There will be a horizontal tangent at the point with parameter value t if dx
= 0,
dy
dθ dy dx
i.e., if dx
= 0, implying that dθ
must be 0, provided dθ
, 0 at the same value of t.

Information for Students in MATH 141 2009 01 3080

(This last requirement is subtle, and you weren’t expected to actually check it. It is
because of this restriction that I explicitly excluded the origin from consideration.)
The equations we have to solve are

cos θ = 1

and
1
cos θ = − .
2
They are not being solved simultaneously: we are looking for points t that satisfy
at least one — meaning, here, either — of the equations.
(c) The first of these equations is satisfied when θ is an even integer multiple of π. But
this parameter value corresponds always to the origin, which was excluded from
consideration.
(d) The second is satisfied when θ is of the form
1
(2n + 1)π ± π .
3
(e) The functions defining this curve are all periodic with period 2π. Thus we can
study the curve completely by examining its behavior for parameters θ chosen over
an interval of length 2π, e.g. 0 ≤ θ < 2π. There are precisely 3 points here where
dy

= 0:

(x(0), y(0)) = (0, 0)


! !!  √ 
2π 2π  a 3 3a 
x ,y = − , 
3 3 4 4
! !!  √ 
4π 4π  a 3 3a 
x ,y = − , −  .
3 3 4 4

Of these, you were specifically instructed to exclude the first, which is the origin.

2. [5 MARKS] Showing detailed work determine the total length of the portion of the
following curve which is in the first quadrant: x = a cos3 θ, y = a sin3 θ, where a is a
positive constant.
Solution:
dx
= −3a cos2 θ · sin θ

dy
= 3a sin2 θ · cos θ

Information for Students in MATH 141 2009 01 3081

!2 !2  
dx dy
⇒ + = 9a2 sin2 θ · cos2 θ · cos2 θ + sin2 θ
dθ dθ
= 9a2 sin2 θ cos2 θ .

The curve is in the first quadrant when both coordinates are positive; as each of these is
a cube of a sine or cosine, this means that the portion of the curve in the first quadrant is
that given by 0 ≤ θ ≤ π2 . The length of the arc is
Z π
p Z π " # π2
2
2
2 sin2 θ 3a
9a2 sin θ cos2 θ dθ = 3a sin θ · cos θ dθ = 3a = .
0 0 2 0 2

3. [10 MARKS] Find the area of the region that lies inside both of the following curves
r = a + 2 sin θ, r = a − 1, where a is a suitable positive constant.
Solution:

(a) Determination of the limits of integration: we need first to locate where the curves
cross. We begin by solving the two given equations, and find that
1 7π 11π
sin θ = − ⇒ θ = or
2 6 6
or any angle obtained from these by adding an integer multiple of 2π. This yields
the points ! !
7π 11π
a − 1, , a − 1, .
6 6
Students weren’t expected to pursue this question further. Strictly speaking, they
should then have solved r = a + 2 sin θ, r = −(a − 1), which would have yielded no
points; then solved r = −a + 2 sin θ, r = a − 1, which would again yield no points;
then r = −a + 2 sin θ, r = −(a − 1), which would have yielded the 2 points already
found.
(b) This problem could then be approached in several ways. To find the area “directly”
would require finding the sum of the integrals
Z 7π
1 6
(a − 1)2 dθ
2 −pi
6

and Z 11π
1 6
(a + 2 sin θ)2 dθ .
2 7pi
6
Information for Students in MATH 141 2009 01 3082

2π(a−1)2
The first of these is just 2/3 of the area of a disk of radius a − 1, i.e., 3
. The
second is
Z 11π
1 6
(a + 2 sin θ)2 dθ
2 7pi6
Z 11π
6  
1
= a2 + 4a sin θ + 2(1 − cos 2θ) dθ
2 7pi6
1h 2 i 11π
= a θ − 4a cos θ + 2θ − sin 2θ 7π6
2 6

1π 2 √ 2π
= · a − 2 3a + .
3 3
(Another way to solve this would be to find the area of the disk of radius a − 1 and
subtract from it the portion that is cut off.
These integrals could have been slightly more efficiently computed by taking only
the area up to the y-axis and doubling it.)
Hence the net area of the region inside both of the curves is
!
5π 2 4π √ 4π
a − +2 3 + .
9 3 3

4. [5 MARKS] Showing detailed work, determine whether the following series is conver-
gent or divergent. If it is convergent, find its sum. Otherwise explain why it diverges:
X

an + bn
,
n=1
(ab)n

where a, b are integers greater than 1.


Solution: The nth term is !n !n
1 1
an = + .
b a
The nth partial sum is, therefore, the sum of the partial sums of two geometric series;
both of the geometric series are convergent, since the common ratios are less than 1
in magnitude. Since the two separate partial sums approach a limit, the sum of these
sequences approaches as its limit
 the sum of the limits of the two sequences. In the case
1 n
of the series whose nth term is b , the limit of the partial sums is
1
b 1
1
= ;
1− b
b−1
Information for Students in MATH 141 2009 01 3083

1
similarly, the second series sums to a−1 . Hence the given series sums to the sum of these
limits, i.e.
1 1 a+b−2
+ = .
b − 1 a − 1 (a − 1)(b − 1)

Thursday version

1. [10 MARKS] The curve x = a(1 − 2 cos2 t), y = (tan t)(1 − 2 cos2 t), where a is a
given positive integer, crosses itself at some point (x0 , y0 ). Showing all your work, find
the point of crossing, and the equations of both tangents at the point. (In determining
the point of crossing you are expected to investigate the parametric functions: it is not
sufficient to simply plot a finite number of points on the curve.)
Solution:

(a) Since the functions are all periodic with period π, it suffices to take an interval of
this length for t, and that will reveal all aspects of the behavior of this curve. (More
precisely, the tangent function has period π, and, while the cosine function has
period 2π, its square has period π.) So, without limiting generality, let’s consider
− π2 ≤ t ≤ π2 : we have to exclude both end points of this interval, since the tangent
function is not defined at either of them.
Suppose that the curve crosses itself at the points with parameter values t = t1 and
t = t2 ; without limiting generality, we can assume that these parameter values have
been so labelled that t1 < t2 . Since the x-coordinates will need to be the same,

a(1 − 2 cos2 t1 ) = a(1 − 2 cos2 t2 ) (101)

so
cos t1 = ± cos t2 . (102)
Since the y-coordinates must also coincide, we also have

(tan t1 )(1 − 2 cos2 t1 ) = (tan t2 )(1 − 2 cos2 t2 ) (103)

which implies that either


1
cos2 t1 = cos2 t2 = , (104)
2
or
tan t1 = tan t2 . (105)
In the interval we have chosen for t, the cosines are always positive; the only solu-
tion to (104) is t1 = − π4 , t2 = + π4 . In that same interval for t there will be no solu-
tions to (105), since the tangent function is increasing there. Thus the only possible
Information for Students in MATH 141 2009 01 3084

crossing points are t = ± π4 , and the point of crossing is the origin, (x, y) = (0, 0).
While it isn’t required in the solution, note that as t → ± π2 , x → 1: this curve is
asymptotic to the vertical line x = 1.
(b) Tangent at the point with parameter value t = − π4 :
dx
= 4a(cos t)(sin t) = 2a sin 2t = −2a
dt
y = (tan t)(1 − 2 cos2 t) = tan t − sin 2t
dy
= sec2 t − 2 cos 2t = 2 − 0
dt
dy
dy dt 2 1
= dx
=− =− ,
dx dt
2a a
and the tangent has equation y = − ax .
(c) Tangent at the point with parameter value t = π4 :
dx
= 4a(cos t)(sin t) = 2a sin 2t = 2a
dt
dy
= sec2 t − 2 cos 2t = 2 − 0
dt
dy
dy dt 2 1
= dx = = ,
dx dt
2a a
and the tangent has equation y = ax .
2. [5 MARKS] Showing detailed work, find the surface area generated by rotating the
following curve about the y-axis.
x = at2 , y = bt3 , 0 ≤ t ≤ 5.

Solution:
dx
= 2at
dt
dy
= 3bt2
dt
!2 !2
dx dy
+ = 4a2 t2 + 9b2 t4
dt dt
Z 5 √
area about y-axis = 2π at2 4a2 t2 + 9b2 t4 dt
0
Z 5 √
= 2πa t3 4a2 + 9b2 t2 dt
0
Information for Students in MATH 141 2009 01 3085

Under the substitution u = 4a2 + 9b2 t2 ,

du = 18b2 t dt
Z 5 √ Z 4a2 +225b2
3 aπ 3 1
2πa 2 2 2
t 4a + 9b t dt = 4
(u 2 − 4a2 u 2 ) du
0 81b 4a2
" #4a2 +225b2
πa 2 52 8a2 32
= u − u
81b4 5 3 4a2
2πa  
2 2 23 2 2 5
= (4a + 225b ) (2a + 675b ) − 16a
(81)(15)b4
= ...

3. [10 MARKS] There is a region in the first quadrant that is bounded by arcs of both of
the following curves. Showing your work in detail, find the area of the region:

r2 = a sin 2θ r2 = a cos 2θ .

Solution:
(a) The given curves are expressed only in terms of sine and cosine of 2θ. The given
functions are periodic with period π. When, for either of these curves, we permit
θ to range over an interval of length π, we will trace out the entire curves. The
intersections of the curves in the first quadrant will be where sin 2θ = cos 2θ is
positive: thus the only point we have found by this algebraic solution of the two
equations is at θ = π8 .
However there are other ways in which curves can intersect, since points have
infinitely many different sets of polar coordinates. If we transform either of the
given equations under the substitution (r, θ) → (−r, θ + π), we find that there is
no change in the equation. Thus we haven’t missed any points because of the
convention that permits the first coordinate to be negative.
But there is another situation that leads to multiple sets of coordinates; that is at the
pole, where the second — angular — coordinate is totally arbitrary; the pole can
lie on a curve simply because of the fact that its distance coordinate r = 0, with no
reference to θ. To determine whether the pole lies on a curve we must investigate
whether the equation is satisfied by r = 0 with any value of θ. We find the curve
r2 = a sin 2θ does contain the pole: when r = 0 the equation is satisfied by any θ
such that sin 2θ = 0; so two solutions are θ = 0 and θ = π2 . Similarly, the pole lies
on the curve r2 = a cos 2θ with θ = π4 . I have given only the coordinates in the first
quadrant. To summarize: there are 2 intersection points in the first quadrant:
 1 1 π
(r, θ) = a 2 2− 4 ,
4
Information for Students in MATH 141 2009 01 3086

where the point lies on both of the curves with the same pair of coordinates; and
the pole, which lies on the two curves with different sets of coordinates.
√ 
(b) We can find the area by joining the point √4 a , π8 to the pole and calculating the
2
sum of two integrals:
Z π Z π
πa 4 πa 2
sin 2θ dθ + cos 2θ, dθ
2 0 2 π4
πa π πa π πa πa πa
= [− cos 2θ]04 + [sin 2θ] π2 = + = .
4 4 4 4 4 2
4. [5 MARKS] Showing detailed work, express the number below as a ratio of integers.

0.ab = 0.abababab...

where a, b are any two digits. You are expected to simplify your answer as much as
possible.
Solution: The repeating decimal is the sum of an infinite series
! ! ! !
10a + b 10a + b 1 10a + b 1 10a + b 1
+ + + + ...
100 100 100 100 1002 100 1003
XN !
10a + b 1
= lim
N→∞
n=0
100 100n
! XN !n
10a + b 1
= lim
100 N→∞ n=0 100
!  N+1
1
10a + b 1 − 100
= lim 1
100 N→∞ 1 − 100
!
10a + b 1
= 1
100 1 − 100
!
10a + b 100 10a + b
= =
100 99 99

D.2 MATH 141 2008 01


D.2.1 Draft Solutions to Quiz Q1
Release Date: Mounted on the Web on Friday, 01 February, 2008 (but subject to correction)
Information for Students in MATH 141 2009 01 3087

There were four different types of quizzes, for the days when the tutorials are scheduled. Each
type of quiz was generated in multiple varieties for each of the tutorial sections. The order of
the problems in the varieties was also randomly assigned. All of the quizzes had a heading that
included the instructions

• Time = 30 minutes

• No calculators!

• Show all your work: marks are not given for answers alone.

• Enclose this question sheet in your folded answer sheet.

In the following I will either provide a generic solution for all varieties, or a solution to one
typical variety.

Monday version
Zc Zc Zb
1. [5 MARKS] If f (x) dx = k and f (x) dx = `, find f (x) dx. Show your work.
a b a
Solution:

(a)
Zc Zb Zc
f (x) dx = f (x) dx + f (x) dx .
a a b

(b) Hence
Zc Zc Zb
f (x) dx = f (x) dx − f (x) dx .
b a a

(c)
= k −`.
Z a √
2. [5 MARKS] Find an antiderivative of the integrand of the integral x dx, and then
0
use the Fundamental Theorem of Calculus to evaluate the integral. You are not expected
to simplify your numerical answer, but no marks will be given unless all your work is
clearly shown.
Information for Students in MATH 141 2009 01 3088

1
(a) One antiderivative of x 2 is
1 1 2 3
1
· x 2 +1 = x 2 .
2
+1 3

(b) Z " #a
a √ 2 32 2  23  2 3
x dx = x = a − 0 = a2 .
0 3 0 3 3

Zx4
1
3. [10 MARKS] Showing all your work, differentiate the function g(x) = √ dt.
2 + t2
tan x
Solution:
(a) First split the interval of integration into 2 parts at a convenient place:

Z0 Zx4
1 1
g(x) = √ dt + √ dt .
2 + t2 2 + t2
tan x 0

(b) Then reverse the limits in the first summand and change its sign, so that the variable
limit is the upper one:

Z
tan x Zx4
1 1
g(x) = − √ dt + √ dt .
2 + t2 2 + t2
0 0

(c) Denote the upper limit of the first integral by u = tan x. Then
Z
tan x Zu
d 1 d 1 du
√ dt = √ dt ·
dx 2 + t2 du 2 + t2 dx
0 0
1
= √ · sec2 x
2+ u2
2 sec x · tan x 2 sec x · tan x
= √ = √ .
2 + tan2 x 1 + sec2 x

(d) Denote the upper limit of the second integral by v = x4 . Then

Zx4 Zv
d 1 d 1 dv
√ dt = √ dt ·
dx 2 + t2 dv 2 + t2 dx
0 0
Information for Students in MATH 141 2009 01 3089

1
= √ · 4x3
2+v 2

4x3
= √ .
2 + x8

(e) Hence
d 2 sec x · tan x 4x3
g(x) = − √ + √ .
dx 2 + tan2 x 2 + x8
Z x Z t2
a + ub
4. [10 MARKS] If F(x) = f (t) dt, where f (t) = du and a, b are constants,
1 1 u
find F 00 (2).
Solution:

(a) Applying Part 1 of the Fundamental Theorem yields


Z x2
0 a + ub
F (x) = f (x) = du .
1 u

(b) A second application of Part 1 of the Fundamental Theorem yields


Z x2
00 0d a + ub
F (x) = f (x) = du .
dx 1 u

(c) Denote the upper index of the last integral by v = x2 .


(d)
Z x2 Z v
d a + ub d a + ub
du = du
dx 1 u dx 1 u
Z v
d a + ub dv
= du ·
dv 1 u dx
b
a+v dv
= ·
v dx
b
a+v
= · 2x
v
a + x2b
= 2
· 2x
x 
2 a + x2b
= .
x
Information for Students in MATH 141 2009 01 3090

Tuesday version

1. [5 MARKS] Use Part 1 of the Fundamental Theorem of Calculus to find the derivative
of the function Z a
g(x) = b tan(t) dt ,
x
(where a and b are constants). Then use Part 2 of the Fundamental Theorem to evaluate
g(x), by first verifying carefully that ln | sec x| is an antiderivative of tan x.
Solution:

(a) Part 1 of the Fundamental Theorem gives the derivative of a definite integral as a
function of its upper index of integration. Here the variable is the lower index of
integration.
Z a
d
b tan(t) dt
dx x
Z x !
d
= − b tan(t) dt
dx a
Z x
d
= − b tan(t) dt
dx a
= −b tan x .

Some students may quote a variant of Part 1 which gives the derivative of a definite
integral with respect to the lower index, and this should be accepted if work has
been shown.
(b) Students were expected to first find the derivative of ln | sec x|. Since this is a com-
position of 2 functions, the Chain Rule will be needed. Let u = sec x. Then
d d d
ln | sec x| = ln |u| · secx
dx du dx
1
= · sec x tan x
u
1
= · sec x tan x
sec x
= tan x .

Hence
sec a cos x
g(x) = b ln | sec t|ax = b ln = b ln .
sec x cos a
Information for Students in MATH 141 2009 01 3091

2. [5 MARKS] Evaluate the limit by first recognizing the sum as a Riemann sum for a
function defined on [0, 1]:
r r r r 
1  7 14 21 7n 
lim  + + + ... + .
n→∞ n n n n n

Solution:

(a) We are told to take the interval of integration to be [0, 1]; when this is divided into
1
n equal parts, each has length ∆x = . Such a factor has been explicitly written in
n
the sum.
r
7i
(b) The typical summand is — aside from the common factor 1n — of the form .
n
Since the distance of the left end-point of the ith subinterval from 0 is i∆x = ni , we
r
7i √
may interpret = 7x .
n
(c) Thus the limit must be equal to
Z 1√ #
√ 2 3 1 2√
7x dx = 7 · · x 2 = 7.
0 3 0 3

3. [10 MARKS] Use Part 1 of the Fundamental Theorem of Calculus to find the derivative
of the function Z √x
cos t
g(x) = b dt ,
a t
where a, b are constants.
Solution:

(a) Denote the upper index of the integral by u(x) = x.
(b) Then
Z √x
d d cos t
g(x) = b dt
dx dx a t
Z u(x)
d cos t
= b dt
dx a t
Z u(x)
d cos t du(x)
= b dt ·
du a t dx
cos u du(x)
= b ·
u dx
Information for Students in MATH 141 2009 01 3092

cos u 1
= b · √
u 2 x

cos x 1
= b √ · √
x 2 x

cos x
= b
2x
4. [10 MARKS] Showing all your work, determine all values of x where the curve y =
Zx
1
dt is concave downward, where a, b are constants.
1 + at + bt2
0
Solution:

(a) By Part 1 of the Fundamental Theorem,


1
y0 (x) = .
1 + ax + bx2
(b) Differentiating a second time yields
!
00 d 1
y (x) =
dx 1 + ax + bx2
 
 1  d  2

= −   
 · 1 + ax + bx
1 + ax + bx2 2 dx
a + 2bx
= −  .
1 + ax + bx2 2

(c) The curve is concave downward where y00 < 0:


a + 2bx
− 2 > 0 ⇔ −(a + 2bx) > 0
1 + ax + bx2
since the denominator is a square, hence positive
⇔ 2bx < −a



 x < − 2ba when b > 0
 a
⇔ 
 x>− when b < 0

 never concave2bupward when b = 0

Wednesday version
Information for Students in MATH 141 2009 01 3093

π
Zb
1. [5 MARKS] Evaluate the integral sin t dt.
π
a

Solution:

(a) An antiderivative of sin t is − cos t.


(b)
π
Zb
π
sin t dt = [− cos t] πb
a
π
a

(c) Your answer should be simplified as much as possible.

2. [5 MARKS] Evaluate the following limit by first recognizing the sum as a Riemann
Xn
i8
sum for a function defined on [0, 1]: lim 9
. A full solution is required — it is not
n→∞
i=1
n
sufficient to write only the value of the limit.
Solution:

(a) We are told to take the interval of integration to be [0, 1]; when this is divided into
1
n equal parts, each has length ∆x = .
n
1  i 8
(b) The typical summand is — aside from the common factor — of the form .
n n
i
Since the distance of the left end-point of the ith subinterval from 0 is i∆x = , we
 i 8 n
8
may interpret =x .
n
(c) Thus the limit must be equal to
Z 1 #1
1 1
x dx = x9
8
= .
0 9 0 9

3. [10 MARKS] Use Part 1 of the Fundamental Theorem of Calculus to find the derivative
Zbx
of the function cos (tc ) dt, where a, b, c are real numbers.
cos x
Solution:
Information for Students in MATH 141 2009 01 3094

(a) The Fundamental Theorem gives the derivative of a definite integral with respect to
the upper limit of integration, when the lower limit is constant. The given integral
must be expressed in terms of such specialized definite integrals.
Zbx Z0 Zbx
cos (tc ) dt = cos (tc ) dt + cos (tc ) dt
cos x cos x 0
Z
cos x Zbx
= − cos (tc ) dt + cos (tc ) dt
0 0

Zbx
(b) For the summand cos (tc ) dt, let u = bx. Then
0

Zbx Zu
d c d
cos (t ) dt = cos (tc ) dt
dx dx
0 0
Zu
d du
= cos (tc ) dt ·
du dx
0
= cos (uc ) · b
= cos ((bx)c ) · b
Z
cos x

(c) For the summand cos (tc ) dt, let v = cos x.


0

Z
cos x Zv
d d
cos (tc ) dt = cos (tc ) dt
dx dx
0 0
Zv
d dv
= cos (tc ) dt ·
dv dx
0
= cos (vc ) · (− sin x)
= cos (cosc x) · (− sin x)
(d)
Zbx
cos (tc ) dt = − cos (cosc x) · (− sin x) + cos ((bx)c ) · b .
cos x
Information for Students in MATH 141 2009 01 3095




 0 if x < 0

 Z x

 x if 0 ≤ x ≤ a
4. [10 MARKS] Let f (x) =   and g(x) = f (t) dt, where a is


 2a − x if a < x < 2a 0

 0 if x > 2a
a positive constant. Showing all your work, find a formula for the value of g(x) when
a < x < 2a.
Solution:

(a) The interval where we seek a formula is the third interval into which the domain
has been broken. For x in this interval the integral can be decomposed into
Z x Z a Z x
f (t) dt = f (t) dt + f (t) dt .
0 0 a

The portion of the definition of f for x < 0 is of no interest in this problem, since
we are not finding area under that portion of the curve; the same applies to the
portion of the definition for x > 2a.
(b)
Z a Z a
f (t) dt = t dt
0 0
" #
2 t=a
t a2
= = .
2 t=0 2

(c)
Z x Z x
f (t) dt = (2a − t) dt
a a
" #t=x
t2
= 2at −
2 t=a
! !
x2 2 a2
= 2ax − − 2a − .
2 2

(d) ! !
a2 x2 2 a2 x2
g(x) = + 2ax − − 2a − = 2ax − − a2 .
2 2 2 2

Thursday version
Information for Students in MATH 141 2009 01 3096

1. [5 MARKS] Use Part 2 of the Fundamental Theorem of Calculus to evaluate the integral
Zbπ
cos θ dθ, where a, b are given integers. No marks will be given unless all your work

is clearly shown. Your answer should be simplified as much as possible.
Solution:

(a) One antiderivative of cos θ is sin θ.


(b)
Zbπ
cos θ dθ = [sin θ]bπ
aπ = sin(bπ) − sin(aπ) .

(c) Students were expected to observe that the value of the sine at the given multiples
of π is 0, so the value of the definite integral is 0.
X
n
2. [5 MARKS] Express lim axi sin xi ∆x as a definite integral on the interval [b, c],
n→∞
i=1
which has been subdivided into n equal subintervals.
Solution: Z c
ax sin x dx .
b

3. [10 MARKS] Use Part 1 of the Fundamental Theorem of Calculus to find the derivative
of the function Z bx 2
t +c
g(x) = 2
dt ,
ax t − c
where a, b, c are positive integers.
Solution:

(a) The Fundamental Theorem gives the derivative of a definite integral with respect to
the upper limit of integration, when the lower limit is constant. The given integral
must be expressed in terms of such specialized definite integrals.
Z bx 2 Z 0 2 Z bx 2
t +c t +c t +c
g(x) = 2
dt = 2
dt + dt
ax t − c ax t − c 0 t2 − c
Z ax 2 Z bx 2
t +c t +c
=− 2
dt + dt
0 t −c 0 t2 − c
Information for Students in MATH 141 2009 01 3097

Zbx
t2 + c
(b) For the summand dt, let u = bx. Then
t2 − c
0

Zbx Zu
d t2 + c d t2 + c
dt = dt
dx t2 − c dx t2 − c
0 0
Zu
d t2 + c du
= 2
dt ·
du t −c dx
0
2
u + c du
= ·
u2 − c dx
u2 + c
= 2 ·b
u −c
(bx)2 + c
= ·b
(bx)2 − c
Z ax 2
t +c
(c) For the summand dt, let u = ax. Then, analogously to the preceding
0 t2 − c
step, Z ax 2
d t +c (ax)2 + c
dt = · a.
dx 0 t2 − c (ax)2 − c
(d)
(bx)2 + c (ax)2 + c
g0 (x) = · b − · a.
(bx)2 − c (ax)2 − c
Zx3

4. [10 MARKS] Find the derivative of the function f (x) = t cos t dt.

x

Solution:

(a) First split the interval of integration into 2 parts at a convenient place:

Z0 Zx3
√ √
f (x) = t cos t dt + t cos t dt

x 0
Information for Students in MATH 141 2009 01 3098

(b) Then reverse the limits in the first summand and change its sign, so that the variable
limit is the upper one:

Zx Zx3
√ √
f (x) = − t cos t dt + t cos t dt .
0 0


(c) Denote the upper limit of the first integral by u = x. Then

Zx Zu
d √ d √
t cos t dt = t cos t dt
dx dx
0 0
Zu
d √ du
= t cos t dt ·
du dx
0
√ du
= u cos u ·
q dx
√ √ 1
= x cos x · √
2 x

cos x
= 1
.
2x 4

(d) Denote the upper limit of the second integral by v = x3 . Then

Zx3 Zv
d √ d √
t cos t dt = t cos t dt
dx dx
0 0
Zv
d √ dv
= t cos t dt ·
dv dx
0
√ dv
= v cos v ·
√  dx
= x cos x3 · 3x2
3

7
 
= 3x 2 cos x3

(e) Hence √
d cos x 7
 
f (x) = − + 3x 2 cos x3 .
dx 1
2x 4
Information for Students in MATH 141 2009 01 3099

D.2.2 Draft Solutions to Quiz Q2


Distribution Date: Mounted on the Web on Monday, March 03rd, 2008
Caveat lector! There could be misprints or errors in these draft solutions.

There were four different types of quizzes, for the days when the tutorials are scheduled. Each
type of quiz was generated in multiple varieties for each of the tutorial sections. The order of
the problems in the varieties was also randomly assigned. All of the quizzes had a heading that
included the instructions

• Time = 30 minutes

• No calculators!

• Show all your work: marks are not given for answers alone.

• Enclose this question sheet in your folded answer sheet.

In the following I will either provide a generic solution for all varieties, or a solution to one
typical variety.

Monday version

1. [10 MARKS] Showing all your work, find the volume of the√solid obtained by rotating
about the line y = 1 the region bounded by the curves y = n x and y = x, where n is a
given positive integer.
Solution: A favoured method of solution was not prescribed.

Using the method of “washers”: (a) The solution I am giving is for the case where n
is even.
(b) Find the intersections of the curves bounding the region. Solving the 2 equa-
tions yields the points (x, y) = (0, 0), (1, 1).
(c) It’s not clear from the wording of the problem whether it was intended, in the
case of odd n, to permit the second intersection point (x, y) = (−1, −1); the
decision was left to the individual TA’s. The remainder of this solution covers
the case of even n; for odd n this solution does not consider the solid generated
by rotating the region with vertices (x, y) = (−1, −1), (0, 0).
(d) Find the inner and outer dimensions of the washer. Since the axis of revolution
is a horizontal line, the element of area being rotated is vertical. For√arbitrary
x the lower point on the element is (x, x); the upper point is (x, n x). √ The
distances of these points from the axis are, respectively 1 − x and 1 − n x.
Information for Students in MATH 141 2009 01 3100

(e) The volume of the “washer” is, therefore,


 √ 
π −(1 − x)2 + (1 − n x)2 ∆x .

(f) Correctly evaluate the integral:


Z 1
√ 
π −(1 − x)2 + (1 − n x)2 dx
0
Z 1 
1 2
= π −2x + x2 + 2x n − x n dx
0
" #1
2 x2 2n n+1 n n+2
= π −x + + xn − xn
3 n+1 n+2 0
!
1 2n n (n − 1)(n + 4)π
= π −1 + + − =
3 n+1 n+2 3(n + 1)(n + 2)

Using the method of cylindrical shells: (a) The solution I am giving is for the case
where n is even.
(b) Find the intersections of the curves bounding the region. Solving the 2 equa-
tions yields the points (x, y) = (0, 0), (1, 1).
(c) It’s not clear from the wording of the problem whether it was intended, in the
case of odd n, to permit the second intersection point (x, y) = (−1, −1); the
decision was left to the individual TA’s. The remainder of this solution covers
the case of even n; for odd n this solution does not consider the solid generated
by rotating the region with vertices (x, y) = (−1, −1), (0, 0).
(d) Find the inner and outer dimensions of the washer. Since the axis of revolution
is a horizontal line, the element of area being rotated is also horizontal. For
arbitrary y the left endpoint on the element is (yn , y); the right endpoint is (y, y).
The length of the element is, therefore, y−yn ; the distances of the element from
the axis of symmetry is 1 − y.
(e) The volume of the cylindrical shell element of volume is, therefore,

2π(1 − y) · (y − yn ) · ∆y .

(f) Correctly evaluate the integral:


Z 1
2π (1 − y)(y − yn ) dy
0
Z 1 
= 2π −yn + yn+1 + y − y2 dy
0
Information for Students in MATH 141 2009 01 3101

" #1
1 n+1 1 n+2 1 2 1 3
= 2π − y + y + y − y
n+1 n+2 2 3 0
!
1 1 1 1
= 2π − + + − −0
n+1 n+2 2 3
!
1 1 (n − 1)(n + 4)π
= 2π − =
6 (n + 1)(n + 2) 3(n + 1)(n + 2)
Z
2. [5 MARKS] Showing all your work, evaluate the integral (a − t)(b + t2 ) dt.

Solution:

(a) Expand the product in the integrand:


Z Z  
2
(a − t)(b + t ) dt = ab − bt + at2 − t3 dt .

(b) Integrate term by term:


Z   b a 1
ab − bt + at2 − t3 dt = ab · t − · t2 + · t3 − · t4 + C .
2 3 4

3. [10 MARKS] Showing all your work, determine a number b such that the line x = b
divides into two regions of equal area the region bounded by the curves x = ay2 and
x = k.
Solution: The solution is analogous (under the exchange x ↔ y) to that given for Prob-
lem 1 of the Tuesday quiz.

4. [5
Z MARKS] Showing all your work, use a substitution to evaluate the indefinite integral
ex
dx, where a is a non-zero real number.
ex + a
Solution:

(a) Try the substitution u = e x + a, so du = e x dx.


(b) Z Z
ex du
x
dx = = ln |u| + C = ln |e x + a| + C .
e +a u
(If the constant a is positive, then the absolute signs are not required.)
Information for Students in MATH 141 2009 01 3102

Tuesday version

1. [10 MARKS] Showing all your work, find a number b such that the line y = b divides
the region bounded by the curves y = ax2 and y = k into two regions with equal area,
where a, k are given positive constants.
Solution:

(a) Determine the range of values for integration by finding the r intersections
 of the
 k 
bounding curves: solving the equations yields the points ∓ , k .
a 
(b) Determine the portion of the full area which is below the line y = b. We begin by
repeating
 r the calculation of the preceding part: the corner points have coordinates
 
∓ b , b . The area is
a

Z √ ba " # √ ba r
2 ax3 4 b
√ (b − ax ) dx = 2 bx − = b .
− ba 3 0 3 a

(c) As a special case of the foregoing, or r by a separate calculation, we can conclude


4 k
that the area of the entire region is k .
3 a
(d) The condition of the problem is that
r r
4 b 1 4 k
b = · k
3 a 2 3 a
which is equivalent to 4b3 = k3 , and implies that the line should be placed where
2
b = 2− 3 k.

2. [10 MARKS] The region bounded by the curves y = 5 and y = x2 − ax + b is rotated


about the axis x = −1. Showing all your work, find the volume of the resulting solid.
Solution: Because there are constraints on the constants, I will work just one variant,
with a = 3, b = 7.

Using the method of cylindrical shells: (a) To find the extremes of integration, we
solve the equations y = 5 and y = x2 − 3x + 7, obtaining (x, y) = (1, 5), (2, 5).
(b) The height of a vertical element of area which generates a cylindrical shell is,
at horizontal position x, 5 − (x2 − 3x + 7) = −x2 + 3x − 2.
(c) The distance of that vertical element of area from the axis of revolution is 1+ x.
Information for Students in MATH 141 2009 01 3103

(d) The volume is given by the integral


Z 2
2π (1 + x)(−x2 + 3x − 2) dx
1

(e) Evaluating the integral:


Z 2
2π (1 + x)(−x2 + 3x − 2) dx
1
Z 2 
= 2π −x3 + 2x2 + x − 2 dx
1
" #2
1 4 2 3 1 2
= 2π − x + x + x − 2x
4 3 2 1
!
16 1 2 1
= 2π −4 + +2−4+ − − +2
3 4 3 2

=
6
Using the method of “washers” (a) To find the lowest point on the parabola, we solve
x2 − 3x + 7 ≥ 0. This can be done by completing the square, ! or by using the
3 19
calculus to find the local minimum. We find it to be , .
2 4
(b) The horizontal element generating the “washer” at height y extends between
the solutions in x to the equation y = x2 − 3x + 7; these are
p
3 ± 4y − 19
x= .
2
(c) The volume of the “washer” at height y is, therefore,
 p 2  p 2 
 3 + 4y − 19 
 
 3 − 4y − 19  
π 1 +  − 1 +
    ∆y

2 2
p
= 5π 4y − 19 ∆y
Z 5p
(d) The volume is given by the integral 5π 4y − 19 dy.
19
4
(e) Evaluation of the integral:
Z 5p " #5
2 1 3 5π
5π 4y − 19 dy = 5π · · (4y − 19) 2 = .
19
4
3 4 19 6
4
Information for Students in MATH 141 2009 01 3104

3. [5
Z MARKS] Showing all your work, use a substitution to evaluate the indefinite integral
cosn x sin x dx, where n is a fixed, positive integer.

Solution:

(a) Use new variable u, where du = − sin x dx; one solution is u = cos x.
(b)
Z Z
n
cos x sin x dx = − un du
un+1
= − +C
n+1
1
= − cosn+1 x + C
n+1
Z !
1
4. [5 MARKS] Showing all your work, evaluate the integral xb + a + dx, (where
x2 + 1
a and b are given positive integers).
Solution:
Z
xb+1
(a) xb dx = + C1 ,
b+1
Z
(b) a dx = ax + C2
Z
1
(c) dx = arctan x + C3
x2 + 1
Z !
b 1 xb+1
(d) x +a+ 2 dx = + ax + arctan x + C.
x +1 b+1

Wednesday version

1. [10 MARKS] Showing all your work, find the volume of the solid obtained by rotating
about the line x = −1 the region bounded by y = xn and x = yn , where n is a given
positive integer.
Solution:

Case I: n is even
Using the method of “washers”: (a) Find the intersections of the curves bound-
ing the region. Solving the 2 equations yields the points (x, y) = (0, 0), (1, 1).
Information for Students in MATH 141 2009 01 3105

(b) Find the inner and outer dimensions of the washer. Since the axis of revo-
lution is a vertical line, the element of area being rotated is horizontal. For
1
arbitrary y the farther endpoint on the element is (y n , y); the nearer end-
point is (yn , y). The distances of these points from the axis are, respectively

1 + n y and 1 + yn .
(c) The volume of the “washer” is, therefore,
 √ 
π −(1 + yn )2 + (1 + n y)2 ∆y .
(d) Correctly evaluate the integral:
Z 1
√ 
π −(1 + y)2 + (1 + n y)2 dy
0
Z 1 
1 2
= π 2y n + y n − 2yn − y2n dy
0
" #1
2n n+1 n n+2 2 n+1 1 2n+1
= π yn + yn − y − y
n+1 n+2 n+1 2n + 1 0
2
2(n − 1)(3n + 7n + 3)π
=
(n + 1)(n + 2)(2n + 1)
Using the method of cylindrical shells: (a) Find the intersections of the curves
bounding the region. Solving the 2 equations yields the points (x, y) =
(0, 0), (1, 1).
(b) Since the axis of revolution is a vertical line, the element of area being
rotated is also vertical. For arbitrary x the top endpoint on the element is
1
(x, x n ); the lower endpoint is (x, xn ). The length of the element is, there-
1
fore, x n − xn ; the distance of the element from the axis of symmetry is
1 + x.
(c) The volume of the cylindrical shell element of volume is, therefore,
 1 
2π(1 + x) · x n − xn .
(d) Correctly evaluate the integral:
Z 1
1
2π (1 + x)(x n − xn ) dx
0
Z 1 
1 n+1
= 2π x n − xn + x n − xn+1 dx
0
!
n 1 n 1
= 2π − + −
n + 1 n + 1 2n + 1 n + 2
2(n − 1)(3n2 + 7n + 3)π
=
(n + 1)(n + 2)(2n + 1)
Information for Students in MATH 141 2009 01 3106

Case II: n is odd


Using the method of “washers”: (a) Find the intersections of the curves bound-
ing the region. Solving the 2 equations yields the points (x, y) = (0, 0), (±1, ±1).
Here there is an issue of interpretation. The textbook usually permits the
word region to apply to one that may have more than one component;
some authors would not wish to apply the term in such a situation. I will
follow the textbook, and permit a region here to have two components.
(b) Find the inner and outer dimensions of the washer. Since the axis of rev-
olution is a vertical line, the element of area being rotated is horizontal.
But there are two kinds of elements, depending on whether y is positive
or negative. For arbitrary, positive y the farther endpoint on the element is
1
(y n , y); the nearer endpoint is (yn , y). The distances of these points from

the axis are, respectively 1 + n y and 1 + yn . For arbitrary, negative y the
1
nearer endpoint on the element is (y n , y); the farther endpoint is (yn , y).

The distances of these points from the axis are, respectively 1 + n y and
1 + yn (both of which are less than 1).
(c) The volume of the “washer” is, therefore,

π −(1 + yn )2 + (1 + n y)2 ∆y .
(d) Correctly evaluate the integral:
Z 1
π −(1 + y)2 + (1 + √n y)2 dy
−1
Z 1 
1 2
= π 2y n + y n − 2yn − y2n dy
0
Z 0 
1 2
+π −2y n − y n + 2yn + y2n dy
−1
" #1
2n n+1 n n+2 2 n+1 1 2n+1
= π y +n y −
n y − y
n+1 n+2 n+1 2n + 1 0
" #0
2n n+1 n n+2 2 n+1 1 2n+1
+π − yn − yn + y + y
n+1 n+2 n+1 2n + 1 −1
2(n − 1)(3n2 + 7n + 3)π 2(n − 1)(n2 + 3n + 1)π
= +
(n + 1)(n + 2)(2n + 1) (n + 1)(n + 2)(2n + 1)
4(n − 1)π
= .
n+1
Using the method of cylindrical shells: (a) Find the intersections of the curves
bounding the region. Solving the 2 equations yields the points (x, y) =
(0, 0), (±1, ±1).
Information for Students in MATH 141 2009 01 3107

(b) Since the axis of revolution is a vertical line, the element of area being
rotated is also vertical. For arbitrary, positive x the top endpoint on the el-
1
ement is (x, x n ); the lower endpoint is (x, xn ); for arbitrary, negative x the
1
bottom endpoint on the element is (x, x n ); the upper endpoint is (x, xn ).
1
The length of the element is, therefore, x n − xn ; the distance of the ele-
ment from the axis of rotation is 1 + x.
(c) The volume of the cylindrical shell element of volume is, therefore,
1
2π(1 + x) · x n − xn .

(d) Correctly evaluate the integral:


Z 1 1
2π (1 + x) x n − xn dx
−1
Z 1
1 n n+1
n+1
= 2π x − x + x − x dx
n n

−1
!
n 1 n 1
= 2π − + −
n + 1 n + 1 2n + 1 n + 2
!
n 1 n 1
+2π − − +
n + 1 n + 1 2n + 1 n + 2
4(n − 1)π
=
n+1
Z
sin 2t
2. [5 MARKS] Showing all your work, evaluate the indefinite integral dt.
cos t
Solution:
(a) Apply a “double angle” formula:
Z Z Z
sin 2t 2 sin t cos t
dt = dt = 2 sin t dt .
cos t cos t
(b) Complete the integration:
Z
2 sin t dt = −2 cos t + C .

3. [10 MARKS] Showing all your work, find the area of the region bounded by the parabola
y = x2 , the tangent line to this parabola at (a, a2 ), and the x-axis, (where a is a given real
number).
Solution: This area can be computed by integrating either with respect to y or with
respect to x.
Information for Students in MATH 141 2009 01 3108

Integrating with respect to y: (a) Since y0 = 2x, the tangent line through (a, a2 ) has
equation
y − a2 = 2a(x − a) ⇔ y = 2ax − a2 .
(b) To integrate with respect to y we need to express the equations of the parabola
and the line in the form
x = function of y .

The branch of the parabola to the right of the y-axis is x = y. The line has
y a
equation x = + .
2a 2 !
y + a2 √
(c) The area of the horizontal element of area at height y is − y ∆y.
2a
(d) The area is the value of the integral
Z a2 !
y + a2 √
− y dy .
0 2a
(e) Integration yields
" 2 #a2 !
y ay 2 32 1 1 2 3 1 3
+ − y = + − a = a .
4a 2 3 0 4 2 3 12

Integrating with respect to x: (a) As above, the tangent line is y = 2ax − a2 . Its
a
intercept with the x-axis is at x = .
2
a  
(b) The area of the vertical element of area at horizontal position x ≤ is x2 − 0 ∆x.
2  
(c) The area of the vertical element of area at horizontal position x ≥ 2a is x2 − (2ax − a2 ) dx =
(x − a)2 ∆x.
(d) The area of the region is the sum
Z a2 Z a
2
x dx + (x − a)2 dx .
a
0 2

(e) Integration yields


" # a2 " #a
x3 (x − a)3 a3
+ − = .
3 0 3 a 12
2
Z
4. [5 MARKS] Showing all your work, evaluate the indefinite integral seca x tan x dx,
where a is a constant, positive integer.
Solution:
Information for Students in MATH 141 2009 01 3109

(a) Try the substitution given by du = sec x·tan x dx, of which one solution is u = sec x.
(b) Z Z
a ua seca x
sec x tan x dx = ua−1 du = +C = +C.
a a
Some students may have integrated by sight.

Thursday version

1. [10 MARKS] Showing all your work, use the method of cylindrical shells to find the
volume
√ generated by rotating about the axis x = b the region bounded
√ by the curves
y = x − 1, y = 0, x = a, where a, b are fixed real constants. b ≥ a − 1 .
Solution:

(a) Solve
√ equations to determine the limits of integration.
 √  Solving x = a with y =
x − 1 yields the single point of intersection a, a − 1 .
(b) The horizontal element of area at height y which generates the cylindrical shell has
left endpoint (1 + y2 , y) and right endpoint (a, y), so its length is a − (1 + y2 ).
(c) The distance of the horizontal element of area which generates the shell from the
axis of rotation is b − y.
(d) Set up the integral for the volume by cylindrical shells:
Z √
a−1  
(b − y) a − (1 + y2 ) dy .
0

(e) Evaluate the integral


Z √
a−1  
(b − y) a − (1 + y2 ) dy
0
Z √
a−1  
= b(a − 1) − (a − 1)y − by2 + y3 dy
0
" # √a−1
a−1 2 b 3 1 4
= b(a − 1)y − y − y + y
2 3 4 0
3 a − 1 b 3 1
= b(a − 1) 2 − (a − 1) − (a − 1) 2 + (a − 1)2
2 !3 4
3 2 1 √
= (a − 1) 2 ·b− a−1 .
3 4
Information for Students in MATH 141 2009 01 3110

R b
2. [5 MARKS] Showing all your work, use a substitution to evaluate the integral (1+ax)3
dx ,
where a, b are non-zero constants.
Solution:

(a) A substitution which suggests itself is u = 1 + ax, implying that du = a dx, so


dx = 1a du.
(b) Z Z
b b du b b
3
dx = 3
= − u−2 + C = − +C.
(1 + ax) a u 2a 2a(1 + ax)2
3. [5
Z MARKS] Showing all your work, use a substitution to evaluate the indefinite integral
 
t2 cos a − t3 dt, (where a is a given real number).

Solution:

(a) Try the substitution u = t3 .


(b) du = 3t2 dt ⇒ t2 dt = 13 du.
(c)
Z   Z
2 3 1
t cos a − t dt = cos(a − u) du
3
1
= − sin(a − u) + C
3
1
= − sin(a − t3 ) + C.
3
(Some students may wish to employ a second substitution v = a − u. Alternatively,
a better substitution for the problem would have been to take u = a − t3 .)

4. [10 MARKS] Showing all your work, determine the area of the region bounded by the
parabola x = y2 , the tangent line to this parabola at (a2 , a), and the y-axis, where a is a
fixed, positive real number.
Solution: The solution is analogous (under the exchange x ↔ y) to that given for Prob-
lem 3 of the Wednesday quiz.

D.2.3 Draft Solutions to Quiz Q3


Release Date: Mounted on the Web on 05 April, 2008
These draft solutions could contain errors, and they must be subject to correction. Caveat
lector!
Information for Students in MATH 141 2009 01 3111

There were four different types of quizzes, for the days when the tutorials are scheduled. Each
type of quiz was generated in multiple varieties for each of the tutorial sections. The order of
the problems in the varieties was also randomly assigned. All of the quizzes had a heading that
included the instructions
• Time = 30 minutes
• No calculators!
• Show all your work: marks are not given for answers alone.
• Enclose this question sheet in your folded answer sheet.
In the following I will either provide a generic solution for all varieties, or a solution to one
typical variety.

Monday version
Z
1. [5 MARKS] Showing all your work, evaluate the integral x5 ln(20x) dx.
Solution:
dx x6
(a) I will integrate by parts, setting u = ln(20x), dv = x5 dx. Then du = ,v= .
x 6
(b)
Z ! Z
5 x6 1
x ln(20x) dx = (ln(20x)) · − x5 dx
6 6
x6 ln(20x) x6
= − +C.
6 36
Z
1 − cot2 x
2. [5 MARKS] Showing all your work, evaluate the integral dx.
csc2 x
Solution:
Z Z
1 − cot2 x sin2 x − cos2 x
dx = 2
dx
csc2 x Z  sin x · csc2x

= sin2 x − cos2 x dx
Z
= (− cos 2x) dx
1
= − sin 2x + C .
2
Information for Students in MATH 141 2009 01 3112

3. [10 MARKS] Use a substitution to transform this integral into the integral of a rational
function; then integrate, and express your answer in terms of x:
Z
1
dx
e−3x − e−x

Solution:

(a) I would try the substitution u = e−x , so du = −e−x dx = −u dx.


(b) Then
Z Z
1 du
dx = du
e − e−x
−3x u2 (1 − u2 )
Z  1 1 
 1 
=  2 + 2
+ 2  du
u 1+u 1−u
1 1 1
= − + ln |1 + u| − ln |1 − u| + C
u 2 2
1 1 1 − u
= − − ln +C
u 2 1 + u
r
x
1 − e−x
= −e − ln +C
1 + e−x

There are other, equivalent ways in which this last class of antiderivatives can be ex-
pressed. (For example, if we define K = eC , so C = ln K, we can bring the logarithm
terms together.)
Z
x2
4. [10 MARKS] Showing all your work, evaluate the integral  3 dx.
4 − x2 2
Solution:

(a) Use a trigonometric substitution, e.g., x = 2 cos θ, i.e., θ = arccos 2x . Then dx =


−2 sin θ dθ.
(b)
Z Z
x2 cos2 θ sin θ
 32 dx = − sin3 θ

4 − x2
Z
= − cot2 θ dθ
Z  
= − csc2 θ − 1 dθ
Information for Students in MATH 141 2009 01 3113

= cot θ + θ + C
cos θ
= +θ+C
sin θ
cos θ x
= √ + arccos + C
1 − cos2 θ 2
x
x
= q 2 + arccos + C
2 2
1 − x4
x x
= √ + arccos + C .
4 − x2 2

Tuesday version
Z
1. [10 MARKS] Showing all your work, evaluate the integral e5x cos(2x) dx .

Solution:

(a) Use integration by parts. In this case the factors e5x and cos 2x are rendered neither
“more complicated” nor “simpler” under either integration or differentiation. Two
applications of integration by parts, with appropriate choices of functions, will
yield an equation that can be solved for the value of the indefinite integral. For the
first application take, for example, u = e5x and dv = cos 2x. Then du = 5e5x dx,
and v = 21 sin 2x.
(b) Z Z
5x 15x 5
e cos(2x) dx = e · sin 2x − e5x sin 2x dx . (106)
2 2
R
(c) Now we apply integration by parts to evaluate e5x sin 2x dx, taking U = e5x and
dV = sin 2x. Then dU = 5e5x dx, and V = − 21 cos 2x.
(d) Z ! Z
5x 5x 1 5
e sin(2x) dx = e · − cos 2x + e5x cos 2x dx . (107)
2 2
(e) Combining equations (106), (107) yields
Z Z !
5x 5x 1 5 5x 1 5 5x
e cos(2x) dx = e · sin 2x − −e · · cos 2x + e sin 2x dx
2 2 2 2
! Z
5x 1 5 25
= e · · sin 2x + cos 2x − e5x cos(2x) dx .
2 4 4
Information for Students in MATH 141 2009 01 3114

(f) Solving the last equation yields


Z
1
e5x cos(2x) dx = e5x · (2 sin 2x + 5 cos 2x) + C .
29

2. [10 MARKS] Showing all your work, use a substitution to change this integral into the
integral of a rational function; then integrate and express your solution in terms of x:
Z
1 + 7e x
dx .
1 − ex

Solution:

(a) Let u = e x , so du = e x dx.


Z Z
1 + 7e x 1 + 7u du
(b) Then x
dx = · and we proceed to expand the integrand into
1−e 1−u u
a sum of partial fractions.
1 + 7u A B
(c) Assuming that = + , and multiplying both sides by u(1 − u), we
u(1 − u) u 1−u
obtain the identity in u 1 + 7u = A(1 − u) + Bu. Setting u = 0 and u = 1 yields the
equations 1 = A and 8 = B. We can now continue integration.
(d)
Z Z !
1 + 7u 1 8
du = + du
1−u u 1−u
= ln |u| − 8 ln |1 − u| + C
= ln e x − 8 ln |1 − e x | + C
= x − 8 ln |1 − e x | + C

which solution can be checked by differentiation.


Z
dx
3. [10 MARKS] Showing all your work, evaluate the integral  dx.
x2 + 8x + 17 2
Solution:

(a) Completing the square of the polynomial in the denominator, we obtain


!2  !2 
2 8  8  2
x + 8x + 17 = x + + 17 −  = (x + 4) + 1 .
2 2

Accordingly, we can simplify the integral by taking u = x + 4, du = dx.


Information for Students in MATH 141 2009 01 3115

(b) The preceding substitution is not sufficient, however. All we obtain is


Z Z
dx du
 2
dx =  .
x2 + 8x + 17 u2 + 1 2
We can simplify this further by taking u = tan θ, i.e., by taking θ = arctan u, so
du
dθ = .
1 + u2
(c)
Z Z
du sec2 θ
2 = dθ
1 + u2 sec4 θ
Z
= cos2 θ dθ
Z
1 + cos 2θ θ sin 2θ
= dθ = + +C
2 2 4
1 u
= arctan u + +C
2 2(1 + u2 )
!
1 x+4
= arctan(x + 4) + 2 +C
2 x + 8x + 17

which can be verified by differentiation. You should always verify this type of
integration by differentiation, in order to locate silly algebra mistakes (or worse).

Wednesday version
Z
1. [5 MARKS] Showing all your work, evaluate the indefinite integral x cos(18x) dx.

Solution:

(a) This can be solved using integration by parts. Define u = x, dv = cos(18x) dx, so
sin(18x)
du = dx and v = .
18
(b)
Z Z
sin(18x) sin(18x)
x cos(18x) dx = x · −
18 18
x sin(18x) cos(18x)
= + +C,
18 182
which can be verified by differentiation.
Information for Students in MATH 141 2009 01 3116

Z
1 − sin x
2. [5 MARKS] Showing all your work, evaluate the integral dx.
cos x
Solution:
Z Z
1 − sin x
dx = (sec x − tan x) dx
cos x
= ln | sec x + tan x| + ln | cos x| + C
= ln |(sec x + tan x) · (cos x)| + C
= ln |1 + sin x| + C = ln(1 + sin x) + C .

Note that the absolute signs are not needed, since 1 + sin x cannot be negative.
π
Z3
ln tan x
3. [10 MARKS] Showing all your work, evaluate the integral dx.
(sin x) · (cos x)
π
6

Solution:

(a) In view of the complicated nature of the integrand, one would be advised to seek
a substitution that could render it more amenable. But the integrand involves both
sines and cosines. However, note that
tan x tan x
(sin x)(cos x) = (tan x)(cos2 x) = = .
sec2 x tan2 x + 1
π √ π
Taking u = tan x, we have du = sec2 x dx. When x = , u = 3; when x = ,
3 6
1
u= √ .
3
(b)
π
Z3
ln tan x
dx
(sin x) · (cos x)
π
6

Z3
ln u du
= u
u2 +1
u2 + 1
√1
3

Z3
ln u
= du
u
√1
3
Information for Students in MATH 141 2009 01 3117

(c) Now the integral looks as though it could be simplified by a substitution v = ln u,


du √
so dv = . When u = 3, v = ln23 ; when u = √13 , v = − ln23 .
u
(d)

Z3 Z ln 2
ln u 2
du = v dv
u − ln23
√1
3
" # ln23
v2
= = 0.
2 − ln23

Z √
4. [10 MARKS] Showing all your work, evaluate the integral et 49 − e2t dt.

Solution:

(a) Clearly a substitution of the form u = et is indicated, in order to simplify the


integrand. We find that du = et dt.
Z √ Z √
t
(b) We obtain e 49 − e2t dt = 49 − u2 du. Now a trigonometric substitution
is indicated. Take u = 7 sin θ — more precisely, θ = arcsin u7 (the inverse cosine
could also have been used), so du = 7 cos θ dθ:
Z √ Z
49 − u du = 49 cos2 θ dθ.
2

(c)
Z Z
2 49
49 cos θ dθ = (1 + cos 2θ) dθ
2
49
= (θ + sin θ · cos θ) + C
2
49 u u√
= arcsin + 49 − u2 + C
2 7 2 √
49 et et 49 − e2t
= arcsin + +C.
2 7 2

Thursday version
Z
1. [5 MARKS] Showing all your work, evaluate the integral sin3 9x dx.
Information for Students in MATH 141 2009 01 3118

Solution: The integrand is an odd power of the sine function. I will substitute u = cos 9x,
so du = −9 sin 9x dx.
Z Z   1!
3 2
sin 9x dx = 1−u du
9
u u3
= − + +C
9 27
cos x cos3 x
= − + +C.
9 27
π
Z2
2. [5 MARKS] Showing all your work, evaluate the integral cot2 x dx. Your answer
π
4
should be simplified as much as possible; the instructors are aware that you do not have
the use of a calculator.
Solution:
d
(a) Recall that cot2 x = csc2 x − 1, and that cot x = − csc2 x.
dx
(b)
π π
Z2 Z2  
cot2 x dx = csc2 x − 1 dx
π π
4 4
π
= [− cot x − x] π4 .
2

π π π
(c) = − + 1 + = 1 − .
2 4 4
Z √
9 − x2
3. [10 MARKS] Showing all your work, evaluate the integral dx.
x
Solution:

(a) To simplify the square root, substitute x = 3 cos θ, i.e., θ = arccos 3x . Then dx =
−3 sin θ dθ.
(b)
Z √ Z
9 − x2 sin2 θ
dx = −3 dθ
x cos θ
Information for Students in MATH 141 2009 01 3119

Z
1 − cos2 θ
= −3 dθ
Z cos θ
= −3 (sec θ − cos θ) dθ
= −3 ln | sec θ + tan θ| + 3 sin θ + C

(c)
q
1 + 1 − x2 r
−3 ln | sec θ + tan θ| + 3 sin θ + C = −3 ln
9 + 3 1 − x2 + C
x 9
3

3 + 9 − x2 √
= −3 ln + 9 − x2 + C
x

4. [10 MARKS] Showing all your work, determine whether the following integral is con-
vergent or divergent. Evaluate it if it is convergent; in such a case you are expected to
simplify your answer as much as is consistent with not having the use of a calculator:
Z 1 !
ln 7x
6 √ dx
0 x

Solution:
Z
ln 7x
(a) Let’s look first at the associated indefinite integral, √ dx. The integrand
x
can be expressed as a product, in which one of the factors “simplifies” upon dif-
ferentiation, while the other does not become significantly “more difficult” upon
1
integration. So we will integrate√by parts, taking u = ln 7x, and dv = x− 2 dx. Then
du = 1x , and we may take v = 2 x.
(b)
Z Z
ln 7x √ dx
√ dx = (ln 7x)(2 x) − 2 √
x x
√ √
= (ln 7x)(2 x) − 4 x + C .

(c) The integrand is not defined at x = 0 in the given interval of integration. By the
definition of an improper integral, we have
Z 1 Z 1
ln 7x ln 7x
√ dx = lim+ √ dx
0 x a→0 a x
Information for Students in MATH 141 2009 01 3120

h √ √ i1
= lim+ (ln 7x)(2 x) − 4 x
a→0 a
h √ i
= lim+ (2 ln 7 − 4) − a · (2 ln(7a) − 4)
a→0

= 2 ln 7 − 4 − lim+ (ln 7a)(2 a)
a→0

(d) The limit can be expressed as that of a ratio where numerator and denominator both
become infinite. Thus l’Hospital’s Rule may be used:
√ ln 7a
lim+ (ln 7a)(2 a) = 2 lim+ 1
a→0 a→0 a− 2
1
a
= 2 lim+ 3
a→0 − 21 a− 2

= 2 lim+ (−2 a) = 0.
a→0

Thus the original integral is convergent, and its value is 6(2 ln 7 − 4).

Problems not used


1. [10 MARKS] Make a substitution to express the integrand as a rational function, and
then evaluate the integral. Z 4 √
x
dx .
1 x − 16

Solution:

(a) Start by substituting u = x, so x = u2 , dx = 2u du. When x = 1, u = 1; when
x = 4, u = 2.
(b)
Z 4 √ Z 2
x 2u2
dx = du
1 x − 16 1 u2 − 36
Z 2 !
72
= 2+ 2 du
1 u − 36
Z 2 !
6 6
= 2+ − du
1 u−6 u+6
h i2
= u2 + 6 ln |u − 6| − 6 ln |u + 6|
1
7
= 3 + 6 ln .
10
Information for Students in MATH 141 2009 01 3121

D.2.4 Draft Solutions to Quiz Q4


Release Date: Mounted on the Web on Wednesday, April 9th, 2008
These are draft solutions that were prepared when the quizzes were being designed. It was in-
tended that Teaching Assistants would consult these draft solutions when they graded their stu-
dents’ quizzes, and would report any errors or omissions. As the Teaching Assistants may be-
lieve that they are inhibited from communicating with the instructor who manages this course,
it is not clear that the solutions have been thoroughly checked. The solutions are being released
with the cautionary warning, Caveat lector! — Let the reader beware! Use them at your own
risk.
There were four different types of quizzes, for the days when the tutorials are scheduled.
Each type of quiz was generated in multiple varieties for each of the tutorial sections. The
order of the problems in the varieties was also randomly assigned. All of the quizzes had a
heading that included the instructions.
• Time = 45 minutes
• No calculators!
• Show all your work: marks are not given for answers alone.
• Enclose this question sheet in your folded answer sheet.
In the following I will either provide a generic solution for all varieties, or a solution to one
typical variety.

Monday version
x5 1
1. [10 MARKS] Showing all your work, find the length of the curve y = + 3 (1 ≤
30 2x
x ≤ 2). Simplify your answer as much as possible; the instructors are aware that you do
not have the use of a calculator.
Solution:
dy x4 3
= − 4
s dx 6 2x
!2 r
dy x8 9 1
⇒ 1+ = 1+ + 8−
dx 36 4x 2
r s
8
!2
x 9 1 x4 3
= + + = +
36 4x8 2 6 2x4
4
x 3
= + 4 .
6 2x
Information for Students in MATH 141 2009 01 3122

The absolute signs may be dropped, since the given square root is a sum of positive
multiples of even powers, and must be non-negative. The length is
Z 2 ! " 5 #2
x4 3 x 1
+ dx = −
1 6 2x4 30 2x3 1
353
= .
240

2. [10 MARKS] Showing all your work, find the area of the surface obtained by rotating
1 2 3
x = y + 2 2 (7 ≤ y ≤ 10) about the x-axis. Simplify your answer as much as
2
possible; the instructors are aware that you do not have the use of a calculator.
Solution:
dx 1 3  2 1 p
= · · y + 2 2 · 2y = y y2 + 2 .
dy 3 2
Thus s
!2
dx p
1+ = 1 + y2 (2 + y2 ) = y2 + 2 .
dy
The area of the surface is
Z 10   " #10
2 y2 y4
1 + y · (2πy) dy = 2π +
7 2 4 7
7701π
= .
2

3. [10 MARKS] Showing all your work, find the area enclosed by the curve (in polar
coordinates) r = 7 + 2 sin 6θ.
Solution: The curve surrounds the pole, and is periodic with period 2π. The area may be
expressed as an integral over an interval of length 2π; for example, as
Z Z
1 2π 1 2π
2
(7 + 2 sin 6θ) dθ = (49 + 28 sin 6θ + 4 sin2 6θ) dθ
2 0 2 0
Z
1 2π
= (49 + 28 sin 6θ + 2(1 − cos 12θ)) dθ
2 0
" #2π
1 28 2
= 49θ − cos 6θ + 2θ − sin 12θ
2 6 12 0
1
= (51)(2π) = 51π .
2
Information for Students in MATH 141 2009 01 3123

4. [10 MARKS] Showing all your work, find the exact length of the curve x = 6 + 3t2 ,
y = 2 + 2t3 (0 ≤ t ≤ 1).
Solution:
dx
= 6t
dt
dy
= 6t2
dt
dy
dy
= dt = t
dx dx
s dt
!2 √
dy
⇒ 1+ = 1 + t2
dx
s
Z 1 !2 !2
dx dy
Arc Length = +
0 dt dt
Z 1√
= 36t2 + 36t4 dt
0
Z 1 √
= 6 t 1 + t2 dt
0
1 2   3 1
2 2
  3 1
2 2
= 6· · 1+t =2 1+t
 23 3  √ 0 0

= 2 22 − 1 = 4 2 − 2 .

5. [10 MARKS] Showing all your work, sum a series in order to express the following
number as a ratio of integers: 0.35 = 0.35353535 . . ..
Solution:
!
35 1 1 1
0.35 = 1+ + + + ...
100 100 1002 1003
35
100 35
= 1
= .
1− 100
99

Tuesday version
1√
1. [10 MARKS] Showing all your work, find the length of the curve x = y · (y − 3)
3
(49 ≤ y ≤ 64). Simplify your answer as much as possible; the instructors are aware that
you do not have the use of a calculator.
Information for Students in MATH 141 2009 01 3124

Solution:
!
dx 1 3√ 3 1 1 1 √ 1
= · y − · y− 2 = y− √
dy 3 2 3 2 2 y
s s
!2 !
dx 1 1
⇒ 1+ = 1+ y+ −2
dy 4 y
s !
1 1
= y+ +2
4 y
s !2
1 √ 1
= y+ √
4 y

1 √ 1
= y + √ .
2 y

The absolute signs may be dropped, since the square root is non-negative. The length is
Z ! " #64 " #64
1 64 √ 1 1 2 32 √ 1 32 √
y + √ dy = ·y +2 y = ·y + y
2 49 y 2 3 49 3 49
172
= .
3

2. [10 MARKS] Showing all your work, find the area of the surface obtained by rotating
the curve x = 6 + 2y2 (0 ≤ y ≤ 3) about the x-axis.
Solution:
dx
= 4y
dy
s
!2
dx p
⇒ 1+ = 1 + 16y2
dy
Z 3p
⇒ Area = 1 + 16y2 · 2πy dy
0
π h i
3 3
= (1 + 16y2 ) 2
24 0
π  3

= (145) 2 − 1 .
24

3. [15 MARKS] Showing all your work, find the area of the region that lies inside the curve
r = 15 cos θ, and outside the curve r = 5 + 5 cos θ.
Information for Students in MATH 141 2009 01 3125

Solution: The first curve is a circle; the second is a cardioid whose axis of symmetry
is the initial ray. If we solve the equations we find that the curves intersect at θ =
± arccos 12 = ± π3 . They also intersect at the pole, which appears on the circle when
θ = π2 , etc., and on the cardioid when θ = π, etc. The region whose area we seek lies
between the two curves when − π3 ≤ θ ≤ π3 and r is positive. Integration shows the area
to be
Z π
1 3 
(15 cos θ)2 − (5 + 5 cos θ)2 dθ
2 − π3
Z π
25 3  
= 8 cos2 θ − 2 cos θ − 1 dθ
2 − π3
Z π
25 3
= (4(1 + cos 2θ) − 2 cos θ − 1) dθ
2 − π3
Z π
25 3
= (3 + 4 cos 2θ − 2 cos θ) dθ
2 − π3
π
= 25 [3θ + 2 sin 2θ − 2 sin θ]03
 √ √ 
= 25 π + 3 − 3 = 25π .

4. [15 MARKS] Showing all your work, find the length of the loop of the curve x = 18t −
6t3 , y = 18t2 .
Solution:

dx
= 18 − 18t2
dt
dy
= 36t
s dt
!2 !2 q  
dx dy 
+ = 182 1 − t2 2 + 362 t2 = 18 1 + t2 .
dt dt

We must determine where the curve crosses itself. The student was expected to show that
she knew how to find this crossing point systematically, not just by guessing or exam-
ining a rough graph. The crossing point can be found by solving for distinct parameter
values t1 , t2 the equations x = 18t1 − 8t13 = 18t2 − 8t23 , y = 18t12 = 18t2 . Collecting
terms and factorizing yields the system of equations
  
(t1 − t2 ) 18 − 6 t12 + t1 t2 + t22 = 0,
18 (t1 − t2 ) (t1 + t2 ) = 0 .
Information for Students in MATH 141 2009 01 3126

Since we are looking for a solution where t1 , t2 , we may divide by t1 − t2 , which cannot
equal 0, and obtain the system
 
3 − t12 + t1 t2 + t22 = 0 ,
t1 + t2 = 0 .
From the second equation we see that t2 = −t1 , and then the first equation yields 3 = t12 ,

so the solutions
√ are t1 = −t2 = ± 3; we may take
√ the loop as beginning with parameter
value − 3 and ending with parameter value + 3. The length of the arc will be
Z + √3  
2
√ 18 1 + t dt
− 3
Z √3  
= 2 18 1 + t2 dt
0
since the integrand is even and the interval is symmetric around 0
" # √3
13 √
= 36 t + t = 72 3 .
3 0

Wednesday version
1. [10 MARKS] Showing all your work, find the length of the curve y = ln sec x. Simplify
your answer as much as possible.
Solution:
dy 1
= · (sec x tan x) = tan x
s dx sec x
!2 p
dy
⇒ 1+ = 1 + tan2 x = | sec x| .
dx
In the following integral I will drop the absolute signs because the secant is positive over
the entire interval of integration; the length is
Z π4 Z π4
| sec x| dx = sec x dx
− π4 − π4
π
= [ln | sec x + tan x|]−4 π
√ √4
= ln( 2 + 1) − ln( 2 − 1)
√ √
2+1 ( 2 + 1)2
= ln √ = ln
2−1 2−1
 √  √
= ln ( 2 + 1)2 = 2 ln( 2 + 1) .
Information for Students in MATH 141 2009 01 3127

It would be sufficient for a student to obtain the different of logarithms above. The
subsequent steps simplify the argument, and would be useful if the user did not have the
use of a calculator.

2. [10 MARKS] The curve y = 3 x (1 ≤ y ≤ 3) is rotated about the y-axis. Showing all
your work, find the area of the resulting surface.
Solution: The data are given partly in terms of x and partly in terms of y, so some care is
needed. Since the limits are given in terms of y, I will integrate with respect to y; it will
be convenient to rewrite the equation of the curve as x = y3 .
dx
= 3y2
dy
s
!2
dx p
⇒ 1+ = 1 + 9y4
dy
Z 3p
Area = 1 + 9y4 · 2πy3 dy
1
"  3 #3
1 1 2 4 2
= · · 2π · 1 + 9y
9 4 3 1
π  3 3

= 730 2 − 10 2 .
27
3. [10 MARKS] Showing all your work, find the area of the region enclosed by the inner
loop of the curve r = 9 + 18 sin θ.
Solution: The function 9 + 18 sin θ is periodic with period 2π, so the entire curve is
traced out as θ passes through an interval of that length. If, for example, we consider the
interval 0 ≤ θ ≤ 2π, we find that the curve passes through the pole only at θ = 7π 6
and
11π
at θ = 6 . Between these values the smaller loop is traced out; the larger loop is traced
out, for example, for − π6 ≤ θ ≤ 7π 6
. We can find the area of the small loop by integrating
between the appropriate limits; the area is
Z 11π
1 6
(9 + 18 sin θ)2 dθ
2 7π6
Z 11π
81 6
= (1 + 4 sin θ + 4 sin2 θ) dθ
2 7π6
Z 11π
81 6
= (1 + 4 sin θ + 2(1 − cos 2θ)) dθ
2 7π6
Z 11π
81 6
= (3 + 4 sin θ − 2 cos 2θ) dθ
2 7π6
Information for Students in MATH 141 2009 01 3128

81 11π
= [3θ − 4 cos θ − sin 2θ] 7π6
2 6
! !
81 11π 11π 11π 81 7π 7π 7π
= − 4 cos − sin − − 4 cos − sin
2 2 6 3 2 2 6 3
 √   √ 
81  11π √ − 3  81  7π √ 3 
=  −2 3−  −  + 2 3 − 
2 2 2 2 2 2

81 √ 243 3
= (2π − 3 3) = 81π − .
2 2

4. [10 MARKS] Showing all your work, find the area of the surface obtained by rotating
the curve x = 3t − t3 , y = 3t2 (0 ≤ t ≤ 4) about the x-axis.
Solution:

dx
= 3 − 3t2
dt
dy
= 6t
s dt
!2 !2 q  
dx dy 
+ = 32 1 − t2 2 + 62 t2 = 3 1 + t2 .
dt dt

The area of the surface of revolution about the x-axis will be


Z 4     Z 4 
2 2
3 1 + t · 2π 3t dt = 18π t2 + t4 dt
0 0
" #4
13 15 6 × 64 × 53 20352
= 18π t + t = = .
3 5 0 5 5

X

5. [10 MARKS] Showing all your work, determine the value of c, if it is known that (5+
n=2
c)−n = 2.
Solution: We are told that the geometric series converges; this implies that its common
ratio is less than 1 in magnitude, i.e., that |5 + c| < 1, which implies that −1 < 5 + c <
1, equivalently, that −6 < c < −4. The sum of the geometric series on the left is
1
(5+c)2
1
· 1−1 1 = (5+c)(4+c) . Equating this to 2, we obtain c2 + 9c + 18 = 0, implying that
5+c
(c + 3)(c + 6) = 0, so c = −3, −6. Of these two values, −6 lies outside of the admissible
interval, and would yield a divergent series. Thus c can be equal only to −3.
Information for Students in MATH 141 2009 01 3129

Thursday version
1. [10 MARKS] Showing all your work, find the exact length of the polar curve r = 7e4θ
(0 ≤ θ ≤ 2π).
Solution:
dr
= 28e4θ
s dθ
!2 q
2
dr 
r + = 72 + 282 e8θ


= 7 17e4θ .

The length of the arc is then


√ 2π
√ Z 2π
7 17 
7 17 e4θ dθ = e4θ 
0 4 0

7 17 8π 
= e −1 .
4

2. [10 MARKS] The curve y = 4 − x2 (1 ≤ x ≤ 3) is rotated about the y-axis. Showing all
your work, find the area of the resulting surface.
Solution:
s
!2 √
dy dy
= −2x ⇒ 1 + = 1 + 4x2
dx dx
Z 3√
Area = 1 + 4x2 · 2πx dx
1
1 1 2   3 3
2 2
= 2π · · · · 1 + 4x
4 2 3 1
π  32 3

= 37 − 5 2 .
6

3. [10 MARKS] Showing all your work, find the area of the region enclosed by the outer
loop of the curve r = 9 + 18 sin θ: this region will include the entire inner loop.
Solution: The function 9 + 18 sin θ is periodic with period 2π, so the entire curve is
traced out as θ passes through an interval of that length. If, for example, we consider the
interval 0 ≤ θ ≤ 2π, we find that the curve passes through the pole only at θ = 7π 6
and
11π
at θ = 6 . Between these values the smaller loop is traced out; the larger loop is traced
Information for Students in MATH 141 2009 01 3130

out, for example, for − π6 ≤ θ ≤ 7π


6
. We can find the area of the large loop by integrating
between those limits, and the area will include the entire smaller loop. The area is
Z 11π
1 6
(9 + 18 sin θ)2 dθ
2 7π
6
Z 7π
81 6
= (1 + 4 sin θ + 4 sin2 θ) dθ
2 −π
6
Z 7π
81 6
= (1 + 4 sin θ + 2(1 − cos 2θ)) dθ
2 −π
6
Z 7π
81 6
= (3 + 4 sin θ − 2 cos 2θ) dθ
2 −π6
81 7π
= [3θ − 4 cos θ − sin 2θ]−6 π
2 6
!
81 7π 7π 7π 81  π π π
= − 4 cos − sin − − − 4 cos + sin
2 2 6 3 2 2 6 3
 √   √ 
81  7π √ 3  81  π √ 3 
=  + 2 3 −  − − − 2 3 + 
2 2 2 2 2 2

243 3
= 162π + .
2

4. [10 MARKS] Showing all your work, sum a series in order to express the following
number as a ratio of integers: 4.645 = 4.645454545 . . ..
Solution:
!
1 45 1 1 1
4.645 = 4.6 + 1+ + + + ...
10 100 100 1002 1003
45
1000
= 4.6 + 1
1 − 100
46 1 45 511
= + · = .
10 10 99 110

5. [10 MARKS] Showing all your work, find an equation for the tangent to the curve x =
cos θ + sin 7θ, y = sin θ + cos 2θ (−∞ < θ < +∞) at the point corresponding to θ = 0.
Solution: The slope of the tangent is
dx
= − sin θ + 7 cos 7θ

Information for Students in MATH 141 2009 01 3131

dy
= cos θ − 2 sin θ

dy
dy cos θ − 2 sin θ 1
= dθ = = when θ = 0 .
dx dx − sin θ + 7 cos 7θ 7

1 1
The line through (x(0), y(0)) = (1, 1) with slope 7
has equation y − 1 = (x − 1), i.e.,
7
x − 7y = −6.

Problems prepared but not used


p
1. The curve x = c2 − y2 (0 ≤ y ≤ 2c ) is rotated about the y-axis. (c is a fixed real number.)
Showing all your work, find the area of the resulting surface.
Solution:

dx 1 1 y
= · p · (−2y) = − p
dy 2 c2 − y2 c2 − y2
s s
!2
dx y2 |c|
1+ = 1+ 2 = p
dy c − y2 c2 − y2
Z 2c p
|c|
Area = p · 2π c2 − y2 dy
0 c2 − y2
Z 2c
= 2π|c| dy = πc2 .
0

2. Showing all your work, find the slope of the tangent line to the curve with equation in
12
polar coordinates r = , at the point corresponding to θ = π.
θ
Solution:

dr 12
= − 2
dθ θ
dr
dy sin θ · dθ + r · cos θ
= dr
dx cos θ · dθ − r · sin θ
− sin θ + θ cos θ
= .
− cos θ − θ sin θ
At the point θ = π this ratio is equal to −π.
Information for Students in MATH 141 2009 01 3132

π
3. Showing all your work, find the area of one of the regions bounded by the line θ = 2
and
the closed curve r = 8 + 6 sin θ.
Solution: (The actual wording of the problem referred to a figure which it is not conve-
nient to include in these notes.) The region can be interpreted as being swept out by a
radius vector from the pole moving between − π2 and + π2 . The area is this
Z π Z + π2  
1 +2 2
(8 + 6 sin θ) dθ = 32 + 48 sin θ + 18 sin2 θ dθ
2 − π2 − π2
Z + π2
= (32 + 48 sin θ + 9(1 − cos 2θ)) dθ
− π2
Z + π2
= (41 + 48 sin θ − 9 cos 2θ) dθ
− π2
" #+ π2
9
= 41θ − 48 cos θ − sin 2θ = 41π .
2 − π2

4. Showing all your work, find the area enclosed by the curve (in polar coordinates) r =
9 + cos 2θ.
Solution: The curve surrounds the pole, and is periodic with period 2π. The area may be
expressed as an integral over an interval of length 2π; for example, as
Z Z
1 2π 2 1 2π
(9 + cos 2θ) dθ = (81 + 18 cos 2θ + cos2 2θ) dθ
2 0 2 0
Z
1 2π 1 + cos 4θ
= (81 + 18 cos 2θ + ) dθ
2 0 2
" #2π
1 1 1
= 81θ + 9 sin 2θ + θ + sin 4θ
2 2 8 0
163 163π
= · 2π = .
4 2
5. Showing all your work, find the exact length of the polar curve r = 4θ2 (0 ≤ θ ≤ 2π).
Solution:
dr
= 8θ
s dθ
!2 √
2
dr
r + = 16θ4 + 64θ2


= 4|θ| 4 + θ2 .
Information for Students in MATH 141 2009 01 3133

Over the interval in question θ is positive, and the absolute signs may be dropped. The
length of the arc is
Z 2π √
4h i
3 2π
4θ 4 + θ s dθ = (4 + θ2 ) 2
0 3 0

32  3

= (1 + π2 ) 2 − 1 .
3

6. Showing all your work, find equations of the tangents to the curve x = 3t2 +4, y = 2t3 +3
that pass through the point (7, 5).
Solution: We might, in error think that we need first to determine the parameter value
associated with the given point. We would then solve the system of equations

3t2 + 4 = 7
2t3 + 3 = 5

to obtain t = +1. This would be an error. It happens that the given curve passes through
the point (7, 5), but that is fortuitous: we want the tangents to pass through the point, not
the curve! And we can’t find the points of contact of the tangents directly. So let’s first
determine the general tangent to the curve, at the point with parameter value t.
dx
= 6t
dt
dy
= 6t2
dt
dy
dy 6t2
= dt = = t,
dx dx 6t
dt
2 3
so the slope of the tangent at the point
 (3t +4, 2t + 3) on the curve is t; the equation of
that tangent is y − 2t3 + 3 = t x − 3t2 + 4 , or

y = tx − t3 − 4t + 3 . (108)

We now impose the condition that this line pass through the point (x, y) = (7, 5), i.e., that
its equation be satisfied by (x, y) = (7, 5), obtaining t3 − 3t + 2 = 0, whose left member
factorizes to (t − 1)2 (t + 2) = 0, so the points of contact of the tangents are t = 1 and
t = −2. The equations of the tangents through the given point are found by giving the
parameter t these two values in equation (108):

y= x−2 and y = −2x + 19 .


Information for Students in MATH 141 2009 01 3134

7. Showing all your work,


 use methods
 of polar coordinates to find the length of the polar

curve r = 15 sin θ 0 ≤ θ ≤ 15 .
Solution:

dr
= 15 cos θ
s dθ
!2
dr
r2 + = 15 .

The length of the arc is, therefore


Z 4π
15
15 dθ = 4π .
0
Information for Students in MATH 141 2009 01 3135

E Final Examinations from Previous Years


E.1 Final Examination in Mathematics 189-121B (1996/1997)
1. [4 MARKS] Find the derivative of the function F defined by
Z x4 √
F(x) = sin t dt .
x2

Z π
2. [4 MARKS] Evaluate f (x) dx , where
−π
2

( −π
cos x, ≤ x ≤ π3
f (x) = 3
2
π .
π
x + 1, 3
<x≤π
Z
3. [7 MARKS] Evaluate x sin3 x2 cos x2 dx .
Z
4. [7 MARKS] Evaluate (x5 + 4−x ) dx .

5. [10 MARKS] Calculate the area of the region bounded by the curves x = y2 and
x − y = 2.

6. [10 MARKS] The region bounded by f (x) = 4x − x2 and the x-axis, between
x = 1 and x = 4 , is rotated about the y-axis. Find the volume of the solid that is
generated.
Z
7. [6 MARKS] Evaluate x ln x dx .
Z
8. [6 MARKS] Evaluate sin2 x cos5 x dx .

9. [6 MARKS] Determine the partial fraction decomposition of the following ratio of poly-
nomials:
x5 + 2
.
x2 − 1
10. [4 MARKS] Determine whether or not the following sequence converges as n→∞.
If it does, find the limit: ( )
x 3n
1+ .
n
Information for Students in MATH 141 2009 01 3136

11. [4 MARKS] Determine the following limit, if it exists:



x
lim+ √ √ .
x→0 x + sin x

X

2
12. [6 MARKS] Determine whether the series ke−k converges or diverges.
k=2

13. [6 MARKS] Test the following series for

(a) absolute convergence,


(b) conditional convergence.

X

(−1)k
√ .
k=10 k(k + 1)

14. [10 MARKS] Find the area of the region that consists of all points that lie within the
circle r = 2 cos θ , but outside the circle r = 1 .

15. [10 MARKS] Determine the length of the curve

r = 5(1 − cos θ) , (0 ≤ θ ≤ 2π) .

E.2 Final Examination in Mathematics 189-141B (1997/1998)


1. [10 MARKS]

(a) Sketch the region bounded by the curves

y = x2 and y = 3 + 5x − x2 .

(b) Determine the area of the region.

2. [10 MARKS] The triangular region bounded by the lines


3 x
y = x, y= − , and y=0
2 2
is revolved around the line y = 0. Determine the volume of the solid of revolution which
is generated.
Information for Students in MATH 141 2009 01 3137

x2 √
3. [10 MARKS] Find the length of the curve y = − ln 4 x from x = 1 to
2
x = 2.

4. [5 MARKS] Determine, at x = 12 , the value of the function sin−1 x and the slope of its
graph.
x3 − 8
5. [5 MARKS] Evaluate lim .
x→2 x4 − 16
6. [5 MARKS] Showing all your work, evaluate lim x x .
x→0+
Z
2
7. [5 MARKS] Evaluate x3 e−x dx .
Z
x3 − 1
8. [10 MARKS] Evaluate dx .
x3 + x
Z
x3
9. [10 MARKS] Evaluate √ dx , where |x| < 1 .
1 − x2
10. [10 MARKS] Find the area of the region that lies within the limaçon r = 1 + 2 cos θ
and outside the circle r = 2 .

11. [5 MARKS]
Z x Showing all your work, obtain a second-degree Taylor polynomial for
f (x) = et(1−t) dt at x = 0 .
0

12. [5 MARKS] Showing all your work, determine whether the following infinite series
converges or diverges. If it converges, find its sum.
X

3n − 2n
n=0
4n

13. [5 MARKS] Showing all your work, determine whether or not the following series con-
verges:
X ∞ 1
2n
n=1
n2

14. [5 MARKS] Showing all your work, determine whether the following series converges:
X

1
n=1
n · 2n
Information for Students in MATH 141 2009 01 3138

E.3 Supplemental/Deferred Examination in Mathematics 189-141B (1997/1998)


1. [10 MARKS]

(a) Sketch the region bounded by the curves


8
y= and x + y = 4.
x+2
(b) Determine the area of the region.

2. [10 MARKS] The triangular region bounded by the lines


3 x
y = x, y= − , and y=0
2 2
is revolved around the line y = 0. Determine the volume of the solid of revolution which
is generated.

3. [10 MARKS] Find the area of the surface of revolution generated by revolving the curve
1 x 
y= e + e−x (0 ≤ x ≤ 1)
2
about the x-axis.

4. [5 MARKS] Determine, at x = 12 , the value of the function cos−1 x and the slope of its
graph.
x − 2 cos πx
5. [10 MARKS] Evaluate lim .
x→2 x2 − 4
! x4
1
6. [5 MARKS] Evaluate lim cos 2 .
x→∞ x
Z
e2x
7. [5 MARKS] Evaluate dx .
1 + e4x
Z
8. [5 MARKS] Evaluate x2 cos x dx .
Z
x3 − 1
9. [10 MARKS] Evaluate dx .
x3 + x
Z √
10. [10 MARKS] Evaluate a2 − u2 du , where |u| < a.
Information for Students in MATH 141 2009 01 3139

11. [10 MARKS] Find the area of the region that lies within the limaçon r = 1 + 2 cos θ
and outside the circle r = 2 .

12. [5 MARKS]
Z x Showing all your work, obtain a second-degree Taylor polynomial for
f (x) = e s(1−s) ds at x = 0 .
0

13. [5 MARKS] Showing all your work, determine whether the following infinite series
converges or diverges. If it converges, find its sum.
X

1 + 2n + 3n
n=0
5n

14. [5 MARKS] Showing all your work, determine whether or not the following series con-
verges.
X∞
ln n
n=1
n

15. [5 MARKS] Showing all your work, determine whether the following series convereges.
X

n2 + 1
n=1
en (n + 1)2

E.4 Final Examination in Mathematics 189-141B (1998/1999)


1. [8 MARKS] Find the area of the region bounded by the curves y2 = x and (y−1)2 = 5− x.

2. [8 MARKS] Find the volume of the solid of revolution generated by revolving about the
line x = 1 the region bounded by the curve (x − 1)2 = 5 − 4y and the line y = 1 .

3. [8 MARKS] Find the volume of the solid generated by revolving about the line x = 0
the region bounded by the curves

y = sin x
y = −2
x = 0
and x = 2π .

4. [8 MARKS]
√ Find the area of the surface obtained by revolving the curve y = x2 (0 ≤
x ≤ 2) about the y-axis.
Information for Students in MATH 141 2009 01 3140

Zx
3
5. Define the function F by F(x) = et dt .
0

(a) [4 MARKS] Showing all your work, explain clearly whether or not the following
inequalities are true.
3
e < F(e) < ee +1 .
d
(b) [4 MARKS] Determine the value of F(x3 ) at each of the
dx
following points:
i. at x = 0 .
ii. at x = 2 .

6. [4 MARKS] Showing all your work, evaluate


Z
sin3 πx dx .

7. [4 MARKS] Showing all your work, evaluate


Z
x2 e−x dx .

8. [4 MARKS] Showing all your work, evaluate


Z
x−1
dx .
x − x2 − 2x
3

9. [4 MARKS] Showing all your work, evaluate


Z 3
x + x2 + x − 1
dx .
x2 + 2x + 2

10. [8 MARKS] Find the area of the region inside the curve r = 3 sin θ and outside the curve
r = 2 − cos θ.

11. Showing all your work, determine whether each of the following integrals is convergent
or divergent:
Z∞
(a) [4 MARKS] sin x dx .
0
Information for Students in MATH 141 2009 01 3141

Z2
dx
(b) [4 MARKS] .
1 − x2
0

12. Showing all your work, determine whether each of the following sequences is convergent
or divergent.
 π
(a) [4 MARKS] n sin
 n
(b) [4 MARKS] (2 n + 1) e−n
13. Showing all your work, determine whether each of the following infinite series is con-
vergent or divergent:
X∞
1
(a) [4 MARKS] 3
.
n=1
4n
X∞ !
1 1
(b) [4 MARKS] + .
n=1
n n2

14. Showing all your work, determine whether each of the following series is convergent,
divergent, conditionally convergent and/or absolutely convergent.
X

n+2
(a) [4 MARKS] (−1)n .
n=1
n(n + 1)
X

cos n
(b) [4 MARKS] (−1)n .
n=1
n2

E.5 Supplemental/Deferred Examination in Mathematics 189-141B (1998/1999)


1. [8 MARKS] Find the area of the region bounded by the curves y2 = x and y = 6 − x.
2. [8 MARKS] Find the volume of the solid of revolution generated by revolving about the
line x = 0 the region bounded by the curve y = 4 − x2 and the lines x = 0 and
y=0 .
3. [8 MARKS] Find the volume of the solid generated by revolving about the line x = 0
the region bounded by the curves
y = sin x
y = 2
x = 0
and x = 2π .
Information for Students in MATH 141 2009 01 3142

4. [8 MARKS]
√ Find the area of the surface obtained by revolving the curve y = −x2 (0 ≤
x ≤ 2) about the y-axis.
Zx
5. Define the function F by F(x) = sin10 t dt .
0

(a) [4 MARKS] Showing all your work, explain clearly whether or not the following
inequalities are true.
0 < F(e) < e .
d
(b) [4 MARKS] Determine the value of F(x) at each of the
dx
following points:
i. at x = 0 .
π
ii. at x = .
2
6. [8 MARKS] Showing all your work, evaluate
Z
2
x5 e−x dx .

7. [4 MARKS] Showing all your work, evaluate


Z 3
x − x2 + x + 1
dx .
x2 − 2x + 2

8. [8 MARKS] Find the area of the region inside the curve r = 6 sin θ and outside the curve
r = 4 − 2 sin θ.

9. Showing all your work, determine whether each of the following integrals is convergent
or divergent:
Z∞
(a) [4 MARKS] cos x dx .
0

Z4
dx
(b) [4 MARKS] .
4 − x2
0

10. Showing all your work, determine whether each of the following sequences is convergent
or divergent.
Information for Students in MATH 141 2009 01 3143

 π
(a) [4 MARKS] n sin
n

(b) [4 MARKS] (2 n + 1) e−n

11. Showing all your work, determine whether each of the following infinite series is con-
vergent or divergent:
X∞
1
(a) [4 MARKS] 5
.
n=1
4n
X∞ !
1 1
(b) [4 MARKS] − 3 .
n=1
n n

E.6 Final Examination in Mathematics 189-141B (1999/2000)


1. [11 MARKS] Find the area of the region bounded by the curves x = y2 and x=
−y2 + 12y − 16 .

2. [11 MARKS] Let C denote the arc of the curve y = cosh x for −1 ≤ x ≤ 1 . Find
the volume of the solid of revolution generated by revolving about the line x = −2
e2 + 1
the region bounded by C and the line y = .
2e
3. (a) [5 MARKS] Showing all your work, evaluate
Z 9

2
3
6t − t2 dt .
2

(b) [6 MARKS] Showing all your work, evaluate


Z 3π

4
π
1 − sin u du .
4

4. (a) [7 MARKS] Showing all your work, determine a reduction Zformula which ex-
presses, for any integer n not less than 2, the value of xn sin 2x dx in
Z
terms of xn−2 sin 2x dx.
Z
(b) [4 MARKS] Use your reduction formula to determine the indefinite integral x2 sin 2x dx.
Information for Students in MATH 141 2009 01 3144

5. [11 MARKS] Showing all your work, evaluate


Z
8x2 − 21x + 6
dx .
(x − 2)2 (x + 2)

6. [11 MARKS] Find the area of the region inside the curve
r = 1 + cos θ and outside the curve r = 1 − cos θ .

7. [11 MARKS] Determine whether the following integral is convergent or divergent. If it


is convergent, find its value. Show all your work.
Z 3
1
4
dx
0
(x − 1) 5

8. [11 MARKS] Showing all your work, determine whether the following infinite series is
X

2
convergent or divergent: n! e−(n − 1) .
n=1

9. Showing all your work, determine whether each of the following series is convergent,
divergent, conditionally convergent and/or absolutely convergent.
X
∞ √ √ 
(a) [6 MARKS] (−1)n n+2− n .
n=1
X

n
(b) [6 MARKS] (−1)n .
n=1
ln n2

E.7 Supplemental/Deferred Examination in Mathematics 189-141B (1999/2000)


1. [11 MARKS] Determine the area of the region bounded by the curves y = x4 and
y = 2 − x2 .

2. [11 MARKS] Determine the volume of the solid generated by rotating the region bounded
by the curves y = 2x2 and y2 = 4x around the x-axis.

3. Evaluate the integrals:


Z
x7
(a) [5 MARKS] √ dx .
1 − x4
Z
x2
(b) [6 MARKS] √ dx .
4 − x2
Information for Students in MATH 141 2009 01 3145

Z π/2
4. [11 MARKS] Showing all your work, find e2x sin 3x dx .
0
Z
6x3 − 18x
5. [11 MARKS] Determine dx .
(x2 − 1)(x2 − 4)
6. [11 MARKS] Find the area of the region inside the curve r = 2 + 2 sin θ and outside
r = 2.
Z 1
ln x
7. [11 MARKS] Determine whether the following improper integral converges: dx .
0 x2
8. [11 MARKS] Showing all your work, determine whether the following infinite series
X∞
1
converges: √ .
n=1 15n3 + 3
9. Showing all your work, determine, for each of the following series, whether it is conver-
gent, divergent, conditionally convergent and/or absolutely convergent.
X

(−1)n ln n
(a) [6 MARKS] .
n=1
n
X

cos nπ
(b) [6 MARKS] .
n=1
n

E.8 Final Examination in Mathematics 189-141B (2000/2001)


1. Showing all your work, determine, for each of the following infinite series, whether or
not it converges.
X

n
(a) [3 MARKS] .
i=1
n3 + 1
X∞  n 
(b) [3 MARKS] ln .
n=1
3n + 1
X

(−1)n (3n + 1)4
(c) [6 MARKS] .
n=2
5n

2. [12 MARKS] Determine the volume of the solid of revolution generated by revolving
about the y-axis the region bounded by the curves
2
y = e−x ,
Information for Students in MATH 141 2009 01 3146

y = 0,
x = 0,
x = 1.

3. [12 MARKS] Determine the area of the surface of revolution generated by revolving
about the x-axis the curve
 π
y = cos x , 0≤x≤ .
6
[Hint: You may wish to make use of the fact that
Z
2 sec3 θ dθ = sec θ tan θ + ln | sec θ + tan θ| + C .]

4. [12 MARKS] Find the area that is inside the circle


r = 3 cos θ and outside the curve r = 2 − cos θ .

5. [14 MARKS] Evaluate the integral


Z
x
dx .
(x − 1)(x2 + 4)

6. For the curve given parametrically by x = t3 + t2 + 1 , y = 1 − t2 , determine

(a) [6 MARKS] The equation of the tangent line at the point


(x, y) = (1, 0) , written in the form y = mx + b , where m and b are con-
stants;
d2 y
(b) [6 MARKS] the value of at the point (x, y) = (1, 0) .
dx2
7. (a) [10 MARKS] Use integration by parts to determine the value of
Z
e x cos x dx .

Z 0
(b) [4 MARKS] Evaluate e x cos x dx .
−∞

8. [12 MARKS] Find the area of the region bounded by the curves y = x2 − 4 and
y = −2x2 + 5x − 2 .
Information for Students in MATH 141 2009 01 3147

E.9 Supplemental/Deferred Examination in Mathematics 189-141B (2000/2001)


1. (a) [6 MARKS] Showing all your work, find F 0 (1) when
Z 2t
x
F(t) = 3
dx .
1 x +x+7
R6
(b) [6 MARKS] Showing all your work, evaluate 0
|x − 2| dx .

2. Showing all of your work, evaluate each of the following integrals:


Z
x+1
(a) [4 MARKS] √ dx;
9 − x2
Z
1
(b) [4 MARKS] 3
dx;
2x + x
Z
(c) [4 MARKS] sin2 2x cos2 2x dx;
Z
(d) [4 MARKS] ln x dx

3. [15 MARKS] Showing all your work, find the area of the region bounded below by the
1 1
line y = , and above by the curve y = .
2 1 + x2
4. [15 MARKS] Showing all your work, find the volume generated by revolving about the
y-axis the smaller region bounded by the circle x2 + y2 = 25 and the line x = 4 .

5. Showing all your work,

(a) [2 MARKS] sketch the curve r = 1 − sin θ ;


(b) [6 MARKS] find the length of the portion of the curve that lies in the region given
π π
by r ≥ 0 , − ≤ θ ≤ ;
2 2
(c) [5 MARKS] find the coordinates of the points on the curve where the tangent line
is parallel to the line θ = 0 .

6. For each of the following integrals, determine whether it is convergent or divergent; if it


is convergent, you are expected to determine its value. Show all your work.
Z 2
1
(a) [7 MARKS] 3
dx ;
−1 x
Z ∞
2
(b) [7 MARKS] xe−x dx .
1
Information for Students in MATH 141 2009 01 3148

7. Showing all your work, determine, for each of the following series, whether or not it
converges:
X

1
(a) [5 MARKS] ;
n=2
n(ln n)2
X
∞ !
n n2 − 1
(b) [5 MARKS] (−1) ;
n=1
n2 + 1
X

n+1
(c) [5 MARKS] .
n=1
3n

E.10 Final Examination in Mathematics 189-141B (2001/2002)


1. Showing all your work, evaluate each of the following indefinite integrals:
Z
x3
(a) [3 MARKS] √ dx
4 − x2
Z
1
(b) [3 MARKS] p dy
y ln y
Z
sec u
(c) [3 MARKS] · tan u du
1 + sec u
Z
et
(d) [3 MARKS] dt
1 + e2t
2. Let K denote the curve
y = x2 , (0 ≤ x ≤ 1) .

(a) [6 MARKS] Determine the area of the surface of revolution generated by revolving
K about the y-axis.
(b) [6 MARKS] Determine the volume of the solid of revolution formed by revolving
about the line y = 0 the region bounded by K and the lines x = 1 and
y = 0.

3. Consider the arc C given by r = θ2 (0 ≤ θ ≤ π).

(a) [4 MARKS] Express the length of C as a definite integral. Then evaluate the inte-
gral.
(b) [4 MARKS] Determine the area of the region subtended by C at the pole — i.e. of
the region bounded by the arc C and the line θ = 0.
Information for Students in MATH 141 2009 01 3149

(c) [4 MARKS] The given curve can be represented in cartesian coordinates paramet-
rically as x = θ2 cos θ, y = θ2 sin θ. Determine the slope of the tangent to this curve
 2
at the point (x, y) = 0, π2 .

4. [12 MARKS] Showing all your work, evaluate the integral


Z
40 − 16x2
 dx .
1 − 4x2 (1 + 2x)
5. [12 MARKS] Showing all your work, determine the area of the region bounded by the
curves y = arctan x and 4y = π x in the first quadrant.
6. (a) [4 MARKS] Showing all your work, determine the value of
Z
sin3 x cos2 x dx .

(b) [4 MARKS] Showing all your work, determine the value of


Z
tan4 x dx .
π
Z2
(c) [4 MARKS] Investigate the convergence of the integral tan4 x dx .
0

7. [12 MARKS] Showing all your work, determine the value of


Z x Z e2t √ 

d2   dt

 u + 1 du
dx2 0 1

when x = 0.
8. Showing all your work, determine, for each of the following infinite series, whether it is
absolutely convergent, conditionally convergent, or divergent.
X

n2 − 1
(a) [4 MARKS] (−1)n .
n=5
6n2 + 4
X∞
(−1)n
(b) [4 MARKS] .
n=2
n(ln n)2
X
∞ n(n+1)
(−1) 2
(c) [4 MARKS] .
n=2
2n
X∞
n+5
(d) [4 MARKS] .
n=0
2n
Information for Students in MATH 141 2009 01 3150

E.11 Supplemental/Deferred Examination in Mathematics 189-141B (2001/2002)


1. Showing all your work, evaluate each of the following, always simplifying your answer
as much as possible:
Z
(a) [3 MARKS] e x sin x dx
Z 12  Z 12 
sin−1 y  arcsin y 
(b) [3 MARKS] p dy equivalently, p dy .
0 1 − y2 0 1 − y2
Z
(c) [3 MARKS] (u2 + 2u)e−u du
Z
1 + cos t
(d) [3 MARKS] dt
sin t
2. Let K denote the curve !
√ 1
y = 2x − x2 , 0≤x≤ .
2
(a) [6 MARKS] Showing all your work, use an integral to determine the area of the
surface of revolution generated by revolving K about the x-axis.
(b) [6 MARKS] Determine√
the volume of the solid of revolution formed by revolving
3
about the line y = 2 the region bounded by K and the lines x = 0 and

y = 23 .

(You may assume that


Z √
x−1√ 1
2x − x2 dx = 2x − x2 + arccos(1 − x) .)
2 2

3. A curve C in the plane is given by parametric equations

x = t3 − 3t2
y = t3 − 3t .

(a) [6 MARKS] Showing all your work, determine all points (x, y) on C where the
tangent is horizontal.
d2 y
(b) [6 MARKS] By determining the value of as a function of t, determine all
dx2
points (x, y) on C at which the ordinate (y-coordinate) is a (local) maximum, and
all points at which the ordinate is a (local) minimum.
Information for Students in MATH 141 2009 01 3151

4. [12 MARKS] Showing all your work, evaluate the indefinite integral
Z
4x3
 dx .
x2 − 9 (3x + 9)

5. [12 MARKS] Showing all your work, determine the area of the region bounded by the
curves x − 2y + 7 = 0 and y2 − 6y − x = 0 .

6. (a) [6 MARKS] Showing all your work, evaluate


Z
sin4 x cos2 x dx .

(b) [4 MARKS] Showing all your work, evaluate


Z
tan5 x dx .

π
Z2
(c) [4 MARKS] Investigate the convergence of the integral tan5 θ dθ .
π
4

7. [12 MARKS] Showing all your work, determine the value of


Z x Z π3 p !
d2
4 + sin(−2u) du dt
dx2 0 −2t

when x = π4 . Your answer should be simplified, if possible.

8. Showing all your work, determine, for each of the following infinite series, whether it is
absolutely convergent, conditionally convergent, or divergent.
X

1
(a) [4 MARKS] (−1)n √ .
n=5
n+1
X∞
(−1)2n
(b) [4 MARKS] .
n=2
n(ln n)3
X∞ !3n
2n
(c) [4 MARKS] .
n=2
1 + 5n
 
X∞ sin 1
n 1
(d) [4 MARKS]  · .
n=1 cos
1 n
n
Information for Students in MATH 141 2009 01 3152

9. [10 MARKS] Prove or disprove the following statement: The point with polar coordi-
nates

r = 2( 2 − 1)

θ = −π + arcsin(( 2 − 1)2 )

lies on the intersection of the curves with polar equations

r2 = 4 sin θ,
r = 1 + sin θ .

You are expected to justify every statement you make, but you do not need to sketch the
curves.

E.12 Final Examination in MATH 141 2003 01


1. [10 MARKS] Find the area of the region bounded in the first quadrant by the curves

y = ex , y = e−x , y = e2x−3 .

Simplify your answer as much as possible. (Your instructors are aware that you do not
have the use of a calculator.)

2. Showing all your work, evaluate each of the following indefinite integrals:
Z
1
(a) [5 MARKS] 2
dx
x + 2x + 17
Z
ln(ln x)
(b) [5 MARKS] dx
x
3. [12 MARKS] For each of the following integrals,

(a) [2 MARKS] Explain why the integral is improper.


(b) [10 MARKS] Determine its value, or show that the integral does not converge.

Show all your work.


Z ∞ Z 1
2(x2 − x + 1) 2(x2 − x + 1)
I1 = 2
dx , I2 = dx
2 (x − 1)(x + 1) 0 (x − 1)(x2 + 1)
p
4. Let Ω denote the region in the first quadrant bounded by the curves x = 16 + y2 , y = 0,
x = 0, and y = 3.
Information for Students in MATH 141 2009 01 3153

(a) [5 MARKS] Showing all your work, determine the volume of the solid of revolu-
tion obtained by rotating Ω about the y-axis.
(b) [7 MARKS] Showing all your work, determine the area of the surface of revolution
obtained by rotating the arc
p
x = 16 + y2 , (0 ≤ y ≤ 3)

about the y-axis. You may assume that


d
(sec θ tan θ + ln |sec θ + tan θ|) = 2 sec3 θ .

.

5. Consider the arc C given parametrically by


 Z tp 


 



 x = 4(1 − cos θ)θ2 dθ 


 0 
 (−π ≤ t ≤ 2π) .


 y = cos t + t sin t 

Showing all your work

(a) [4 MARKS] Find the slope of the tangent to C at the point with parameter value

t= .
2
(b) [6 MARKS] Find the length of C.

6. Give, for each of the following statements, a specific example to show that the statement
is not a theorem:
P

(a) [3 MARKS] If {an }∞
n=0 is a sequence such that lim an = 0, then an converges.
n→∞ n=0
P
∞ P
∞ P

(b) [3 MARKS] If the series an and bn are both divergent, then (an + bn ) is
n=0 n=0 n=0
divergent.
P
∞ P

(c) [3 MARKS] If a series an converges, then a2n converges.
n=0 n=0

7. Showing all your work, determine, for each of the following infinite series, whether it is
absolutely convergent, conditionally convergent, or divergent.
X∞
(−1)n
(a) [4 MARKS] .
n=0
4n2 + 1
Information for Students in MATH 141 2009 01 3154

X∞ !n2
n−1
(b) [4 MARKS]
n=2
n
X

1
(c) [4 MARKS] √ .
n=2 n(n + 1)

8. [10 MARKS] Showing all your work, determine the area of the part of one “leaf” of the
“4-leafed rose” r = 2 cos(2θ) that is inside the circle r = 1.

E.13 Supplemental/Deferred Examination in MATH 141 2003 01


1. [10 MARKS] Find the area of the region bounded by the curves

y = e x − 1, y = x2 − x, x = 1.

2. Showing all your work, evaluate each of the following:


Z 1
ex
(a) [5 MARKS] dx
x2
Z 5
(b) [5 MARKS] |x2 − 4x| dx
2

3. [12 MARKS] For each of the following integrals,

(a) [2 MARKS] Explain precisely whether the integral is improper.


(b) [10 MARKS] Determine its value, simplifying as much as possible; or show that
the integral does not converge. (The examiners are aware that you do not have
access to a calculator.)

Show all your work.


Z 1 Z 3
2x 2x
I1 = 2
dx , I2 = dx
0 (x − 1)(x + 1) 2 (x − 1)(x2 + 1)

4. Let Ω denote the region bounded by the curves y = sin x, y = 0, x = π2 , x = π.

(a) [6 MARKS] Showing all your work, determine the volume of the solid of revolu-
tion obtained by rotating Ω about the y-axis.
(b) [6 MARKS] Showing all your work, determine the volume of the solid of revolu-
tion obtained by rotating Ω about the x-axis.
Information for Students in MATH 141 2009 01 3155

5. Consider the arc C given parametrically by


 
 2 
 x = 2t(t − 3)

 



 
 (−1 ≤ t ≤ 1) .

 y = 6t(−t) 

Showing all your work

(a) [4 MARKS] Find the slope of the tangent to C at the point with parameter value
t = − 12 .
(b) [6 MARKS] Find the area of the surface obtained by rotating the curve about the
x-axis.

6. Give, for each of the following statements, a specific example to show that the statement
is not a theorem:
P

(a) [3 MARKS] If {bn }∞
n=0 is a sequence such that lim bn = 1, then bn converges.
n→∞ n=0
P
∞ P

(b) [3 MARKS] If a series b2n converges, then bn converges.
n=0 n=0
P
∞ P
∞ P

(c) [3 MARKS] If the series an and bn are both divergent, then (an bn ) is di-
n=0 n=0 n=0
vergent.

7. Showing all your work, determine, for each of the following infinite series, whether it is
absolutely convergent, conditionally convergent, or divergent.
X

(−1)n
(a) [4 MARKS] 1
.
n=3 en
X
∞ !
1
(b) [4 MARKS]
n=0
(n + 5)(n + 6)
X

2
(c) [4 MARKS] (−1)n−1 √ .
n=2
n − 1

8. [10 MARKS]Showing all your work, find the area of the region that lies inside the curve
r = 2 − 2 sin θ and outside the curve r = 3 .

E.14 Final Examination in MATH 141 2004 01


One of several versions
Information for Students in MATH 141 2009 01 3156

1. BRIEF SOLUTIONS
[3 MARKS EACH] Give the numeric value of each of the following limits, sums, inte-
grals. If it does not exist write “DIVERGENT”.
X

1 + 2n−1
(a) =
n=1
3n

ANSWER ONLY

Z ∞
2
(b) xe x dx =
−∞

ANSWER ONLY

P
n    
2 2i 2 2i 5
(c) The limit of the Riemann sum lim 3+ −6 3+ n =
n→∞ i=1 n n

ANSWER ONLY

Z −∞
2
(d) xe−x dx =

ANSWER ONLY

X

4
(e) =
n=3
(2n + 1)(2n + 3)

ANSWER ONLY

2. BRIEF SOLUTIONS
[3 MARKS EACH] Simplify your answers as much as possible.
Information for Students in MATH 141 2009 01 3157

 
(a) For the point with polar coordinates 3, π7 give another set of polar coordinates
(r, θ) in which r < 0 and θ > 2.

ANSWER ONLY

(b) Determine the length of the arc of the curve r = θ2 from (0, 0) to (1, 1).

ANSWER ONLY

Rt 2
R4 2
(c) A curve is given parametrically by x(t) = 0 e−u du, y(t) = t eu du. Find the
slope of the tangent to the curve at (x(1), y(1)).

ANSWER ONLY

(d) Give a definite integral whose value is the area of the surface obtained by rotating
y3 1 1 
the curve x = + 2
≤ y ≤ 1 about the y-axis. You need not evaluate the
6 2y
integral.

ANSWER ONLY

(e) On the interval 0 ≤ x ≤ 4 the average value of the function


( √
1 − x if 0 ≤ x ≤ 1
f (x) = is
x−2 if 1 < x ≤ 4
Information for Students in MATH 141 2009 01 3158

ANSWER ONLY

3. BRIEF SOLUTIONS
[3 MARKS EACH] Give the value of each of the following indefinite integrals:
Z
x
(a) 2
dx =
3x + 1
ANSWER ONLY

Z √
(b) e x 1 + e x dx =

ANSWER ONLY

Z
(c) (sin2 x − 3 cos2 x) dx =

ANSWER ONLY

Z
(d) tan2 3x dx =

ANSWER ONLY
Information for Students in MATH 141 2009 01 3159

Z
(e) sec3 x tan3 x dx =

ANSWER ONLY

4. SHOW ALL YOUR WORK!


Let R be the finite region bounded by the curves x = y2 and x = 4 − 3y4 .

(a) [5 MARKS] Find the area of R.


(b) [5 MARKS] Find the volume of the solid generated by revolving R about the y-axis.

5. SHOW ALL YOUR WORK!


[12 MARKS] Evaluate the definite integral
Z 12 4x 2x2 − x − 2
dx .
− 21 (x2 + 1)(x2 − 1)

6. SHOW ALL YOUR WORK!

(a) [4 MARKS] Show that, for any positive integer n,


Z Z
(ln x) dx = x(ln x) − 2n (ln x)2n−1 dx
2n 2n

(b) [7 MARKS] Evaluate the integral


Z 1
y
p dy .
0 2y − y2

7. SHOW ALL YOUR WORK!


[4 MARKS EACH] Determine for each of the following series whether it

• converges absolutely;
• converges conditionally; or
• diverges.
Information for Students in MATH 141 2009 01 3160

X
∞ !2
1 −n
(a) 1+ e
n=1
n
X∞ √
(−1)n 2n
(b) √
n=10
1+2 n
X∞ √ √
n + 2 − n−1
(c) (−1)n ·
n=2
n
X

2π + cos n
(d)
n=0
6n

8. SHOW ALL YOUR WORK!


[6 MARKS] Find the area bounded by one loop of the curve r = cos 3θ.

Another version
1. BRIEF SOLUTIONS
[3 MARKS EACH] Give the numeric value of each of the following limits, sums, inte-
grals. If it does not exist write “DIVERGENT”.
X

1 + 3n−1
(a) =
n=1
4n

ANSWER ONLY

Z ∞
2
(b) yey dy =
−∞

ANSWER ONLY

P
n    5 
2 2i 2
(c) The limit of the Riemann sum lim 4+ − 7 4 + 2in =
n→∞ i=1 n n

ANSWER ONLY
Information for Students in MATH 141 2009 01 3161

Z −∞
2
(d) ye−y dy =

ANSWER ONLY

X

4
(e) =
n=3
(2n − 1)(2n + 1)

ANSWER ONLY

2. BRIEF SOLUTIONS
[3 MARKS EACH] Simplify your answers as much as possible.
(a) On the interval 0 ≤ x ≤ 4 the average value of the function
( √
1 − x if 0 ≤ x ≤ 1
f (x) = is
x−2 if 1 < x ≤ 4

ANSWER ONLY

 
(b) For the point with polar coordinates 3, π5 give another set of polar coordinates
(r, θ) in which r < 0 and θ > 2.

ANSWER ONLY

(c) Determine the length of the arc of the curve r = θ2 from (0, 0) to (1, 1).

ANSWER ONLY
Information for Students in MATH 141 2009 01 3162

Rt 2
R4 2
(d) A curve is given parametrically by x(t) = 0 e−v dv, y(t) = t ev dv. Find the
slope of the tangent to the curve at (x(1), y(1)).

ANSWER ONLY

(e) Give a definite integral whose value is the area of the surface obtained by rotating
y3 1 1 
the curve x = + ≤ y ≤ 1 about the y-axis. You need not evaluate the
6 2y 2
integral.

ANSWER ONLY

3. BRIEF SOLUTIONS
[3 MARKS EACH] Give the value of each of the following indefinite integrals:
Z
(a) sec3 x tan3 x dx =

ANSWER ONLY

Z
x
(b) dx =
5x2 + 1
ANSWER ONLY

Z √
(c) e x 1 + e x dx =
Information for Students in MATH 141 2009 01 3163

ANSWER ONLY

Z
(d) (3 sin2 x − cos2 x) dx =

ANSWER ONLY

Z
(e) tan2 4x dx =

ANSWER ONLY

4. SHOW ALL YOUR WORK!


Let S be the finite region bounded by the curves y = x2 and y = 4 − 3x4 .
(a) [5 MARKS] Find the area of S .
(b) [5 MARKS] Find the volume of the solid generated by revolving S about the x-
axis.
5. SHOW ALL YOUR WORK!
[12 MARKS] Evaluate the definite integral
Z 12 4x 4x2 − 2x − 4
dx .
− 12 (x2 + 1)(x2 − 1)

6. SHOW ALL YOUR WORK!

(a) [4 MARKS] Show that, for any positive integer m,


Z Z
(ln y) dy = y(ln y) − 2m (ln y)2m−1 dy
2m 2m
Information for Students in MATH 141 2009 01 3164

(b) [7 MARKS] Evaluate the integral


Z 1
x
√ dx .
0 2x − x2

7. SHOW ALL YOUR WORK!


[4 MARKS EACH] Determine for each of the following series whether it

• converges absolutely;
• converges conditionally; or
• diverges.
X∞ √
(−1)n 2n
(a) √
n=10
1+2 n
X∞ !2
1 −n
(b) 1+ e
n=1
n
X

(cos n) − 2π
(c)
n=0
4n
X
∞ √ √
n n+2− n−1
(d) (−1) ·
n=2
n

8. SHOW ALL YOUR WORK!


[6 MARKS] Find the area bounded by one loop of the curve r = cos 3θ.

E.15 Supplemental/Deferred Examination in MATH 141 2004 01


1. BRIEF SOLUTIONS
[3 MARKS EACH] Give the numeric value of each of the following limits, sums, inte-
grals. If it does not exist write “DIVERGENT”.
X

en − e−n
(a) =
n=1
3n

ANSWER ONLY
Information for Students in MATH 141 2009 01 3165

Z 0
(b) xe x dx =
−∞

ANSWER ONLY

π X 2  iπ 
n
(c) The limit of the Riemann sum lim sin =
n→∞ n n
i=1

ANSWER ONLY

X

4
(d) =
n=0
(2n + 1)(2n + 3)

ANSWER ONLY

2. BRIEF SOLUTIONS
[3 MARKS EACH] Simplify your answers as much as possible.
 
(a) For the point with polar coordinates (r, θ) = − 10π 3
, − π
6
give another set of polar
coordinates (r1 , θ1 ) in which r1 > 0 and θ1 > 2 .

ANSWER ONLY

(b) Find all points — if there are any — where the curves
r = 1 − cos θ and r = − 32 intersect.
Information for Students in MATH 141 2009 01 3166

ANSWER ONLY

(c) Find the exact length of the curve r = e2θ , (0 ≤ θ ≤ π) .

ANSWER ONLY

(d) On the interval ln 12 ≤ x ≤ π the average value of the function


(
sinh x if ln 12 ≤ x ≤ 0
f (x) = is
sin x if 0<x≤π

ANSWER ONLY

3. BRIEF SOLUTIONS
[3 MARKS EACH] Give the value of each of the following indefinite integrals:
Z
1
(a) dx =
4x2 + 1
ANSWER ONLY
Information for Students in MATH 141 2009 01 3167

Z
dx
(b) √ =
x2 − 25
ANSWER ONLY

Z
(c) (cos x + 1)(cos x − 2) dx =

ANSWER ONLY

Z
(d) tan 3x dx =

ANSWER ONLY

4. SHOW ALL YOUR WORK!


1 p 2 3 √
Let C be the arc x = y +2 , (− 2 ≤ y ≤ 0) .
3
(a) [6 MARKS] Find the area of the surface obtained by revolving C about the x-axis.
(b) [6 MARKS] Find the volume of the solid generated by revolving√about the y-axis
the region bounded by C, the coordinate axes, and the line y = − 2.

5. SHOW ALL YOUR WORK!


[12 MARKS] Evaluate the indefinite integral
Z
x3 − 8x − 1
dx .
(x2 − 1)(x + 1)
Information for Students in MATH 141 2009 01 3168

6. SHOW ALL YOUR WORK!


Simplify your answers as much as possible.
Z √
(a) [6 MARKS] Evaluate the indefinite integral 1 − 9x2 dx .
Z √
− 3
(b) [6 MARKS] Evaluate the definite integral arctan x dx .
0

7. SHOW ALL YOUR WORK!


[4 MARKS EACH] Determine for each of the following series whether it

• converges absolutely;
• converges conditionally; or
• diverges.
X
∞ √
nn ln
(a) (−1)
n=2
ln(n2 )
X

1 · 3 · 5 · . . . · (2n − 1)
(b)
n=1
3n n!
X∞
sin 2n
(c)
n=1
1 + 2n
X
∞ √
n n
(d) (−1)
n=1
3n − 1

8. SHOW ALL YOUR WORK!


[12 MARKS] Use polar coordinates — no other method will be accepted — to find the
1
area of the region bounded by the curve r = 2 and the line r = , and containing the
cos θ
pole.

E.16 Final Examination in MATH 141 2005 01


1. SHOW ALL YOUR WORK!
Z 3
(a) [4 MARKS] Evaluate |x − 1| dx .
0
Information for Students in MATH 141 2009 01 3169

Z 5 √
d
(b) [3 MARKS] Evaluate 4 + t2 dt .
dx x
x Z 2
d
(c) [3 MARKS] Evaluate sec t dt .
dx 2π
Z  √3 
(d) [3 MARKS] Evaluate x5 x3 + 1 dx .

SHOW ALL YOUR WORK!

2. For each of the following series you are expected to apply one or more tests for conver-
gence or divergence and determine whether the series is convergent. In each case you
must answer 3 questions:

• Name the test(s) that you are using.


• Explain why the test(s) you have chosen is/are applicable to the given series.
• Use the test(s) to conclude whether or not the series is convergent.
X

2 − cos n
(a) [4 MARKS]
n=2
n
X

n(−3)n
(b) [4 MARKS]
n=0
4n
X

1
(c) [4 MARKS]
n=2
n ln n

3. BRIEF SOLUTIONS Express the value of each of the following as a definite integral
or a sum, product, or quotient of several definite integrals, but do not evaluate the inte-
gral(s). It is not enough to quote a general formula: your integrals must have integrand
and limits specific to the given problems:

(a) [6 MARKS] The area of the region bounded by the parabola y = x2 , the x-axis, and
the tangent to the parabola at the point (1, 1).
DEFINITE INTEGRAL(S) ONLY (DO NOT EVALUATE)
Information for Students in MATH 141 2009 01 3170

(b) [3 MARKS] The volume of the solid obtained


p by rotating about the line y = 4 the
region bounded by x = 0 and the curve x = sin y (0 ≤ y ≤ π).
DEFINITE INTEGRAL(S) ONLY (DO NOT EVALUATE)

(c) [3 MARKS] The area of the surface obtained by revolving about the y-axis the
curve y = e x , 1 ≤ y ≤ 2.
DEFINITE INTEGRAL(S) ONLY (DO NOT EVALUATE)

2x
(d) [2 MARKS] The average value of the function over the interval 0 ≤ x ≤
(1 + x2 )2
2.
DEFINITE INTEGRAL(S) ONLY (DO NOT EVALUATE)

4. SHOW ALL YOUR WORK!


[12 MARKS] Evaluate the indefinite integral
Z
2x3 + 3x2 + 3
dx .
x2 + x − 12

5. SHOW ALL YOUR WORK!


(a) [9 MARKS] Use integration by parts to
prove that, for integers
m ≥ 2, Z Z
m 1 m−1 m−1
cos x dx = cos x · sin x + cosm−2 x dx
m m
Information for Students in MATH 141 2009 01 3171

(b) [3 MARKS] Showing all your work, use the formula you have proved to evaluate
Z π2
cos6 x dx.
0

6. SHOW ALL YOUR WORK!


Consider the curve C defined by
x = 2 cos t − cos 2t
y = 2 sin t − sin 2t .
(a) [8 MARKS] Determine the points where the arc of the curve given by
π 7π
≤t≤
4 4
has a vertical tangent.
(b) [4 MARKS] Determine the length of the arc of the curve given by
0 ≤ t ≤ 2π .

7. SHOW ALL YOUR WORK!

(a) [5 MARKS] Determine whether the following integral is convergent; if it is con-


vergent, determine its value: Z 1
dx

−1 1 − x2
(b) [5 MARKS] Determine whether the following series is conditionally convergent,
absolutely convergent, or divergent.
X

n!
(−1)n
n=1
nn

(c) [3 MARKS] Determine whether the sequence an = ln(n + 1) − ln n is convergent;


if it is convergent, carefully determine its limit.

8. SHOW ALL YOUR WORK!


[12 MARKS] Find the area of the region bounded by the curves
r = 4 + 4 sin θ
r sin θ = 3
which does not contain the pole.
Information for Students in MATH 141 2009 01 3172

E.17 Supplemental/Deferred Examination in MATH 141 2005 01


Instructions
1. Fill in the above clearly.

2. Do not tear pages from this book; all your writing — even rough work — must be handed in.
You may do rough work for this paper anywhere in the booklet.

3. Calculators are not permitted.

4. This examination booklet consists of this cover, Pages 1 through 7 containing questions; and
Pages 8, 9, and 10, which are blank.

5. There are two kinds of problems on this examination, each clearly marked as to its type.

• Most of the questions on this paper require that you SHOW ALL YOUR WORK!
Their solutions are to be written in the space provided on the page where the question
is printed. When that space is exhausted, you may write on the facing page . Any
solution may be continued on the last pages, or the back cover of the booklet, but you
must indicate any continuation clearly on the page where the question is printed!
• Some of the questions on this paper require only BRIEF SOLUTIONS ; for these you
are expected to write the correct answer in the box provided; you are not asked to show
your work, and you should not expect partial marks for solutions that are not correct.

You are expected to simplify your answers wherever possible.


You are advised to spend the first few minutes scanning the problems. (Please inform the
invigilator if you find that your booklet is defective.)

6. A TOTAL OF 85 MARKS ARE AVAILABLE ON THIS EXAMINATION.

1. SHOW ALL YOUR WORK!


Z 0
(a) [4 MARKS] Evaluate |x2 − 1| dx .
−2
Z 4
d 2
(b) [3 MARKS] Evaluate et dt .
dx x
x Z 2
d dt
(c) [3 MARKS] Evaluate .
dx 1 1 + t5
Z
(d) [3 MARKS] Evaluate x sin(x2 ) dx .

SHOW ALL YOUR WORK!


Information for Students in MATH 141 2009 01 3173

2. For each of the following series you are expected to apply one or more tests for conver-
gence or divergence and determine whether the series is convergent. In each case you
must answer 3 questions:

• Name the test(s) that you are using.


• Explain why the test(s) you have chosen is/are applicable to the given series.
• Use the test(s) to conclude whether or not the series is convergent.
X
∞ !n
2n
(a) [4 MARKS]
n=2
3n − 1
X∞
(−1)n
(b) [4 MARKS] √
n=2
n−1
X∞ !
(−4)n
(c) [4 MARKS]
n=2
3n + 2n

3. BRIEF SOLUTIONS Express the value of each of the following as a definite integral
– possibly improper — or a sum, product, or quotient of several such integrals, but do
not evaluate the integral(s). It is not enough to quote a general formula: your integrals
must have integrand and limits specific to the given problems:
 
(a) [6 MARKS] The area of the infinite region containing the point 0, 21 bounded by
the curve y = e x , the x-axis, and the tangent to the curve at the point (1, e).
DEFINITE INTEGRAL(S) ONLY (DO NOT EVALUATE)

(b) [3 MARKS]
√ The volume generated by rotating the region bounded by the curves
y = x − 1, y = 0, x = 5 about the line y = 3.
DEFINITE INTEGRAL(S) ONLY (DO NOT EVALUATE)
Information for Students in MATH 141 2009 01 3174

(c) [3 MARKS] The area of the surface obtained by revolving about the x-axis the
curve x = ln y, 1 ≤ x ≤ 3.
DEFINITE INTEGRAL(S) ONLY (DO NOT EVALUATE)

4. SHOW ALL YOUR WORK!


[12 MARKS] Evaluate the indefinite integral
Z 3
x − x2 − 2x − 2
dx .
x(x2 + x + 1)

5. SHOW ALL YOUR WORK!

(a) [9 MARKS] Use integration by parts to prove that, for integers


m , 1,
Z Z
m 1 m−2 m−2
sec x dx = sec x · tan x + secm−2 x dx
m−1 m−1

(b) [3 MARKS] Showing all your work, use the formula you have proved to evaluate
Z π3
sec3 x dx.
0

6. SHOW ALL YOUR WORK!


The curve C has equations x = t3 + 4t, y = 6t2 .

(a) [8 MARKS] Determine the points on C where the tangent is parallel to the line
with equations x = −7t, y = 12t − 5.
(b) [4 MARKS] Determine a definite integral whose value is the length of the arc of
C between the points with parameter values t = 1 and t = 2. YOU ARE NOT
EXPECTED TO EVALUATE THE INTEGRAL!

7. SHOW ALL YOUR WORK!


[12 MARKS] Find the area of the region between the inner loop and the outer loop of
the curve r = 1 − 2 cos θ.
Information for Students in MATH 141 2009 01 3175

E.18 Final Examination in MATH 141 2006 01 (One version)


Instructions
1. Fill in the above clearly.

2. Do not tear pages from this book; all your writing — even rough work — must be handed
in. You may do rough work for this paper anywhere in the booklet.

3. Calculators are not permitted. This is a closed book examination. Regular and transla-
tion dictionaries are permitted.

4. This examination booklet consists of this cover, Pages 1 through 8 containing questions;
and Pages 9, 10, and 11, which are blank. Your neighbour’s version of this test may be
different from yours.

5. There are two kinds of problems on this examination, each clearly marked as to its type.

• Most of the questions on this paper require that you SHOW ALL YOUR WORK!
Their solutions are to be written in the space provided on the page where the ques-
tion is printed. When that space is exhausted, you may write on the facing page.
Any solution may be continued on the last pages, or the back cover of the booklet,
but you must indicate any continuation clearly on the page where the question is
printed!
• Some of the questions on this paper require only BRIEF SOLUTIONS ; for these
you are expected to write the correct answer in the box provided; you are not asked
to show your work, and you should not expect partial marks for solutions that are
not correct.

You are expected to simplify your answers wherever possible.


You are advised to spend the first few minutes scanning the problems. (Please inform
the invigilator if you find that your booklet is defective.)

6. A TOTAL OF 100 MARKS ARE AVAILABLE ON THIS EXAMINATION.

1. SHOW ALL YOUR WORK!


Z 2
(a) [4 MARKS] Evaluate |x| dx .
−1

Ze3
dt
(b) [3 MARKS] Evaluate √ .
t 1 + ln t
1
Information for Students in MATH 141 2009 01 3176

Z x2
d 2
(c) [3 MARKS] Evaluate et dt .
dx 0
(d) [4 MARKS] Evaluate
 !7 !7 !7 !7 
1  0 1 2 n − 1 
lim  + + + ... +  .
n→∞ n n n n n

SHOW ALL YOUR WORK!

2. For each of the following series you are expected to apply one or more tests for conver-
gence or divergence and determine whether the series is convergent. In each case you
must answer 3 questions:

• Name the test(s) that you are using.


• Explain why the test(s) you have chosen is/are applicable to the given series.
• Use the test(s) to conclude whether or not the series is convergent.
X

1
(a) [4 MARKS]
n=1
(tanh n)2 + 1
X

2
(b) [4 MARKS] n2n e−n
n=1
X

n2 − 85n + 12
(c) [4 MARKS]
n=1
n(n + 6)2

3. BRIEF SOLUTIONS Express the value of each of the following as a definite integral
or a sum, product, or quotient of several definite integrals, but do not evaluate the inte-
gral(s). It is not enough to quote a general formula: your integrals must have integrand
and limits specific to the given problems, and should be simplified as much as possible,
except that you are not expected to evaluate the integrals.

(a) [3 MARKS] Expressed as integral(s) along the x-axis only, the area of the region
bounded by the parabola y2 = 2x + 6 and the line y = x − 1. An answer involving
integration along the y-axis will not be accepted.
(b) [3 MARKS] The volume of the solid obtained by rotating about the line y = 1 the
region bounded by the curves y = x3 and y = x2 . For this question you are to use
only the method of “washers”.
Information for Students in MATH 141 2009 01 3177

(c) [3 MARKS] The volume of the solid obtained by rotating about the line y = 1 the
region bounded by the curves y = x3 and y = x2 . For this question you are to use
only the method of “cylindrical shells”.
(d) [3 MARKS] The length of the curve whose equation is

x2 y2
+ = 1.
4 9

4. SHOW ALL YOUR WORK!


[12 MARKS] Evaluate the indefinite integral
Z 5
x +x
dx .
x4 − 16

5. SHOW ALL YOUR WORK!


Showing all your work, evaluate each of the following:
Z
(a) [4 MARKS] cos x · cosh x dx

Z1 √
(b) [5 MARKS] x2 + 2x + 5 dx
−3
Z
(c) [4 MARKS] sin2 x · cos2 x dx

6. SHOW ALL YOUR WORK!


Consider the curve C defined by

x = x(t) = 10 − 3t2
y = y(t) = t3 − 3t ,

where −∞ < t < +∞.


d2 y
(a) [8 MARKS] Determine the value of 2 at the points where the tangent is horizon-
dx
tal.
(b) [4 MARKS] Determine the area of the surface of revolution about the x-axis of the
arc n √ o
(x(t), y(t)) : − 3 ≤ t ≤ 0 .
Information for Students in MATH 141 2009 01 3178

7. SHOW ALL YOUR WORK!

(a) [5 MARKS] Showing detailed work, determine whether the following integral is
convergent; if it is convergent, determine its value:
Z 0
dx
2
.
−1 x 3

(b) [5 MARKS] Determine whether the following series is conditionally convergent,


absolutely convergent, or divergent.
X∞
(−1)n
n=1
n − ln n

(c) [3 MARKS] Give an example of a sequence {an } with the property that lim an = 0
n→∞
X

but an = +∞. You are expected to give a formula for the general term an of
n=1
your sequence.

8. SHOW ALL YOUR WORK!


[12 MARKS] The arc
r = 1 − cos θ (0 ≤ θ ≤ π)
divides the area bounded by the curve

r = 1 + sin θ (0 ≤ θ ≤ 2π)

into two parts. Showing


  all your work, carefully find the area of the part that contains
the point (r, θ) = 12 , π2 .

E.19 Supplemental/Deferred Examination in MATH 141 2006 01


Instructions
1. Fill in the above clearly.

2. Do not tear pages from this book; all your writing — even rough work — must be handed in.
You may do rough work for this paper anywhere in the booklet.

3. Calculators are not permitted. This is a closed book examination. Regular and translation
dictionaries are permitted.
Information for Students in MATH 141 2009 01 3179

4. This examination booklet consists of this cover, Pages 1 through 8 containing questions; and
Pages 9, 10, and 11, which are blank.

5. There are two kinds of problems on this examination, each clearly marked as to its type.

• Most of the questions on this paper require that you SHOW ALL YOUR WORK!
Their solutions are to be written in the space provided on the page where the question
is printed. When that space is exhausted, you may write on the facing page . Any
solution may be continued on the last pages, or the back cover of the booklet, but you
must indicate any continuation clearly on the page where the question is printed!
• Some of the questions on this paper require only BRIEF SOLUTIONS ; for these you
are expected to write the correct answer in the box provided; you are not asked to show
your work, and you should not expect partial marks for solutions that are not correct.

You are expected to simplify your answers wherever possible.


You are advised to spend the first few minutes scanning the problems. (Please inform the
invigilator if you find that your booklet is defective.)

6. A TOTAL OF 100 MARKS ARE AVAILABLE ON THIS EXAMINATION.

1. SHOW ALL YOUR WORK!


Simplify your answers as much as possible.
Z ee
dt
(a) [4 MARKS] Evaluate .
e2 t ln t
Z + π2
(b) [4 MARKS] Evaluate | cosh x| dx .
− π2

(c) [4 MARKS] Evaluate


 !2 !2 !2 !2 
1  0 1 2 n − 1 
lim  4 + + 4+ + 4+ + ... + 4 +  .
n→∞ n n n n n
Z −3x √
d
(d) [3 MARKS] Evaluate 1 + t2 dt when x = 1.
dx 0

SHOW ALL YOUR WORK!

2. For each of the following series you are expected to apply one or more tests for conver-
gence or divergence and determine whether the series is convergent. In each case you
must answer 3 questions:

• [1 MARK] Name the test(s) that you are using.


Information for Students in MATH 141 2009 01 3180

• [1 MARK] Explain why the test(s) you have chosen is/are applicable to the given
series.
• [2 MARKS] Use the test(s) to conclude whether or not the series is convergent.
X

n
(a) [4 MARKS] (−1)n
n=1
n2 + 4
X
∞  n 
(b) [4 MARKS] ln
n=1
3n + 2
X

3n + 6
(c) [4 MARKS]
n=1
5n

3. BRIEF SOLUTIONS Express the value of each of the following as a definite integral
or a sum, product, or quotient of several definite integrals, but do not evaluate the inte-
gral(s). It is not enough to quote a general formula: your integrals must have integrand
and limits specific to the given problems, and should be simplified as much as possible,
except that you are not expected to evaluate the integrals.

(a) [4 MARKS] The length of the curve whose equation is

x = 1 + et , y = t2 , (−3 ≤ t ≤ 3).

(b) [4 MARKS] The volume of the solid obtained by rotating about the x-axis the
region bounded by the curves y = x and y = x2 . For this question you are expected
to use only the method of “cylindrical shells”.
(c) [4 MARKS] The volume of the solid obtained by rotating about the line y = 2 the
region bounded by the curves y = x and y = x2 . For this question you are expected
to use only the method of “washers”.

4. SHOW ALL YOUR WORK!


[12 MARKS] Evaluate the indefinite integral
Z
x3
dx .
(x2 + 4)(x − 2)

5. SHOW ALL YOUR WORK!


Showing all your work, evaluate each of the following. Simplify your answers as much
as possible.
Information for Students in MATH 141 2009 01 3181

Z
(a) [4 MARKS] 8x cos 2x dx

Z 2
2−
1
(b) [4 MARKS] √ dx
4x − x2
0
Z
(c) [4 MARKS] e x sin x dx

6. SHOW ALL YOUR WORK!


Consider the arc C defined by

x = x(t) = 3t − t3
y = y(t) = 3t2 ,

where 0 ≤ t ≤ 1.
d2 y 1
(a) [6 MARKS] Determine the value of 2
at the point with parameter value t = .
dx 2
(b) [6 MARKS] Determine the area of the surface of revolution of C about the x-axis.

7. SHOW ALL YOUR WORK!

(a) [5 MARKS] Showing detailed work, determine whether the following integral is
convergent; if it is convergent, determine its value:
Z 6
x
xe 3 dx .
−∞

(b) [4 MARKS] Give an example of a series which is convergent but not absolutely
convergent. Justify all of your statements.
(c) [4 MARKS] Give an example of 2 divergent sequences {an }, {bn } with the property
that the sequence {an bn } is convergent. You are expected to give formulas for the
general terms an , bn of both of your sequences.

8. SHOW ALL YOUR WORK!


[12 MARKS] The curves r = 2 cos 2θ and r = 2 sin θ define a number of regions in the
plane. Let R denote the region containing the point (r, θ) = (1, 0), bounded by arcs of
both of the curves. Showing all your work, carefully find the area of R.
Information for Students in MATH 141 2009 01 3182

E.20 Final Examination in MATH 141 2007 01 (One version)


Instructions
1. Do not tear pages from this book; all your writing — even rough work — must be handed in.
You may do rough work for this paper anywhere in the booklet.

2. Calculators are not permitted. This is a closed book examination. Regular and translation
dictionaries are permitted.

3. This examination booklet consists of this cover, Pages 1 through 8 containing questions; and
Pages 9, 10, and 11, which are blank. Your neighbour’s version of this test may be different
from yours.

4. There are two kinds of problems on this examination, each clearly marked as to its type.

• Most of the questions on this paper require that you SHOW ALL YOUR WORK!
Their solutions are to be written in the space provided on the page where the question
is printed. When that space is exhausted, you may write on the facing page . Any
solution may be continued on the last pages, or the back cover of the booklet, but you
must indicate any continuation clearly on the page where the question is printed!
• Some of the questions on this paper require only BRIEF SOLUTIONS ; for these you
are expected to write the correct answer in the box provided; you are not asked to show
your work, and you should not expect partial marks for solutions that are not correct.

You are expected to simplify your answers wherever possible.


You are advised to spend the first few minutes scanning the problems. (Please inform the
invigilator if you find that your booklet is defective.)

5. A TOTAL OF 100 MARKS ARE AVAILABLE ON THIS EXAMINATION.

1. SHOW ALL YOUR WORK!


Your answers must be simplified as much as possible.
Z 2
(a) [2 MARKS] Evaluate |x|2 dx .
−1
Z0
t4 dt
(b) [2 MARKS] Evaluate √ .
t5 + 1
1
(c) [3 MARKS] Determine the value of
 !3 !3 !3 !3 
1  0 1 2 n − 1 
 + + + ... +  .
n n n n n
Information for Students in MATH 141 2009 01 3183

(d) [3 MARKS] Suppose it is known that f 0 (x) = 4 cosh x for all x. Showing all your
work, determine the value of f (1)− f (−1), expressed in terms of the values of either
exponentials or hyperbolic functions.
Z x2
d t
(e) [4 MARKS] Evaluate et dt when x = 1 .
dx 12

SHOW ALL YOUR WORK!

2. For each of the following series you are expected to apply one or more tests for conver-
gence or divergence to determine whether the series is absolutely convergent, condition-
ally convergent, or divergent. All tests used must be named, and all statements must be
carefully justified.
X

(−n − 2)n (n − 2)n
(a) [4 MARKS]
n=1
(2n2 + 1)n
X

n!
(b) [4 MARKS] (−1)n+1
n=1
n2 2n
X

1
(c) [4 MARKS] (−1)n sin
n=1
n

3. BRIEF SOLUTIONS Express each of the following as a definite integral or a sum,


product, or quotient of several definite integrals, simplified as much as possible; you are
not expected to evaluate the integrals.
R is defined to be the region enclosed by the curves x + y = 6 and y = x2 ; C is the arc
y = 3 x (−1 ≤ x ≤ 2).

(a) [3 MARKS] The region R is rotated about the x-axis. Give an integral or sum of
integrals whose value is the volume of the resulting solid.
DEFINITE INTEGRAL(S) ONLY (DO NOT EVALUATE)

(b) [3 MARKS] The region R is rotated about the line x = 5. Give an integral or sum
of integrals whose value is the volume of the resulting solid.
Information for Students in MATH 141 2009 01 3184

DEFINITE INTEGRAL(S) ONLY (DO NOT EVALUATE)

(c) [3 MARKS] Express in terms of integrals — which you need not evaluate — the
average length that R cuts off from the vertical lines which it meets.
DEFINITE INTEGRAL(S) ONLY (DO NOT EVALUATE)

(d) [2 MARKS] Give an integral whose value is the length of C; you need not evaluate
the integral.
DEFINITE INTEGRAL(S) ONLY (DO NOT EVALUATE)

(e) [3 MARKS] Given an integral whose value is the area of the surface generated by
rotating C about the line y = −1; you need not evaluate the integral.
DEFINITE INTEGRAL(S) ONLY (DO NOT EVALUATE)

4. SHOW ALL YOUR WORK!


[12 MARKS] Evaluate the indefinite integral
Z x  x2 − 4 (x − 2) + 4
 dx .
x2 − 4 (x − 2)

5. SHOW ALL YOUR WORK!


Showing all your work, evaluate each of the following:
Information for Students in MATH 141 2009 01 3185

Z
(a) [4 MARKS] e−x · cos x dx
5
Z2
x
(b) [5 MARKS] √ dx
8 + 2x − x2
− 21
Z !
2 1
(c) [4 MARKS] cos x + · tan2 x dx
cos2 x

6. SHOW ALL YOUR WORK!


Consider the arc C defined by
x = x(t) = cos t + t sin t
y = y(t) = sin t − t cos t ,
where 0 ≤ t ≤ π2 .
d2 y
(a) [6 MARKS] Determine as a function of t the value of .
dx2
(b) [6 MARKS] Determine the area of the surface generated by revolving C about the
y-axis.
7. SHOW ALL YOUR WORK!

(a) [5 MARKS] Showing detailed work, determine whether the following integral is
convergent; if it is convergent, determine its value:
Z π
sec x dx .
π
2

X

4
(b) [5 MARKS] Showing all your work, carefully determine whether the series
n=3
n ln n
is convergent.
(c) [3 MARKS] Showing all your work, determine whether the following sequence
converges; if it converges, find its limit:
a1 = 1.
a2 = 1.23
a3 = 1.2345
a4 = 1.234545
a5 = 1.23454545
a6 = 1.2345454545
Information for Students in MATH 141 2009 01 3186

etc., where each term after a2 is obtained from its predecessor by the addition on
the right of the decimal digits 45.

8. SHOW ALL YOUR WORK!


[10 MARKS] The polar curves

r = 2 + 2 sin θ (0 ≤ θ ≤ 2π)
and
r = 6 − 6 sin θ (0 ≤ θ ≤ 2π)

divide the plane into several regions. Showing all your work, carefully find the area of
the region bounded by these curves which contains the point (r, θ) = (1, 0).

E.21 Supplemental/Deferred Examination in MATH 141 2007 01 (One


version)
Instructions
1. Fill in the above clearly.

2. Do not tear pages from this book; all your writing — even rough work — must be handed in.
You may do rough work for this paper anywhere in the booklet.

3. The use of calculators is not permitted. This is a closed book examination. Use of regular
and translation dictionaries is permitted.

4. This examination booklet consists of this cover, Pages 1 through 8 containing questions; and
Pages 9, 10, and 11, which are blank. Your neighbour’s version of this examination may be
different from yours.

5. There are two kinds of problems on this examination, each clearly marked as to its type.

• Most of the questions on this paper require that you SHOW ALL YOUR WORK!
Their solutions are to be written in the space provided on the page where the question
is printed. When that space is exhausted, you may write on the facing page . Any
solution may be continued on the last pages, or the back cover of the booklet, but you
must indicate any continuation clearly on the page where the question is printed!
• Some of the questions on this paper require only BRIEF SOLUTIONS ; for these you
are expected to write the correct answer in the box provided; you are not asked to show
your work, and you should not expect partial marks for solutions that are not correct.
Information for Students in MATH 141 2009 01 3187

You are expected to simplify your answers wherever possible.


You are advised to spend the first few minutes scanning the problems. (Please inform the
invigilator if you find that your booklet is defective.)

6. A TOTAL OF 100 MARKS ARE AVAILABLE ON THIS EXAMINATION.

1. SHOW ALL YOUR WORK!


Z 0
1
(a) [2 MARKS] Evaluate √ dx .
− √1 1 − x 2
2
Z  
(b) [2 MARKS] Evaluate t3 cosh t4 dt .

(c) [3 MARKS] Determine one antiderivative of x ln x.


Z x
2
(d) [3 MARKS] Evaluate the integral tet dt.
−x
Z 1
d π
(e) [4 MARKS] Evaluate (ln | sec t + tan t|) dt when x = .
dx sin x 4

SHOW ALL YOUR WORK!

2. For each of the following series you are expected to apply one or more tests to determine
whether the series is convergent or divergent. All tests used must be named, and all
statements must be carefully justified.
X

1
(a) [4 MARKS]
n=3
(n + 2)(n − 2)
∞ 
X∞ X
 
(b) [4 MARKS]  3 
−i

n=1 i=n

X
∞ !n2
n+1
(c) [4 MARKS]
n=1
n

3. BRIEF SOLUTIONS Express each of the following as a definite integral or a sum,


product, or quotient of several definite integrals, simplified as much as possible; you are
not expected to evaluate the integrals.
R is defined to be the region enclosed by the curves y − x = 9 and y = (x + 3)2 ; C is the
arc x = t, y = e3t (−2 ≤ t ≤ 1).
Information for Students in MATH 141 2009 01 3188

(a) [3 MARKS] The region R is rotated about the y-axis. Give an integral or sum of
integrals whose value is the volume of the resulting solid.
DEFINITE INTEGRAL(S) ONLY (DO NOT EVALUATE)

(b) [3 MARKS] The region R is rotated about the line y = −3. Give an integral or sum
of integrals whose value is the volume of the resulting solid.
DEFINITE INTEGRAL(S) ONLY (DO NOT EVALUATE)

 
(c) [3 MARKS] Let f (t) denote the vertical distance of the point t, e3t from the x-
axis. Express in terms of integrals — which you need not evaluate — the average
value of f (t) over the interval −2 ≤ t ≤ 1.
DEFINITE INTEGRAL(S) ONLY (DO NOT EVALUATE)

(d) [2 MARKS] Give an integral whose value is the length of C; you need not evaluate
the integral.
DEFINITE INTEGRAL(S) ONLY (DO NOT EVALUATE)

(e) [3 MARKS] Given an integral whose value is the area of the surface generated by
rotating C about the line x = 1; you need not evaluate the integral.
Information for Students in MATH 141 2009 01 3189

DEFINITE INTEGRAL(S) ONLY (DO NOT EVALUATE)

4. SHOW ALL YOUR WORK!


[12 MARKS] Evaluate the indefinite integral
Z x3  x4 − 4x2  − 16
dx .
x4 − 4x2

5. SHOW ALL YOUR WORK!


Z
(a) [4 MARKS] Showing all your work, evaluate sin2 (3x) cos2 (3x) dx .
Z
1
(b) [4 MARKS] Showing all your work, evaluate √ dx .
x2 9x2 − 16
(c) [5 MARKS] Assume that Z x
f (x) = sec100 t dt
0
Z x
is known. Showing all your work, express the value of sec102 t dt in terms of
0
f (x). (You are not expected to determine f (x) explicitly.)

6. SHOW ALL YOUR WORK!


Consider the closed arc C defined by

x = x(t) = 3t2
y = y(t) = t3 − 3t ,
√ √
where − 3 ≤ t ≤ 3.

(a) [3 MARKS] Determine the area bounded by C.


(b) [3 MARKS] Determine the equation of the tangent to C at the point with parameter
1
value t = .
2
(c) [6 MARKS] Determine the area of the surface generated by revolving C about the
y-axis.
7. SHOW ALL YOUR WORK!

(a) [6 MARKS] Showing detailed work, determine whether the following integral is
convergent; if it is convergent, determine its value:
Z ∞
x
2
dx .
−∞ x + 4

(b) [7 MARKS] Showing all your work, carefully determine whether the series
X∞ √
n ln n
(−1)
n=3
n

is conditionally convergent, absolutely convergent, or divergent.

8. SHOW ALL YOUR WORK!


[10 MARKS] Find the area inside the larger loop and outside the smaller loop of the
limaçon r = 2 sin θ − 1.

E.22 Final Examination in MATH 141 2008 01 (one version)


This examination was written during a labour disruption, when the services of Teaching As-
sistants were not available for grading purposes. The following additional instructions were
distributed with the examination.

VERSION n
McGILL UNIVERSITY
FACULTY OF SCIENCE
FINAL EXAMINATION
IMPORTANT ADDITIONAL INSTRUCTIONS
MATHEMATICS 141 2008 01CALCULUS 2
EXAMINER: Professor W. G. Brown DATE: Monday, April 14th, 2008
ASSOCIATE EXAMINER: Mr. S. Shahabi TIME: 09:00 – 12:00 hours

A. Part marks will not be awarded for any part of any question worth [4 MARKS] or less.
Information for Students in MATH 141 2009 01 3191

B. To be awarded part marks on a part of a question whose maximum value is 5 marks or


more, a student’s answer must be deemed to be more than 75% correct.

C. While there are 100 marks available on this examination 80 MARKS CONSTITUTE A
PERFECT PAPER. You may attempt as many problems as you wish.

All other instructions remain valid. Where a problem requires that all work be shown, that
remains the requirement; where a problem requires only that an answer be written in a box
without work being graded, that also remains the requirement.
Students are advised to spend time checking their work; for that purpose you could verify
your answers by solving problems in more than one way. Remember that indefinite integrals
can be checked by differentiation.

W. G. Brown, Examiner.

Instructions
1. Fill in the above clearly.

2. Do not tear pages from this book; all your writing — even rough work — must be handed in.
You may do rough work for this paper anywhere in the booklet.

3. Calculators are not permitted. This is a closed book examination. Regular and translation
dictionaries are permitted.

4. This examination booklet consists of this cover, Pages 1 through 9 containing questions; and
Pages 10, 11, and 12, which are blank. Your neighbour’s version of this test may be different
from yours.

5. There are two kinds of problems on this examination, each clearly marked as to its type.

• Most of the questions on this paper require that you SHOW ALL YOUR WORK!
Their solutions are to be written in the space provided on the page where the question
is printed; in some of these problems you are instructed to write the answer in a box,
but a correct answer alone will not be sufficient unless it is substantiated by your work,
clearly displayed outside the box. When space provided for that work is exhausted, you
may write on the facing page . Any solution may be continued on the last pages, or the
back cover of the booklet, but you must indicate any continuation clearly on the page
where the question is printed!
• Some of the questions on this paper require only BRIEF SOLUTIONS ; for these you
are expected to write the correct answer in the box provided; you are not asked to show
your work, and you should not expect partial marks for solutions that are not correct.
Information for Students in MATH 141 2009 01 3192

You are expected to simplify your answers wherever possible.


You are advised to spend the first few minutes scanning the problems. (Please inform the
invigilator if you find that your booklet is defective.)

6. A TOTAL OF 100 MARKS ARE AVAILABLE ON THIS EXAMINATION.

1. SHOW ALL YOUR WORK!


Your answers must be simplified as much as possible.
Z
4 − 6x
(a) [2 MARKS] Evaluate dx .
1 + x2

ANSWER (SHOW YOUR WORK OUTSIDE THE BOX)

Z2 p
(b) [3 MARKS] Evaluate y2 y3 + 1 dy .
0
Information for Students in MATH 141 2009 01 3193

ANSWER (SHOW YOUR WORK OUTSIDE THE BOX)

Z
(c) [3 MARKS] Evaluate sin(18 θ) cos(30 θ) dθ .

ANSWER (SHOW YOUR WORK OUTSIDE THE BOX)

2. SHOW ALL YOUR WORK!


Z √
3
d
(a) [3 MARKS] Simplifying your answer as much as possible, evaluate earcsin z dz .
dx −x
Information for Students in MATH 141 2009 01 3194

ANSWER (SHOW YOUR WORK OUTSIDE THE BOX)

(b) [4 MARKS] For the interval 2 ≤ x ≤ 5 write down the Riemann sum for the
function f (x) = 3 − x, where the sample points are the left end-point of each of n
subintervals of equal length.

ANSWER ONLY

(c) [4 MARKS] Determine the value of the preceding Riemann sum as a function of
n, simplifying your work as much as possible. (NOTE: You are being asked to
determine the value of the sum as a function of n, not the limit as n → ∞.)
Information for Students in MATH 141 2009 01 3195

ANSWER (SHOW YOUR WORK OUTSIDE THE BOX)

3. SHOW ALL YOUR WORK!


For each of the following series determine whether the series diverges, converges condi-
tionally, or converges absolutely. All of your work must be justified; prior to using any
test you are expected to demonstrate that the test is applicable to the problem.
X ∞ !
1
(a) [4 MARKS] √
n=3 n ln n
X ∞ √
4n + 5
(b) [4 MARKS] (−1)n+1
n=1
3n + 10
X ∞ ! !!
−1 1 −1 1
(c) [4 MARKS] cot − cot
n=1
n + 1 n

4. BRIEF SOLUTIONS R is defined to be the region in the first quadrant enclosed by the
curves 2y = x, y = 2x, and x2 + y2 = 5.
(a) [4 MARKS] The region R is rotated about the line x = −1. Give an integral or sum
of integrals whose value is the volume of the resulting solid.
DEFINITE INTEGRAL(S) ONLY (DO NOT EVALUATE)

(b) [4 MARKS] Let L(a) denote the length of the portion of line y = a which lies inside
R. Express in terms of integrals — which you need not evaluate — the average of
the positive lengths L(a).
DEFINITE INTEGRAL(S) ONLY (DO NOT EVALUATE)
Information for Students in MATH 141 2009 01 3196

(c) [4 MARKS] Let C1 be the curve x(t) = t, y(t) = cosh t (0 ≤ t ≤ ln 2). Simplifying
your answer as much as possible, find the length of C1 .
ANSWER ONLY

5. SHOW ALL YOUR WORK!

(a) [8 MARKS] Evaluate the indefinite integral


Z
36
dx .
(x + 4)(x − 2)2
Z∞
36
(b) [4 MARKS] Determine whether dx converges.
(x + 4)(x − 2)2
3
If it converges, find its value.

6. SHOW ALL YOUR WORK!


Showing all your work, evaluate each of the following:
Z p

(a) [4 MARKS] e x dx

ANSWER (SHOW YOUR WORK OUTSIDE THE BOX)


Information for Students in MATH 141 2009 01 3197

Z0
x
(b) [5 MARKS] √ dx
3 − 4x − 4x2
− 21

ANSWER (SHOW YOUR WORK OUTSIDE THE BOX)

Z π
(c) [4 MARKS] sin2 t cos4 t dt .
0

ANSWER (SHOW YOUR WORK OUTSIDE THE BOX)

7. SHOW ALL YOUR WORK!


Information for Students in MATH 141 2009 01 3198

Consider the curve C2 defined by x = x(t) = 1 + e−t , y = y(t) = t + t2 .

(a) [2 MARKS] Determine the coordinates of all points where C2 intersects the x-axis.

ANSWER (SHOW YOUR WORK OUTSIDE THE BOX)

(b) [2 MARKS] Determine the coordinates of all points of C2 where the tangent is
horizontal.

ANSWER (SHOW YOUR WORK OUTSIDE THE BOX)


Information for Students in MATH 141 2009 01 3199

(c) [6 MARKS] Determine the area of the finite region bounded by C2 and the x-axis.

ANSWER (SHOW YOUR WORK OUTSIDE THE BOX)

8. SHOW ALL YOUR WORK!

(a) [5 MARKS] Showing all your work, determine whether the series
X

√ √ √ 
n n+2− n−2
n=2

is convergent or divergent.
(b) [5 MARKS] Showing all your work, determine whether the following sequence
converges; if it converges, find its limit:
a1 = 3.
a2 = 3.14
a3 = 3.1414
a4 = 3.141414
a5 = 3.14141414
a6 = 3.1414141414
etc., where each term after a2 is obtained from its predecessor by the addition on
the right of the decimal digits 14.
Information for Students in MATH 141 2009 01 3200

9. SHOW ALL YOUR WORK!


Curves C3 and C4 , respectively represented by polar equations
r = 4 + 2 cos θ (0 ≤ θ ≤ 2π) (109)
and
r = 4 cos θ + 5 (0 ≤ θ ≤ 2π) , (110)
divide the plane into several regions.
(a) [8 MARKS] Showing all your work, carefully find the area of the one region which
is bounded by C3 and C4 and contains the pole.
(b) [4 MARKS] Find another equation — call it (110*) — that also represents C4 , and
has the property that there do not exist coordinates (r, θ) which satisfy equations
(109) and (110*) simultaneously. You are expected to show that equations (109)
and (110*) have no simultaneous solutions.

E.23 Supplemental/Deferred Examination in MATH 141 2008 01 (one


version)
Instructions
1. Fill in the above clearly.

2. Do not tear pages from this book; all your writing — even rough work — must be handed in.
You may do rough work for this paper anywhere in the booklet.

3. The use of calculators is not permitted. This is a closed book examination. Use of regular
and translation dictionaries is permitted.

4. This examination booklet consists of this cover, Pages 1 through 8 containing questions; and
Pages 9, 10, and 11, which are blank. Your neighbour’s version of this examination may be
different from yours.

5. There are two kinds of problems on this examination, each clearly marked as to its type.

• Most of the questions on this paper require that you SHOW ALL YOUR WORK!
Their solutions are to be written in the space provided on the page where the question
is printed. When that space is exhausted, you may write on the facing page . Any
solution may be continued on the last pages, or the back cover of the booklet, but you
must indicate any continuation clearly on the page where the question is printed!
• Some of the questions on this paper require only BRIEF SOLUTIONS ; for these you
are expected to write the correct answer in the box provided; you are not asked to show
your work, and you should not expect partial marks for solutions that are not correct.
Information for Students in MATH 141 2009 01 3201

You are expected to simplify your answers wherever possible.


You are advised to spend the first few minutes scanning the problems. (Please inform the
invigilator if you find that your booklet is defective.)

6. A TOTAL OF 100 MARKS ARE AVAILABLE ON THIS EXAMINATION.

1. SHOW ALL YOUR WORK!


Z 0
1
(a) [2 MARKS] Evaluate √ dx .
− √1 1 − x 2
2
Z  
(b) [2 MARKS] Evaluate t3 cosh t4 dt .

(c) [3 MARKS] Determine one antiderivative of x ln x.


Z x
2
(d) [3 MARKS] Evaluate the integral tet dt.
−x
Z 1
d π
(e) [4 MARKS] Evaluate (ln | sec t + tan t|) dt when x = .
dx sin x 4

SHOW ALL YOUR WORK!

2. For each of the following series you are expected to apply one or more tests to determine
whether the series is convergent or divergent. All tests used must be named, and all
statements must be carefully justified.
X

1
(a) [4 MARKS]
n=3
(n + 2)(n − 2)
∞ 
X∞ X
 
(b) [4 MARKS]  3 
−i

n=1 i=n

X
∞ !n2
n+1
(c) [4 MARKS]
n=1
n

3. BRIEF SOLUTIONS Express each of the following as a definite integral or a sum,


product, or quotient of several definite integrals, simplified as much as possible; you are
not expected to evaluate the integrals.
R is defined to be the region enclosed by the curves y − x = 9 and y = (x + 3)2 ; C is the
arc x = t, y = e3t (−2 ≤ t ≤ 1).
Information for Students in MATH 141 2009 01 3202

(a) [3 MARKS] The region R is rotated about the y-axis. Give an integral or sum of
integrals whose value is the volume of the resulting solid.
DEFINITE INTEGRAL(S) ONLY (DO NOT EVALUATE)

(b) [3 MARKS] The region R is rotated about the line y = −3. Give an integral or sum
of integrals whose value is the volume of the resulting solid.
DEFINITE INTEGRAL(S) ONLY (DO NOT EVALUATE)

 
(c) [3 MARKS] Let f (t) denote the vertical distance of the point t, e3t from the x-
axis. Express in terms of integrals — which you need not evaluate — the average
value of f (t) over the interval −2 ≤ t ≤ 1.
DEFINITE INTEGRAL(S) ONLY (DO NOT EVALUATE)

(d) [2 MARKS] Give an integral whose value is the length of C; you need not evaluate
the integral.
DEFINITE INTEGRAL(S) ONLY (DO NOT EVALUATE)

(e) [3 MARKS] Given an integral whose value is the area of the surface generated by
rotating C about the line x = 1; you need not evaluate the integral.
Information for Students in MATH 141 2009 01 3203

DEFINITE INTEGRAL(S) ONLY (DO NOT EVALUATE)

4. SHOW ALL YOUR WORK!


[12 MARKS] Evaluate the indefinite integral
Z x3  x4 − 4x2  − 16
dx .
x4 − 4x2

5. SHOW ALL YOUR WORK!


Z
(a) [4 MARKS] Showing all your work, evaluate sin2 (3x) cos2 (3x) dx .
Z
1
(b) [4 MARKS] Showing all your work, evaluate √ dx .
x2 9x2 − 16
(c) [5 MARKS] Assume that Z x
f (x) = sec100 t dt
0
Z x
is known. Showing all your work, express the value of sec102 t dt in terms of
0
f (x). (You are not expected to determine f (x) explicitly.)

6. SHOW ALL YOUR WORK!


Consider the closed arc C defined by

x = x(t) = 3t2
y = y(t) = t3 − 3t ,
√ √
where − 3 ≤ t ≤ 3.

(a) [3 MARKS] Determine the area bounded by C.


(b) [3 MARKS] Determine the equation of the tangent to C at the point with parameter
1
value t = .
2
(c) [6 MARKS] Determine the area of the surface generated by revolving C about the
y-axis.
Information for Students in MATH 141 2009 01 3204

7. SHOW ALL YOUR WORK!

(a) [6 MARKS] Showing detailed work, determine whether the following integral is
convergent; if it is convergent, determine its value:
Z ∞
x
2
dx .
−∞ x + 4

(b) [7 MARKS] Showing all your work, carefully determine whether the series
X∞ √
ln n
(−1)n
n=3
n

is conditionally convergent, absolutely convergent, or divergent.

8. SHOW ALL YOUR WORK!


[10 MARKS] Find the area inside the larger loop and outside the smaller loop of the
limaçon r = 2 sin θ − 1.
Information for Students in MATH 141 2009 01 4001

F WeBWorK
F.1 Frequently Asked Questions (FAQ)
F.1.1 Where is WeBWorK?
WeBWorK is located on Web servers of the Department of Mathematics and Statistics, and is
accessible at the following URL’s:

http://msr04.math.mcgill.ca/webwork/m141w08
or
http://msr05.math.mcgill.ca/webwork/m141w08

If your student number ends with an odd digit — 1, 3, 5, 7, or 9 — you should access the URL

http://msr05.math.mcgill.ca/webwork/m141w08;

if your student number ends with an even digit — 0, 2, 4, 6, or 8 — you should access

http://msr04.math.mcgill.ca/webwork/m141w08.

If you access WeBWorK through WebCT, the link on your page will have been programmed
to take you to the correct WeBWorK server automatically.

F.1.2 Do I need a password to use WeBWorK?


You will need a user code and a password.

Your user code. Your user code will be your 9-digit student number.

Your password. The WeBWorK system is administered by the Mathematics and Statistics
Department, and is not accessible through the myMcGill Portal; your initial password will
be different from your MINERVA password, but you could change it to that if you wish.
Your initial password will be your 9-digit student ID number. You will be able to change
this password after you sign on to WeBWorK.61
61
If you forget your password you will have to send a message to Professor Brown so that the system adminis-
trator may be instructed to reset the password at its initial value.

UPDATED TO April 12, 2009


Information for Students in MATH 141 2009 01 4002

Your e-mail address. The WeBWorK system requires each user to have an e-mail address.
After signing on to WeBWorK, you should verify that the e-mail address shown is the one that
you prefer. You should endeavour to keep your e-mail address up to date, since the instructors
may send messages to the entire class through this route.
We suggest that you use either your UEA62 or your po-box address. You may be able to
forward your mail from these addresses to another convenient address, (cf. §4.)

F.1.3 Do I have to pay an additional fee to use WeBWorK?


WeBWorK is available to all students registered in the course at no additional charge.

F.1.4 When will assignments be available on WeBWorK?


Each assignment will have a begin date and a due date. The assignment is available to you
after the begin date; solutions will be made available soon after the due date.

F.1.5 Do WeBWorK assignments cover the full range of problems that I should be able
to solve in this course?
The questions on the WeBWorK assignments (A1 through A7 ) are a sampling of some types
of problem you should be able to solve after successfully completing this course. Some types
of calculus problems do not lend themselves to this kind of treatment, and may not appear on
the WeBWorK assignments. Use of WeBWorK does not replace studying the textbook —
including the worked examples, attending lectures and tutorials, and working exercises
from the textbook — using the Student Solutions Manual [9] to check your work. Students
are cautioned not to draw conclusions from the presence, absence, or relative frequencies of
problems of particular types, or from particular sections of the textbook. Certain sections
of the textbook remain examination material even though no problems are included in the
WeBWorK assignments.

F.1.6 May I assume that the distribution of topics on quizzes and final examinations will
parallel the distribution of topics in the WeBWorK assignments?
No! While the order of topics on WeBWorK assignments should conform to the order of the
lectures, there are some topics on the syllabus that will not appear in WeBWorK questions.
Use WeBWorK for the areas it covers, and supplement it by working problems from your
textbook. Also, remember that WeBWorK — which checks answer only — cannot ascertain
whether you are using a correct method for solving problems. But, if you write out a solution
to an odd-numbered textbook problem, you can compare it with the solution in the Solutions
62
Uniform E-mail Address
Information for Students in MATH 141 2009 01 4003

Manual; and, if in doubt, you can show your work to a Teaching Assistant at one of the many
office hours that they hold through the week.

F.1.7 WeBWorK provides for different kinds of “Display Mode”. Which should I use?
“Display mode” is the mode that you enter when you first view a problem; and, later, when you
submit your answer. You may wish to experiment with the different formats. The default is im-
ages mode, which should look similar to the version that you print out (cf. next question). The
lowest quality is text mode, which is essentially the way the author of the problem entered his
data into the system; this mode is related to the TEX and LATEX systems that mathematicians use
in typesetting their documents; the notes that you are reading here were prepared using LATEX;
it contains formatting instructions in a “markup” language, and is difficult for inexperienced
readers. The mode called jsMath requires the presence of certain TEX fonts on your computer,
and should be avoided unless you are using your own computer, and are prepared to follow
the instructions to install the new fonts. This requires a major investment of time, and is not
recommended unless there are other reasons why you need to work with TEX or LATEX.

F.1.8 WeBWorK provides for printing assignments in “Portable Document Format”


(.pdf), “PostScript” (.ps) forms. Which should I use?
Most newer home computers have already been loaded with the Acrobat Reader for .pdf files;
if the Reader has not been installed on your computer63 , you will find instructions for down-
loading this (free) software in §1.5.5 of these notes. If you are not happy with .pdf files, and
wish to print and view PostScript files, you may require such (free) software as Ghostscript
and Ghostview, available at

http://www.cs.wisc.edu/∼ghost/gsview/index.html

Most computers available to you on campus should be capable of printing in either of .pdf
and PostScript formats.

F.1.9 What is the relation between WeBWorK and WebCT?


There is none. WebCT is the proprietary system of Web Course Tools that has been imple-
mented by McGill University. You may access the web page for this course, and WeBWorK
through your WebCT account64 , and WebCT will link you to the appropriate server for WeB-
WorK. If you follow this route to WeBWorK, you will still have to log in when you reach
the WeBWorK site. At the present time we will be using WebCT primarily for the posting of
63
At the time these notes were written, the latest version of the Reader was 7.0.8, but recent, earlier versions
should also work properly.
64
http://webct.mcgill.ca
Information for Students in MATH 141 2009 01 4004

grades, and as a convenient repository for links to notes and announcements in the course. We
are not planning to use the potential WebCT sites that exist for the tutorial sections: use only
the site for the lecture section in which you are registered.

F.1.10 Which browser should I use for WeBWorK?


We recommend that you use Internet Explorer, Netscape, or Mozilla. While other browsers
may give satisfactory results, your instructors and tutors do not have time to correct errors in
your WeBWorK records that could be attributed to idiosyncracies in another browser. Infor-
mation about browsers supported by WebCT may be obtained at
http://www.mcgill.ca/webct/

F.1.11 What do I have to do on WeBWorK?


After you sign on to WeBWorK, and click on “Begin Problem Sets”, you will see a list of As-
signments, each with a due date. Since there is no limit to the number of attempts at problems
on P0 or the other “Practice” assignments, you may play with these assignments to learn how
to use the WeBWorK software.
You may print out a copy of your assignment by clicking on “Get hard copy”. This is
your version of the assignment, and it will differ from the assignments of other students in
the course. You should spend some time working on the assignment away from the computer.
When you are ready to submit your solutions, sign on again, and select the same assignment.
This time click on “Do problem set”. You can expect to become more comfortable with the
system as you attempt several problems; but, in the beginning, there are likely to be situations
where you cannot understand what the system finds wrong with some of your answers. It is
useful to click on the Preview Answers button to see how the system interprets an answer that
you have typed in. As the problems may become more difficult, you may have to refer to the
“Help” page, and also to the “List of functions” which appears on the page listing the problems.
Don’t submit an answer until you are happy with the interpretation that the Preview Answers
button shows that the system will be taking of your answer.

F.1.12 How can I learn how to use WeBWorK?


As soon as your instructor announces that the WeBWorK accounts are ready, sign on and try
assignment P0 , which does not count. The system is self-instructive, so we will not burden
you with a long list of instructions.

You will need to learn how to enter algebraic expressions into WeBWorK as it is coded to
read what you type in a way that may different from what you expect. For example, the symbol
ˆ is used for writing exponents (powers). If you type 2ˆ3, WeBWorK will interpret this as
Information for Students in MATH 141 2009 01 4005

23 = 8. However, if you type 2ˆ3+x, WeBWorKwill interpret it as 23 + x, i.e. as 8 + x; if you


wish to write 23+x , you have to type 2ˆ(3+x). You may obtain more information from the List
of Available Functions, available online, or at

http://webwork.math.rochester.edu/webwork_system_html
/docs/docs/pglanguage/availablefunctions.html

F.1.13 Where should I go if I have difficulties with WeBWorK ?


If you have difficulties signing on to WeBWorK, or with the viewing or printing functions on
WeBWorK, or with the specific problems on your version of an assignment, you may send
an e-mail distress message directly from WeBWorK by clicking on the Feedback button.
You may also report the problem to your instructor and/or your tutor, but the fastest way of
resolving your difficulty is usually the Feedback . Please give as much information as you
can. (All of the instructors and tutors are able to view from within WeBWorK the answers
that you have submitted to questions.)
If your problem is mathematical, and you need help in solving a problem, you should
consult one of the tutors at their office hours; you may go to any tutor’s office hours, not only
to the hours of the tutor of the section in which you are registered.

F.1.14 Can the WeBWorK system ever break down or degrade?


Like all computer systems, WeBWorK can experience technical problems. The systems man-
ager is continually monitoring its performance. If you experience a difficulty when online,
please click on the Feedback button and report it. If that option is not available to you,
please communicate with either instructor by e-mail.
If you leave your WeBWorK assignment until the hours close to the due time on the due
date, you should not be surprised if the system is slow to respond. This is not a malfunction,
but is simply a reflection of the fact that other students have also been procrastinating! To
benefit from the speed that the system can deliver under normal conditions, do not delay your
WeBWorK until the last possible day! If a systems failure interferes with the due date of an
assignment, arrangements could be made to change that date, and an e-mail message could
be broadcast to all users (to the e-mail addresses on record), or a message could be posted on
WeBCT or the WeBWorK sign-on screen.65

F.1.15 How many attempts may I make to solve a particular problem on WeBWorK?
Practice Assignments P1 — P6 are intended to prepare you for Assignments A1 — A6 , and
permit unlimited numbers of attempts; your grades on these “Practice” do not count in your
65
But slowness of the system just before the due time will not normally be considered a systems failure.
Information for Students in MATH 141 2009 01 4006

term mark. For the problems on assignments A1 — A6 you will normally be permitted about 5
tries: read the instructions at the head of the assignment.

F.1.16 Will all WeBWorK assignments have the same length? the same value?
The numbers of problems on the various assignments may not be the same, and the individual
problems may vary in difficulty. Assignments A1 — A6 will count equally in the computation
of your grade.

F.1.17 Is WeBWorK a good indicator of examination performance?


A low grade on WeBWorK has often been followed by a low grade on the examination.
A high grade on WeBWorK does not necessarily indicate a likely high grade on the exam-
ination.
To summarize: WeBWorK alone is not enough to prepare this course; but students who
don’t do WeBWorK appear to have a poor likelihood of success in MATH 141: that is one
reason why we have made the WeBWorK assignments compulsory.
Information for Students in MATH 141 2009 01 5001

G Contents of the DVD disks for


Larson/Hostetler/Edwards
These excellent disks were produced to accompany the textbook, Calculus of a Single Vari-
able: Early Transcendental Functions, 3rd Edition[28] (called LHE in the charts below). The
correspondence shown to sections of [7] are only approximate. (NOTE THAT THIS BOOK
DOES NOT FOLLOW STEWART’S CONVENTIONS FOR INVERSE SECANT/COSECANT!)
[All references in this table are to the 5th edition of Stewart, [7].]
DVD LHE Stewart
# Section Subject Minutes Section
1 P Chapter P: Preparation for Calculus
1 P.1 Graphs and Models 45
1 P.2 Linear Models and Rates of Change 27 A10
1 P.3 Functions and Their Graphs 48 1.1
1 P.4 Fitting Models to Data 21 1.2
1 P.5 Inverse Functions 48 1.6
1 P.6 Exponential and Logarithmic Functions 30 1.5
DVD LHE Stewart
# Section Subject Minutes Section
1 1 Chapter 1: Limits and Their Properties
1 1.1 A Preview of Calculus 11 2.1
1 1.2 Finding Limits Graphically and Numerically 25 2.2, 2.4
1 1.3 Evaluating Limits Analytically 28 2.3
1 1.4 Continuity and One-Sided Limits 22 2.5
1 1.5 Infinite Limits 18 2.6
DVD LHE Stewart
# Section Subject Minutes Section
1 2 Chapter 2: Differentiation
1 2.1 The Derivative and the Tangent Line Problem 68 2.1
1 2.2 Basic Differentiation Rules and Rates of 34 2.3
Change
1 2.3 The Product and Quotient Rules and Higher 25 3.2, 3.7
Order Derivatives
Information for Students in MATH 141 2009 01 5002

DVD LHE Stewart


# Section Subject Minutes Section
2 2 Chapter 2 (continued): Differentiation
2 2.4 The Chain Rule 44 3.5
2 2.5 Implicit Differentiation 50 3.6
2 2.6 Derivatives of Inverse Functions 17 3.5, 3.8, 3.9
2 2.7 Related Rates 34 3.10
2 2.8 Newton’s Method 26 4.9
DVD LHE Stewart
# Section Subject Minutes Section
2 3 Chapter 3: Applications of Differentiation
2 3.1 Extrema on an Interval 41 4.1
2 3.2 Rolle’s Theorem and the Mean Value Theo- 15 4.2
rem
2 3.3 Increasing and Decreasing Functions and the 19 4.3
First Derivative Test
2 3.4 Concavity and the Second Derivative Test 24 4.3
2 3.5 Limits at Infinity 23 2.6
2 3.6 A Summary of Curve Sketching 43 4.5
2 3.7 Optimization Problems 37 4.7
2 3.8 Differentials 51 3.11
DVD LHE Stewart
# Section Subject Minutes Section
3 4 Chapter 4: Integration
3 4.1 Antiderivatives and Indefinite Integration 40 4.10
DVD LHE Stewart
# Section Subject Minutes Section
4 7 Chapter 7: Integration by Parts Trigonomet-
ric Substitution Partial Fractions L’Hôpital’s
Rule
4 7.7 Indeterminate Forms and L’Hôpital’s Rule 22 4.4
(The coverage extends to part of the material for Math 141 as well.)
Information for Students in MATH 141 2009 01 6001

H References

H.1 Stewart Calculus Series


[1] J. Stewart, Single Variable Calculus (Early Transcendentals), Sixth Edition. Thomson *
Brooks/Cole (2008). ISBN 0-495-01169-X.

[2] J. Stewart, Calculus (Early Transcendentals), Sixth Edition. Thomson * Brooks/Cole


(2008). ISBN 0-495-01166-5.

[3] D. Anderson, J. A. Cole, D. Drucker, Student Solutions Manual for Stewart’s Sin-
gle Variable Calculus (Early Transcendentals), Sixth Edition. Thomson * Brooks/Cole
(2008). ISBN 0-495-01240-8.

[4] J. Stewart, Single Variable Calculus (Early Transcendentals), Sixth Edition. Thomson *
Brooks/Cole (2008); bundled with Student Solutions Manual for Stewart’s Single Vari-
able Calculus (Early Transcendentals), Sixth Edition. Thomson * Brooks/Cole (2008).
ISBN 0-495-42966-X.

[5] R. St. Andre, Study Guide for Stewart’s Single Variable Calculus (Early Transcenden-
tals), Sixth Edition. Thomson * Brooks/Cole (2008). ISBN 0-495-01239-4.

[6] J. Stewart, Multivariable Calculus (Early Transcendentals), Sixth Edition. Thomson *


Brooks/Cole (2008). ISBN 0-495-?????-?.

[7] J. Stewart, Single Variable Calculus (Early Transcendentals), Fifth Edition. Thomson *
Brooks/Cole (2003). ISBN 0-534-39330-6.

[8] J. Stewart, Calculus (Early Transcendentals), Fifth Edition. Thomson * Brooks/Cole


(2003). ISBN 0-534-39321-7.

[9] D. Anderson, J. A. Cole, D. Drucker, Student Solutions Manual for Stewart’s Sin-
gle Variable Calculus (Early Transcendentals), Fifth Edition. Thomson * Brooks/Cole
(2003). ISBN 0-534-39333-0.

[10] J. Stewart, Single Variable Calculus (Early Transcendentals), Fifth Edition. Thomson *
Brooks/Cole (2003); bundled with Student Solutions Manual for Stewart’s Single Vari-
able Calculus (Early Transcendentals), Fifth Edition. Thomson * Brooks/Cole (2003).
ISBN 0-17-6425411.

[11] J. Stewart, Single Variable Essential Calculus (Early Transcendentals). Thomson *


Brooks/Cole (2006). Thomson * Brooks/Cole (2003). ISBN 0-495-10957-6.
Information for Students in MATH 141 2009 01 6002

[12] J. Stewart, Calculus (Early Transcendentals), Fifth Edition. Thomson * Brooks/Cole


(2003); bundled with Student Solutions Manual for Stewart’s Single Variable Calculus
(Early Transcendentals), Fifth Edition. Thomson * Brooks/Cole (2003). ISBN 0-534-
10307-3.

[13] R. St. Andre, Study Guide for Stewart’s Single Variable Calculus (Early Transcenden-
tals), Fifth Edition. Thomson * Brooks/Cole (2003). ISBN 0-534-39331-4.

[14] Video Outline for Stewart’s Calculus (Early Transcendentals), Fifth Edition. Thomson
* Brooks/Cole (2003). ISBN 0-534-39325-X. 17 VCR tapes.

[15] Interactive Video Skillbuilder CD for Stewart’s Calculus: Early Transcendentals, 5th
Edition. Thomson * Brooks/Cole (2003). ISBN 0-534-39326-8.

[16] H. Keynes, J. Stewart, D. Clegg, Tools for Enriching Calculus, CD to accompany [7]
and [8]. Thomson * Brooks/Cole (2003). ISBN 0-534-39731-X.

[17] J. Stewart, Single Variable Calculus (Early Transcendentals), Fourth Edition.


Brooks/Cole (1999). ISBN 0-534-35563-3.

[18] J. Stewart, Calculus (Early Transcendentals), Fourth Edition. Brooks/Cole (1999). ISBN
0-534-36298-2.

[19] D. Anderson, J. A. Cole, D. Drucker, Student Solutions Manual for Stewart’s Single
Variable Calculus (Early Transcendentals), Fourth Edition. Brooks/Cole (1999). ISBN
0-534-36301-6.

[20] J. Stewart, L. Redlin, S. Watson, Precalculus: Mathematics for Calculus, Enhanced


Review Edition. Thomson * Brooks/Cole. (2006). ISBN: 0-495-39276-6.

[21] J. Stewart, Trigonometry for Calculus. Thomson * Brooks/Cole. ISBN: 0-17-641227-1.

H.2 Other Calculus Textbooks


H.2.1 R. A. Adams

[22] R. A. Adams, Calculus, Single Variable, Fifth Edition. Addison, Wesley, Longman,
Toronto (2003). ISBN 0-201-79805-0.

[23] R. A. Adams, Calculus of Several Variables, Fifth Edition. Addison, Wesley, Longman,
Toronto (2003). ISBN 0-201-79802-6.
Information for Students in MATH 141 2009 01 6003

[24] R. A. Adams, Calculus: A Complete Course, Fifth Edition. Addison, Wesley, Longman,
Toronto (2003). ISBN 0-201-79131-5.
[25] R. A. Adams, Student Solution Manual for Adams’, Calculus: A Complete Course, Fifth
Edition. Addison, Wesley, Longman, Toronto (2003). ISBN 0-201-79803-4.
[26] R. A. Adams, Calculus: A Complete Course, with Solution Manual, Fifth Edition. Ad-
dison, Wesley, Longman, Toronto (2003). ISBN 0-131-30565-4.
[27] R. A. Adams, Calculus: A Complete Course Manual, Sixth Edition. Addison, Wesley,
Longman, Toronto (2006). ISBN 0-321-27000-2.

H.2.2 Larson, Hostetler, et al.

[28] Calculus Instructional DVD Program, for use with (inter alia) Lar-
son/Hostetler/Edwards, Calculus of a Single Variable: Early Transcendental Functions,
Third Edition [29]. Houghton Mifflin (2003). ISBN 0-618-25097-2.
[29] R. Larson, R. P. Hostetler, B. H. Edwards, D. E. Heyd, Calculus, Early Transcenden-
tal Functions, Third Edition. Houghton Mifflin Company, Boston (2003). ISBN 0-618-
22307-X.

H.2.3 Edwards and Penney

[30] C. H. Edwards, Jr., and D. E. Penney, Single Variable Calculus, Early Transcendentals,
Sixth Edition. Prentice Hall, Englewood Cliffs, NJ (2002). ISBN 0-13-041407-7.
[31] C. H. Edwards, Jr., and D. E. Penney, Calculus with Analytic Geometry, Early Tran-
scendentals Version, Fifth Edition. Prentice Hall, Englewood Cliffs, NJ (1997). ISBN
0-13-793076-3.
[32] C. H. Edwards, Jr., and D. E. Penney, Student Solutions Manual for Calculus with Ana-
lytic Geometry, Early Transcendentals Version, Fifth Edition. Prentice Hall, Englewood
Cliffs, NJ (1997). ISBN 0-13-079875-4.
[33] C. H. Edwards, Jr., and D. E. Penney, Single Variable Calculus with Analytic Geome-
try, Early Transcendentals Version, Fifth Edition. Prentice Hall, Englewood Cliffs, NJ
(1997). ISBN 0-13-793092-5.
[34] C. H. Edwards, Jr., and D. E. Penney, Student Solutions Manual for Single Variable
Calculus with Analytic Geometry, Early Transcendentals Version, Fifth Edition. Pren-
tice Hall, Englewood Cliffs, NJ (1997). ISBN 0-13-095247-1.
Information for Students in MATH 141 2009 01 6004

H.2.4 Others, not “Early Transcendentals”

[35] G. H. Hardy, A Course of Pure Mathematics, 10th edition. Cambridge University Press
(1967).

[36] H. S. Hall, S. R. Knight, Elementary Trigonometry, Fourth Edition. Macmillan and


Company, London (1905).

[37] S. L. Salas, E. Hille, G. J. Etgen, Calculus, One and Several Variables, 10th Edition.
John Wiley & Sons, Inc. (2007). ISBN 0471-69804-0.

H.3 Other References


[38] G. N. Berman, A Problem Book in Mathematical Analysis. Mir Publishers, Moscow,
(1975) 1977

[39] D. Ebersole, D. Schattschneider, A. Sevilla, K. Somers, A Companion to Calculus.


Brooks/Cole (1995). ISBN 0-534-26592-8.

[40] McGill Undergraduate Programs Calendar 2007/2008. Also accessible at


http://coursecalendar.mcgill.ca/ugcal200708/wwhelp/wwhimpl/js/html/wwhelp.htm

You might also like